You are on page 1of 286

,Pad 9 2217 6510 .

'

+ G R< UI usmle rx com/ap l#Qmaxl + D :


Item 1 of 45
QID 2600
L

J.
• Mark -<l
PreYlDu,
t>-
NeKt
all
lab"" ues
~.
Notes
~'t
c .. kul~tor

· 1
· 2
•• The accompanying image shows a relatively common congenital anomaly.

·3
· 4
••
·5
· 6 ••
· 7
·8
••
.,0 •
· 9
.
· 11 •
· 12 •
· 13 •
· 14 •
· 15 •

·16.
· 17 •
· 18 •
· 19 •

Which of the following is the embryologic explanation for this malformation?

•••
· 2
134415: zheng
·3 A. Expansion of the foramen cecum
· 4 We value your feedback!
·5
· 6
•• B. Failure of fusion of the lateral palatine processes

C. Failure of fusion of the medial palatine processes


· 7
·8
•• O. Failu re of fusion of the nasal processes

.,0 •
· 9 E. Incomplete formation of the nasal septum
.
· 11 •
· 12 •
· 13 •
The correct answer is D. 39% chose this.
· 14 •
The image shows a unilateral cleft lip. This results from the failure of fusion of the maxillary and
medial nasa l processes. The maxillary process is a derivative of the first branchia l (pharyngea l)
· 15 •
arch, and the medial nasa l processes develop from mesenchymal accumulation on the medial
·16.
· 17 •
side of the nasal pJacodes. Their failure to fuse together causes a so-called cleft in the palate
that makes the nasal cavity and the mouth contiguous.
· 18 • A is not correct. 3% chose this.
· 19 . The foramen cecum is the remnant of the thyroglossal duct, located at the back of the tongue.
The closest pathology to an "expansion of the foramen cecum" would be a thyroglossal duct
cyst, formed when the duct persists. This would not cause a cleft lip or cleft palate.
,Pad 9 2217 65« . '

+ G R< UI usmle rx com/ap l#qmaxl + D :


Item ' 1 of 45
QID 2600 i
L • Mark --<l
Pre Ylous
t:>
Next
_!
la b~ ues
~.
Notes
C';'\I
Cakul~ tor

· 1
· 2
•• B is not correct. 21% chose this.
Failure affusion of only the lateral palatine processes would produce a cleft palate without a
·3
· 4
•• cleft lip and thus cannot be the explanation for the condition shown in the image. The primary
pa late is formed by the intermaxillary segment, and the secondary palate is formed from the
lateral and median palatine processes and the nasal septum.
·5
· 6
•• Cleft Up

C is not correct. 33% chose this.


· 7
·8
•• A cleft palate, not a cleft lip, is due to failu re of fusion of the lateral palatine processes, the
nasal septum, and/or the medial palatine process.

.,0 •
Cleft Pa late Cleft Lip

".
· 9
. E is not correct. 4% chose this .
The formation of the nasal septum is involved in a cleft pa late, not a cleft lip.
.· 12 • Cleft Pa late Cleft lip

· 13 •
· 14 • Bottom line:
·,5 .
·16.
· 17 •
The maxillary process is a derivative of the first branchial arch .

· 18 •
ul;f,.·,.·1 for year: 2015
·19. FIlST -,.0 F-'CfS

•••
An infant is born with multiple congenita l defects. Her mother did not drink alcohol or use
· 2 any drugs during pregnancy, but she did travel throughout Asia during her first trimester. It
·3 is determined that she was infected with rubella (German measles) during her pregnancy.
· 4

·5
· 6
•• Which of the following congenital defects is most likely to be seen in this infant?
134415 ; zheng
· 7
·8
•• A. Bone deformity of the nasal bridge
We value your feedback!

• B. Brain calcifications

".
· 9
. ,0 •

.· 12 •
C. Chondrodystrophy

D. Cryptorchidism
l
· 13 • " E. Pulmonary artery stenosis
· 14 •

·,5 .
·16.
· 17 •
The correct answer is E. 46% chose this.
Congenital rubella infection can result in deafness, patent ductus arteriosus, pulmonary artery
· 18 •

·19. stenosis, cataracts, and microcephaly.


Patent ductus arter iosus M icrocephilly CongenItal Rubella Syndrome Oeafr.ess Bilateral cataracts (disorder)

A is not correct. 13% chose this.


Bone deformity of the nasal bridge, or saddle nose, is seen in congenita lly acquired
,..3·2.
,Pad 9 2217 65« . '

+ G R< UI usmle rx com/ap l#qmaxl + D :


Item' 2 of 45 L • Mark --<l t:> _! ~. C';'\I
.
QID 1847 i
. .. Pre Ylous
. . Next la b~ ues Notes Cakul~ tor

Bone deformity of the nasal bridge, or saddle nose, is seen in congenitally acqui red syphilis.

.·5
4.. Deform ity of bone Acqu ired syphilis

B is not correct. 19% chose this .

·6..
Brain calcifications may result from maternal cytomegalovirus infection and Toxoplasmosis

.7
.,
gondii infection, not German measles.
Toxoplasmosis Measles

C is not correct. 12% chose this .

., Chrondrodystrophy is an autosomal recessive disorder.

.".
. ,0 . D Is not correct. 10% chose this .
Cryptorchidism occurs when the testes fail to descend into the scrotum during fetal
development. Cryptorchidism is a rare congenital defect associated with maternal Epstein-Barr
· 12 • virus infection, not German meas les.
· 13 • Cryptorchidism Scrotum Epstein·Barr Virus Infections Measles

· 14 •

·,5 . Bottom line:


·16.
· 17 •
Rubella (German measles) is one of the ToRCHeS infections that can lead to congenital
anomalies including patent ductus arteriosus, pulmonary artery stenosis, cataracts, and
· 18 • deafness. The ToRCHeS infections can cross the placental barrier, often result in congenital

·19. abnormalities, and include the following: Toxoplasmosis, Rubella, Cytomegalovirus,


Herpes/ HIV, and Syphilis.
Patent duo:tus arteriOSUS Rubella Measles Congenita l musculOSkeletal anomalies Bilateral cataracts (disorder) Deafness

•••
Rubella (German measles) is one of the ToRCHeS infections that can lead to congenital
· 2 anomalies including patent ductus arteriosus, pulmonary artery stenosis, cataracts, and
·3 deafness. The ToRCHeS infections can cross the placental barrier, often result in congenital
· 4 abnormalities, and include the following: Toxoplasmosis, Ru bella, Cytomegaloviru s,
·5
· 6
•• Herpes/HIV, and Syphil is.
Patent duo:tus arteriOSUS Rubella Measles Congenita l musculoskeletal anomalies Bilateral cataracts (disorderl Deafness

., •
· 7

., • Ul;fi'·'I.' for year: 2015


.,0 •
~IRST " 10 ~" CTS

.".
· 12 •
· 13 •
· 14 •
.
Congenital cardiac
defect associations t\ lcohol exposu re in utero (fet al alcohol
synd rome)
Conge nita l rubell a
DHl(t

VSD. PDA. t\ SD, tctralogy of F'a llot


FAI5,p . 2901

Sept a) defcds, PO" , pulmonary artery stenosis


Down synd ro me t\V s('ptal defect (e ndocardi al c nsh ion defeel),
·,5.
·16.
VSD, ASD
Infant of d iabeti c moth cr Tmnsposition of grea t vcsscls
· 17 • " 'larfan synd rome M VP.t homcic aortic aneurysm and dissec tion,
aortic rcgurgitat ion
· 18 •

·19. I'rcn ata l1ith ium exposurc


TUfllc r s)'ndrom c
Williams synd ro me
Ebstci n a nomaly
Bicuspid aortic l"dIvc, coa rel<1tion of aorta
Supm\'ail-u iar aorti c stc nosis
22qll synd ro mes Trunc us artcriosus, tctm log), of Fa llot
,Pad 9 2218 65« . '

+ G R< UI usmle rx com/ap l#qmaxl + D :


Item ' 3 of 45
QID 2737 i
L • Mark --<l
Pre Ylous
t:>
Next
_!
la b~ ues
~.
Notes
C';'\I
Cakul~ tor

· 1
· 2
•• An 85-year-old man is diagnosed with an abdominal aortic aneurysm when a pulsating
abdomina l mass is pa lpated. The patient is sent by his physician to a surgeon, who decides
IAA[
·3
· 4
•• to perform endovascular surgery by inserting a stent into the femoral artery and threading it
upward, toward the aortic defect.

·5
· 6
•• The surgeon must access the femora l artery through a funnel-shaped fascia l tube that also
· 7
·8
•• contains which of the following structures?

We value your feedback!


134415 : zheng

.,0 •
A. External iliac lymph nodes
· 9

.".
· 12 •
· 13 •
· 14 •
. B. Femora l nerve

C. Femora l vein

D. Inguinal ligament

E. Superficial inguinal lymph nodes


·,5 .
·16.
· 17 •
· 18 • The correct answer is C. 62% chose this.
· 19 . Flexing, abducting, and laterally rotating the thigh will reveal a triangular-shaped depression
immediately inferior to the inguinal ligament. This depression marks the borders of the femora l
triangle, which is a subfascial space bounded superiorly by the inguinal ligament, I
the adductor longus muscle, and laterally by the sartorius muscle. The femoral

Flexing, abducting, and laterally rotating the thigh will reveal a triangular-shaped depression

•••
· 2 immediately inferior to the ingu inal ligament. This depress ion marks the borders of the femoral
·3 triangle, which is a subfascial space bounded superiorly by the inguinal ligament, medially by
the adductor longus muscle, and laterally by the sartorius muscle. The femoral triangle contains
· 4
important neurovascular structures servicing the lower limbs. The location of these structures
·5
· 6 •• within the triangle can be recalled by use of the mnemonic NAVEL; the structures of the femoral
triangle, from lateral to medial, are N erve (femoral nerve), Artery (femoral artery), Vein (femora l
· 7
·8
•• vein), Em pty space (potential space in the femora l canal), and l ymphatics (lymphatic vessels,
deep ingu inal lymph nodes. To access the femoral artery, the surgeon must open the femoral
sheath, which is a funnel-shaped fascial tube contained within the femoral triangle. The femoral

.,0 •
· 9 sheath is derived from the transversalis and iliopsoas fascia and is subdivided into three
.
.".
compartments that house the femoral artery, femoral vein, and femora l canal, as shown in the
image. The femoral canal contains a potential space and loose subcutaneous tissue that
accommodate expansion of the femoral vein when there is increased venous flow; it also
· 12 • houses lymphatic vessels and the deep inguinal lymph nodes. In a femoral hernia, the femoral
· 13 • canal is the space into which abdominal contents herniate.
Endoll1f!tnal mtraep!thellal neoplas ia IngUInal hgament Structure of femoral artery EntIre lymphatIc vessel HernIa, Femoral
· 14 •

·,5 .
·16.
· 17 •
· 18 •
· 19 •
,Pad 9 2220 6510 . '

+ G R< UI usmle rx com/ap l#Qmax1 + D :


Item ' 3 of 45
QID 2737
L

J.
• Mark -<l
PreYlDu,
t>-
NeKt
all
lab"" ues
~.
Notes
~'t
c .. kul~tor

· 1
· 2
•• Endometrial mtrilepl1heh~1 neoplilsia Inguinililigament Structure of femoral artery Entire lymphatic vessel Heml". Femorill

·3
· 4
••
·5
· 6 ••
· 7
·8
••
.,0 •
· 9
.
· 11 •
· 12 •
· 13 •

"..
· 14 •
· 15 •

.· 17 •
· 18 •

." Image courtesy of Dr. Craig Goodmurphy


A is not correct. 4% chose this.
The external iliac lymph nodes lie along the external iliac vessels, far from the femora l

A is not correct. 4% chose this.

•••
· 2 The external iliac lymph nodes lie along the external iliac vessels, far from the femora l sheath.
·3 They receive drainage from the genitouri nary system as well as from the superficial inguinal
· 4
lymph nodes.
·5
· 6
•• B is not correct. 19% chose this.
The femora l nerve, although located in the femoral triangle, is located outside the femoral
· 7
·8
•• sheath.
o is not correct. 10% chose this.

.,0 •
The inguinal ligament forms the superior border of the femoral triang le. It is formed by the
· 9
. external abdomina l oblique aponeuros is and is not considered part of the femoral sheath .
lnguinal l'9"ment Aponeurosis structure

· 11 •
E is not correct. 5% chose this.
· 12 •
The superficial inguinal lymph nodes arise from the superficial lymphatic vessels that
· 13 • accompany the superficial veins of the lower extremities, such as the great saphenous vein.

".
· 14 • They also can receive drainage from the umbilicus and nearby abdomina l wall, external
· 15 •
genitalia, and internal anal canal. Although these lymph nodes can be found in the area of the
femoral triangle, they are not contained within the femoral sheath .
Great saphenous vein structure tymph nodes
.· 17 •

.".
· 18 •
Bottom Line:
A mnemonic for remembering the contents of the femoral triangle is NAVEL, laterally to
medially: Nerve, Artery, Vein, Empty space, and Lymphatics. The last four structures in
mnemonic are contained within the femoral sheath.
,Pad 9 2220 65« . '

..·2,3..
+
Item ' 3 of 45
QID 2737
G
i
R<
L

-

UI usmle rx com/ap l#qmax1

Mark

- -
--<l
Pre Ylous
. -
t:>
Next
_!
la b~ ues
~.
Notes
C';'\I
Cakul~ tor

The inguinal ligament forms the superio r border of the femoral triang le. It is formed by the
external abdomina l oblique aponeuros is and is not considered part of the femoral sheath .
+ D :

.4
·5 .
.
Inguinal ligament Aponeurosis structure

.
E Is not correct. 5% chose this.

·6
.7
.,
. The su perficial inguinal lymph nodes arise from the superficial lymphatic vessels that
accompany the su perficial veins of the lower extremities, such as the great saphenous vein.
They also can receive drainage from the umbilicus and nearby abdomina l wall, external
genitalia, and internal anal canal. Although these lymph nodes can be found in the area of the
., femoral triangle, they are not contained within the femoral sheath .
Great saphel1Oll5 vein structure lymph nodes

.".
.,0 .
· 12 •
· 13 •
· 14 •
Bottom Line:
A mnemonic for remembering the contents of the femoral triangle is NAVEL, laterally to
medially: Nerve, Artery, Vein, Empty space, and l ymphatics. The last four structures in the
I-

mnemonic are contained with in the femoral sheath.


·,5 . Endometriallntraeplthe llal neo~asia

·16.
· 17 •
· 18 • Iii lif, , ., .. , for year: 2015
fIRST ""D f ... CTS
· 19 .
FA1S, p . 350.2

•••
A new small+molecule drug for the treatment of HIV has just been discovered. Th is small
· 2 molecu le appears to bind to a newly G+coupled surface protein receptor, TIY2, and sets off a
·3 cascading signa l that leads to apoptosis of the cell. The TIY2 receptor is expressed in cells
· 4 that have been infected by HIV but is not expressed in noninfected cells. Using in vitro
·5
· 6 •• experiments, scientists have been able to show that binding of the TIY2 receptor will act on
phospholipase C, thereby leading to an increase in intracellular calcium.

., •
· 7
The TIY2 receptor is similarto which of the following receptors?

., •
• A.
We value your feedback!
134415 : zheng

".
Q,
. ,0 •

.· 12 • c. ~2
· 13 •
· 14 •
D. ~3
·,5 . E. Dopamine,

·16.
· 17 •
· 18 •
· 19 • The correct answer is A. 66% chose this.
When activated, the Gq class of G-coupled protein receptors results in activation of protein
kinase C and an increase in intracellula r calcium. Thus, like the TIY2 receptor, the
receptor is a I G-coupled protein receptor. Other receptors incl
,Pad 9 2221 65« . '

+ G R< UI usmle rx com/ap l#qmaxl + D :


Item' 4 of 45
QID 1319 i
L • Mark --<l
PreYlous
t:>
Next
_!
la b~ ues
~.
Notes
C';'\I
Cakul~tor

· 1
· 2
•• The correct answer is A. 66% chose this.
When activated, the Gq class of G-coupled protein receptors results in activation of protein
·3
· 4
•• kinase C and an increase in intracellular calcium. Thus, like the TTY2 receptor, the Q1-adrenergic
receptor is a Gq-class G-coupled protein receptor. Other Gq-class receptors include muscarinic1,

·5
· 6
•• mU5carinic3, and histamine,.
C"leium HiStilmir.e

· 7
·8
•• B is not correct. 11% chose this.
The Q2-adrenergic receptor is a Gj-class G-coupled protein receptor. When activated, the Gi

. ,0 •
receptor leads to a decrease in cyclic adenosine monophosphate and a subsequent decrease in
· 9 protein kinase A. Other Gj-class receptors include muscarinic2 and dopaminez receptors .

.".
· 12 •
· 13 •
· 14 •
. Adenosine

C is not correct. 12% chose this.


The !32-adrenergic receptor is a Gs-class G-coupled protein receptor. The Gs receptor activates
adenyl cyclase, which converts ATP to cyclic adenosine monophosphate and leads to the
activation of protein kinase A.
Adenosine
·,5 .
·16.
· 17 •
o is not correct. 5% chose this.
The !33-adrenergic receptor is a Gs-class G-coupled protein receptor. The Gs receptor activates
adenyl cyclase, which converts ATP to cyclic adenosine monophosphate and leads to the
· 18 •
activation of protein kinase A. Other Gs-class receptors include fh dopamine" and histaminez.
·19. Adenosine HisLlmine

E is not correct. 6% chose this.

•••
· 2 The !33-adrenergic receptor is a Gs-class G-coupled protein receptor. The Gs receptor activates
adenyl cyclase, which converts ATP to cyclic adenosine monophosphate and leads to the
·3
activation of protein kinase A. Other Gs-class receptors include fh dopamine" and histaminez.
· 4

••
Adenosine HisLlmine

·5 E is not correct. 6% chose this.


· 6
· 7
·8
•• The dopamine, -adrenergic receptor is a Gs-class G-coupled protein receptor. The Gs receptor
activates adenyl cyclase, which converts ATP to cyclic adenosine monophosphate and leads to
the activation of protein kinase A.

.,0 •
· 9

.".
· 12 •
· 13 •
· 14 •
. Bottom Line:
Gq-class receptors include muscarinic" muscarinic3, and histamine,. Gq-class receptors
activate phospholipase C, causing an increase in inositol triphosphate (which leads to ca lcium
release in the celJ ) and diacylg lycerol, which leads to protein kinase C activation.
InoSitol HisLlmine C.. lc ium
r I

·,5.
·16.
· 17 •
ul;fi.·,.·1 for year: 2015
FIIUT "10 F"CTS

· 18 •

·19. FA15. p. 246 .1


,Pad 9 2221 65« . '

+ G R< UI usmle rx com/ap l#qmaxl + D :


Item ' 5 of 45
QID 2093 i
L • Mark --<l
Pre Ylous
t:>
Next
_!
la b~ ues
~.
Notes
C';'\I
Cakul~ tor

· 1
· 2
•• A 32-year-old man who is HIV-positive informs his physician that he has recently been
experiencing severe headaches accompanied by some nausea. He also had one episode
·3
· 4
•• du r ing which he passed out on the floor and his partner noticed he was jerking his hands
and legs. Laboratory studies show a CD4+ cell count of 75/mm 3 ; this is the first time it has dropped

·5
· 6
•• below 100/mm 3 • On further questioning the patient says that over the past year he started eating
steak tarta re, a dish made with raw beef.

· 7
·8
•• Which of the following is another (ommon source of this parasite?
rt---,-----:-,---,---
.,0 •
134415 : zheng
· 9 We value your feedback!
A. Anopheles mosquito bite

.".
· 12 •
· 13 •
· 14 •
.
B. Cat feces

C. Fema le sandflies

D. Reduviid bugs
I lCDii
·,5 . E. Water containing the protozoal cysts

·16.
· 17 •
· 18 •

· 19 . The correct answer is B. 63% chose this.


Toxoplasma gondil infection common ly presents with brain abscesses in HIV-positive patients,
and with birth defects if infection occurs during pregnancy Toxoplasmosis is one of the
ToRCHeS organisms, which include Toxoplasmosis, Ru bella, Cytomegalovirus, He rpesvi

••
· 2 Toxoplasma gondii infection commonly presents with brain abscesses in HIV-positive patients,
·3 and with birth defects if infection occurs during pregnancy Toxoplasmosis is one of the
· 4

·5 •• ToRCHeS organisms, which include Toxoplasmosis, Ru bella, Cytomegalovirus, He rpesvirus/H IV,


and Syphilis. T. gondii is transmitted via cysts in raw meat or cat feces. The definitive stage
(sexual stage) occurs in cats . Microscopically, acid-fast-staining cyst s are found . Sulfadiazine and
· 6
· 7
•• pyrimethamine are used to t reat toxoplasmosis.
Pregnancy ToxoplasmosIs Pynmetham lne BramAbscess

·8

.,0 •
· 9 •. A is not correct. 4% chose this.
Malaria, caused by Plasmodium parasites, causes cyclic fever, shaking, and chills. It is

.".
transmitted by the female Anopheles mosqu ito. Cerebral malaria is a feared complication of the
Plasmodium falciparum species. It ca n cause nystagmus, seizures, impaired consciousness, and,
in the worst cases, coma. Cerebral malaria is always fatal without treatment. However, the
· 12 • patient in this question does not live in an endem ic area, so ma laria is not the most li kely cause
· 13 • of his symptoms.
Malaria Seizures Pe l Ebstem fever Malana, Cerebra l Nystagmus
· 14 •

·,5 . C is not correct. 6% chose this.

·16.
· 17 •
Leishmania donovani infection (leishmaniasis) is transmitted by the bite of the female sandfly.
Patients with this infection present with hepatosplenomegaly, malaise, anemia, and weight
loss. Microscopically, macrophages containing amastigotes can be seen . Sodi um stibogluconate
· 18 • is used to treat L. donovani infection.
· 19 • LeishmaniaSIS Anem ia Sod,um

o Is not correct. 7% chose this.


,Pad 9 2222 6.\« • •

..3·2,..
+
Item ' 5 of 45
QID 2093
G
i
R<
L •

ryp
UI usmle rx com/ap l#qmax1

Mark --<l
Pre Ylous
t:>
Next

p •• y •
_!
la b~ ues

••
~.
Notes

9
C';'\I
Cakul~ tor

an imals or humans who have circu lating parasites. T. cruzl causes Chagas' disease, a condition
• 9 ••
+ D :

.·5
4..
in which the heart is enla rged and flaccid. Microscopic examination reveals flagellated
trypomastigotes in the blood, and nonmotile amastigotes in tissue culture . T. cruzi infection is
treated with nifurtimox .

.·6 . .
Nifurtimox Chagas Disease

7
.,
E Is not correct. 20% chose this .
Giardia lamblfa infection follows the ingestion of hardy cysts that colonize the small intestine
and release flagellated tropholoites. Giardia is spread via fecal-ora l transmission, usually
., through contaminated water. Patients with G. JambJia infection present with bloating,

".
fl atulence, foul-smelling diarrhea, and light-colored fatty stools On microscopy, one observes
.,0 . tea rdrop-shaped trophozoites with a ventral sucking disc or cysts. Metron idazole is used to
treat G. fambfia infection .
.· 12 • Water Oiarrhea Microscopy Metronida~ole entire small intestine

· 13 •
· 14 • Bottom Li ne:
·,5 . Toxoplasmosis infection presents in HIV patients with brain abscesses. Sulfadiazine and

·16.
· 17 •
pyrim etham in e are used to treat T. gondii infections.
Toxoplasmosis Pyrimetham ine BrainAbscess

· 18 •

· 19 .
ul;fi"'''' for year: 2015
FIJI.ST A'D FA CTS

•••
An 18-month-old chil d with a rare hereditary disease presents to genetics clinic. She is small
· 2 for her age, with club feet, coarse facial features, and gingival hypertrophy. She is unable to
·3 roll or sit unsupported. She said her first word at 17 months, and her parents report that her
· 4 voice is consistently hoarse. The geneticist explains that this child's disease is caused by the failure
·5
· 6 •• of a process that occurs in the cellu lar structure shown in the below.

., •
· 7

., •

".
. ,0 •

.· 12 •
· 13 •
· 14 •

·,5 .
·16.
· 17 •
· 18 •
· 19 • Image courtesy of Louisa Howard

The organelle pictured is essential for what cellular process?


,Pad 9 2222 6.\« • •

+ G R< UI usmle rx com/ap l#qmax1 + D :


Item ' 6 of 45
QID 3704 i
L • Mark --<l
Pre Ylous
t:>
Next la b~
_! ues
~.
Notes
C';'\I
Cakul~ tor

· 1
· 2
•• The organelle pictured is essential for what cellular process?

••
134415 : zheng
We value your feedback! ]
·3 A. ~-Oxidation of long-chain fatty acids
· 4

·5
· 6
•• 8. Degradation of cellul ar materials

C. Lipid and membrane biosynthesis


ltl~ J
· 7
·8
•• D. Protein modification

• E. Protein synthesis

".
· 9
. ,0 •

.· 12 •
The correct answer Is D. 47% chose this.
· 13 •
The Goigi complex adds ol igosaccharides to proteins and lipids. The oligosaccharide tags allow
· 14 •
processed final products to be accurately sorted for transport to secretory vesicles or
·,5 . Iysosomes. Lysosoma l hydrolases are tagged with mannose-6-phosphate. The patient in the
·16.
· 17 •
stem has mucolipidosis II, also known as I-cell disease. In this condition, enzymes within the
Golgi complex fails to attach mannose-6-phosphate to hydrolases. As a result, the hydrolases
are directed extracellularly, leaving Iysosomes without the enzymes required to hydrolyze their
· 18 •

·19. contents. Lysosomes fill with undigested molecules, creating visible cellular inclusions.
Symptoms of the disease are caused by lysosomal dysfunction and by the actions of
extracellular hydrolases. I-cell disease is characterized by growth, motor, and speech
club foot. joint contractures, umbilical and inguinal hernias, coarse facial featu res, i
hVI)ertrc)D~1V. and hoarse voice. This condition usuali results in death in I I

•••
· 2
A is not correct. 11% chose this.
Peroxisomes contain oxidases involved in catabolic pathways, such as l3-ox idation of long-chain
·3
fatty acids. Peroxisomes also contain catalase, an enzyme that uses hydrogen perox ide to
· 4 ox idize toxic substances. Peroxisome defects are exceed ingly rare, but can cause a constellation
·5
· 6
•• of symptoms that include liver dysfunction and skeletal deformities.
CATALASe peroxisome Hydrogen Peroxide liver Dysfurl(tion

· 7
·8
•• B is not correct. 15% chose this.
Lysosomes contain hydrolases that participate in cell product degradation. Primary Iysosomes

.,0 •
· 9 are the storages sites of lysosomal hydrolases, and secondary Iysosomes, also called

.". . phagolysosomes, are involved in the catalytic process. Gaucher disease is the most common
lysosomal storage disorder and results from the absence of glucocerebrosidase in Iysosomes,
leading to a buildup of glucocerebros id e. Symptoms are variable, but include hepatomegaly,
· 12 • bone pain, delayed growth, and neuropathy.
lysosomes G<llKher Disease lysosomal Storage Diseases
· 13 •
· 14 • C is not correct. 15% chose this.
·,5 . Smooth endoplasmic reticulum (SER) lacks ribosomes and functions primarily in lipid

·16.
· 17 •
biosynthesis and membrane synthesis and repa ir. In liver cells, SER plays a major role in
detoxification of various noxious metabolic byproducts.
Ribosomes Detox ification procedure Smooth Er.do~asmic Retkulum
· 18 •

·19. E is not correct. 12% chose this.


Rough endoplasmic reticulum (RER) contains ribosomes, which are essential for protein
synthesis. RER initiates the processing of finalized protein products.
fljbosomes Rough er.do~<lsmlc reticu lum
,Pad 9 2222 6.\« • •

+ G R< UI usmle rx com/ap l#qmax1 + D :


Item ' 6 of 45
QID 3704 i
L • Mark --<l
Pre Ylous
t:>
Next
_!
la b~ ues
~.
Notes
C';'\I
Cakul~ tor

· 1
· 2
•• C is not correct. 15% chose this.
Smooth endoplasm ic reticulum (SER) lacks ribosomes and functions primarily in lipid
·3
· 4
•• biosynthesis and membrane synthesis and repair. In liver celis, SER plays a major role in
detoxification of various noxious metabolic byproducts.

••
IUbosomes Deloxir,ca!ion procedure Smooth Endopasmic Reticulum

·5 E is not correct. 12% chose this.


· 6
· 7
·8
•• Rough endoplasm ic reticulum (RER) contains ribosomes, which are essential for protein
synthesis. RER initiates the processing of finalized protein products.
Ribosomes Rough endopl<lsmic reticu lum

.,0 •
· 9

.".
· 12 •
· 13 •
· 14 •
. Bottom Line:
The Goigi apparatus add s oligosaccharides to proteins and lipids, tagging them for delivery to
vesicles and Iysosomes. In I-cell disease, this process is disrupted and Iysosomes are left
witho ut necessary enzymes, leading to cellula r inclusions and death in early ch ildhood.
lipids me~surem'mt lysosomes G~gi Appilr~tus

·,5 .
·16.
· 17 • ul;fi.·,.·1 for year: 2015
FOUT .... 0 F"'CTS
· 18 •

·19. Cell trafficking


FA15, p. 13.1

Colgi is the distrihution ccnter for protei ns and lipids from the ER to the vesicles and plasma
membrane. Modifies N-oligosacclm rides on aspa ragine. Adds O-oligosaccharides on serine and
thrt'(lninc. Adds lllannose-6-phosphatc to I·

•••
A 24-year-old law student has been experiencing frequent headaches over the past several
· 2 months, for wh ich he has been taking increasing doses of medication. After a long night of
·3 studies, he comes into the university medical center complaining of ringing in his ears. He
· 4 later becomes disoriented and confused, and then experiences a seizure in the examination room.
·5
· 6
•• Which of the following is the most appropriate treatment?
· 7
·8
• • A. Bicarbonate
We val ue your feedback! 134415: zheng

.,0 •
· 9 B. Glucagon

.".
· 12 •
· 13 •
· 14 •
.
C. N-Acetylcysteine

D. Protamine

E. Vitamin K

·,5 .
·16.
· 17 •
The correct answer is A_ 50% chose this.
· 18 •

·19. Administration of bicarbonate alkal inizes the urine, thereby promoting the exc retion of acidic
drugs such as aspirin and not all owing them to be reabsorbed. Highly basic urine de proto nates
aci ds such as salicylates within the renal tubules, resulting in an ionic charge. In general,
charged molecules are cleared, while uncharged molecules are easily reabsorbed from
tubules. Thus bicarbonate administration is indicated because it promotes "trapping "
,Pad 9 2223 6.\« • •

+ G R< UI usmle rx com/ap l#qmax1 + D :


Item' 7 of 45
QID 249<1 i
L • Mark --<l
PreYlous
t:>
Next
_!
la b~ ues
~.
Notes
C';'\I
Cakul~tor

· 1
· 2
•• Admin istration of bicarbonate alkal inizes the urine, thereby promoting the excretion of acidic
drugs such as aspirin and not all owing them to be reabsorbed. Hi ghly bas ic urine deprotonates
·3
· 4
•• acids such as salicylates within the renal tubules, resulting in an ionic charge. In general,
charged molecules are cleared, while uncharged molecules are easily reabsorbed from the
tubules. Thus bicarbonate administration is indicated because it promotes "trapping" of
·5
· 6
•• salicylates in the urine and their subsequent excretion.
Aspirin

· 7
·8
• • B is not correct. 8% chose this.
Glucagon is used to treat J3-blocker toxicity.

.,0 •
glucagon (rONA)

".
· 9
. C is not correct. 35% chose this .
N-Acetylcysteine is used to treat acetam in ophen toxicity.
.· 12 • Acety1cysteme Acetaminophen

· 13 • D is not correct. 5% chose this.


· 14 • Protamine is used to treat heparin toxicity.
·,5 . Heparin

·16.
· 17 •
E is not correct. 2% chose this.
Vitam in K is used to treat warfarin toxicity.
· 18 • Vitamin K

·19. Bottom line:


Bicarbonate alkalinizes urine, aiding in removal of acid (such as aspirin) from the body.

•••
· 2
E is not correct. 2% chose this.
·3 Vitamin K is used to treat warfarin toxicity.
· 4 Vitamin K

·5
· 6
••
•••
Bottom li ne:
· 7
Bicarbonate alkalinizes urine, aiding in removal of acid (such as aspirin) from the body.
·8
ASprin

".
· 9
. ,0 •
ul;fi.·,.·1 for year: 2015
.· 12 • FIIU T ". 0 F" CTS

· 13 •
FA15,p , 4Q7,1
· 14 •
Aspirin
·,5 . Irrcl'crsih ly inhibits cyclooxygcnasc (both COX·1 and COX·2) cnzymc by cOI'alcnt acetylation.

·16.
M(HAJlISM
Pbtelets ca nn ot synthesize n ew enzyme, so effect bsts unt il new platelets a re prod uced·
I bleeding time. I TXA 2 ,mel prost'lglaudiu s. No effect on PT or fYTT.
· 17 •
{lIN I(Al U5( All tipyrc1ic, analgesic. :mb.in flammatory. antiplatclct (I aggregation).
· 18 •

·19.
TOI IClTY Gastric ulcerati on. tinnitus (eN V III ). Chronic usc can lead to acute renal failurc. interstitial
ncphritis. and upper G I bleed ing. Rcre .lyndrome in children with \'irdl infcction. O vcrdosc
initially C3U= h)'lJCrlTntiia l ion and respiratory alka losis. but tran sitions to mi.xed metabolic
acidosis-respiratory alkalosis.
,Pad 9 2223 6.\« • •

+ G R< UI usmle rx com/ap l#qmaxl + D :


Item ' 8 of 45
QID 1937 i
L • Mark --<l
Pre Ylous
t:>
Next
_!
la b~ ues
~.
Notes
C';'\I
Cakul~ tor

· 1
· 2
•• A constant infusion of drug Y is begun. Drug Y has a volume of distribution of 5 L and a
clea rance of 0.05 Uh.
IAA[
·3
· 4
•• Approximately how long will it take for drug Y to reach steady state?
·5
· 6
•• A. 70 hours
We value your feedback!
134415 : zheng

· 7
·8
•• B. 100 hours

.,0 •
· 9 C. 112 hours

.".
· 12 •
· 13 •
· 14 •
.
" D. 280 hours
E. 420 hours

·,5 .
·16.
· 17 •
The correct answer is O. 38% chose this.
To answer this question, you must first determine the half-life (t1/2) of drug Y. The formula for
determining tl/2 is: (0.7 x volume of distribution} I (clearance) = (0.7 x 5 L) 1(0.05 Uh) = 70 hours.
· 18 •

·19. A drug infused at a constant rate takes four to five half-lives to reach steady state ; so in this
case, about 280 hours. Students should note that this relationship holds true only for drugs with
first-order elimination. With first-order elimination, a constant fraction of the drug is eliminated
over time because the elimination rate is proportional to the drug concentration. Steady
achieved because the drug accumulates with in the body in a pred ictable manner.

•••
· 2 To answer this question, you must first determine the half-life (t 1l2) of drug Y. The formula for
·3 determ ining t1l2 is: (0.7 x volume of distribution) I (clearance) = (0.7 x 5 L) 1(0.05 Uh) = 70 hours.
· 4 A drug infused at a constant rate takes four to five half-lives to reach steady state ; so in this

·5
· 6
•• case, about 280 hours. Students should note that this relationship holds true only for drugs with
first-order elimination. With first-order elimination, a constant fraction of the drug is eliminated
over time because the elimination rate is proportional to the drug concentration. Steady state is
· 7
·8
•• achieved because the drug accumulates within the body in a pred ictable manner.
A is not correct. 16% chose this.

.,0 •
Ca lculation error.

".
· 9
. B is not correct. 29% chose this.
Calculation error.
.· 12 • e is not correct. 10% chose this.
· 13 • Ca lculation error.
· 14 • E is not correct. 7% chose this.
·,5. Ca lculation error.

·16.
· 17 • Bottom II ne:
· 18 • The t' /2 of a drug is calculated as: (0.7 x the volume of distribution) divided by clea rance or (0.7

·19. x volume of distribution) I clearance. It takes four to five half-lives for a drug to reach steady
state.
,Pad 9 2223 6.\« • •

+ G R< UI usmle rx com/ap l#qmax1 + D :


Item ' 8 of 45
QID 1937 i
L • Mark --<l
Pre Ylous
t:>
Next la b~
_! ues
~.
Notes
C';'\I
Cakul~ tor

· 1
· 2
•• Bottom Line:
·3
· 4
•• The t1/2 of a drug is ca lculated as: (0.7 x the volume of distribution) divided by clearance or (O.7
)( volume of distribution) I clearance. It takes four to five half-lives for a drug to reach steady

·5
· 6
•• state.

· 7
·8
•• hiif." ·,,,, for year: 2015
fIRST AID FAC TS

.,0 •
· 9

.". . FA15, p . 243.1

Pharmacokinetics
Bioa vailability (F ) Fraction of administered drug reaching sys temic circulatio n ullchanged. For an IV dose. F = 100%.
· 12 • Orally: Ii t)'llicall \,< 100% dllC to incomplete absorpt ion and first-pass metabolism.

· 13 • Volume of distribution Thmrctical I'olllme occupied Ill' the lola l amount of dru g in th e booy relati l'e 10 ils plasma
(Vd) concentrdtion . App;lrent Vd of plasma protein-bound drugs can he allerI'd b)' lil'er and kid ne)'
· 14 • di sease (I protein binding. I Vd). Dm gs ma l' di strilmte in more lhan one compa rtment
·,5 . V ;lmount of drug in the bo d)'

·16.
· 17 •
d

'.Low
plasma drugcollcclltratioll
(OMPARTMEHT

Blood Largclchargcd molecules; plasma protein bound


· 18 •

·19.
Medium ECF Small hrdrophilic moleculcs
High All tissues including Small lipophilic molecules, espedall)' if boun{]
r" to li ssue protein
Clearance (Cl) T he I'olumc of plasma cleared of drug per uniltime. Clearancc may he impaired wilh defects in
ca rdiac . hepatic. or rena l fUlldioll

•••
A 73-year-old woman is admitted to the hospital because of an acute exacerbation of
· 2 congestive heart fai lure. A central ve nous catheter is inserted on the second hospital day.
·3 hospital day 6, the nurse notes that the patient can no longer recall her name and is
· 4 confused about where she is. The patient'S temperature is 3B.9°( (102°F). Blood cu lture reveals a
·5
· 6
•• gram -negative rod that produces a red pigment.

· 7
·8
•• Which of the following orga ni sms is the most likely ca use of this patient's infection?

We value your feedback!


134415 : zheng

.,0 •
A. Escherichia coli
· 9

.".
· 12 •
· 13 •
· 14 •
. B. Klebsiella pneumoniae

C. Proteus mirabilis

D. Serratia marcescens

E. Staphylococcus saprophyticus
·,5 .
·16.
· 17 •
· 18 • The correct answer is D. 68% chose this.

·19. This patient likely has a nosocomial infection related to her central veno us cat heter (CVC), as
indicated by the change in mental stat us and her fever without another obvious cause. Serratia
marcescens is the most likely culprit here, in light of the blood culture findings: It is a
negative rod that produces red pigment (th ink of red maraschino cherries).
,Pad 9 2223 6.\« • •

+ G R< UI usmle rx com/ap l#qmax1 + D :


Item ' 9 of 45
QID 2046 i
L • Mark --<l
Pre Ylous
t:>
Next
_!
la b~ ues
~.
Notes
C';'\I
Cakul~ tor

· 1
· 2
•• This patient likely has a nosocomial infection related to her central venous catheter (CVe). as
indicated by the change in mental status and her fever without another obvious cause. Serratia
·3
· 4
•• marcescens is the most likely culprit here, in light of the blood culture findings: It is a gram-
negative rod that produces red pigment (think of red maraschino cherries).

·5
· 6
•• Blood cu lture

A is not correct. 7% chose this.


· 7
·8
•• Escherichia coli is the most common cause of urinary tract infection. It is a gram-negative rod,
but it produces no pigments.

. ,0 •
Urin;,'Y t'<lct infection
· 9

.". . B is not correct. 14% chose this .


Klebsiella pneumoniae is a gram-negative rod that is responsible for approximately 8% of
nosocomial infections. It is a significant cause of urinary tract infection and pneumonia in
· 12 • hospitalized and ambulatory patients, especially in alcoholic and diabetic ones. It produces no
· 13 • pigments.
Pneumonia Pneumonia due to Klebsiella pneumoniae UriMry tract infection
· 14 •

·,5 . C is not correct. 7% chose this.

·16.
· 17 •
Proteus mirabilis is a gram-negative bacillus that is a frequent cause of nosocomial urinary tract
infections. It produces the enzyme urease, which serves to create a more alkaline environment
for itself. The ha llmark of a urease-producing organism such as Proteus is the production of
· 18 •

·19. kidney stones along with infection. It does not produce pigment.
UriMry Iract infecllon

E is not correct. 4% chose this.


Staphylococcus saprophyticus is the second most common cause of urinary tract infection i

•••
· 2 Proteus mirabilis is a gram-negative bacillus that is a frequent cause of nosocomial urinary tract
·3 infections. It produces the enzyme urease, which serves to create a more alkaline environment
· 4
for itself. The ha llmark of a urease-producing organism such as Proteus is the production of

·5
· 6
•• kidney stones along with infection. It does not produce pigment.
UriMry Iract infecllon

· 7
·8
•• E is not correct. 4% chose this.
Staphylococcus saprophyticus is the second most common cause of urinary tract infection in
young women. It is a gram-positive coccus and does not produce any pigments.


".
· 9 UriMry Iract infection
. ,0 •
I
Bottom Li ne:
.· 12 •
Serratia marcescens is a common cause of catheter·related infection.
· 13 •
· 14 •

·,5. 141 ;fi , .j 1.1 for year:


·16.
· 17 •
FIII.ST A I D FAC TS
2015

FAtS, p . 174.2
· 18 •

·19. UTI bugs


SlWES
Escherichia coli
fEAtURES {OMMENTS
Diagnostic Ill arkers:
® uukoc)'te estcrasc ,. cvidcncc of WBC
,Pad 9 2224 6.\« • •

+ G R< UI usmle rx com/ap l#qmax1 + D :


Item ' 10of45 L • Mark --<l t:> _! ~. C';'\I
QID 1173 i Pre Ylous Next la b~ ues Notes Cakul~ tor

· 1
· 2
•• A town with 1000 citizens has a 10% prevalence of disease X. A screening test for disease X
was just developed, with a sensitivity of 80% and a specificity 0(70%.
IAA[
·3
· 4
•• How many people without disease X will be falsely diagnosed positive by this screening test?
·5
· 6
•• A. 20
We value your feedback!
134415 : zheng

· 7
·8
•• B. 80

.,0 •
· 9 c. 100

.".
· 12 •
· 13 •
· 14 •
.
" 0. 270
E. 630

·,5 .
·16.
· 17 •
The correct answer is O. 54% chose this.
The question is asking for the number of false-positives.
Specificity = True-negatives/(True-negatives + False-positives). False-positive signifies the
· 18 •

·19. number of people without disease X who will be fa lsely diagnosed by the screening test. Tn this
case, 900 people do not have the disease, represented by true-negatives + false-positives. Using
a speci fi city of 70%, the number of true-negatives is 630, whereas the number of fa lse-positives
is 270. Thus, 270 people without disease X will be falsely diagnosed with t his screening
they will be false-positives). The Punnett squa re illustrates the concept.

•••
· 2
The correct answer is O. 54% chose this.
Oist-as!'
·3
· 4

·5
· 6
•• TP
80
FP
270
350

· 7
·8
•• FN
20
TN
630
650

.,0 •
"'00
· 9 "'" 900

.".
· 12 •
· 13 •
· 14 •
. The question is asking for the number of false-positives. Specificity =True-negatives/(True-
negatives + False-positives). Fa lse-positive signifies the number of people without disease X who
will be falsely diagnosed by the screening test. I n this case, 900 people do not have the disease,
represented by true-negatives + false-positives. Using a speci ficit y of 70%, the number of true-
negatives is 630, whereas the number of false-positives is 270. Thus, 270 people without disease
X will be falsely diagnosed with this screening test (ie, they will be false-positives). The Punnett
·,5 . square illustrates the concept.
·16.
· 17 •
A is not correct. 21% chose this.
The figure 20 is the number of people with the disease who will have an incorrect negative
· 18 • screen ing test result (ie, fa lse-negatives).

·19. You know that the disease prevalence is 10%, so in a town of 1000 people 100 will have disease
X and 900 will not. You can use these numbers with 80% sensitivity and 70% specificity to make
the 2 x 2 table.
,Pad 9 2224 6.\« • •

+ G R< UI usmle rx com/ap l#qmax1 + D :


Item ' 10of45 L • Mark --<l t:> _! ~. C';'\I

--12..
QID 1173 i
- -
Pre Ylous
-
Next la b~ ues Notes Cakul~ tor

..43.. The figure 20 is the number of people with the disease who will have an incorrect negative
screen in g test result (ie, fa lse-negatives) .
You know that the disease prevalence is 10%, so in a town of 1000 people 100 will have disease

''65.. X and 900 will not. You can use these numbers with 80% sensitivity and 70% specificity to make

.,
'7
. ,0 .
.
., .
the 2 x 2 table.

B is not correct. 11% chose this.


The figure 80 is the number of people who will have a correct positive screen ing test result (ie,
true-positives) .
C is not correct. 9% chose this .
The figure 100 is the number of people in the town with disease X (ie, the prevalence of disease

"'.
· 12 •
· 13 •
· 14 •
Xl.
E is not correct. 5% chose this.
The figure 630 is the number of people who wi ll have a correct negative screening test resu lt (ie,
true-negatives).

"5
'16..
· 17 •
Bottom Line:
The formulas for figuring out sensitivity, specificity, positive predictive value, and negative
· 18 •
pred ictive value are important, and all of thei r formulas can be rearranged to solve for true-

'19. positive, false-positive, false-negative, and true-negative. Making a 2 x 2-cell chart can make it
much easier to vi sualize what the question is asking.

C is not correct. 9% chose this.

•••
· 2 The figure 100 is the number of people in the town with disease X (ie, the prevalence of disease
·3 Xl·
· 4 E is not correct. 5% chose this.
·5
· 6
•• The figure 630 is the number of people who will have a correct negative screening test resu lt (ie,
true-negatives).

·, •
•7

·, •
Bottom Line:

.,0.•
The formulas for figuring out sensitivity, specificity, po sitive pred ictive value, and negative
pred ictive value are important, and all of thei r formulas can be rearranged to solve for true-

"'.
positive, false-positive, false-negative, and true-negative. Making a 2 x 2-cell chart can make it
much easier to visualize what the question is asking .
· 12 •
· 13 •
· 14 •
141;1','·'1.' for year: 2015
~tRST ... to ~"' CT S
" 5 .

'16.
· 17 •
Evaluation of Uscs 2 x 2 ta ble comparing lcsl rcs ults wil h th c
FAIS. p. 49.1

· 18 • diagn05tic te5t5 achwl pfeseucc of dist"ase. TP = lrnc positive;

'19. ~ : I-I-:-+--~-'-I
FP = f~ l sc po.Iilh·c; T N = truc ncg.11i\c; FN =
Mse ncgali\·e.
Sensitil'it )' and specifi cit), <Ife fi ~ed properties
of a tes t rrv and NPV v;lr}, depending on
disease prevalence
,Pad 9 2224 6.\« • •

+ G R< UI usmle rx com/ap l#qmax1 + D :


Item ' 11 of 45
QID 1S48 i
L • Mark --<l
Pre Ylous
t:>
Next la b~
_!
ues
~.
Notes
C';'\I
Cakul~ tor

· 1
· 2
•• Reporting of disease provides information useful for the study of both disease epidemiology
and pathology. Many diseases are reportable in all of the United States.
IAA[
·3
· 4
•• Which of the following positive resu lts is required to be reported in all 50 states?
·5
· 6
•• 134415 : zheng

· 7
·8
•• A. Positive IgG antibody to hepatitis B surface antigen

.,0 •
· 9 8. Positive 19M antibody to hepatitis B core antigen
. I ~

.".
· 12 •
· 13 •
C. Positive purified protein derivative oftubercuHn in someone who has received
baci llus Calmette-Guerin
D. Positive rapid plasma reag i n in someone who describes flu-like symptoms

E. Positive sputum cu lture of Streptococcus pneumoniae


I-
I --'

I~

· 14 •

·,5 .
·16.
· 17 •
We value your feedback!

· 18 •

·19.

•••
· 2 The presence of IgM for hepatitis B core antigen corresponds to an active infection with
·3 hepatitis B virus (HBV); this is in contrast to having IgG antibodies to hepatitis B surface
· 4 antigen, which may suggest past infection or prior immunization with HBV. All states require

·5
· 6
•• the reporting of cases of HBV infection, as well as cases of varicella, measles, mumps, rubella,
hepatitis A, syphilis, tuberculosis, gonorrhea, shigellosis, salmonellosis, and HIV infection.
TuberculOSis Gonorrhea Hepatitis 8 Vacc ine Hepatitis B surface antigen measurement Measles Mumps Rubel la

· 7
·8
•• A is not correct. 11% chose this.

. ,0 •
Positive IgG to hepatitis B surface antigen can occur in someone who has been infected with
· 9 hepatitis B, but would also be seen in someone who had been immunized .
.
.".
Hepatitis a surface antigen measurement Hepatitis a Vaccine

C is not correct. 21% chose this.


· 12 • A positive purified protein derivative of tuberculin is a common false-positive result in someone
· 13 • who has received the bacillus Cal mette-Guerin vaccine.
Pur ified Protem DerIVative ofTuberculin
· 14 •

·,5 . D is not correct. 17% chose this.

·16.
· 17 •
A positive rapid plasma reagin test is sens itive enough to rule out syph ilis; however, the test is
not specific enough to diagnose syph ilis. Fluorescent treponemal antibody absorption is the
specific serum test used to diagnose syphilis. A positive treponemal antibody test must be (is
· 18 •

·19. requi red to be) reported by all 50 states.


Rapid Plasma Reagin Measurement

E is not correct. 4% chose this.


,Pad 9 2225 6.\« • •

+ G R< UI usmle rx com/ap l#qmax1 + D :


Item ' 11 of 45
QID 1S48 i
L • Mark --<l
Pre Ylous
t:>
Next
_!
la b~ ues
~.
Notes
C';'\I
Cakul~ tor

· 1
· 2
•• o is not correct. 17% chose this.
A positive rapid plasma reag in test is sens itive enough to rule out syph ilis; however, the test is
·3
· 4
•• not specific enough to diagnose syph ilis. Fluorescent treponema I antibody absorption is the
specific serum test used to diagnose syphilis. A positive treponema I antibody test must be (is
·5
· 6
•• required to be) reported by all 50 states.
Rapid Plasma R~a9in Measurement

· 7
·8
•• E Is not correct. 4% chose this.
Streptococcus pneumoniae infection is not a reportable disease.

.,0 •
Pneumococca l Infections
· 9
.
.".
· 12 •
· 13 •
Bottom line:
Cases of varicella infection, measles, mumps, rubella, hepatitis A, hepatitis B, syphilis,
tuberculosis, gonorrhea, shigellosis, salmonellosis, and HIV must be reported in all 50 states .
Of note, AIDS has been reclassified as HIV stage III by the CDC.
r

· 14 •
Tuberculos is Gonorrhea Meas les Mumps Rubel l., Hep.Wtls B Vaccine
·,5 .
·16.
· 17 •
hli1',"''''
fIRS T A I D fAC TS
for year: 2015
· 18 •

·19. Hepatitis serologic markers


FA15. p. 167.1

•••
An 18-year-old woman presents to her pri mary care physician with a 3-day history of
· 2 headaches and a nonproductive cough. On examination, the patient is found to have a low-
·3 grade fever, diffuse interstitial infiltrates, and an increased reticulocyte count. The physician
· 4 chooses an antibiotic that inhibits RNA-dependent protein synthesis by blocking translocation
·5
· 6
•• within the ribosome.

· 7
·8
•• Which of the following antibiotics is the physician most likely to choose for this patient?

We value your feedback!


134415 : zheng

.,0 •
A. Erythromycin
· 9

.".
· 12 •
· 13 •
· 14 •
. B. Lincomycin

C. Quinupristin/dalfopristin

D. Tetracycline

E. Tobramycin
·,5 .
·16.
· 17 •
· 18 • The correct answer is A. 59% chose this.
·19. Erythromycin is a macrolide antibiotic used to treat Legionella, Chlamydia, Mycoplasma, and
Neisseria species infections. It is typically used to treat atypical pneumonias and upper
respiratory infections. Erythromycin toxicity causes gastrointestinal discomfort, acute
cholestatic hepatitis, eosinophi lia, and skin rash. The mechanism of action of mthr·onlYCi
,Pad 9 2225 6.\« • •

+ G R< UI usmle rx com/ap l#qmax1 + D :


Item ' 12 of 45
QID 1718 i
L • Mark --<l
Pre Ylous
t:>
Next
_!
la b~ ues
~.
Notes
C';'\I
Cakul~ tor

· 1
· 2
•• The correct answer is A. 59% chose this.
Erythromycin is a macrolide antibiotic used to treat Legionella, Chlamydia, Mycoplasma, and
·3
· 4
•• Neisseria species infections. It is typically used to treat atypical pneumonias and upper
respiratory infections. Erythromycin toxicity causes gastrointestinal discomfort, acute
·5
· 6
•• cholestatic hepatitis, eosinophi lia, and skin rash. The mechanism of action of erythromycin is
inhibition of translocation, during which the growing peptide chain is transferred from the
amino acid- binding site to the peptide-binding site .
· 7
·8
•• Erythromycin Mycoplasma Infections Primary aty~cal pneumonia Disorder characterized by eosinophIlia

B is not correct. 7% chose this.

.,0 •
".
· 9
Lincomycin is a seldom-used antibiotic that acts by interfering with binding of aminoacyl-tRNA
. to the 50S subunit. It is not common ly used to treat Mycoplasma species infection .
Lincomycin Mycoplasma Infections

.· 12 • C is not correct. 6% chose this.


· 13 • Quinupristin/dalfopristin inhibits the 50S subun it, blocking protein synthesis. It is used to treat
· 14 • vancomyc in-resistant enterococcus bacteremia or compl icated skin infections due to
methicillin-resistant Staphylococcus aureus or Streptococcus pyogenes.
·,5 .
·16.
· 17 •
daifopristin 6acterem ia MRSA Methicillin reSistant Staphy1oc:occvs aurevs infection Skin DiSeaSeS. InfeCtiOUs

o Is not correct. 20% chose this.


Tetracycline is a bacteriostatic antibiotic that binds to the 305 subunit and prevents attachment
· 18 •
of the aminoacyl-tRNA. Tetracycline is used to treat infections caused by Vibrio, Chlamydia,
· 19 . Mycoplasma, and Rickettsia species as well as Lyme disease and tularemia. Although tetracycline
may be used to treat this patient's infection, it does not block chain translocation.
Tetracyc lu-.e Mycoplasma Infections Tularemia Lyme Disease Vacc ine

Tetracycline is a bacteriostatic antibiotic that binds to the 305 subunit and prevents attachment

•••
· 2 of the aminoacyl-tRNA. Tetracycline is used to treat infections caused by Vibrio, ChlamydIa,
Mycoplasma, and Rickettsia species as well as Lyme disease and tularem ia. Although tetracycline
·3
may be used to treat this patient's infection, it does not block chain translocation.
· 4

••
Tetracyc lIne Mycoplasma Infections TularemIa Lyme Disease Vacc ine
·5 E is not correct. 8% chose this.
· 6
· 7
·8
•• Tobramycin is an aminoglycoside antibiotic that inhibits the formation of the initiation complex
necessary to read mRNA. Aminoglycosides are used to treat gram -negative infections and are
synergistic with 13-lactam antibiotics. Adverse effects include nephrotoxicity and ototoxic ity.
· 9

".
. ,0 •

.· 12 •
· 13 •
· 14 •
Bottom Line:
Walking pneumonia is classically caused by Mycoplasma pneumoniae and is treated with
macrolides such as azithromycin and erythromycin, bacteriostatic antibiotics that bind to the
235 rRNA of the 50S ribosomal subunit and block ribosomal translocation (and thus protein
·,5 . synthesis).

·16.
· 17 •
Walj(jng (activity) Pneumonia N:lthromycin Erythromycin Mycoplasma pneumonia

· 18 •
liil;fi""" foryear: 2015
· 19 • FIRST AIO FACTS
,Pad 9 2225 6.\« • •

+ G R< UI usmle rx com/ap l#qmax1 + D :


Item ' 13 of 45
QID 1553 i
L • Mark --<l
Pre Ylous
t:>
Next
_!
la b~ ues
~.
Notes
C';'\I
Cakul~ tor

· 1
· 2
•• The statistical distribution of two studies is shown below. The mean is equal to the median
and the mode in the first curve (labeled A),
IAA[
·3
· 4
••
·5
· 6
••
· 7
·8
••

".
· 9
. ,0 •

.· 12 •
· 13 •
Which of the following correctly describes the mean, median, and mode in the seco nd curve
· 14 • (labeled B)?
·,5 .
·16. We value your feedback!
134415 : zheng

"
A. Mean < median < mode
· 17 •

IlDii
· 18 •
B. Mean < mode < median

·19. c. Median < mean < mode

D. Median < mode < mean


~

•••
· 2
The first curve, with mean = median = mode, represents a normal Gaussian di stribution. The
second curve represents a negative skew. The mean is equal to the center of the graph. The
·3 mode is equal to the most (ommon result. This is represented at the top of the curve. The
· 4 median is the middle value if the value were ordered sequentia lly. It turns out that during either
·5
· 6
•• a po sitive skew or a negative skew, the median is in between the mean and the mode.
Therefore, mean < median < mode.

· 7
·8
•• B is not correct. 7% chose this.
In Gaussian distributions, the median is always between the mode and the mean.

. ,0 •
C is not correct. 11% chose this.

".
· 9
. In Gaussian distributions, the median is always between the mode and the mean .
D is not correct. 7% chose this .
.· 12 • In Gaussian distributions, the median is always between the mode and the mean.
E is not correct. 5% chose this.
· 13 •
In Gaussian distributions, the median is always between the mode and the mean.
· 14 •
F is not correct. 13% chose this.
·,5 .
·16.
· 17 •
Th is wou ld be the case in a positively skewed data distribution, rather than a negative skew.

· 18 • Bottom line:

·19. In a negatively skewed distribution, the mode is greater than the mean; in a positively skewed
distribution, the mode is less than the mean. In either case, the median is in between the
mode and the mean.
A 22 Sis
v 17 KS/s

8
l ock
0
End Block
,Pad 9 2225 6.\« • •

+ G R< UI usmle rx com/ap l#qmax1 + D :


Item ' 13 of 45
QID 1553 i
L • Mark --<l
Pre Ylous
t:>
Next
_!
la b~ ues
~.
Notes
C';'\I
Cakul~ tor

· 1
· 2
•• D is not correct. 7% chose this.
In Gaussian distributions, the median is always between the mode and the mean.
·3
· 4
•• E is not correct. 5% chose this.
In Gaussian distributions, the median is always between the mode and the mean.
·5
· 6
•• F is not correct. 13% chose this.
This would be the case in a positively skewed data distribution, rather than a negative skew.
· 7
·8
•• Bottom Line:

.,0 •
".
· 9 In a negatively skewed distribution, the mode is greater than the mean; in a positively skewed
. distribution, the mode is less than the mean. In either case, the median is in between the
mode and the mean .
.· 12 •
· 13 •
· 14 •
h!;fj,·, ... for year: 2015
fIRS T ... I D f ... CTS
·,5 .
·16.
· 17 •
Statistical distribution
Measures of central l'I'lea n = (sum of I'alucs)!\;total r1l1miJer of ,·alu es).
FAI5, p , 53.1

Most affected b)' outliers (cxtrcmc ,·aJucs).


· 18 •

·19. tendency i\'iediHn" middl e I'alue of a list of data sorted


from Icasl to grea tcst
Mode = most common I'aluc.
If th ere is an even number of nducs.the media n
will bc thc m·crage of thc middle two 11IIues
Least affeckd b)' outliers.
St,mdard dc,·iation = hoI\' mllch I'ariability a = SD; n = sa mplc size

•••
A mass is felt in the groin of an infant girl during a physical examination. Surgica l resectio n
· 2 shows that it is a testicle. The baby is diagnosed with testicular feminization synd rome. In
·3 this syndrome. androgens are produced but cells fail to respond to the stero id hormones
· 4 because they lack appropriate intracellular receptors.
·5
· 6
•• After binding intracellular receptors, steroids regulate the rate of whic h of the following?
· 7
·8
•• A. I niti ation of protein synthesis
We value your feedback!
134415: zheng

.,0 •
· 9 B. mRNA degradation

.".
· 12 •
· 13 •
· 14 •
.

"
C. mRNA processing

D. Protein translation

E. Transcription of genes

·,5 .
·16.
· 17 •
The correct answer is E. 75% chose this.
· 18 •

·19. Steroid hormones enter cells and bind to receptor proteins. The receptor-hormone complex
binds to spec ifi c response elements, or the regulatory region of DNA. and activates gene
transcription.
A is not correct. 9% chose this.
,Pad 9 2226 6.\« • •

+ G R< UI usmle rx com/ap l#qmax1 + D :


Item ' 14of45 L • Mark --<l t:> _! ~. C';'\I
QID 1218 i Pre Ylous Next la b~ ues Notes Cakul~ tor

· 1
· 2
•• The correct answer is E. 75% chose this.

·3
· 4
•• Steroid hormones enter cells and bind to receptor proteins. The receptor-hormone com plex
binds to spec ific response elements, or the regulatory region of DNA, and activates gene
transcription.
·5
· 6
•• A is not correct. 9% chose this.
Steroid hormones do not regulate t he initiation of protein synthes is.
· 7
·8
•• B is not correct. 2% chose this.
Steroid hormones do not regulate mRNA degradation.

.,0 •
C is not correct. 6% chose this .

".
· 9
. Steroid hormones do not regulate mRNA processing .
D is not correct. 8% chose this.
.· 12 • Steroid hormones regulate gene transcri ption, not t ranslation.
· 13 •
· 14 • Bottom Line:
·,5 . Steroid hormones are lipophilic and easily pass t hrough the cell's lipid bilayer. They exert t heir
·16.
· 17 •
effect by binding to intracell ular receptors. Once bound to their ligand, steroid hormone
receptors relocate to the nucleus and act as transcription factors to affect gene express ion.
Lipid Bilayers Gene Expression
· 18 •

·19. ul;f.""'0 ·,,,, for year: 2015


fIRST f"C TS

•••
C is not correct. 6% chose this.
· 2
Steroid hormones do not regulate mRNA processing.
·3
D is not correct. 8% chose this.
· 4

••
Steroid hormones regulate gene transcri ption, not t ranslation.
·5
· 6
· 7
·8
•• Bottom Line:
Steroid hormones are lipophilic and easily pass th rough the cell's lipid bilayer. They exert their

. ,0 •
effect by bi nding to intracellular receptors. Once bou nd to their ligand, steroid hormone
· 9 receptors relocate to the nucleus and ac t as transcription factors to affect gene expression .

.".
· 12 •
· 13 •
· 14 •
. Lipid Bilayers Gene Expression

ul;f.,,·,!·, for year: 2015


fIRST ""0 F"CTS

·,5 . FA1S. p. 321.3

·16.
· 17 •
Signaling pathway of
steroid hormones
Sicroid hormoncs ,nc lipophilic and Ihereforc
mnst circulatc bound to spccific binding
globulins. which I lhcir soillbility.
/ -
· 18 • 111 'nell, I sex hormonc- hindi ng globulin

·19. (SI IBC) lowcrs free t c~to\lerone


- grn~~omast ia.
In women. I SHBG raises frce lestosterone
-him,tislll.
,Pad 9 2226 6.\« • •

+ G R< UI usmle rx com/ap l#qmax1 + D :


Item ' 1Sof45 L • Mark --<l t:> _! ~. C';'\I

.".
QID 1931 i Pre Ylous Next la b~ ues Notes Cakul~ tor

A total of 50 mg of drug X is administered to a patient during a constant intravenous


·,2 . infusion, yielding a steady-state plasma concentration of 5.0 mg/L.
· 13 •
.,4 . If the half-li fe of drug X is 100 min, which of the followi ng is the clearance of drug X?
·,5 . We value your feedback!
134415 : zheng
·,6 . A. 0.0007 Um in
· 17 •
B. 0.007 Umin
· 18 •
· ,9 • C. 0.01 Umin

. 20 . 0 .0.07 Umi n
· 21 •
E. 0.10 Umin
· 22 •
. 23 .
· 24 .
· 25 •
The correct answer is 0 . 41 % chose this .
. 26 .
The 50-mg total drug amount in the body corresponds to the loading dose: 50 mg =(C p )( Vd) IF,
· 27 • where Cp is the plasma drug concentrat ion, Vd is the volume of distribution, and F is the
· 28 • bioava ilability. Because the infusion was IV, F = 1. Rearrange t his equation to calculate the
· 29 . volume of distribution: Vd = (amount of drug in the body) 1 (plasma drug concentrati on) = (50
mg) 1 (5.0 mg/L) = 10 L. With the volume of distribution calculated, and knowing the half-life
(t,d, cleara nce (CL) can be ca lculated as: t'/2 = (0.7 x Vd) I CL; then CL = (0.7 x Vd) 1 t1l2 =
L) 1100 min = 0.07 Umin. Remember that loading dose is in mass units (grams), but

".
.· 12 •
.,3 .
.,4 .
The correct answer is 0 . 41 % chose this .
The 50-mg total drug amount in the body corresponds to the loading dose: 50 mg =(C p x Vd) 1 F,
where Cp is the plasma drug concentrat ion, Vd is the volume of distribution, and F is the
bioava ilability. Because the infusion was IV, F = 1. Rearrange t his equation to calculate the
volume of distribution: Vd = (amount of drug in the body) 1 (plasma drug concentrati on) = (50
·,5 . mg) 1 (5.0 mg/L) = 10 L. With the volume of distribution calculated, and knowing the half-life
·,6 . (t,d, cleara nce (CL) can be ca lculated as : t'/2 = (0.7 x Vd) I CL; then CL = (0.7 x Vd) 1 t1l2 = (0.7 x 10
L) 1100 min = 0.07 Umin. Remember that loading dose is in mass units (grams), but
· 17 •
ma int enance dose is a rat e with units of mass per time (grams/ho ur).
· 18 •
A Is not correct. 6% chose this.
·,9 . Ca lculation error.
. 20 .
B is not correct. 14% chose this.
· 21 •
Ca lculat ion error.
· 22 •
e is not correct. 20% chose this .
. 23 .
Ca lculation error.
· 24 . E is not correct. 19% chose this.
· 25 •
Ca lculation error.
. 26 .
· 27 •
Bottom Line:
· 28 •
The ha lf-life of a drug is calcu lated as: (0.7 x t he volume of distribution) divided by clearance,
· 29 . or t'/2 = (0.7 x Vd) 1 CL.
,Pad 9 2226 6.\« • •

+ G R< UI usmle rx com/ap l#qmax1 + D :


--<l t:> _! ~. C';'\I

".
Item ' 1Sof45 L • Mark
la b~ Cakul~ tor
QID 1931 i
.. Pre Ylous
, . Next ues Notes

Ca lculation error.
·,2.
.
· 13 • Bottom Line:
.,4 . The half-life of a drug is calcu lated as: (0.7 )( the volume of distribution) divided by cleara nce,
·,5. or t'/2 = (0.7)( Vd) I CL.
·,6.
· 17 •
· 18 • I ill ;fi , .j 1.1 for year: 2015
F1II.5T AID FACTS
· ,9 •
. 20 .
FA15,p . 2"'3.1
· 21 • Pharmacokinetics
· 22 • Bioavail abil ity (F ) Fraction of administered drug reaching systemic circliblioll ullchan ged. For an IV dose. F '" 100%.
Omll)': F t)'llicall\, < 100% title 10 incompl ete absorpt ion and first-pass metabolism .
. 23 .
Vol u me o f d istribut ion Theoretical I'Oluml' occupied by the lolal amollnt of drug in th e 1>0(1), reblil'l' 10 it s plasma
· 24 . (Vd) concentratioll , App:lrcnt Vo of plasma prolcill-houud drugs c~n he ahered b)' li\'er ,1",1 kiduey
· 25 • disease (I proleill hilldillg. t Vd). Dru gs may di slrilmle ill more lhall olle COlllparhnell t
V ;llI1ount of drug ill the hod),
. 26 .
d pl~sll'" drugcullcelltr;ltiou
· 27 • (OMPARlM!HI [JIUX; TYPES
· 28 •
"Low Blood Large/c harged moleculcs; plasma protcin oouud

· 29 . Medium ECF Small hydrophilic molccules


High Small lipophilic luoleClll es. espcdall)' if hmllHI

.".
· 12 •
.,3.
. ,4 .
An 8-year-old boy comes to the physician 's office compla ining of a sore throat. He and his
family recently immigrated to the United States from Cameroon, and his parents do not
know if he has had any vacc inations. Currently, he is having difficulty breathing and
swallowi ng. A grayish-white muco us film is noted on the boy's oropharynx .

·,5 . Which of the follo wing best explains this patient's symptoms?
·,6. 134415: zheng
· 17 • We value your feedba ck!
A. I mpaired initiation of tran slation
· 18 •
· ,9 • B. I mpaired translocation
. 20 . C. I nactivated 30S ribosome
· 21 •
D. I nactivated 80S ribosome
· 22 •
. 23 . E. I nhibition of acetylcholine release
· 24 .
· 25 •
. 26 .
· 27 • The correct answer is B. 42% chose this.
· 28 • This boy has diphtheria. It is caused by Corynebacterium diphtheriae, a bacterium that produces
· 29 . diphtheria toxin. Diphtheria toxin inhibits elongation fa cto r EF2- a. EF2-a is an important
molecule in protein synthesis. It co mplexes with guanosine triphosphate and binds to the
ribosome, ini tiating the translocation step of protein synthesis.
,Pad 9 2226 6.\« • •

+ G R< UI usmle rx com/ap l#qmax1 + D :


--<l t:> _! ~. C';'\I

".
Item' 16of 45 L • Mark
QID 1211 i PreYlous Next la b~ ues Notes Cakul~tor

The correct answer is B. 42% chose this .


This boy has diphtheria. It is caused by Corynebacterium diphtheriae, a bacterium that produces
·,2.
. diphtheria toxi n. Diphtheria toxin inhibits elongation factor EF2- a. EF2-Q is an important
· 13 • molecule in protein synthes is. It complexes with guanosine triphosphate and binds to the
. ,4 . ribosome, initiating the translocation step of protein synthesis .
·,5. Oip(11heri~

·,6 . A is not correct. 33% chose this.


· 17 • Initiation of translation involves eukaryotic initiation factor 2. Diphtheria toxin inhibits
· 18 •
elongation factor EF2-a, which is involved in the translocation step of protein synthesis. In
diphtheria, initiation occu rs norma lly.
·,9. Oiphlheri~

. 20 .
C is not correct. 13% chose this.
· 21 •
Antibiotics suc h as tetracycline and aminog lycosides inhibit the 305 subunit of ribosomes found
· 22 •
in prokaryotes. Diphtheria toxin is causing the patient's symptoms, not a drug .
. 23 . Tetr~cyc l H)e Ribosomes Olphtherla

· 24 . D is not correct. 9% chose this.


· 25 •
This boy has diphtheria. Diphtheria toxin does not inhibit ribosomes. Instead, it inhibits the
.26 . trans location of proteins being synthesized.
· 27 • Oipfltheria Ribosomes

· 28 • E is not correct. 3% chose this.


· 29 . Inhibiti on of acetylcholine release is ca used by a toxin released by Clostridium botulinum.
Botulism causes flaccid pa ralysis .
Botulism Acetylchol ine Flaccid paralysis

· 2

·3 • • D is not correct. 9% chose this.


This boy has diphtheria. Diphtheria toxin does not inhibit ribosomes. Instead, it inhibits the

·5 •
trans location of proteins being synthesized.
·4
·6 •
Oipfltheria Ribosomes

•••
E is not correct. 3% chose this.
Inhibiti on of acetylchol ine release is caused by a toxin released by Clostridium botulinum.
· 7 Botulism causes flaccid para lysis .
·8 Botulism Acetylchol ine Flaccid paralysis

.,0 •
·9

.".
· 12 •
. ,3.
·,4 .
. Bottom line:
Corynebacterium diphtheriae produces diphtheria toxin. Diphtheria toxin inhibits elongation
factor EF2-a, in hibiting translocation and protein synthesis .
Oiphtheria
j
·,5.
·,6. ul;fii,' .. , for year: 2015
FIII.5T ""D F"' CTS
· 17 •
· 18 • FA1S. p . 130.3
·,9. Corynebacterium Corrue = cluh ~ h apcd.
diphtheriae Hlad: colollics 011 c),slillc-Icl lurilc ag;.r.
AB C DEFC:
,Pad "" 2234 63« _ .

+ G R< UI usmle rx c om/ap l#qmax1 + III :


Item ' 17 of 45
QID 1719 i
L • Mark --<l
Pre Ylous
t:>
Next
_!
la b~ ues
~.
Notes
C';'\I
Cakul~ tor

··1213""
· 14 "
A 95-year-ol d woman is transferred to the intensive ca re unit after a 3-day history of cough
and declining mental status. Her blood pressure is 85/50 mm Hg, pulse is 124/min,
temperature is 39.8°( (103.6°F), and respiratory rate is 27/min. Crackles are heard at the left

·15 "
· 16 "
lower lung base, and the patient is sufferi ng from rigors intermittently. Blood and sputum cultures
drawn at the onset of symptoms grow strains of Klebsiella pneumoniae resistant to all antibiotics
except polymyxin B.

··17"
18"
· 19 "
Which of the following is a serious adverse effect of polymyxin B?
134415 : zheng
· 20 "

··2223 ""
• 21 "
A. Gran ul ocytopenia

8. Hearing loss
· 24 "
C. Hemolysis in patients with glucose-6-phosphate dehydrogenase deficiency
· 25 "
· 26 " D. Numbness of the extremities

·27 "
· 28 "
E. Severe vomiting

We value your feedback!

Polymyxins bind to gram-negative bacteria l cell membrane phospholipids and destroy the
· 13 " membrane by acting like a detergent and are predominantly used to treat severe gram-negative
· 14 " infections that are resistant to less toxic antimicrobials. Numbness of the extrem ities is an
· 15 " adverse effect of polymyxins; other adverse effects include neph rotoxicity, dizziness,
drowsiness, confusion, nystagmus, and blurred vision.
· 16 "

·17"
· 18 "
Phospholipids Confusion Numbness Dizziness Drowsiness Nystagmus

A is not correct. 29% chose this.


Granulocytopenia is not associated with polymyxin B. Antibiotics associated with
· 19 "
granu locytopenia include trimethoprim and dapsone.
· 20 " Dilpsone

··2223 ""
• 21 " B is not correct. 20% chose this.
Ototoxicity is a common adverse effect of aminoglycosides and vancomycin, particularly when
the two agents are used in combination, but it is not an adverse effect of polymyxins.
· 24 " Ototoxicity

· 25 "
C is not correct. 14% chose this.
· 26 " Patients with glucose-6-phosphate dehydrogenase deficiency are predisposed to hemolys is,
·27 "
· 28 "
particularly when takin g drugs with a high redox potential. These incl ude primaqu in e and
sulfonamides but not polymyxins.
P,imilquine Deficiency of glucose-6·phosphate dehydrogenase

E is not correct. 7% chose this.


Vomiting is an adverse effect of many antibiotics, but it is not a common adverse effect of
polymyxins.
,Pad "" 2234 63« _ .

+ G R< UI usmle rx c om/ap l#qmax1 + III :


Item ' 17 of 45
QID 1719 i
L • Mark --<l
Pre Ylous
t:>
Next
_!
la b~ ues
~.
Notes
C';'\I
Cakul~ tor

··1213""
Patients with glucose-6-phosphate dehydrogenase deficiency are predisposed to hemolys is,
particularly when taking drugs with a high redox potential. These include primaquine and
sulfonamides but not polymyxins.
· 14 "

·15 "
· 16 "
PrirThlquine Deficiency of glucose-6-p!1ospl!ate dehydrogenase

E is not correct. 7% chose this.

··17"
Vomiting is an adverse effect of many antibiotics, but it is not a common adverse effect of

18"
· 19 "
polymyxins.
Vom iting

· 20 "
Bottom II ne:

··2223 ""
• 21 "
Polymyxins are detergent-like antibacterial agents with gram-negative coverage that bind to
and cause destruction of cell membranes. Major toxicities include numbness of the
extremities, confusion, drowsiness, nystagmus, and blurred vision.
· 24 " (oofusion Numbness Drowsiness Nystagmus

· 25 "
· 26 "

·27 "
· 28 "
I il1ifi'·'I.) for year:
FIR S T AID FA CTS
2015

FA15, p. 138.2

K/ebs;ello An int estin al Aora that callSes loha r pne un lOn i" .. A'~ of KlebsiellA-
in alcoholics and diabetics wh en aspirakd .
Verr Illncoid colonies ca used br abu ndant
polysaccharid c ca pslllcs. Dink rcd "c urrant

A 35-year-old man presents to his primary care physician with weakness and fatigue. The
· 13 " patient has a 20-year history of smoking but quit 1 month ago, when cigarettes began to
· 14 " make him nauseated. On physical examination, the patient has mild abdominal tenderness
· 15 "
in the right upper quadrant, and his skin tone is slightly yellow. Laboratory studies are shown in
the list. The primary care physician decides to start the patient on lamivudine and a second
· 16 " antiviral therapy that acts on RNA viruses and plays a role in natural killer cell and macrophage
·17"
· 18 "
activation.
HIV ELISA negative
HCV PCR negative
· 19 "
HBsAg positive
· 20 " HBsAb negative
HBcAg positive
HBcAb negative

··2223 ""
• 21 "
HBeAg positive

· 24 " Which of the following is a dose-limiting adverse effect of this second agent?
· 25 " 13441 5 : zhe ng
We value your feedback!
· 26 " A. Hemolytic anemia

·27 "
· 28 "
B. Nephrotoxicity

C. Neutropenia

O. Peripheral neuropathy

E. Retinopathy
,Pad "" 2235 63« _ .

+ G R< UI usmle rx c om/ap l#q max1 + III :


Item ' 18 of45 L • Mark --<l t:> _! ~. C';'\I
QID 17S4 i Pre Ylous Next la b~ ues Notes Cakul~ tor
, "
, 12 "
The physician most likely prescribed interferon therapy for hepatits B infection. Interferons are
, 13 " glycoproteins produced by human leukocytes in response to a viral infection. a-Interferon acts
, 14 " by degrading viral messenger RNA and inhibits protein synthesis. Interferons also stimulate the
, 15 " host immune system, particularly natural killer cells and cytotoxic T lymphocytes. a-Interferon is
effective in the treatment of hepatitis B and hepatitis C as well as some cancers, but many
' 16 "
adverse effects can occur. Flu-like symptoms, psychiatric disturbances, and pancytopenia are
, 17 " common. The neutrophil count drops during therapy, returning to normal 4 weeks after
, 18 " cessation of therapy. Most patients can tolerate this drop, but it can be dose-limiting.
Immune 5)'5tem Leukocyte5 Natural Killer Cell5 Cytoto~ i ( T·Lymphocyte5 Influenza·like 5ymptom5
, 19 "
, 20 " A is not correct. 16% chose this.
, 21 " Hemolytic anemia results from prematu re breakdown of RBCs in the cytoplasm. A variety of
, 22 " etiologies for hemolysis exist, including autoimmune, congenital (such as hereditary
spherocytosis), and drug induced. In hemolytic anemia, the reticu locyte count shou ld be
, 23 "
increased appropriately along with an increase in bilirubin and LDH. This is not seen with G-
, 24 " interferon use.
' 25 " Cytopla5m Hereditary 5pherocyt05i5

, 26 " B Is not correct. 24% chose this.


, 27 " Nephropathy is a side effect of many medications, manifested by a decrease in glomerular
, 28 " filtration rate (GFR). Increases in creatinine and blood urea nitrogen are often used as markers
for nephropathy and indicate a decrease in GFR. Doubling the plasma creatinine indicates that
the GFR has halved. Nephropathy is not associated with interferon therapy.
Glomeru lar filtration Creatinine

, 13 " o Is not correct. 14% chose this.


, 14 " Peripheral neuropathy includes sensory and motor loss in the extremities with relative
, 15 " preservation of function in the trunk. It is commonly seen in patients with HIV due to the effects
of HIV and to the adverse effects of high ly active antiretroviral therapy.
' 16 "
Peripheral Neuropathy Antiretroviral Therapy. Highfy Active
, 17 "
E is not correct. 4% chose this.
, 18 "
Retinopathy is a rare adverse effect of antimicrobial therapy. It commonly occurs with use of
, 19 "
cidofovir, an antiviral, and rifabutin, which is used to treat resistant antimicrobials. It is not
, 20 " associated with interferon therapy.
, 21 " Cidofovir Rifabutin

, 22 "
, 23 "
Bottom Line:
, 24 "
Interferon therapy results in dose-dependent neutropenia during the first 4 weeks of therapy.
' 25 " NelJlropenia
, 26 "
, 27 "
, 28 "
141 ;fi , .j 1.1 for year: 2015
Ftll.ST A I D FACTS

FA1S.p. I95.1
,Pad "" 2235 63« _ .

+ G R< UI usmle rx c om/ap l#q max1 + III :


Item ' 190f45 L • Mark --<l t:> _! ~. C';'\I
QID 1367 i Pre Ylous Next la b~ ues Notes Cakul~ tor

··1213""
· 14 "
The enzyme hexokinase cata lyzes the first step of glycolysis, the phosphorylation of glucose
in the 6-carbo n position. In the liver, a different ki nase called glucokinase cata lyzes this step.
A resea rcher wishes to determine whether other tissues in addition to the liver express
IAA[

·15 "
· 16 "
glucokinase .

··17"
Which of the following techni ques would be most useful for this researcher?

18"
· 19 "
A. Northern blot analysis to detect DNA
We value your feedback! / 134415 : zheng

· 20 " B. Northern blot analysis to detect RNA

··2223 ""
• 21 " C. Southern blot analysis to detect DNA

D. Southern blot analysis to detect RNA


· 24 "
E. Western blot ana lys is to detect RNA
· 25 "
· 26 "

·27 "
· 28 "
The correct answer is B. 55% chose this.
A Northern blot analysis detects RNA usi ng a DNA probe to bind to the sample RNA (DNA-RNA
hybridization). In this example, detection of gl ucokinase messenger RNA from the liver sample
would demonstrate that the liver celis are expressing glucokinase.

A Northern blot analysis detects RNA usi ng a DNA probe to bind to the sample RNA (DNA-RNA
· 13 "
hybridization). In t his example, detection of glucokinase messenger RNA from the liver sa mple
· 14 " would demonstrate that the liver celis are expressing glucokinase.
· 15 "
A Is not correct. 4% chose this.
· 16 "

·17"
· 18 "
Although Northern blot analysis is the app ropriate technique for use in this example, it rel ies on
the use of a DNA probe to det ect sample RNA, not DNA.
C is not correct. 17% chose this.
· 19 " Southern blot analysis detects DNA with the use of a DNA probe to bind to the sam ple DNA. In
· 20 " this case, DNA sequences would not demonstrate a change in expression between two different
tissue types. Instead, RNA should be studied with the use of the Northern blot techni que.

··2223 ""
• 21 "
o is not correct. 8% chose this.
Southern blot analysis is used to detect DNA, not RNA, from a sample.
· 24 " E is not correct. 16% chose this.
· 25 "
Western blot analysis is used to detect protein, not RNA.
Western Blot
· 26 "

·27 "
· 28 " Bottom Line:
The Northern blot involves DNA- RNA hybrid ization usin g DNA probes for detection of specifi c
RNA sequences and is the appropriate molecular biological method with wh ich to distingu ish
expression patterns between tissues in this example.
,Pad "" 2235 63 « _ .

+ G R< UI usmle rx c om/ap l#q max1 + III :


Item ' 190f45 L • Mark --<l t:> _! ~. C';'\I
QID 1367 i Pre Ylous Next la b~ ues Notes Cakul~ tor

··1213""
Southern blot analysis is used to detect DNA, not RNA, from a sa mple.
E is not correct. 16% chose this.
· 14 " Western blot analysis is used to detect protein, not RNA.

·15 "
· 16 "
Western Blot

··17"
Bottom II ne:
18"
· 19 "
The Northern blot involves DNA-RNA hybridization using DNA probes for detection of specific
RNA sequences and is the appropriate molecular biological method with which to distinguish
· 20 "
expression patterns between tissues in this example.

··2223 ""
• 21 "

Ul;f1'·'I"
FIRST AID FACTS
for year: 2015
· 24 "
FA15, p. 78. '
· 25 "
Blotting proce dures
· 26 "

·27 "
· 28 "
Southern bl ot t\ DNA sample is cn'.r"\;1t;c~ 11)' dca,-ed into
sl11Jll cr pieces, electrophoresed on a gel. and
the n tran sferred to a filter. T he filter is then
SN(lW DRoP·
S(lut hern '" D NA
Nort hern '" RNA
waked in a denaturant and subsequently \\ 'e skrn '" Protein
exposed to a radi olabd ed DNA probe that
reeognizes and anneals to its C"Omplement ary
strand. The resulting doublc-stranded. 1abcled
pieee of DN,\ is "iSU;lli1.e(1 wheu the filter is
exp(lsetl t(l film.

A concerned mother brings her S-year-old so n to the pediatrician's office. She explains that
· 13 " the child has been healthy since birth, but has recently begun to have difficulty standing up.
· 14 " In the office, the physician notices that when the child wants to sta nd up, he uses his hands
· 15 "
and arms to push himself to a sta nding position, as shown in the image. He is diagnosed with a
genetic condition.
· 16 "

·17"
· 18 "
· 19 "
· 20 "

··2223 ""
• 21 "
Image courtesy of Wikimedia Commons

· 24 " Which other disorder shares the same inheritance pattern as this patient's genetic disease?
· 25 " 134415: zheng
We value your feedback! ]
· 26 " A. Cystic fibrosis

·27 "
D~ J
B. Hemochromatosis
· 28 "
C. Hemophilia A

0_ Marfan's syndrome

E. Neurofibromatosis type 1
,Pad "" 2236 62« _ .

+ G R< UI usmle rx c om/ap l#q max1 + III :


Item' 20 of 45
QID 4460 i
L • Mark --<l
Pre Ylous
t:>
Next
_!
la b~ ues
~.
Notes
C';'\I
Cakul~ tor

··1213""
The correct answer is C. 60% chose this.
The disease in the question stem is Duchenne's muscular dystrophy . This is caused by an x -
· 14 " linked frame-shift mutation and results in accelerated muscle breakdown. Calf muscles become

·15 "
· 16 "
en larqed due to fatty replacement of muscle, called calf pseudohypertrophy. Hem ophilia A and
B are inherited in the same manner .

··17"
Muscu lar Dys{roph~ Readlng-frame-shift mutation test resu lt HemophIlia A

18"
· 19 "
A Is not correct. 11% chose this .
Cystic fibrosis is an autosomal recessive disorder.
B is not correct. 8% chose this.
· 20 "
Hem och romatosis is caused by abnormally high storage of iron. It is an autosoma l recess ive
disorder.

··2223 ""
• 21 "
Hemochromatosis

D Is not correct. 10% chose this.


· 24 "
Marfan's syndrome has an autosomal dominant inheritance pattern. It is caused by a mutation
·25 "
· 26 "
in fibrillin and results in tall individuals with long extremities; pectu s excavatum;
hyperextensible joints; long, tapering fingers and toes (arachnodactyly); cystic medial necrosis

·27 "
· 28 "
of the aorta; and a floppy mitral valve.
Mutation Arachnodactyly Congenital funnel chest Toes Aona Mitral Valve Prolapse Syndrome

E is not correct. 11% chose this.


Neurofibromatosis types 1 and 2 share an autosoma l dominant inheritance pattern. This
disorder is characterized by cafe-au-Iait spots, neural tumors, and Lisch's nodules.
Neurofibromatosis I

in fibrillin and results in tall individuals with long extremities; pectus excavatum;
· 13 " hyperextensible j oi nts; long, tapering fingers and toes (arachnodactyly); cystic medial necrosis
· 14 " of the aorta; and a floppy mitral valve.

·15 "
· 16 "
Mutation Arachnodactyly Congenita l fUrlnel chest Toes Aorta Mitral Valve Prolapse Syndrome

E is not correct. 11% chose this.


·17"
· 18 "
Neurofibromatosis types 1 and 2 share an autosoma l dominant inheritance pattern. This
di sorde r is characterized by cafe-au-Iait spots, neural tumors, and lisch's nodul es.
Neurofibromatosis 1
· 19 "
· 20 "
Bottom line:

··2223 ""
• 21 "
Duchenne's muscular dystrophy and hemophilia A are X-linked recessive disorders.
MusCUlar Dystrophy Hemophilia A

· 24 " -
·25 "
· 26 "
iii lif, ,
FlII. S T AI O FA CTS
., 1.1 for year: 2015
·27 "
· 28 " FAtS. p. 84.3

X-linked recessive BTIlton ag:nnm aglobn linemia. Wiskolt-t\ldri ch Be Wise, Fool"s COLD I leeds Silly II O pe
disorders syndrome. Fabry diseasc. C 6PD dcfieicne)"
Oen br ~lbinism. Lesch-N),han s),ndromc.
Duchenne (and Bccker) muscular dystrophy.
It unlcr Syndrome. Ilcmoph ilia A and B.
O rnithine trJnscarbam),lase deficiency.
·1 1 i- I a
,Pad "" 2236 62« _ .

+ G R< UI usmle rx c om/ap l#q max1 + III :


Item' 21 of45 L • Mark --<l t:> _! ~. C';'\I
QID 2537 i Pre Ylous Next la b~ ues Notes Cakul~ tor

··1213""
· 14 "
A 34-year-ol d woman present s to her primary care phys ician wit h a lump on the front of her
neck, at the midline. A fine-needle biopsy reveals pa pillary thyroid cancer. The patient elects
to have a total thyroidectomy. The surgery is uneventful. but the patient finds that after

·15 "
· 16 "
surqery her voice has changed, and is now hoarse and raspy. On laryngoscopy, it is noted that the
right vocal cord does not move.

··17"
18"
· 19 "
The most likely damaged structure courses around wh ich of the following structures?

We va lue your feedback!


134415 : zheng
A. Ligamentum arteriosum
· 20 "
B. Right brachiocephalic artery
• 21 "

··2223 ""
· 24 "
C. Right subclavian artery

D. Right superior thyroid artery


· 25 " E. Right suprascap ula r artery
· 26 "

·27 "
· 28 "

The correct answer is C. 42% chose this .


Damaqe to one or both of the recurrent laryngea l nerves is a complication of thyroid, lung, or
heart surgery . I nj ury can also occu r during placement of an endotrac heal tube or by masses or
swollen lymph nodes in the mediastinum. The most common symptoms of recurrent
nerve damage are hoarseness, difficulty speaking, and dysphagia. The right rec urrent I

· 13 " Damaqe to one or both of the recurrent laryngeal nerves is a compl ication of thyroi d, lung, or
· 14 "
heart surgery . I nj ury can also occur during placement of an endotracheal tube or by masses or
· 15 "
swollen lymph nodes in the mediastinum. The most common symptoms of recu rrent laryngeal
nerve damage are hoarseness, di fficulty speaking, and dysphagia. The right recurrent laryngeal
· 16 " nerve courses around t he right subclavian artery. When bilatera l damage occurs, bilateral vocal
·17"
· 18 "
cord paralysis leads to obstruction of the upper ailWay, and intubation is contraindicated. A
tracheostomy. the creation of a surgical opening into the trachea, must be performed to restore
the ailWay.
· 19 "
Intubat,on Tracheostomy p!"ocedure Recurrent Laryngeal Nerve CardIac Surgery p!"ocedures Trachea
· 20 "
A is not correct. 13% chose this.
• 21 "

··2223 ""
The left recurrent laryngeal nerve cou rses around the ligamentum arteriosum. The same
symptoms of hoarseness and voice change will occur with damage to this nerve, but the left
vocal cord would be pa ralyzed.
· 24 " Left recurrent laryngeal nerve

· 25 "
B is not correct. 14% chose this.
· 26 " The right phren ic nerve passes over the brach iocephalic artery. The phrenic nerve arises from
·27 "
· 28 "
(3-(5 and conta ins motor, sensory, and sympathetic nerve fibers. It supplies the diaphragm,
mediastinal pleura, and pericardium. Injury to this nerve results in paralysis of hal f of the
diaphragm. Remember t he mnemonic "C3, 4,5 keeps the diaphragm alive."
entire phrenic nerve SympathetIC nerve struc:tl.lre

o is not correct. 28% chose this .


The superior thyroid artery lies j ust superfic ial to the externa l larynge.31
I nerve is a terminal branch of the II,,rv,nar,al
,Pad"" 2236 62« _ .

+ G R< UI usmle rx com/ap l#qmax1 + III :


Item ' 21 of45 L • Mark --<l t:> _! ~. C';'\I
QI D 2537 i PreYlous Next la b~ ues Notes Cakul~tor

··1213""
D Is not correct. 28% chose this.
The superior thyroid artery lies j ust superficial to the externa l laryngeal nerve. The external
· 14 " laryngeal nerve is a terminal branch of the superior laryngeal nerve (itself a branch of the vagus).

·15 "
· 16 "
This nerve supplies the cricothyroid muscle, which varies the length and tension of the vocal
cord, and 50 inj ury to this nerve results in a monotone voice.

··17"
Vagus nerve structure Laryngeal Nerves Vocal (ord 5tructure

18"
· 19 "
E is not correct. 3% chose this.
The suprascapular artery passes over the superior transverse scapular ligament, whi le the
suprascapular nerve passes below. The suprascapular nerve passes laterally through the
· 20 " posterior triangle of the neck and innervates the supraspinatus and infraspinatus muscles and
the glenohumeral jOint. Injury to this nerve results in the "waiter's tip position" due to loss of
• 21 "

··2223 ""
lateral rotation of the humerus at the shou ld er.
Structure or suprascapular nerve EntIre supraspinatus Entire infraspinatus

· 24 "
· 25 " Bottom Li ne:
· 26 " The right recurrent laryngeal nerve courses around the right subclavian artery.

·27 "
· 28 "
I ill ;fi , .j 1.1 for year: 2015
FIRST AID FACTS

FA15. p. 330. 1

Thy roid ca ncer

A 3·week·old infant presents with failure to thrive, poor feeding, and lethargy. A physical
· 13 " examination reveals an enlarged liver and jaundice. Laboratory analysis reveals an elevated
· 14 " blood galactitollevel and increased urinary reducing substance .
· 15 "
· 16 " Which of the following could desc ribe the levels of intermediates of galactose metabolism in this

·17"
· 18 "
patient?

We value your f eedback! 13441 5 :zheng


· 19 "
A. Decreased galactose /
· 20 " B. Decreased uridine diphosphoglucose

··2223 ""
• 21 " C. Elevated glucose· '·phosphate

" D. Increased galactose· l -phosphate


· 24 "
E. I ncreased glycogen
· 25 "
· 26 "

·27 "
· 28 "
The correct answer is D. 59% chose this.
Elevated galactitol levels is the cause of the clinical symptoms of galactosemia. The urine
reducing substances assay is used to detect inborn errors of carbohydrate metabolism such as
ga lactosemia. Galactose is converted in two steps to glucose-l-phosphate. The first step is
catalyzed by the enzyme gal acto kinase, which phosphorylates ga lactose to galactose-l -
phosphate. The second step is catalyzed by galactose-l-phosphate uridyltransferase (G
,Pad "" 2236 62« _ .

+ G R< UI usmle rx c om/ap l#q max1 + III :


_!
.
Item' 22 of 45 L • Mark --<l t:> ~. C';'\I
QID 1521 i Pre Ylous Next la b~ ues Notes Cakul~ tor

""
'.

.13"
· 14 "
Elevated galactitol levels is the cause of the clinical symptoms of galactosemia. The urin e
reducing substances assay is used to detect inborn errors of carbohydrate metabolism such as

·15"
· 16 "
ga lactosemia. Galactose is converted in two steps to glucose-l-phosphate. The first step is
catalyzed by the enzyme galactokinase, which phosphorylates ga lactose to galactose-l-
phosphate. The second step is catalyzed by galactose- l -phosphate uridyltransferase (Gl PUR),

··17"
which converts galactose-l-phosphate to glucose-l-phosphate. In the absence of Gl PUR,

18"
· 19 "
upstream intermediates in galactose metabolism, includ ing galactose-1-phosphate and
ga lactitol, accumulate. A deficiency in enzymes involved in other aspects of galactose
metabolism leads to a much milder presentation (ie, only infantile cataracts). Although
· 20 " treatment is not ava ilable, prevention of disease progression involves excluding galactose-
containing foods (eg, breast milk and lactose-containing formulas) from the infant's diet.

··2223 ""
• 21 " Metilbolism Ga lactose Classical g<llactosemia Assay Glucose Breast

A is not correct. 7% chose this.


Galactose is upstream from Gl PUR, and its levels would be elevated.
· 24 "
Galactose
· 25 "
B is not correct. 18% chose this.
· 26 "

·27 "
· 28 "
Classic galactosemia is caused by a deficiency of this uridyltransferase and would theoretically
lead to a build up of galactose-1-phosphate, uridine diphosphoglucose, and galactose, and
decreased glucose-1-phosphate.
Galactose Urid,ne Classical ga lactosemia Glucose

C is not correct. 12% chose this.


Glucose-1-phosphate is a downstream prod uct of G1 PUR and would be decreased in
ga lactosemia.

Galactose Urid,ne Classicalga lac{osemia Glucose


· 13 "
C is not correct. 12% chose this.
· 14 "
Glucose-l-phosphate is a downstream product of G1 PUR and would be decreased in
· 15 "
ga lactosemia.
· 16 " Glucose Classical galactosemia

·17"
· 18 "
E Is not correct. 4% chose this.
Glucose- l -phosphate is an intermediate in glycogen pathways. However, galactosem ia does not
· 19 " affect glycogen levels.
· 20 " Glucose Classical galactosemia

··2223 ""
• 21 "
Bottom Line:
Patients with galactosemia lack the enzyme galactose-l-phosphate uridyltransferase (Gl PUR),
· 24 "
which converts galactose·l -phosphate to glucose·1·phosphate. The absence of this enzyme
· 25 " leads to the accumulation of galactose-l-phosphate and ga lactitol.
Galactose Class ical galactosemia Glucose
· 26 "

·27 "
· 28 "
ul;fi'·i!·' foryear: 2015
FIIU T " 10 FACTS

FAtS, p . 103.2
,Pad "" 2236 62« _ .

+ G R< UI usmle rx c om/ap l#q max1 + III :


Item ' 23 of 45
QID 2693 i
L • Mark --<l
Pre Ylous
t:>
Next
_!
la b~ ues
~.
Notes
C';'\I
Cakul~ tor

··1213""
· 14 "
A 25-year-old man presents with muscle pain, sporadic muscle twitching, weakness, burning
abdomina l discomfort, and decreased libi do. He has a family history of multiple cancers.
Fluorescent in situ hybridization reveals an abnormality of chromosome 11. Laboratory tests
IAA[

·15 "
· 16 "
show:

Calcium: 12.3 mg/dL

··17"
Parathyroid hormone: 125 pg/mL
18"
· 19 "
Albumin: 5.0 g/dL
Gastrin (serum): 200 pg/mL (norma l 0-100 pg/mL)

· 20 "

The most appropriate pharmacologic treatment for this patient's gastric symptoms is which of the

··2223 ""
• 21 "
following?
134415 ; zheng
· 24 "
We va lue your feedback!
A. Bromocriptine
· 25 "
8. Leuprolide
· 26 "

·27 "
· 28 "
C. Omeprazole

D. Phenoxybenzamine

E. PrazQsin

· 13 " This patient presents with multiple endocrine neoplasia type 1 (MEN-1), which is characterized
· 14 " by the "3 Ps ": Parathyroid adenoma (causing weakness and muscle twitching), Pancreatic tumor
· 15 "
(such as a gastrinoma/Zollinger Ellison syndrome, as in this case, causing burning epigastric
discomfort), and Pituitary adenoma (causing hypogonadism, in this case as a result of a
· 16 " prolactinoma). Laboratory findings of increased serum calcium and parathyroid hormone levels,
·17"
· 18 "
as well as elevated gastrin level further support the diagnosis. MEN-1, also known as Wermer's
syndrome, is inherited as an autosomal dominant mutation of the MfN1 gene on chromosome
11. For pancreatic islet cell tumors that produce insulin, glucagon, or vasoactive intestinal
· 19 "
peptide, first-line treatment is surgical resection of the tumor, but for gastrin-producing
· 20 " tumors, medical treatment such as a proton pump inhibitor is indicated.
Hypogonadism Prola(tinoma Mutation Multiple Endocrine Neoplasia Type 1 Weakness Mus(le twitch Zollinger-Ellison syndrome

··2223 ""
• 21 "
Serum cakium measurement Islet Cell Tumor Insulin glucagon (rONA)

A is not correct. 11% chose this.


· 24 "
Although bromocriptine may help with the patient's libido and other symptoms related to the
· 25 " prolactinoma (as a dopamine agonist, it decreases prolactin), it is not the most appropriate
· 26 " treatment for gastric symptoms.

·27 "
· 28 "
Bromocripl:lne Libido Prolactinoma

B is not correct. 9% chose this.


Leuprolide is a gonadotropin-releasing hormone (GnRH) analog used in a continuous fashion to
decrease the secretion of GnRH in prostate cancer. It can also be used in a continuous fashion
to treat uterine fibroids and in a pulsatile fashion to treat infertility
Leuprolide Infertility Malignant neoplasm of prostate Utenne Fibroids
,Pad "" 2236 62« _ .

+ G R< UI usmle rx c om/ap l#q max1 + III :


Item ' 23 of 45
QID 2693 i
L • Mark --<l
Pre Ylous
t:>
Next
_!
la b~ ues
~.
Notes
C';'\I
Cakul~ tor

··1213""
· 14 "
D Is not correct. 6% chose this.
Phenoxybenzamine is a nonselective a blocker used in the treatment of pheochromocytoma.

·15 "
· 16 "
Phenoxybenz<lmHle

E is not correct. 3% chose this.

··17"
Prazosin is an Q1-antagonist used to treat hypertension.

18"
· 19 "
Prazosin

· 20 " Bottom li ne:


MEN- ' (Wermer's syndrome) is inherited as an autosoma l-dominant mutation of chromosome

··2223 ""
• 21 "
11, and causes a tumor in the "3 Ps": Pituitary, Parathyroid, and Pancreas.
Mut<ltion

· 24 "
· 25 "
· 26 "
ul;f.,.·,,,, for year:
FIII.5T "" 0 FACTS
2015

·27 "
· 28 "
FA15, p . 376.3

Proton pump inhibitors Omcprazolc. lansopmzolc. csomepra:w1c. pan[oprazole, dcxlansoprnzolc.


MWWIISM Irre\'ersih ly inhibit H+/ K+ATPase in stomach parietal cells
WN I{Al US< Peptic ulcer, gastrit is, esophageal rcAux, Zollinger-ElIiS<l" syndrome.
1011(111 h,erc;,st>d ri sk of C. Ji{ficile infl><:tio1l. pncI1I1Iol1ia. I 5CTIIIlI Mg!+ with long-terlll lise.

A biotechnology company is developing a small protein to block the cascade by which


· 13 " allergens can cause shock, In this type of shock, patients often d ie of vasodi lation and
· 14 " massive edema; therefore, it is thought that the new protein could lead to a life-saving drug .
· 15 "
· 16 " Which of the following molecules is the most promising ta rget to block the anaphylactic pathway?

·17"
· 18 " A. v-Interferon
We valu e your feedback!
134415; zheng

· 19 "
8. C5b
· 20 "
C. Hageman factor

··2223 ""
• 21 "
X D. IgE surface receptors

E. I nterleukin-'
· 24 "
· 25 " F. Nitric oxide
· 26 "

·27 "
· 28 "
The correct a nswer Is D. 49% chose this.
Anaphylactic shock is a life-threatening disorder that occurs when an allergen activates mast
cells and basophils, lead ing to widespread release of histamines, serotonins, and other
compounds stored in the granules of these immune cells. These compounds lead to
vasod ilation and leaky capillaries. The cross·linking of IgE receptors present on these cell
,Pad "" 2237 62« _ .

+ G R< UI usmle rx c om/ap l#q max1 + III :


Item ' 24 of 45 L • Mark --<l t:> _! ~. C';'\I
QID 3233

··1213 ""
i
. . ."
Pre Ylous
, ".
Next
.
la b~ ues Notes Cakul~ tor

Anaphylactic shock is a life-threatening d isorder that occurs when an allergen activates mast
cells and basophil s, lead in g to widespread release of histamines, serotonins, and other
· 14 " compounds stored in the granules of these immune cells. These compounds lead to
·15 "
· 16 "
vasod ilation and leaky capillaries . The cross-linking of IgE receptors present on these cells
causes the activation of these cell types. By blocking this cross-reaction, the anaphylactic

··1718 "" pathway can be stymied.


a<l5ophils ma51 (ell

A is not correct. 7% chose this.


· 19 "
v-Interferon is produced by T-Iymphocytes and, among other functions, activates tumoricida l
· 20 "
macrophages.
• 21 " B is not correct. 9% chose this.
· 22 " The C5b component of the complement cascade functions in cytolysis via the membrane attack
·23 "
· 24 "
complex (MAC). This is not the mechan ism involved in anaphylaxis.
anaphylaxis

· 25 " C is not correct. 7% chose this.


· 26 " Endotoxin can directly activate Hageman factor, activati ng the coagu lation cascade and leading
·27 "
· 28 "
to disseminated intravascu lar coagulation (DIC). However, the mechanism of shock through
anaphylaxis is not related to DIe.
a naphylaxis Disseminated Intravascular Coagulation

E Is not correct. 12% chose this.


The cytokine interleukin-l, released by macrophages activated by endotoxin, causes fever. Fever
is absent in anaphylax is.

·13 "
· 14 "
E is not correct . 12% chose this.
The cytokine interleukin·l , released by macrophages activated by endot oxin, causes fever. Fever
is absent in anaphylaxis.
· 15 " aNphyliI)(JS

· 16 " F is not correct. 16% chose this.


·17 "
· 18 "
Nitric ox ide, released by macrophages activated by endotoxin, causes hypotension (shock).
However, this is not the mechanism of anaphylactic shock.
NItriC OXide Hypotension
· 19 "
· 20 "

• 21 " Bottom line:


The cross-linking of IgE present on mast cells and basophils leads to the release of vasoactive
·23 "
· 22 "

· 24 "
compounds that then lead t o anaphylactic shock .
Bilsoptnls mast cell

· 25 "
· 26 "

·27 "
· 28 "
iii I jfi ,
fIRST "10 f"CTS
., 1.1 for year: 2015

FAI S, p. 211 .1
Hypersensitivity types
Type I Anaphylact ic a nd atopic-frce antige n CfOs~· First (Iype) and FOist (a naph) lax is). Types I. II.
lin ks IgE on prescnsitized m ast cell s and and III arc all ant ibody med iated.

,
basoph ils. t i ' immtxlia te release of Test: skin lest for speci fic IgE.
,Pad "" 2237 62« _ .

+ G R< UI usmle rx c om/ap l#q max1 + III :


Item' 2S of 45
QID 4488 i
L • Mark --<l
PreYlous
t:>
Next
_!
la b~ ues
~.
Notes
C';'\I
Cakul~tor

··1213""
· 14 "
A 16-year-o ld girl in a rural community has swollen, painful lymph nodes in her right axilla.
Physical examination reveals multiple healed scratches on her right arm with a papule
associated with one of the scratch marks. She states that the scratches occurred about 5

·15 "
· 16 "
weeks ago.

··17"
What organism is the most likely cause of this patient's infection?

18"
· 19 "
134415 : zheng

· 20 " A. Gram-negative coccobacillus, oxidase and catalase positive

··2223 ""
• 21 " B. Gram-negative proteobacterium; oxidase, cata lase, and urease positive

" C. Gram-negative proteobacterium; visualized with Warth in-Starry stain


· 24 "
O. Gram-positive rod, spore producing
· 25 "
· 26 " E. Yellow-white round worm

·27 "
· 28 "
We value your feedback!

The correct answer is C. 49% chose this.


· 13 "
OrOYil Fever Angiomiltos is. Bilcillary Ent ire Jymphatk vessel Silver prepilriltion
· 14 "
· 15 "
· 16 "

·17"
· 18 "
· 19 "
· 20 "
Image courtesy of CDC

··2223 ""
• 21 "
Bartonelfa hense/ae is a very small, gram-negative bacterium that is closely related to Rickettsia,
although it is able to grow on lifeless media. It is the cause of cat scratch fever (a local, chronic
· 24 " lymphadenitis most commonly seen in children) and bacillary angiomatosis (seen particularly in
· 25 "
patients with AIDS). In this latter patient population, the organism causes proliferation of blood
and lymphatic vessels, causing a characteristic "mulberry" lesion in the skin and subcutaneous
· 26 " tissues of the afflicted individual. As shown in the image, Bartonella hense/ae bacteria appear as
·27 "
· 28 "
darkly staining bacilli on a Warthin-Starry stain, which is a silver nitrate-based method.
A is not correct. 24% chose this.
Pasteurelfa multocida is a gram-neqative rod that is normal flora of the oral cavity of dogs and
cats. It often causes a loca l abscess following introduction under a person's skin via an animal
bite. Most cases occur in children who are injured while playing with a pet.
,Pad "" 2238 62« _ .

+ G R< UI usmle rx c om/ap l#q max1 + III :


Item' 2S of 4 5 Mark --<l t:> _! ~. C';'\I
. ..
L •

QID 4488

··1213 ""
i
;
PreYlous
, .- Next la b~ ues Notes Cakul~tor

Brucella canis is a gram-negative rod that is a zoonotic agent. Its normal host is the dog. When it
ga ins access to humans, however, it causes an undulating fever with malaise,
· 14 " lymphadenopathy, and hepatosplenomegaly. The normal route of exposure is via ingestion of
·15 "
· 16 "
the organism.
Infection due to Brucella can;s Lymphadenopathy

··1718 "" D is not correct. 9% chose this.


Clostridium tetani is a gram-positive spore-form ing anaerobic rod. It causes tetanus (a spastic
· 19 " pa ralysis caused by tetanospasmin, which blocks the release of the inhibitory neurotransmitters
glycine and y-am in obutyric acid). There may be no lesion at the site of inoculation, and
· 20 "
exudation would be extremely rare.
Tetanus G~cine EXOOatiOfl Spastic paralysis
• 21 "
· 22 "

·23 "
· 24 "
E is not correct. 4% chose this.
Toxocara canis, a common intestinal pa rasite of dogs, is a metazoan parasite that ca uses viscera l
larva migrans. Young child ren are most likely to be affected because they are most likely to
· 25 " ingest soil contaminated with eggs of the parasite.
larva Migrans. Visceral
· 26 "

·27 "
· 28 " Bottom li ne:
Bartonef/a hensefae causes the local lymphadenitis ca ll ed cat scratch fever, commonly in
children. The organism also causes bacillary angiomatosis.
lymphadenitis Oroya Fe~er Ar.giomatos is, Bacillary

.13 "
· 14 "
glycine and y-am inobutyric acid). There may be no lesion at the site of inoculation, and
exudation would be extremely rare.
Tetartus Glycine ExL.dation Spastic paralysis
· 15 "
E Is not correct. 4% chose this.
· 16 "

·17 "
· 18 "
Toxocara canis, a common intestinal parasite of dogs, is a metazoan parasite that ca uses visceral
larva migrans. Young children are most likely to be affected because they are most likely to
ingest soil contami nated with eggs of the parasite.
· 19 " larva M'grans. Visceral

· 20 "

• 21 " Bottom line:

·23 "
· 22 "

· 24 "
Bartonef/a hensefae causes the local lymphadenitis called cat scratch fever, commonly in
children. The organism also causes baci ll ary angiomatosis.
lymphadenitis Oroya Fe~er Ar.giomatO$is. Bacillary

· 25 "
· 26 "

·27 "
· 28 "
Iii lif;
FllST " 10
'I'I.'
F" CTS
for year: 2015

FAI 5,p. 142,1

Zoonotic bacteria Zoonosis: in fectiolls discase transmitted bctween 3ninmls '1nd humans.
lRAlISMI\510N ~D\oU~(

Anaplasma spp. Am1plam10sis Ixodes licks (!i"e on deer and mice)


,Pad 9 2238 6210 . '

+ G R< UI usmle rx com/ap l#Qmax1 + • :


Item" 26 of 45
QID 1&30
L

J.
• Mark -<l
Pre YlDu,
t>-
NeKt
all
la b"" ues
~.
Not es
~'t
c .. k ul~ tor

· 12 •
A homeless man arrives at the clinic with diarrhea, multiple mouth sores, an d a low-grade
· 13 • fever . He complains of intense pain on swa ll owing. Endoscopy reveals multiple diffuse white
.,4 . plaques on the esophagus. Gram-stained sample from scrapings of the plaques appear in
· 15 • the image. A resident remarks that a chron ic syndrome has facilitated the formation of the cu rrent,

"6.
· 17 •
more acute, infection.

· 18 •

"9.
' 20 .
' 21 •
· 22 •
· 23 •

' 24 .
' 25 .
' 26 .
• oJ'
· 27 •
· 2• •
Image courtesy of CDC/Dr. Stuart Brown

Based on recommendations from the Centers for Disease Control and Prevention, which of the
following is requ ired to diagnose the underlying chron ic illness in th is patient?

· 13 • A. A C04+ cell count <200/mm 3 only


.,4 .

."9.
B. Serologic evidence of HIV infection and a C04+ cell count <300/mm3
· 15 •

"6 .
· 17 •
.,
C. Serologic evidence of HIV infection only

D. Serologic evidence of HIV infection, a C04+ cell count <200/mm 3 , and/or one or
more defining opportunist ic infections
E. The presence of two or more defining opportunistic infections
' 20 .
· 21 •
· 22 • We value your feedback!
· 23 •
' 24 .
' 25 •
. 26 .
· 27 •
· 2• •
The correct answer is D. 57% chose this.
The patient in this vignette presents with candida (as shown in the image) esophagitis, whi ch is
an AIDS-defining illness. AIDS results from T-Iymphocyte destruction following infection with
HIV. In 1993 the CDC refined its definition of AIDS to include all HIV-infected persons with either
a C04+ cell count <200/mm 3 OR a C04 percentage <14%,
,Pad "" 2238 62« _ .

+ G R< UI usmle rx c om/ap l#q max1 + III :


Item' 26 of 45
QID 1830 i
L • Mark --<l
Pre Ylous
t:>
Next
_!
la b~ ues
~.
Notes
C';'\I
Cakul~ tor

··1213""
The patient in this vignett e presents with candida (as shown in the image) esophagitis, which is
an AIDS-defin ing illness. AIDS results from T·lymphocyte destruction fo ll owing infection with
HtV. In 1993 the CDC refined its definition of AIDS to include all HIV-infected persons with either
· 14 "

·15 "
· 16 "
a CD4+ cell count <200/mm 3 OR a CD4 percentage <14%,
Alternatively, the CDC gui delines also defined AIDS as the presence of an "AIDS-defining"

··17"
opportunistic infection (eg, candida esophagitis) in the context of a known HIV infection.

18"
· 19 "
Therefore, if serology is positive for HIV i nfection, this patient meets the CDC definition of AIDS
because of the esophageal candid iasis.
hopholgitis Cel l Count Opportunistic Infections C<lndidi<lsis Qf the esoph<lgus
· 20 "
A is not correct. 16% chose this.

··2223 ""
• 21 " A CD4+ cell count of <200/mm3 on ly is not a complete definition of AIDS, because there is no
evidence of HIV infection.
Cell CQum
· 24 "
B is not correct. 10% chose this.
· 25 "
Serologic evidence of HIV infection and a (04+ cell count of <300/mm3 do not constitute a
· 26 " complete definition of AIDS. The definition requ ires a (04+ count of less than 200/mm3. This
·27 "
· 28 "
cut-off is not random--at this level of (04+ depletion, the patient becomes more susceptible to
the AIDS-defining opportunistic infections.
Cell CQunt Opportunistic InfectiQJl!i

C is not correct. 12% chose this.


Serologic evidence of HIV infection only is not a complete definition of AIDS, because the ( 0 4+
count may not (yet) be sufficiently depleted to cause severe immunodeficiency.

· 13 " C is not correct. 12% chose this.


· 14 " Serologic evidence of HIV infection only is not a complete definition of AIDS, because the (04+
· 15 " count may not (yet) be sufficiently depleted to cause severe immunodeficiency.
· 16 " E is not correct. 5% chose this.

·17"
· 18 "
Despite the name, the presence of one or more "AIDS-defining" illnesses does not technically
ensure a diagnosis of AIDS until there is evidence of HIV infection.
· 19 "
· 20 " Bottom Li ne:
AIDS is defined by the CDC by a C04+ count <200/mm 3, (04 percentage <14%, and/or presence

··2223 ""
• 21 "
of one or more defining opportunistic infections, all in the setting of positive HIV serology.
Opportunistic Infections

· 24 "
· 25 "
· 26 "
ul;fi.·,.·1 for year: 2015
FaST -,.0 F-'CTS

·27 "
· 28 "
FAtS, p. 168.2

HIV diagnosis PrCSllmpti" e ,liagnosis IIIade with ELISA ELlSAlWes1cTrl hlot tesls look for antiho<lies
(sensitive, hi gh false-positive rat e ami low to viral proteins; lh('!)e lesls often arc falsely
threshold, rllie Ollt lest); ® results ,Ire then negali"e inlhe firsl 1-2 month s of l-llV
confirmed with Weste rn bioi <lssay (s pecific, infection ,md falsely positive inili;1l1)' in babies
low false-positive rate ,lIld high threshold, TIlle born to il1fccted IllOlhers (anti-gpI20 crosses
in test). placenta),
,Pad 9 2238 6210 . '

+ G R< UI usmle rx com/apl#Qmax1 + • :


Item ' 27 of 45
QID 2387
L

J.
• Mark -<l
PreYlDu,
t>-
NeKt
all
lab"" ues
~.
Notes
~'t
c .. ku l~tor

· 22 )C A biopsy of a lymph node from a 3-year-old who developed paralytic poliomyelitis following
· 23 )C administration of a live atten uated polio vaccine is performed , revealing nodal architecture
that lacks germinal centers (shown below).
· 24 )C
· 25 )C
· 26 )C
· 27 )C
· 28 )C
· 29 )C
· 30 )C
· 31 )C
· 32 )C
· 33 )C
· 34 )C
· 35 )C
· 36 )C This patient wou ld likely respond to treatment with which of the following therapies?
· 37 )C
134415 zheng
· 38 )C A. Cyclosporine
· 39 )C We value your feedback!
· 40 )C B. Regular intramuscular y-globul in injections
· 41 )C C. Sargramostim

· 22 )C This patient wou ld likely respond to treatment with which of the fo ll owi ng therapies?
· 23 )C 134415 zheng

· 24 )C A. Cyclosporine
We value your feedback!
· 25 )C 8. Regular intramuscular y-globul in injections
· 26 )C
C. Sargramostim
· 27 )C
· 28 )C D. Tacrolimus
· 29 )C
E. Thrombopoieti n
· 30 )C
· 31 )C
· 32 )C
· 33 )C The correct answer is 8. 55% chose this.
· 34 )C Th is patient's histologic section demonstrates a lymph node lacking germinal centers.
· 35 )C Activation of the B-Iymphocyte response occurs in the follicular zone in the outer cortex of the
· 36 )C lymph node. Proliferating B cells form clusters, termed germinal centers, where somat ic
hypermutation and affinity maturation take place. Lack of germinal centers and the cl inical
· 37 )C
scenario of disease foll owing vaccination with live pathogen suggest a B-Iymphocyte
immunodeficiency. Thus, the most helpful treatment wo uld be administration of
immunoglobulins .
StrU(ture of germInal center of lymph node Vaccination Lymphocyte Immunoglobu lins

A is not correct. 11% chose this.


,Pad "" 2238 62 « _ .

+ G R< UI usmle rx c om/ap l#q max1 + III :


Item' 27 of 45
QID 2387 i
L • Mark --<l
Pre Ylous
t:>
Next
_!
la b~ ues
~.
Notes
C';'\I
Cakul~ tor

· 22 )(
A is not correct. 11% chose this.
Cyclosporine is an immunosuppressant that increases susceptibility to infection and is
· 23 )(
contraindicated in this patient.
· 24 )( Cydosporine
' 25 ) (
C is not correct. 19% chose thi s .
. 26 )(
Granulocyte-macrophage colony-stimulating factor (GM-CSF) is used to speed recovery of bone
· 27 )( marrow granulocytes and monocytes.
· 28 )( granulocyte Monocytes Bone Marrow

' 29 )(
D is not correct. 11 % chose th is.
' 30 )( Like cyclosporine, tacrolimus is an immunosuppressive agent and is not the appropriate
· 31 )( treatment for a patient suffering from immunodeficiency.
· 32 )( Cy<losporine Ta(rohmus

· 33 )( E is not corre ct. 4% chose thi s.


' 34 )( Used clinically to increase platelet counts, thrombopoietin would not be a useful treatment
· 35 )( option for this patient.
Finding of pI"tele! (Ount
. 36 )(
· 37 )(

Bottom Line :
B-Iymphocyte deficiency is apparent histologically by a lack of germinal centers in lymph
nodes, and may be treated with immunoglobulin injections.
Structure of germlrOlI center of 1ympf1 node Lymphocyte lymph nodes

FindIng of pI"tele! count

' 25 ) ( Bottom Li ne:


. 26 )(
B-Iymphocyte deficiency is appa rent histologically by a lack of germinal centers in lymph
· 27 )( nodes, and may be treated with immunoglobu lin injections.
· 28 )( Structure of germinal center of 1ympf1 node Lymphocyte lymph nodes

' 29 )(
' 30 )(
· 31 )( U!ifil·1!,J for year: 2015
FlII. S T .0.. 0 FA CTS
· 32 )(
· 33 )( FAI5,p . I961

' 34 )( Lymph node t\ 20 lymphoid organ that 113S man y affcrcnts. I or more effe rent s. Encap>lilalcd. with trabeculae.

--...
Function s arc nonspecific filtrati on by macropl13gcs. storage of B and T ce lls. and immune
· 35 )( rcsponse acti vation .
. 36 )( Fo ll icle Site of B-ecillocalization and
· 37 )( proliferJtion. In outer cortex. [0 follicles
arc dense and dormant. 20 fo llicles haw
p~,[e central germinal centers and arc
active
Medulla Consists of ",ednl!;,,), cords (closely
paded ly"'phoc)'tes .. nd pbs",.. cells)
aud medullary sinuscs. ~-k'(lnllary
sinnscs eomnmnieate with effere nl
,Pad "" 2239 62« _ .

+ G R< UI usmle rx c om/ap l#qmax1 + III :


Item' 28 of 45
QID 4442 i
L • Mark --<l
Pre Ylous
t:>
Next
_!
la b~ ues
~.
Notes
C';'\I
Cakul~ tor

· 22 )( A 25-year-old man presents to his primary care physician for a physical examination before
· 23 )( starting a new job. His family history is significant for a father with "c holesterol issues." For
this reason the patient has maintained a healthy diet and exercises 5 days per week . Physical
· 24 )(
examination reveals a well.developed, well-nourished adult man with xanthomas bilaterally along
' 25 ) ( his Ac hilles tendon . Laboratory studies show a total cholesterol level of 300 mg/dL.
. 26 )(
· 27 )( A mutati on in which protein most likely accounts for this patient's dyslipidemia?
· 28 )(
134415 : zheng
We value your feedback!
' 29 )( A. APKD l
' 30 )(
B. Apolipoprotein A·II
· 31 )(
· 32 )( C. Apolipoprotein (-II
· 33 )(
D. Apolipoprotein E
' 34 )(
· 35 )( E. LDL cholesterol receptor
. 36 )( F. Lipoprotein lipase deficiency
· 37 )(

The correct answer is E. 64% chose this .


This patient most likely ha s familial hypercholeste rolemia, an autosomal dominant
a heterozygote of 1 in 500. This disorder is characterized by abnormal LDL

· 22 )( This patient most likely has familial hypercholesterolemia, an autosomal dominant disease with
· 23 )( a heterozygote frequency of 1 in 500. This disorder is characterized by abnormal LDL cholesterol
· 24 )(
receptors. Seventy-five percent of LDl cholesterol receptors are found within the liver. This
receptor binds to apolipoprotein Band apolipoprotein E, which are found on intermediate-
' 25 ) (
density lipoprotein (lDL) and LDL cholesterol particles . Inability of LDL cholesterol to bind these
. 26 )( proteins results in a higher serum concentration of l DL cholesterol and also a higher rate of
· 27 )( conversion of lDL into LDL. Elevated serum LDL cholesterol levels are found on laboratory tests.
Deposition of LD l cholesterol also occurs widely. Skin deposition results in xanthomas, and
· 28 )(
vascu lar deposition resu lts in atherosclerosis .
' 29 )( Apol ipoprote in B Assay Apolipoproteins E measurement (pr<xedure) Atherosclerosis Hypercholesterolemia. Fam ilial
' 30 )(
A is not correct. 2% chose this.
· 31 )(
AKPD1 mutations are responsible for adult polycystic kidney disease. Patients with this disease
· 32 )(
present with pain, hematuria, hypertension, and progressive kidney failu re. None of these
· 33 )( symptoms is present in this patient.
' 34 )( Polycystic Kidney. Autosomal Oominant Kidney Failure

· 35 )( B is not correct. 5% chose this .


. 36 )( Apolipoprotein A·II is a structural apolipoprotein of HDL and would not cause elevated LDL
· 37 )( levels.
C is not correct. 9% chose this .
Apolipoprotein (-II is necessary for lipoprotein lipase activation. Deficiency of this apoprotein is
tantamount to lipoprotein lipase deficiency and therefore results in elevated serum triglyceride
levels.
Lipase Hyperlipoproleinemia Type I
,Pad "" 2239 62« _ .

+ G R< UI usmle rx c om/ap l#qmax1 + III :


Item' 28 of 45
QID 4442 i
L • Mark --<l
Pre Ylous
t:>
Next
_!
la b~ ues
~.
Notes
C';'\I
Cakul~ tor

Apollpoproteln (-II IS necessary for lipoprotein lipase activation. Deficiency of thiS apoprotem IS
· 22 )(
tantamount to lipoprotein lipase deficiency and therefore results in elevated serum triglyceride
· 23 )( levels.
· 24 )( Lipilse Hype rlipoprQteinemi~ Type I

' 25 ) ( o is not correct. 9% chose this .


. 26 )( Apolipoprotein E is required for receptor-mediated clearance of chylomicrons and VLDL
· 27 )( cholesterol. Abnormal apolipoprotein E results in a syndrome known as
· 28 )( dysbetalipoproteinemia, which is characterized by increased levels of IDL cholesterol, VLDL
cholesterol, and chylomicrons. This manifests on laboratory tests as increased triglyceride and
' 29 )(
LDL cholesterol levels.
' 30 )( Apol ipoprote ins E measurement (procedure) Hyperlipoproteinemia Type III Serum VLDL cholesterol measurement Cholesterol

· 31 )(
F is not correct. 11% chose this.
· 32 )(
VLDl cholesterol, an lDL cholesterol precursor, is triglyceride-rich VLDL cholesterol synthesized
· 33 )( by the liver and secreted into the blood. The enzyme lipoprotein lipase converts VlDL into IDl,
' 34 )( which is also an LDL cholesterol precursor. Deficiency of lipoprotein lipase, as seen in
· 35 )(
hyperchylomicronemia, therefore results in elevated serum triglyceride levels.
Serum Vl.DL cholesterol measurement lipase
. 36 )(
· 37 )(
Bottom line:
Familial hypercholesterolemia is an autosomal dominant disease that is due to abnormal lDl
cholesterol receptors .
Hypercholesterolemia, Familial

Apol ipoprote ins E measurement (procedure) Hyperlipoproteinemia Type III Serum VLDL cholesterol measurement Cholesterol

F Is not correct. 11% chose this.


VLDl cholesterol, an LD L cholesterol precursor, is triglyceride-rich VLDL cholesterol synthesized
' 25 ) (
by the liver and secreted into the blood. The enzyme lipoprotein lipase converts VlDL into IDl,
. 26 )( which is al so an LDL cholesterol precursor. Deficiency of lipoprotein lipase, as seen in
· 27 )( hyperchylomicronemia, therefore results in elevated serum triglyceride levels.
Serum Vl.DL cholesterol measurement Lipase
· 28 )(
' 29 )(
' 30 )( Bottom line:
· 31 )( Familial hypercholesterolemia is an autosomal dominant disease that is due to abnormal lDl
· 32 )( cholesterol receptors.
Hypercholesterolemia, Familial
· 33 )(
' 34 )(
· 35 )(
. 36 )( I ill jfj , .j I.' for year: 2015
FIII.S T AI O FAC TS

· 37 )(
FAIS. p. 115.3
Familial dyslipidemias
n" INCREASED BtOOD LEVEL PATIIOI'~OI()(i'

I- hyper- C h)·lom icro ns. T G . choleslcrol Alitosomal recessive. Lipoprok inlipase


chylomicronemia deficie ncy or altered apolipoprolein C_ II.
,Pad "" 2239 62 « _ .

+ G R< UI usmle rx c om/ap l#qmax1 + III :


Item ' 29 of 45
QI D 4606 i
L • Mark --<l
Pre Ylous
t:>
Next
_!
la b~ ues
~.
Notes
C';'\I
Cakul~ tor

· 22 )( A 55-year-old woman who is a 40·pack-year smoker presents to her physician with shortness IAA[
· 23 )( of breath. Pu lmonary function tests are performed, and in conjunction with the patient's
symptoms, history, and physical examination, a tentative diagnosis of chronic bronchitis is
· 24 )(
made.
' 25 ) (
. 26 )( Which kind of study would be best suited for evaluating the risk for developing chronic obstructive
· 27 )( pulmonary disease in smokers relative to nonsmokers?
· 28 )(
134415 : zheng
We value your feedback!
' 29 )( A. Adoption study
' 30 )(
B. Case·control study
· 31 )(
· 32 )( " C. Cohort study
· 33 )(
D. Twin concordance study
' 34 )(
· 35 )(
. 36 )(
· 37 )( The correct answer is C. 65% chose this.
Cohort studies are prospective: they are used to generate relative risk factors based on whether
a cohort of patients with a particu lar ri sk factor will develop a disease.
A Is not correct. 1% chose this.
Adoption studies are best suited for identifying genetic factors contributing to the
of a disease (heritability) and environmental factors.

The correct answer Is C. 65% chose this.


· 22 )(
Cohort studies are prospective: they are used to generate relative risk factors based on whether
· 23 )(
a cohort of patients with a particu lar risk factor will develop a disease.
· 24 )(
A is not correct. 1% chose this.
' 25 ) (
Adoption studies are best suited for identifying genetic factors contributing to the development
. 26 )( of a disease (heritability) and environmental factors.
· 27 )( B is not correct. 31% chose this.
· 28 )( Case-control studies are retrospect ive: they are used to generate odds ratios by evaluating a
' 29 )( patient population that has a disease to one that does not by evaluating the participant's
history to identify risk factors that would increase one's odds of developing the disease.
' 30 )(
· 31 )(
D is not correct. 3% chose this.
Twin concordance st udies are best suited for identifying genetic factors contributing to the
· 32 )(
development of a disease (heritability).
· 33 )(
' 34 )(
Bottom II ne:
· 35 )(
. 36 )( Cohort studies are used to generate relative risk factors based on whether a cohort of pat ients
with a particular risk factor will develop a disease.
· 37 )(

ul;fii,' .. , for year: 2015


FIJI.ST ""D F"'CTS
,Pad "" 2239 62« _ .

+ G R< UI usmle rx c om/ap l#qmax1 + III :


Item ' 30 of 45
QID 204.2 i
L • Mark --<l
Pre Ylous
t:>
Next
_!
la b~ ues
~.
Notes
C';'\I
Cakul~ tor

· 22 )( A 35-year-old man comes to the emergency department after stepping on a rusty nail in his IAA[
· 23 )( backyard. He has not received any immunizations since he was 12 years old, but he states
that he "had all the regular childhood shots,"
· 24 )(
' 25 ) (
If left untreated, which of the following is the patient most likely to develop?
. 26 )(
134415 ; zheng
· 27 )( [ We value your feedback!
A. Crepitus
· 28 )(
' 29 )( 8. Flaccid paralysis
" 0 )( C. Tenesmus
· 31 )(
· 32 )( D. Trismus
· 33 )( E. Watery diarrhea
" 4 )(
· 35 )(
, '6 )(
· 37 )( The correct answer is D. 56% chose this.
Assuming that this man received an initial course of immunizations aga inst tetanus, he must
receive a tetanus booster shot following this puncture wound (tetanus booster shots are good
for only 10 years). Tetanus is a disease caused by an exotoxin (tetanospasmin) released by
Clostridium tetanl. This toxin causes sustained contraction of skeletal muscles (tetany). When
this occurs in the muscles of the jaw, a cond iti on called trismus (or lockjaw) results.

· 22 )( Assuming that this man received an initial course of immunizations aga inst tetanus, he must
· 23 )(
receive a tetanus booster shot following thi s puncture wound (tetanus booster shots are good
for only 10 years). Tetanus is a disease caused by an exotoxin (tetanospasmin) released by
· 24 )(
Clostridium tetani. This toxin causes sustained contraction of skeletal muscles (tetany). When
' 25 ) ( this occurs in the muscles of the jaw, a cond iti on called trismus (or lockjaw) results .
. 26 )( Tetanus Tetany Trismus PUllCture wOI.Hld Skeletal muscle structure Jaw

· 27 )( A Is not correct. 6% chose this.


· 28 )( Crepitus is a grind ing or crunching noise that can be associated with Clostridium perfringens
' 29 )( infection (also known as "gas gang rene"). The organ ism creates pockets of gas in skin, muscle,
and subcuta neous tissues that can be released when the skin is palpated.
" 0 )(
Subcutaneous crepitus Gas Gangrene
· 31 )(
· 32 )( B Is not correct. 9% chose this.
· 33 )(
Flacci d paralysis is characteristic of infection with Clostridium botulinum. Botulinum toxin
prevents the release of acetylcholine from motor end plates, caus ing a flaccid paralysis.
" 4 )( lnjeuion ofOOlu linum toxin Motor Endplate Flaccid paralysis Acetylchol ine
· 35 )(
C is not correct. 27% chose this.
, '6 )(
Tenesmus is the se nsation of the need to empty the bowel accompanied by pain and cramping,
· 37 )(
although there is little stool to pass. It is usually associated with inflammatory (either infectious
or noninfectious) diseases of the bowel.
E is not correct. 2% chose this.
Pseudomembranous colitis is caused by Clostridium difficile, usually after a patient has been
treated with broad·spectrum antibiotics for several days. This organism causes diarrhea,
abdominal and fever via two exotoxins: toxin A and toxin B. Treatment of this i
,Pad 9 2240 62« . '

+ G R< UI usmle rx com/ap l#qmax1 + III :


Item ' 3 0 of 45
QID 204.2 i
L • Mark --<l
Pre Ylous
t:>
Next
_!
la b~ ues
~.
Notes
C';'\I
Cakul~ tor

· 22 )( E Is not correct. 2% chose this.


· 23 )( Pseudomembranous colitis is caused by Clostridium difficife, usually after a patient has been
treated with broad-spectrum antibiotics for several days. This organism causes diarrhea,
· 24 )(
abdominal cramping, and fever via two exotoxins: toxin A and toxin B. Treatment of this illness
' 25 ) ( includes discontinuation of the original antibiotic and administration of oral vancomycin or ora l
. 26 )( or intravenous metronidazole.
· 27 )( Oiarrheil Metron,d.lzole pseudomembraflO\.l5 colni5

· 28 )(
' 29 )( Bottom Line:
" 0 )( After childhood immunization for tetanus, a booster shot should be administered every 10 ,
· 31 )( years to prevent C. tetani-related trismus and muscle spas ms following trauma.
· 32 )( Tetanus Tr ismus

· 33 )(

" 4 )(
· 35 )( I ill jfj , .j 1.1 for year: 2015
FIRST AID FAC TS
, '6 )(
· 37 )( FAI5, p, 131,1

Clostridia (with
exotoxinsl
(.teton! Produccs tetanospasmin. an exot o~ in ca using Tetanus is tet anic paral)'s i ~.
tetanus. Tetanus to~in (and botulinum 'o~in)
arc prote:ISCS that clcJ\'c SNARE prot cins for
Ilcurotnlll sm ittcTS. Hlocks rckOisc of inhihitor)'

· 22 )( Inhibitor X and inhibitor Y bind an enzyme. Inhibitor X directly binds the same site as the
· 23 )( substrate, while inhibitor Y interacts with the enzyme at a different location.
· 24 )( Inhibitor Inhibitor
Choice X V
' 25 ) (
. 26 )( A
1 ~
· 27 )(
· 28 )(
' 29 )(
• M ....".. M ...."..

" 0 )(
· 31 )(
C Mct>onge
1
· 32 )(
· 33 )( 0 M""""" ~
" 4 )(
· 35 )( E
~ ""ct>onge
, '6 )(
· 37 )( F
~ 1
G ~ ~
,Pad 9 2241 62« . '

+ G R< UI usmle rx com/ap l#qmax1 + III :


Item '31of45 L • Mark --<l t:> _! ~. C';'\I
QID 1272 i Pre Ylous Next la b~ ues Notes Cakul~ tor

· 22 )( How is the maximum velocity of the reaction between the enzyme and substrate affected when
· 23 )( inhibitors X and Y bind the enzyme separately?
· 24 )( 134415 : zheng
We value you r feedback!
' 25 ) ( A
. 26 )(
B
· 27 )(
· 28 )( C
' 29 )( D
' 30 )(
· 31 )(
E
· 32 )( F
· 33 )(
G
' 34 )(
· 35 )(
. 36 )(
· 37 )(
The correct answer is O. 55% chose this.
Inhibitor X represents a competitive inhi bitar, whi le inhibitor Y represents a noncompetitive
inhibitor. The binding of a competitive inhibitor lowers the affinity of the enzyme for the
substrate (increased Michaelis-Menten constant, or Km), but the maximum velocity (Vmax) of the
reaction remains unchanged. Increasing the concentration of substrate can overcome the
effects of a competitive inhibitor. Binding of a noncompetitive inhibitor does not change

· 22 )( Inhibitor X represents a competitive inhi bitor, whi le inhibitor Y represents a noncompetitive


inhibitor. The binding of a competitive inhibitor lowers the affinity of the enzyme for the
· 23 )(
substrate (increased Michaelis-Menten constant, or Km), but the maximum velocity CVmax) of the
· 24 )( reaction remains unchanged. Increasing the concentration of substrate can overcome the
' 25 ) ( effects of a competitive inhibitor. Binding of a noncompetitive inhibitor does not change the
. 26 )( affin ity of the enzyme for the su bstrate (Km is unchanged), but it does decrease the maxim um
velocity. The best answer is that compet itive inhibitor X will effect no change in Vmax and that
· 27 )(
the noncompetitive inhibitor Y wi ll decrease Vmax.
· 28 )(
A is not correct. 6% chose this.
' 29 )( Inhibitors do not increase the speed of the reaction, although a cofactor might. This response is
' 30 )( most appropriate for a cofactor (Xl and a noncompetitive inhibitor (Y).
· 31 )( B Is not correct. 6% chose this.
· 32 )( This answer is most appropriate for two competitive inhibitors.
· 33 )( C is not correct. 8% chose this .
' 34 )( Thi s response is most appropriate for a competitive inhibitor (Xl and a cofactor M.
· 35 )( E is not correct. 14% chose this .
. 36 )( This response would be most appropriate if X were a noncompetitive inhibitor and Y were a
· 37 )( competitive inhibitor.
· 38 )( F is not correct. 5% chose this.
. 39 )( This answer is most appropriate for a noncompetitive inhibitor (X) and a cofact or (Y) .
. 40 )( G is not correct. 6% chose this .
. 41 )( This response is most appropriate for two noncompetitive inhibitors .
A 68 Sis
v 27 KS/s

8
l ock
0
End Block
,Pad 9 2241 62 « . '

+ G R< UI usmle rx com/ap l#qmax1 + III :


Item '31of45 L • Mark --<l t:> _! ~. C';'\I
QID 1272 i Pre Ylous Next la b~ ues Notes Cakul~ tor
- - - -
· 22 )( F is not correct. 5% chose thi s.
· 23 )( This answer is most appropriate for a noncompetitive inh ibitor (X) and a cofactor (V).
· 24 )( G Is n ot correct. 6% chose this.
' 25 ) ( This response is most appropriate for two noncompetitive inhibitors .
. 26 )(
· 27 )( Bottom line:
· 28 )(
The binding of a competitive inhibitor lowers the affinity of the enzyme for the substrate, but
' 29 )( the maximum velocity of the reaction remains unchanged. The binding of a noncompetitive
' 30 )( inhibitor does not change the affinity of the enzyme for the substrate, but the maximum
velocity is decreased.
· 31 )(
· 32 )(
· 33 )(
141 ;fi , ., 1.1 for year: 2015
' 34 )( fI RS T AI D FA CTS

· 35 )(
FA1S, p . 242 .1
. 36 )(
Enzyme kinetics
· 37 )(
Michaelis-Menten IS )" concentration of snbstrate; V" \"(~loc it )'. K", is inversely related to the affinity of the
kinetics en z)"nw for its snbstrate.
V" ..., is dircctly proportional to the enzyme
co nccntrJtioll.
1\'lost cn zymatic rcacti ons follow a h)'l}Crbolic
cn fl'e (i.e .. Michaelis-Menten kin etics);

· 22 )( A 28-year-old man presents to his phys ician complaining of worsen ing arthritis in his left
· 23 )( knee. He has a history of spontaneous bleeding episodes (which have led to his
osteoarthritis) and easy bruising . Previous testing has shown a normal platelet count and
· 24 )(
bleeding time but a prolonged activated partial thromboplastin time.
' 25 ) (
. 26 )( This patient is most likely deficient in which of the following factors?
· 27 )(
134415 : zheng
· 28 )( We value your feedback!
A. Factor VII
' 29 )(
' 30 )( B. Factor VIII
· 31 )( C. Factor X
· 32 )(
D. Factor XI
· 33 )(
' 34 )( E. Von Willebrand factor
· 35 )(
. 36 )(
· 37 )(
Th e correct answer i s B_63% chose th is.
This patient has hemophilia A, an X-linked disorder characterized (in moderate to severe
deficiency) by spontaneous bleeding into deep soft tissues (eg, musc le), easy bruising, and
hemarthrosis into weight-bearing joints (hip, knee, and ankle), due to a deficiency of factor VIII.
Repetitive hemarthroses can lead to osteoarthritis of the affected joint, which increases
of subsequent bleeds into that joint. Laboratory testing in patients with this condition
,Pad 9 2241 62« . '

+ G R< UI usmle rx com/ap l#qmax1 + III :


Item ' 32 of 45
QI D 1393 i
L • Mark --<l
Pre Ylous
t:>
Next la b~
_! ues
~.
Notes
C';'\I
Cakul~ tor
. . .. .
· 22 )( This patient has hemophilia A, an X-linked disorder characterized (in moderate to severe
· 23 )( deficiency) by spontaneous bleeding into deep soft tissues (eg, muscle), easy bru ising, and
hemarthrosis into weight-bearing joints (hip, knee, and ank le), due to a deficiency of factor VIII.
· 24 )(
Repetitive hemarthroses can lead to osteoarthritis of the affected joint, which increases the risk
' 25 ) ( of subsequent bleeds into that joint. Laboratory testing in patients with this condition will show
. 26 )( a prolonged activated partial thromboplastin time; prothrombin time is usually normal. Of note,
· 27 )(
deficiency of factor IX (Christmas disease or hemophilia B) is clinically indistinguishable from
factor VIII deficiency.
· 28 )(
H~marthrosis Prothrombin tim~ assay Fa<tor IX H~mophilia A soft tiSSUE' Ankl~ Actrvat~d Partia l Thrombopastin Ti~ m~asur~~nt

' 29 )(
H~moph i l i a B
' 30 )(
A is not correct. 12% chose this.
· 31 )(
Factor VII deficiency is a rare autosomal recessive disorder. Spontaneous or posttraumatic
· 32 )(
bleeding can occur. Hemarthroses are rare.
· 33 )( F~'tor VII Defiderxy
' 34 )(
C is not correct. 6% chose this.
· 35 )(
Factor X deficiency is a rare autosomal recessive disorder. Spontaneous or posttraumatic
. 36 )( bleeding can occur. Hemarthroses are rare.
· 37 )( F~'tor X Deficiency

D is not correct. 8% chose this.


Factor XI deficiency (hemophilia C) is an autosomal recessive bleeding disorder. Unlike
hemophilia A (factor VIII deficiency) or hemophilia B (factor IX deficiency), hemarthroses are
uncommon, there is less spontaneous bl eed ing, and the correlation between the amount of
factor XI and the clin ica l symptoms is not as well defined. Factor XI deficiency is

Factor XI deficiency (hemophilia C) is an autosomal recessive bleeding disorder. Unlike


· 22 )(
hemophilia A (factor VIII deficiency) or hemophilia B (factor IX deficiency), hemarthroses are
· 23 )( uncommon, there is less spontaneous bl eed ing, and the correlation between the amount of
· 24 )( factor XI activity and the clinical symptoms is not as well defined. Factor XI deficiency is
' 25 ) (
common in Ashkenazi Jews.
Hemophilia A Hemophilia B
' 26 )(
· 27 )( E is not correct. 11% chose this.
· 28 )( Von Willebrand factor (vWF) aids in platelet adhesion. A deficiency of vWF causes prolonged
bleeding time, which manifests as increased bleeding after trauma or surgery, nosebleeds, and
· 29 )(
hematomas. Laboratory tests may show a slightly prolonged activated partial thromboplastin
' 30 )( time (due to vWF's action as a stabifizer of factor VIII) and a norma l prothrombin time. A major
· 31 )( variable distinguishing vWF deficiency from a deficiency in factor VIII is bleeding time, which is
· 32 )(
prolonged with vWF deficiency and normal in factor VIII deficiency.
~on Wlilebrand Disease ProthrombIn ume assay Bleed'ng tIme proc:edur~ Activ~led Partia l Thromboplastin TIme measurement
· 33 )(
' 34 )(
· 35 )( Bottom line:
. 36 )( Hemophilia, a group of X-linked recessive hematologic disorders, exists as two forms:
· 37 )( hemophilia A, or Factor VIII deficiency, and hemophilia B, or Factor IX deficiency. It man ifests
clinically as spontaneous hemarthroses, excessive soft-tissue bruising, and spontaneous
· 38 )(
bleeding.
· 39 )( Hemophlh~ A Factor V1t1 Hemophilia B Factor IX
. 40 )(
' 41 )(
,Pad 9 2241 62« . '

+ G R< UI usmle rx com/ap l#qmax1 + III :


Item ' 33 of 45
QID 1216 i
L • Mark --<l
Pre Ylous
t:>
Next la b~
_! ues
~.
Notes
C';'\I
Cakul~ tor

· 22 )( A 74-year-ol d woman (omes to the clin ic compla ining of fever and a productive cough. She
· 23 )( has been coughing for t he past 3 days, and she is able to produce some sputum. Blood
pressure is 130/90 mm Hg, pulse is 8S/min, temperat ure is 38.3°( (101 °F), and respirations
· 24 )(
are 221m in. There are dullness to perc ussion and bronchia l breath sounds on the lower right side,
' 25 ) ( but the remainder of the physical examination is unimpressive. A stain of her yellow-brown
. 26 )( sputum shows gram-positive diplococci. In light of her history of penici llin allergy, she is started
· 27 )(
on azithromycin.

· 28 )(
What is the mechanism of action for the class of drug the patient received?
' 29 )(
' 30 )( 134415; zheng

· 31 )(
· 32 )( A. Binds to the 30S ribosomal subunit of bacteria
· 33 )(
8. I nhibits bindi ng of aminoacyl-transfer RNA to the A site of the ribosome
' 34 )(
· 35 )( C. I nhibits elongation factor 2
. 36 )(
D. I nhibits eukaryotic initiation factor 2
· 37 )(
E. I nhibits translocation of the transfer RNA

We value your feedback!

· 22 )( Azithromycin and other macrolides bind to the 50S subunit of ribosomes and prevent
trans location of transfer RNA.
· 23 )(
Az ithromycin Ribosomes
· 24 )(
A is not correct. 21% chose this.
' 25 ) (
. 26 )( Aminoglycosides and tetracycline act by binding to the ribosomal 30S subunit, thus preventin g
the binding of aminoacyl-transfer RNA to the A site of the ribosome.
· 27 )(
Tetracycline
· 28 )(
B is not correct. 22% chose this.
' 29 )(
Aminoglycosides and tetracycline act by binding to the ribosomal30S subu nit, thus preventing
' 30 )(
the binding of aminoacyl -transfer RNA to the A site of the ribosome.
· 31 )( Tetracyc l'ne
· 32 )(
C is not correct. 7% chose this.
· 33 )(
Diphtheria toxin inhibits elongati on factor 2.
' 34 )( Diphtheria

· 35 )(
o is not correct. 4% chose this .
. 36 )(
Eukaryotic initiation factor 2 is necessary for translation and therefore protein synthesis.
· 37 )( Anyth ing that inhibits eukaryotic initiation factor 2 will be lethal to the organism.

Bottom Line:
Azithromycin and other macrolides bind to the 50S sub unit of ribosomes and act to prevent
translocation of the transfer RNA.
,Pad 9 2241 62« . '

+ G R< UI usmle rx com/ap l#qmax1 + III :


Item ' 34 of 45
QID 1653 i
L • Mark --<l
Pre Ylous
t:>
Next
_!
la b~ ues
~.
Notes
C';'\I
Cakul~ tor

· 22 )( A baby is born to a 42·year-old mother. The baby has wide-spaced eyes and epicantha l fol ds. IAA[
· 23 )( There is one fewer crease in the palm than expected. Eye examination reveals small wh ite
spots on the peripheral iris, and cytogenetic testing shows an extra autosoma l chromosome.
· 24 )(

' 25 ) (
Whic h of the following is associated with the patient's most likely condition?
. 26 )(
134415 ; zheng
· 27 )( We value your feedback!
A. Acquired cardiac valvular disease
· 28 )(
' 29 )( 8. Chronic lymphocytic leukemia
' 30 )(
C. Duodenal atresia
· 31 )(
· 32 )( D. Pick disease
· 33 )( E. Rocker-bottom feet
' 34 )(
· 35 )(
. 36 )(
· 37 )( The correct answer is C. 64% chose this.
The patient in the question has Down syndrome, or trisomy
21. Duodenal atresia, evidenced by the "do uble bubble"
sign (shown in the image), is associated w ith Down
syndrome.

The patient in the question has Down syndrome, or trisomy


21. Duodenal atresia, evidenced by the "double bubble"
sign (shown in the image), is associated with Down
' 25 ) ( syndrome.
' 26 )(
· 27 )(
· 28 )(
· 29 )(
' 30 )(
· 31 )(
· 32 )(
· 33 )(
' 34 )(
· 35 )(
. 36 )(
· 37 )(
· 38 )(
· 39 )( Image copyright © 2008 Mirza et 01;
. 40 )( licensee BioMed Central Ltd

' 41 )(
,Pad 9 2242 62« . '

+ G R< UI usmle rx com/ap l#qmax1 + III :


Item ' 34 of 45
QID 1653 i
L • Mark --<l
Pre Ylous
t:>
Next
_!
la b~ ues
~.
Notes
C';'\I
Cakul~ tor

· 22 )( A is not correct. 13% chose this.


· 23 )( Congenital cardiac disease is commonly associated with trisomy 21; especially common are
endocardial cushion defects.
· 24 )(
B is not correct. 11% chose this.
' 25 ) (
Down syndrome is associated with acute lymphocytic leukemia, not chronic lymphocytic
. 26 )(
leukemia.
· 27 )(
D is not correct. 5% chose this.
· 28 )(
The brains of patients with Down syndrome have changes similar to those of Alzheimer disease,
' 29 )( not Pick disease, after the age of 35.
' 30 )( E Is not correct. 7% chose this.
· 31 )( Rocker-bottom feet are characteristic of trisomy 18 (Edwards syndrome) and trisomy 13 (Patau
· 32 )( syndrome).
· 33 )(
' 34 )( Bottom li ne:
· 35 )( Down syndrome (trisomy 21) results in infants with wide-spaced eyes and epicanthal folds. Its
. 36 )( incidence is highest in mothers with advanced maternal age. Down syndrome has many
· 37 )( associated comorbidities, including duodenal atresia and endocardial cushion defects.

ul;fi.·,.·1 for year: 2015


Fll1.5T ... ID F... CTS

· 22 )( A 56-year-old woman presents to her primary care physician with a painful cut on her lower IAA[
· 23 )(
leg that she sustained when she scraped it on a rusty car door . On physical examination, her
leg is warm and erythematous, with white purulent material oozing from the wo und site .
· 24 )(
Laboratory evaluation of the purulent material shows gram-positive organisms distributed in
' 25 ) ( clumps. Laboratory studies show methicillin susceptibility .
. 26 )(
· 27 )( Which of the following is the best first-line treatment for this patient?
· 28 )(
134415: zheng
We value your feedback!
' 29 )( A. Dicloxacillin
' 30 )(
B. Gentamicin
· 31 )(
· 32 )( C. Penic illin G
· 33 )(
D. Piperacillin-tazobactam
' 34 )(
· 35 )( E. Vancomycin
. 36 )(
· 37 )(

The correct answer is A. 41% chose this.


Staphylococcus aureus infections can have many different manifestations, including skin
infections, osteomyelitis, abscesses, bacteremia, toxic shock syndrome, and pneumonia. Most
S. aureus strains are susceptible to meth icill in, a penic illi nase-resistant penicillin, and
generation cephalosporins, making these drugs the first-line treatment options. Die:IOJeaCi
,Pad 9 2242 62« . '

+ G R< UI usmle rx com/ap l#qmax1 + III :


Item ' 35 of 45
QID 1706 i
L • Mark --<l
Pre Ylous
t:>
Next
_!
la b~ ues
~.
Notes
C';'\I
Cakul~ tor

· 22 )(
Staphylococcus aureus infections can have many different manifestations, including skin
infections, osteomyelitis, abscesses, bacterem ia, toxic shock syndrome, and pneumonia. Most
· 23 )( S. aureus strains are susceptible to methicill in, a penicillinase-resistant penicillin, and first-
· 24 )( generation cephalosporins, making these drugs the first-line treatment options . Dicloxacillin,
' 25 ) (
methicillin, and nafcillin are common penicillinase-resistant penicillins. Infection with strains of
methicillin-resistant S. aureus wou ld require the use of other drugs such as vancomycin .
. 26 )(
Bacteremia Pneumon ia Didoxac iliin Nakil ',n Skin Diseases. Infe<tious
· 27 )(
· 28 )(
B is not correct. 7% chose this.
Gentamicin is used to treat gram-negative m icrobial infections but is not effective for t he
' 29 )(
treatment of Staphylococcus aUfeus. Gentamicin belongs to the aminoglycoside group of
' 30 )( antibiotics.
· 31 )( C is not correct. 26% chose this.
· 32 )( Penicillin G is an intravenous antibiotic that is bactericidal for gram-positive and some gram-
· 33 )( negative organisms. Because most strains of S. aUfeus have penicillinase enzyme, they are
' 34 )( typically not susceptible to penicil lin.
· 35 )( D is not correct. 14% chose this.
. 36 )( Piperacillin-tazobactam is used to treat Pseudomonas infections, resistant Staphylococcus aUfeus,
and many gram-negative infections. It wou ld not be a first-li ne therapy for Staphylococcus DUfeus
· 37 )(
infection.
Piperacil lH'I Pseudomonas Infections

E is not correct. 12% chose this.


Vancomycin is effective for the treatment of gram-positive organisms, including St"p!IY/CICOCCLIS
DUfeus. It is not indicated, however, unless the strai n is methicillin-res ista nt or the
penicillin all ergy.

· 22 )( Vancomycin is effective for the treatment of gram-positive organisms, including Staphylococcus


· 23 )( aUfeus. It is not indicated, however, unless the strai n is methicillin-resistant or the patient has a
· 24 )( pen icil lin allergy.
' 25 ) (
. 26 )( Bottom Li ne:
· 27 )( Staphylococcus DUfeus is involved in many skin infections, and is a gram -positive organism that
· 28 )( appears in clusters on Gram stain. It contains pen icillinase, and so requires a penici ll inase-
resistant antibiotic such as dicloxacillin for first- line treatment (as long as it is methicillin-
' 29 )(
sens itive).
' 30 )( DicloxaCili in Skin Diseases. Infectious
· 31 )(
· 32 )(
· 33 )( I ill ;fi , .j 1.1 for year: 2015
FI RS T AI D FA C TS
' 34 )(
· 35 )( FAI5.p. 181.2
. 36 )( Didoxacillin, nafcillin, oxacillin (penicillinase·resistant penicillins)
· 37 )( ME(H~ISM Sallie <IS penicillin. N<lrrow Spcc1 rt1l1l ;
pellieillillase res iSlanl hec allse hlliky I~ group
blocks ,"ccess of ~- 1 ;1C1 ;lIn;lse 10 ~-bct;1!I1 rillg
(lINICAlUS( S. aurCIlS (e~cept .\ 1RSt\: resist;,," because of "Use n ilf (n"fcil1i n) for d ;lph:'
alte red pellicillin_bindillg protein larget site).
TOII( ITY I-Iyperse nsiti\'ity reactions. interstitial neph riti s.
,Pad 9 2248 61 0 . '

+ G R< UI usmle rx com/ap l#Qmax1 + • :


Item" 36 of 45
QID 2076
L

J.
• Mark -<l
PreYlDu,
t>-
NeKt
all
lab"" ues
~.
Notes
~'t
c .. ku l~tor

· 22 •
· 23 •
A 2·year-old girl presents to the emergency room because of the acute onset of fevers to
400 ( (104°F). On physical exam ination, she refuses to walk and does not move very much. In
IAAI
' 24 . addition a purpuric rash is noted on her legs. Blood for cultures is drawn and empiric
' 25 •
anti biotic treatment is begun. Twenty-four hours later, a Gram stai n of the culture growth is
performed with the results shown in the i mage .
. 26 . ,.
· 27 •
· 28 •
r
' 29 .
' 30 . •
· 31 •
·32 •
· 33 •

' 34 .
· 35 •
. 36 . •

..
· 37 •

'38 . Image courresy of CDC / Or. Brodsky


. 39 •
. 40 . This ch ild's infection is associated with a deficiency of which of the followi ng complement
' 4' •
molecules?

This chi ld's infection is associated wit h a deficiency of which of the following complement
· 23 • molecules?
' 24 . 134415: zheng
We value your feedback!
' 25 • A. (1 est erase
. 26 . B. (2
· 27 •
.28 . C. (4

' 29 . D. (5· (9
. 30 .
E. Decay-accelerating factor
· 31 •
·32 •
· 33 •

' 34 .
The correct answer is D. 66% chose this.
· 35 •
Th is acutely ill child present s with signs and symptoms suggestive of meningococcem ia given
.36 . her acute, pyrexia, petech ial rash, and blood cult ure findings. In addition, the bacteria shown in
· 37 • the image are gram-negative diplococci, co nsistent wit h Neisseria meningitidis. Patients with
'38 . men ingococcemia are less likely to exhibit classic signs of mening it is. A deficiency in
complement molecules (5-(9 can lead specifica lly to Neisseria bact eremia. Proteins (5-(9 are
. 39 .
part of the membrane attack complex and are involved in cytolysis.
. 40 . Men ingoco«emi<l Meninglti5 a<lcteremi<l Blood cu lture

'4' . A is not correct. 13% chose this.


A deficiency in ( 1 esterase inhibitor is one of the causes of hereditary ang ioedema.
,Pad "" 2248 61 « _ .

+ G R< UI usmle rx com/ap l#qmax1 + III :


Item ' 36 of 45 Mark --<l t:> _! ~. C';'\I
.. . .
L •

i la b~ Cakul~ tor
QID 2076

· 22 •
. Pre Ylous Next
-
ues Notes

A deficiency in (1 esterase inhibitor is one of the causes of hereditary angioedema.


· 23 •
Serum (1 onh,bitor antigen measurement Angioedemas, HeredItary
· 24 .
· 25 •
B is not correct. 7% chose this.
(2 deficiency does not lead to any specific disease, alth ough as part of the classic pathway
.26 . (a long with (2, (3, and (4), it will decrease the ability of the immune system to neutralize
· 27 • viruses .
. 28 . Immune system

· 29 . C is not correct. 8% chose this .


.'0 . (4 deficiency does not lead to any specific disease, alth ough as part of the classic pathway
· 31 • (a long with (2, (3, and (1), it will decrease the ability of the immune system to neutralize
·32 • viruses.
Immune system
· 33 •

.'4 . E is not correct. 6% chose this .


· 35 • A deficiency in decay-accelerating factor leads to paroxysmal nocturnal hemoglobinuria. The
norma l role of this molecu le, along with membrane inhibitor of reactive lysis (MIRL), is to
.'6 . prevent complement from attacking and lysing the RBCs. Its lack therefore leads to red blood
· 37 • cell lysis and hematuria.
·'8 . P<lroxysmal nocturnal hemogloblflUri<l Lysis

.'9 •
. 40 . Bottom Line:
·4' . Cocmo,le,ne," omte;,,, (5-C9 are components of the membrane attack complex involved
delrocj,en.:y of these proteins leaves the patient particularly susceptible to i

· 23 •
Paroxysmal nocturnal hemoglobinuria Lysis
· 24 .
· 25 •
.26 . Bottom Line :
· 27 • Complement proteins (5-C9 are components of the membrane attack complex involved in
.28 . cytolysis. A deficiency of these proteins leaves the patient particularly susceptible to infection
with gram-negative bacteria such as Neisseria.
· 29 •

.'0 .
· 31 •
·32 •
14l;f." "", for year: 2015
fiRST ""0 f ... cts

· 33 •

.'4 . Complement disorders


FA1S, p . 206 .2

· 35 •
C1 esterase inhibitor C;I\lses herelli larr ;lIIgioedcllla. AC E inhihi!ors are contr;.indic;.tcd
.'6 . deficiency

· 37 • C3 deficiency In creascs ri s!: of sel'ere, recurrent pyogcnic si nu s and respirator y Irdcl infcelions: I Sl1sceplibilil ), 10
type III hypersensili,·ity reactions
·'8 . CS - C9 deficiencies Te rminal compl ement deficie nc)' i"CTeascs slIsceptibilit y to recurren! Neisseria hacteremia.
.'9 . OAF (GPI·anchored C;lllses complement-medi;llt'<:llysis of RBCs;lIId p,aro~)'smallloctnrn~1 helll ogiohinuria
.40 . enzyme) deficiency

·4' .
,Pad "" 2248 61 « _ .

+ G R< UI usmle rx com/ap l#qmax1 + III :


Item ' 37 of 45
QI D 4025 i
L • Mark --<l
Pre Ylous
t:>
Next la b~
_! ues
~.
Notes
C';'\I
Cakul~ tor

· 22 •
A 3·year-old child is brought to his pediatrician by his mother, who states that he has had
· 23 • watery diarrhea and abdominal pain for 36 hours. The patient attends daycare, and recently
· 24 . many children have had simi lar symptoms. Physical exam is negative except that the child
· 25 •
appears dehydrated with increased capillary refill time. The child has vitals within norma l limits
except for mild hypotension and tachycardia .
. 26 .
· 27 •
Which of the following best describes the most likely pathogen in this case?
.28 .
134415: zheng
· 29 .
A. A double-stranded, linear-segmented RNA virus
.'0 .
· 31 • B. A retrovirus
·32 •
C. A single-stranded enveloped virus with negative polarity
· 33 •

.'4 . D. A single-stranded, enveloped RNA virus with positive polarity


· 35 •
.'6 . We value your feedback!
· 37 •

·'8 •
. '9 •
. 40 .
·4' .

· 23 •
Rotavirus is the number one cause of viral gastroenteritis and fatal diarrhea in children. It is part
of the Reovirus family and is a nonenveloped, double-stranded, linear-segmented RNA virus.
· 24 . rotaviws vaCC ine. tetravalent. live. ora l EnteritiS due to spec ified virus
· 25 •
B is not correct. 9% chose this .
. 26 .
Retroviruses are sing le-stranded, enveloped RNA viruses with pos it ive polarity. They have
· 27 •
reverse transcriptase and include HIV (which causes AIDS) and human T-Iymphocyte lymphoma
.28 . virus (which causes T-Iymphocyte leukemia).
· 29 . lymphoma leukemia T·lymphocyte

.'0 . C is not corre ct. 15% chose this.


· 31 • Rhabdovi ruses are single-stranded enveloped vi ruses with negative polarity. They resu lt in
· 32 • rabies.
R<lbies (disorder)
· 33 •

.'4 . o is not correct. 21 % chose th is.


· 35 • Togaviruses are single-stranded, enveloped RNA viruses with positive polarity. They are
.'6 . responsible for rubella (German measles), Eastern equine encephaliti s, and Western equine
encephalitis.
· 37 •
Rubel la Measles Erxepmlomyelitis. Eastern Equine Erxeph<llomye lltis. Western Equine
·'8 •
.'9 .
.40 . Bottom Line:

·4' . Rotavirus, a double-stranded, linear-segmented RNA virus, is the most prevalent cause of viral
gastroenteritis in children.
,Pad "" 2248 61 « _ .

+ G R< UI usmle rx com/ap l#qmax1 + III :


Item ' 37 of 45
QI D 4025 i
L • Mark --<l
Pre Ylous
t:>
Next
_!
la b~ ues
~.
Notes
C';'\I
Cakul~ tor

· 22 •
· 23 • Bottom line:
· 24 . Rotavirus, a double-stranded, linear-segmented RNA vi rus, is the most prevalent cause of viral
· 25 • gastroenteritis in chil dren.
rotavirus vaccine, (etravalen~ live, oral Enteritis due to specif,ed virus
. 26 .
· 27 •
. 28 .
· 29 . I il1ifi','!,) for year: 2015
FIRS T A I D FAC TS

.'0 . FA15, p. 161.1


· 31 •
RNA vi ruses
·32 •
VIUlfAMllV (I!\I(lOff RHASIRlKlURl (AP'lIOSYMM(l1n' MEOI(AlIMPOIITAN(E
· 33 • Reoviruses No OS linear [cos~he{lral Colti l'irus' - Colomdo tick lever
.'4 . Pic:ornaviruses N,
10- 12 scgmCllts
S5 €£I linear
(douhle)
kosahl><lr..tI
Rota" irns-#l c allse of fal:11 diarrhea ill children
Pulio\'iTl1s-pulio-Sal klS'lbin \';lCC illCS- JI'Vropv
· 35 • Echo\'i rn ~- ~ scptic mcningitis

.'6 . Rhi no" irns- "conllllOll cold"


C oxsackicviTlls- ascptic lIleningitis; hcrpangina
· 37 • (month blisters, fever); han{1. foot. a nd month
·'8 . dise;LSC; lll)ocarditis; pericarditis
II i\V- acnlc ";T'JI hcp.~t ;tis
.'9 . PE RC II
.40 . HepevirU5 No 5S (£I lillcar lcosahedml HEV

·4' . Cilliciviruses N, 55 ® lincar Icns:.h~'{lr:LI Noro\·irus- "iral gastmellk ritis

· 22 •

· 23 •
A 45-year-old woman presents to her primary care phys ician with polydipsia and polyuria.
Laboratory tests reveal that her plasma ADH level is elevated. Her urine is colorless, and her
gAl
. 24 . urine osmolality is 78 mOsm/kg. After a detailed work-up, her physician believes that the
. 25 . patient's symptoms are caused by a medication she has been prescribed .
· 26 .
From which of the following conditi ons does the pati ent most likely suffer?
· 27 •
. 28 . 134415: zheng
We value your feedback!
. 29 . A. Bipolar disorder

·'0 . B. Graves disease


· 31 •
C. Hashimoto disease
· 32 •
.'3 . D. Major depressive disorder
·'4. E. Small cell ca rcinoma of the lung
·'5 .
·'6 .
· 37 •
.'8 . The correct answer is A. 64% chose this.
·'9 . The patient described in the Vignette is suffering from diabetes insipidus (01). The production of
dilute (low osmolarity) urine despite elevated levels of ADH indicates that the cause is
nephrogenic. Lith ium is commonly used in the treatment of bipolar disorder, a mood disorder
characterized by alternat ing periods of mania and depression. Lithium has a number of adverse
effects, including tremor, hypothyroidism, and nephrogenic diabetes insipidus.
umium Central Diabetes Insipidus B.polar Disorder Nephrogenic Diabetes Insipidus
--------------------------------------------------------------------
Item. 38 of 45
QIO 4127 i
~ • Mark <J
Previous
t:>
Nexl
-I
l~ b~ ue s
~. ,
Note s
f:o;:t
C" I'u l~tor

· 22 • The patient described in the vignette is suffering from diabetes insipidus (OI). The production of
dilute (low osmolarity) urine despite elevated levels of ADH indicates that the cause is
· 23 •
nephrogenic. Lithium is commonly used in the treatment of bipolar disorder, a mood disorder
.24 . characterized by alternating periods of mania and depression. Lithium has a number of adverse
· 25 • effects, including tremor, hypothyroidism, and nephrogenic diabetes insipidus.
· 2• • LJtnium Central Diabetes Insipidus Bipolar Disorder Nephrogenic Diabetes Insipidus

· 27 • B Is not correct. 8% chose this .


. 28 . Graves disease is an autoimmune disease in wh ich activating antibodies are generated against

.
.29 . the thyroid-stimulating hormone receptor, leading to hyperthyroidism. Graves disease is treated
with drugs such as propylthiouracil and methimazole, which decrease thyroid hormone
·'0 . synthesis and decrease the peripheral conversion of thyroxine to triiodothyron ine.
· 31 • HyperthyroIdism Propylthiouracil Methimazole Graves DlseClse Thyro~ine Triiodothyronine

."

.
C is not correct. 6% chose this .
. 33 .
Hashimoto thyroiditis is an autoimmune disease that causes hypothyroidism. Therapy for this
·'4 . condition is the administration of synthetic thyroid hormone. Diabetes insipidus is not an
· 35 • adverse effect of thyroid hormone admin istration .
., Cemral Diabetes Insipidus

· 37 • D is not correct. 9% chose this .


. 38 . Depression is treated with a number of medication classes including selective serotonin
·'9 . reuptake inhibitors, monoamine oxidase inhibitors, tricyclic agents, and others. Although many
of these agents have significant adverse effects, none result in nephrogenic diabetes ins ipidus.
Importantly, lithium is rarely used in unipolar depression and is far more frequently prescribed
as a mood sta bilizer in bipolar disorder.
litl1ium Nepllrogenic Diabetes Insipidus Bipolar Disorder

E Is not correct. 13% chose this.

· 22 • litl1ium Nepllrogenic Diabetes Insipidus Bipolar Disorder

· 23 • E is not correct. 13% chose this .


. 24 . Small cell ("oat cell ") lung carcinoma (SCLC) is a neoplasm of the lung that is strongly associated
· 25 • with smoking. This tumor is notable for its ability to produce hormones including ADH and
· 2• • ACTH . Neither, however, would result in the nephrogenic diabetes insipidus (01) described in the
stem. Furthermore, therapies used to treat SCLC are not associated with the development of
· 27 •
nephrogenic Dr.
. 28 . oat al lergenic el<lract Carcinoma of lung lung Neopiasms NephrogenIC Diabetes InSIpidus

.
.29 .
·'0 . Bottom line:
· 31 •

." Lithium is used to treat bipolar disorder, and it has a number of adverse effects, including

.
tremor, hypothyroidism, birth defects, polyuria, and nephrogenic diabetes insipidus due to
· 33 • ADH antagonism.
·'4 . lIthium Polyuria Bipolar Disorder Nephrogenic Diabetes Insipidus

· 35 •
.,
· 37 • hli1'.,1,' .. , for year: 2015
fiRST AID FAC TS
. 38 .
·'9 . FA1S.p.S22.1
LithIum
MICHAHISM Nol established: po$5ibly related to inhibition of L" INO P- Lithium si de effects:
phosphoinositol cascad e. " IOI'cmcllt (Ircmor)
(UHlCAtUS, 'Id ood st;lhilil'.er for hipolar disorder; bl oc!..s Nephrogenic diabetes insipidus
relapse and ;lc lIte Illanic el·c nts. Also SIAOH H)'pO thyroidi sm
Pregnane)· problems
h
---------------------------------------------------------------
Item. 39 of 45
QID 1!)4l i
~ • Mark <J
Previous
t:>
Nexl
-I
l~b~ ues
~. ,
Note s
f:o;:t
C"I'ul~tor

· 22 •

· 23 •
A 54-year-old man with a history of hypertension. congestive heart failure, gastroesophageal
reflux disease, and alcoholism presents to his primary care physician complaining that he is
gAl
.24 . "growing breasts," Physical examination reveals palpable mammary tissue bilaterally under
· 25 •
his nipples.
· 2• •
Which of the following medications could be the cause of this patient's symptoms?
· 27 •
. 28 . 134415 : zheng
We va lue your feedback!

.
.29 . A. Cimetidine

·'0 . B. Furosemide
· 31 •

." C. Hydrochlorothiazide

.
. 33 .

·'4 .
· 35 •
.,
· 37 •
. 38 .
D. Metoprolol

E. Ondansetron

The correct answer is A. 67% chose this.


·'9 . Gynecomastia is caused by an imbalance in the ratio of estrogen to testosterone in the body.
Medications that decrease the amount of testosterone, increase the amount of estrogen, or
decrease the sensitivity of breast tissue to androgens can result in gynecomastia. The H2
receptor antagoni sts cimetidine and, to a lesser extent, ranitidine can cause gynecomastia by
inhibiting the cytochrome P450 system, therefore slowing the breakdown of serum estradiol. It
is also important to rule out other potential causes of gynecomastia such as chronic I
cirrhosis, or other medications. Cardiovascular drugs such as spironolactone, algll,a""

· 22 • Gynecomastia is caused by an imbalance in the ratio of estrogen to testosterone in the body.


· 23 • Medications that decrease the amount of testosterone, increase the amount of estrogen, or
.24 . decrease the sens itivity of breast tissue to androgens can result in gynecomastia. The H2
receptor antagonists cimetidine and, to a lesser extent, ranitidine can cause gynecomastia by
· 25 • inhibiting the cytochrome P450 system, therefore slowing the breakdown of serum estradiol. It
· 2• • is also important to rule out other potential causes of gynecomastia such as chronic alcoholism,
· 27 • cirrhosis, or other medications. Cardiovascular drugs such as spironolactone, digitalis, calcium
channel blockers, and angiotensin·converting enzyme inhibitors have been associated with
. 28 .
gynecomastia. Omeprazole, a proton pump inhibitor used to treat gastroesophageal reflux, has

.
.29 . also been associated with gynecomastia .
·'0 . Gynecomastia Cimetidine Ranitidine Spiro~actone DigItalis preparation Omepralole Testosterone areast Alcoholic Intoxication. Chronic

· 31 • Calcium Gastroesophageal reflux disease

."

.
B Is not correct. 8% chose this.
· 33 • Furosemide is a loop diuretic used to treat hypertension. It acts on the Na+/K+I2Ci- transporter
·'4 . in the thick ascending limb of the loop of Henle. Whi le furosemide is known to cause
· 35 • hypoka lemia, hyperuricemia, hypocalcemia, ototoxicity, and nephrotoxicity, it does not cause
., gynecomastia .
HypokalemIa Hyperuricemia Hypoca lcemIa Ototoxicity Gynecomastia
· 37 •
. 38 .
C is not correct. 8% chose this .
Hydrochlorothiazide is a common diureti c used to treat hypertension. It acts on the
·'9 . sodium/ch loride cotransporter in the distal tubule. Whi le it can cause hypercalcemia,
hyperglycemia, hyperlipidemia, and hyperuricemia, it does not cause gynecomastia.
Hydrochlorothiazide Hypercalcemia Hypergly<emia Hyperlipidemia HyperuricemIa Gynecomastia Sod ium

D is not correct. 5% chose this.


Metoprolol is a ~·b l ocker used to treat hypertension, angina, arrhythmia, and acute mllocarc
infarction. While it can cause depression, impotence, or exacerbation of congestive
---------------------------------------------------------------
Item. 39 of 45 ~ • Ma rk <J t:> -I ~. , f:o;:t
QID 1!)4l

· 22 •
i
-y. .
Previous Nexl
. l~b~ ues
,
Note s C"I'ul~tor
yp
sodium/ch lorid e cotransporter in the distal tubu le. While it can ca use hypercalcemia,
· 23 • hyperglycem ia, hyperlipidemia, and hyperuricemia, it does not cause gynecomastia .
. 24 . Hydrochlorothi<lzide Hyper'<llcemia Hypergly<;emiil Hyperlipidemi<l Hyperuricemiil Gynecoffiilsti" Sodium

· 25 •
o is not correct. 5% chose this.
· 2• • Metoprolol is a ~-b l ocker used to treat hypertension, angina, arrhythmia, and acute myocardial
· 27 • infarction. Wh ile it can cause depression, impotence, or exacerbation of congestive heart
. 28 . failure, it is not a cause of gynecomastia .

.
.29. Metoprolol Gynecom(lsti(l Acute myoc.. rdiill inf(l'ction Congestive heilrt fililure

·'0 . E is not correct. 12% chose this.


· 31 • Ondansetron is a 5-hydroxytryptamine-3 antagonist and is a centrally acting antiemetic used for

." controlling vomiting in postoperative patients. While it is known to cause headache,

.
constipation, and (rarely) ECG changes, it does not cause gynecomastia .
. 33 . He~d"che Gynecom~sti" Vomiting

·'4 .
· 35 •
., Bottom Line:

· 37 • Know the common causes of gynecomastia: Spironolactone, marijuana [Dope], Di gitalis,


Estrogen, Cimetidine, chronic Alcohol use, Heroin, Dopamine D2 antagonists, and
. 38 .
Ketoconazole (remember the mnemonic: Some Dope Dru gs Easily Create Awkward Hai ry DO
·'9. Knockers).
Gynecomasti" MMijuana Abuse

iii I if') , .j I.) for year: 2015


fllI.ST AIO FACTS

· 22 •

· 23 •
A 15-year-old boy was recently diagnosed with pneumonia. He has had multiple episodes of
pneumonia in the past, and a CT scan of his chest reveals prominent bronch iectasis .
gAl
. 24 . Add iti onally, he has had trouble maintaining weight, as he has had intermittent diarrhea for
· 25 •
most of his life.
· 2• •
Which of the following antim icrobials is useful in the treatment of this patient's pneumonia?
· 27 •
. 28 . 13441 5 : zhe ng
We value your feedback!

.
.29. A. Ampicillin

·'0 . B. Erythromycin
· 31 •

." C. Gentamicin

.
· 33 •

·'4 .
· 35 •
.,
· 37 •
. 38 .
D. Metronidazole

E. Rifampin

The correct answer is C. 41% chose this.


·'9. The key to this question is that the patient has cystiC fibrosis «(F). For patients with CF,
Pseudomonas is a very common cause of pneumonia. Gentamicin is from the antim icrobial
family of aminog lycosides, which are bactericidal and act by inhibiting the formation of the
initiation complex and causing misreading of mRNA. They require oxygen for uptake and are
therefore ineffective aga inst anaerobes. They are used to treat severe gram-negative rod
infections, including Pseudomonas, usually in combination with other antibiotics. Toxi i
include nephrotoxicity, ototoxicity, and teratogenicity.
---------------------------------------------------------------
Item. 40 of 45
QID 4005 i
~ • Mark <J
Previous
t:>
Nexl
-I
l~ b~ ue s
~. ,
Note s
f:o;:t
C" I'u l~tor

· 22 • The key to this question is that the patient has cystic fibrosis (CF). For patients with CF,
Pseudomonas is a very common cause of pneumonia. Gentamicin is from the antimicrobial
· 23 •
fam ily of aminog lycosides, which are bactericidal and act by inhibiting the formation of the
.24 . initiation complex and causing misread ing of mRNA. They require oxygen for uptake and are
· 25 • therefore ineffective aga inst anaerobes . They are used to treat severe gram-negative rod
· 2• • infections, including Pseudomonas, usually in combination with other antibiotics. Toxicities
include nephrotoxicity, ototoxicity, and teratogenicity.
· 27 •
Pneumonia Oxygen OtotoxIC ity
.28 .
A is not correct. 17% chose this .

.
. 29.
Ampicillin is an aminopenicillin antimicrobial with a wider spectrum than penicillin. It is
· 30 . pen icillinase-sensitive but is often comb in ed with clavulaniC acid to enhance its spectrum. It is
· 31 • used to treat certain gram-positive and gram -negative infections, including Haemophilus
." in!fuenzae, Escherichia coli, Listeria, Proteus, Salmonella, enterococci, and Shigella, but is not
.33. effective against Pseudomonas. Toxicities include hypersensitivity reactions and
pseudomembranous colitis.
· 34 . Ampicillin Clavulanic Acid Shige lla Infections HypersensItivity pseudomembranous colitis
· 35 •
B is not correct. 27% chose this.
· 3• •
Erythromycin is a macrolide antimicrobial that acts by blocking the 23S-rRNA of the 50S
· 37 •
ribosomal subunit to inhibit translocation. It is used clinically to treat upper respiratory
.38 . infections, pneumonias, sexua ll y transmitted diseases, Mycoplasma, Leglonella, and gram-
· 39 . positive cocci in patients allergic to penicillins. It is not effective against Pseudomonas.
Erythromycin Mycoplasma Infections

D is not correct. 10% chose this.


Metronidazole is an antimicrobial and anti-protozoan that acts by stimu lating the formation of
toxic metabolites in the bacterial cell wall. It is used to treat infections caused by Giardia,
Entamoeba, Trichomonas, Gardnerella vaginaJis, and anaerobes (Bacteroides). It is also used
of the i to treat Helicobacter It is not effective

D Is not correct. 10%


· 23 • Metronidazole is an antimicrobial and anti-protozoan that acts by stimu lating the formation of
.24 . toxic metabolites in the bacterial cell wall. It is used to treat infections caused by Giardia,
Entamoeba, Trichomonas, Gardnerella vaglnal/s, and anaerobes (Bacteroides). It is also used as part
.25. of the triple therapy regimen to treat HeJ/cobacter pylori. It is not effective against Pseudomonas.
· 2• • MetronIdazole Trichomon.1s Infections

· 27 •
E is not correct. 5% chose this.
·28 . Rifampin is an antimicrobial that acts by inhibiting DNA-dependent RNA polymerase. It is used
.29 .

.
to treat Mycobacterium tuberculosis and also as meningococcal prophylaxis and
· 30 . chemoprophylaxis in contacts of children with Haemophllus In/luenzae type B. Rifampin al so
delays resistance to dapsone when used for leprosy. It is not effective against Pseudomonas.
· 31 •
Toxicities include minor hepatotoxicity and drug interactions as a result of stimulating the
." cytoch rome P450 isoenzyme system.
· 33 • lUfampln Tubercu losis Dapsone leprosy HepatotoxicIty Drug InteractIOns

. 34 .
· 35. Bottom line:
· 3• •
Patients with CF are prone to pneumonia infections with Pseudomonas, which are best treated
· 37 • with gentamicin .
. 38 . PneumonIa

· 39 .
· 40 .
141;£0,,·,..1
FHUT "10 F"CTS
for year: 2015
---------------------------------------------------------------
Item. 41 of 45
QID 4126 i
~ • Mark <J
Previous
t:>
Nexl
-I
l~ b~ ue s
~. ,
Note s
f:o;:t
C" I'u l~tor

· 23 • A 26-year-old man is undergoing antimicrobia l treatment for mu ltidrug-resistant


· 24 . Staphylococcus aureU5 endocarditis. Shortly after the antibiotic is injected, the patient
becomes severely flushed around the face and neck.
· 25 •
. 26 .
How could this complication have been avoided?
· 27 •

· 28 . 134415 zheng
A. Desensitization therapy
·29 .
We va lue your feedback!
. 30 . B. Oral administration rather than intravenous
· 31 •
C. Pretreatment with high-dose systemic steroids
· 32 •
· 33 • D. Pretreatment with ipratroprium
.34 . E. Slower infusion of the drug
.35 .
· 36 .
· 37 •

·38 . The correct answer is E. 58% chose this •


. 39 . This patient has "red man" syndrome, which is assoc iated with administration of vancomycin
.40 . due to a secondary release of histam ine. This adverse effect can be avoided by a slow 1- to 2-
·4' . hour infusion of vancomycin and pretreatment with antih istamines.
. 42 . A is not correct. 12% chose this .
.43 . Desensitization therapy plays an important role in the administration of essentia l drugs to
patients with severe allergies, but red man syndrome is not a severe allergy requ iring
· 44 •
desensitization treatment. Rather, it can be prevented by slow infusion and pretreatment

i
due to a secondary release histam ine. Thi s adverse effect can be avoided by a slow 1- to 2-
· 24 . hour infusion of vancomycin and pretreatment with antih istamines .
. 25 . HistamIne

.26 . A is not correct. 12% chose this.


· 27 • Desensitization therapy plays an important role in the administration of essentia l drugs to
· 28 . patients with severe allergies, but red man syndrome is not a severe allergy requiring
·29 . desensitization treatment. Rather, it can be prevented by slow infusion and pretreatment with
antih istamines .
. 30 .
Allergen [mmuootner<lpy
· 31 •
· 32 •
B is not correct. 7% chose this.
Oral vancomycin would not be effective i n treating S. aUfeus endocarditis. Oral vancomycin is
· 33 •
used to treat Clostridium difficile colitis .
. 34 . Clostrid ium dlfficile colitis
.35 . C is not correct. 15% chose this.
· 36 .
"Red man" syndrome can be avoided by slow infusion and administration of antihistamines.
· 37 • High-dose stero ids are used after anaphylactic drug reactions.
·38 . Advf!r'Se reaction to drug

. 39 . D is not correct. 8% chose this .


. 40 . Anticholinergic drugs such as ipratroprium maintain airway tone in patients with chronic
·4' . obstructive pulmonary disease, and play no role in preventing red man syndrome .
. 42 . Chronic Obstructive Airway Disease

.43 .
· 44 •
--------------------------------------------------------------------
Item. 41 of 45
QID 4126 i
~ • Mark <J
Previous
t:>
Nexl
-I
l~ b~ ue s
~. ,
Note s
f:o;:t
C" I'u l~tor

High-dose steroids are used after anaphylactic drug reactions.


· 23 •
Advi!rse reaction to drug
· 24 .
· 25 •
D is not correct. 8% chose this.
.26 . Anticholinergic drugs such as ipratroprium maintain airway tone in patients with chronic
obstructive pulmonary disease, and play no role in preventing red man syndrome.
· 27 •
Chronic ObstructM! Airway DiseaSe
· 28 .
·29 .
Bottom Line:
.30 .
· 31 • Red-man syndrome is a nonallergic release of histamine after vancomycin administration; it
can be prevented by slow infusion and pretreatment with antihistamines.
· 32 •
Hist<lmine
· 33 •
. 34 •
. 35 .
ul;f.,.·,.·1 for year: 2015
· 36 . fiRST ""0 FACts

· 37 •
FA15, p . 183.3
· 38 . V,lIncomydn
. 39 . MWWIISM Inhibils cell wall peptidoglrc'l1l formation b)' binding D-~la O-"b portion of cell wa ll preC Ur$()r$
. 40 . Ibctericidal. Not susce ptible to ~-1actama ses .
Gram-positive bugs only-se rious. nHllhdrllg-res istant organisms. including MRSA. S, epidermidis.
·4' . (lIN I(AlUS<
sc nsiti,'c EnlerOCCOCIlS specics. ,md Clostridium di({icile (oral dose for pscudomcmbranous colitis).
. 42 . IOII(IT~ Wel l tol erated in general- but NOT trouble free. Nephrotox icit y. O totox icity, T hrombophlebitis,
. 43 . diffu se flushing- red man syndrome {can largely prc\'c nt by prctrcatm... nt \\'ith ant ihistamincs
and slo\\' infusion rat ...}.
· 44 •
',\(HAlIISIoIOfmIIIAllH "Pay back 2 O ·;ll as

· 23 •

· 24 .
A 67-year-old man visits his physician to request a drug to prevent the flu because he has an
egg allergy and cannot receive influenza vaccine. The patient states that he was previou sly
gAl
• 25 •
prescribed amantadine when he was exposed to influenza a few years ago .
. 26 .
Amantadine may act as prophylaxis against influenza infection through which of the f ollowing
· 27 •
mechanisms?
· 28 .
134415 : zheng
·29 . A. It blocks late protein synthesis and processing
.30 . We value your feedback!
· 31 • B. It blocks viral adsorption to the cell
· 32 •
C. It blocks viral packaging and assembly
· 33 •
. 34 • D. It prevents early protein synthesis
. 35 . E. It prevents vira l uncoating
· 36 .
· 37 •
· 38 •
. 39 . The correct answer is E. 52% chose this .
. 40 . Amantadine binds to the M2 protein on the influenza A virus, thus preventing viral uncoating.
·4' . Amantadine has been used in the treatment and prophylaxis of influenza A infection. However,
90% of all influenza A strains now are resistant to amantadine, so it is no longer typically used
. 42 . for these purposes in the United States. Amantadine is also used to treat Parkinson's disease .
. 43 . Its mechanism of action in Parkinson 's disease is not well understood, but it is thought that it
· 44 • increases synthesis and release of dopamine from the substantia nigra.
Amant.ld,f'.e Influenza Entire subst.lmia nigra
---------------------------------------------------------------
Item. 42 of 45
QID 1!J16 i
~ • Mark <J
Previous
t:>
Nexl
-I
l~b~ ues
~. ,
Note s
f:o;:t
C"I'ul~tor

· 23 • Amantadine binds to the M2 prot ein on the influenza A virus, thus preventing viral un coating.
· 24 . Amantadine has bee n used in the treatment and prophylaxis of influenza A infection. However,
90% of all in fl uenza A strains now are resistant to amantad ine, so it is no longer typically used
· 25 •
for these pu rposes in the United States. Amantadine is also used to t reat Parkin son's disease .
. 26 . Its mechanism of action in Parkinson's disease is not well understood, but it is th ought that it
· 27 • increases synthesis and release of dopamine from the substantia nigra.
Amantadme InfiuerlZa Entire substantia nigra
· 28 .
·29 . A is not correct. 7% chose this .
. 30 . Late protein synthesis occurs after nucleic acid synthesis. It can be blocked by methimazole and
· 31 • various protease inhibitors.
Methim<lzole
· 32 •
· 33 • B is not correct. 19% chose this.
.34 . Viral adsorpti on is t he first step requ ired for viral repl ication. It ca n be blocked in a nonspecific
manner by the adm inistration of gamma globulins .
. 35 .
C is not correct. 13% chose this.
· 36 .
Viral packaging and assembly occur after late protein synthesis and precede viral release. They
· 37 •
can be blocked by rifampin.
·38 . lUfampln
.39 . D Is not correct. 9% chose this .
. 40 . Early protein synthesis is the step following vira l uncoating. It can be blocked by fom ivirsen,
·4' . which is an antisense ol igonucleotide used to t reat cytomega lovirus infection .
. 42 . fomivir'Sen

.43 .
· 44 •

· 23 • Viral adsorption is the first step requ ired for viral repl ication. It can be blocked in a nonspecific
manner by the adm inistration of gamma globulins.
· 24 .
C is not correct. 13% chose this •
. 25 .
Viral packaging and assembly occur after late protein synthesis and precede viral release. They
.26 . can be blocked by ri fampin.
· 27 • lUfampin
· 28 .
D Is not correct. 9% chose this.
·29 . Early protein synthesis is the step following vira l uncoating. It can be blocked by fom ivirsen,
. 30 . which is an antisense ol igonucleotide used to treat cytomega lovirus infection.
· 31 • fomlYirsen

· 32 •
· 33 •
Bottom Li ne:
.34 .
Amantadine has been used in the treatment and prophylaxis of influenza A. It acts by binding
.35 . to the M2 protein on the virus to prevent vi ral uncoating.
· 36 . AmanL3dine Influenza

· 37 •

·38 .
.39 . hl;*,i.·,.·1 for year: 2015
FaST AID FACTS
.40 .
·4' . FA15. p , 192,4

. 42 . Anti viral the ra py


.43 . HIV ANTIVIR AL OTHER ANTIVIRAL
THERAPY THERAPY
· 44 •
+
,Pad9 ~
G ~ UI usmle rx com/ap l#qmax1
225{)
+ .1 :
",JlO% . '

Item 43 of 45
QI D 2n4 i
~ • Mark <::J
Prcylo us
c:>
Ne '"
al
ta b~ ues
~'
lIIotes
r~".I
ca lcu lato r

.23 . A man is undergoing inguinal hernia repair, which requires isolation of the structu res within I~A r
' 24 . the inguinal canal. While incising the inguinal canal, the surgeon damages an important
· 25 •
structure, alth ough on closer look the spermatic cord appears norma l and intact.

' 26 .
Which of the following symptoms would be most likely in this patient given this structural damage?
· 27 •
· 28 • 134415 : zheng

'29.
' 30 . "
A. I nferti lity

8. Numbness of the scrotum and medial thigh


We value your feedback!

' 31 •
C. Testicular edema
· 32 •
· 33 • D. Testicular ischemia
' 34 .
E. Testicular pain
' 35 •
. 36 •
. 37 .
· 38 •
The correct answer Is B. 54% chose this.
' 39 . The spermatic cord is a thick cord conta i ning the structu res that run to and from the testes. It is
' 40 . covered by three layers: external spermatic fascia derived from the external oblique muscle,
.41 . cremasteric muscle and fascia derived from the internal oblique muscle, and internal spermatic
fascia derived from the transversalis fascia. The ilioinguinal nerve arises from L1, passes
' 42 .
through the inguinal ligament on top of the spermatic cord, and suppl ies cutaneous sensation
to the scrotum/labia and medial of the It is not a of the spermatic cord
I I

covered by three layers: externa l spermatic i derived from the external oblique muscle,
cremasteric muscle and fascia derived from the internal oblique muscle, and internal spermatic
· 25 •
fascia derived from the transversalis fasc ia. The ilioinguinal nerve arises from L1, passes
through the inguinal ligament on top of the spermatic cord, and suppl ies cutaneous sensation
' 26 . to the scrotum/labia and medial aspect of the thigh. It is not a part of the spermatic cord and
· 27 • must be isolated separately from the cord during hernia surgeries to ensure that there is no
· 28 • loss of sensation to these areas.

'29.
' 30 .
Hernlil Strucrure of Crem.l5ter mU5de Inguinallig<lment Scrorum

A is not correct. 11% chose this.


The ductus deferens is the tube that carries sperm from the testes to the ejaculatory duct. It is
' 31 •
at the center of the spermatic cord.
· 32 •
Vas d~fN~M stnxtlJr~
· 33 •

' 34 .
C is not correct. 16% chose this.
. 35 . The pampiniform plexus, or testicular venous plexus, drains blood from the testes and is a part
of the spermatic cord. Additionally, the testes produce lymph that needs to be drained by
. 36 .
lymphatic vessels. Because the spermatic cord contains structures that run away from the
.37 . testes, the testicular lymph vessels are a part of the spermatic cord.
· 38 • EntIre lymphatic ve55e l

' 39 . D is not correct. 14% chose this.


' 40 . The testicular artery supplies blood to the testes and epididymis and runs in the spermatic cord.
.41 . Remember that anything running to or from the testes will be a part of the spermatic cord. Even
if you don't know the layers of the cord, you can eliminate structures that you know have
' 42 .
something to do with testicular function.
+ G ~ UI usmle rx com/ap l#qmax1 + iii :
Item 43 of 45
QI D 2n4 i
~ • Mark <::J
Prcylo us
c:>
Ne '"
i.1
ta b~ ues
~'
Notes
r~".I
ca lcu lato r

The testicular artery supplies blood to the testes and epididymis and runs In the spermatic cord .
. 23 .
Remember that anything running to or from the testes will be a part of the spermatic cord. Even
' 24 . if you don't know the layers of the cord, you can eliminate structures that you know have
· 25 • something to do with testicular function.
epidi<tjmis
' 26 .
· 27 • E is not correct. 5% chose this.
· 28 • Testicular pain can be found in conditions such as testicu lar torsion. This condition is

'29.
' 30 .
characterized by twisting of the spermatic cord and loss of blood flow to the testicles and
requires surgical correction . However, in this vignette the spermatic cord appears normal,
suggesting that testicular torsion is not the underlying pathology.
' 31 • Pain in testicle Spermatic Cord Torsioo Blood now

· 32 •
· 33 •
Bottom li ne:
' 34 .
The ilioinguinal nerve arises from l 1, passes through the inguinal ligament on top of the
. 35 .
spermatic cord, and supplies cutaneous sensation to the scrotum/labia and medial aspect of
. 36 . the thigh .
. 37 . Inguinal ligament Scrotum

· 38 •

' 39 .
' 40 . I iii I jfj , .j I.) for year: 2015
fiRST AID fACTS
. 41 .
' 42 . FAt5,p.351 .t

Inguinal canal

A 46-year-old microbiology professor comes to the clinic with complaints of cough, fever,
· 25 • and dyspnea. His WBC count is elevated, and x-ray of his chest reveals diffuse patchy
. 26 . interstitial inflammat ion . He is determined to have "wa lking pneumonia ." He is prescribed
· 27 • an antibiotic.
' 28 .

'29.
. 30 .
Based on the mechanism of action of the most common ly prescribed class of drugs to treat this
condition, which of the following is a plausible means by which bacteria could achieve resistance to
these drugs?
.31 .
134415 ; zheng
' 32 .
A. Alteration of the penta peptide side chain of N-acetylmuramic acid
· 33 •
. 34 • B. Increased utilization of host cell folate
. 35 . C. Methylation of the 235 ribosomal RNA
' 36 .
D. Mutation of the gene for DNA gyra5e
· 37 •

' 38 . E. Production of an enzyme that cleaves [3-lactam structures


. 39 •
. 40 .
We value your feedback!
' 41 •
. 42 .

' 43 .
· 44 •
+ G ~ UI usmle rx com/ap l#qmax1 + iii :
Item 44 of 45
QI D 4108 i
~ • Mark <::J
Prcylo us
c:>
Ne '"
i.1
ta b~ ues
~'
Notes
r~".I
ca lcu lato r

.25 "
· 24 "

· 26 "
Macrolides function by binding to th e 235 ribosomal RNA (rRNA) of the prokaryotic 50S
riboso me , effectively block ing the translocation step of protein synthesis . In the absence of
synthesis of bact erial protein s, the pathoqens are unable to qrow and divide; thus, the effect of
·27 "
· 28 "
macrolides is bact eriostatic. Methylation of the 235 rRNA in res istant bacteria diminishes the
binding of macrolides, rendering them less effective .
· 29 " A is not correct. 9% chose this.
· 30 " Vancomycin functions by binding to the pentapeptide si de chains of peptid oglycan precursor
molecules, preventing the formation of cross-bridges that are necessary for a functional

··3233 ""
• 31 " bacterial ce ll wall. Resistance to vancomycin has arisen in bacteria that have alt ered the term ini
of the side chains from D-alanine-D-alanine to D-alanine-D-Iactate.
B is not correct. 6% chose this.
· 34 " Su lfa drugs such as sulfamethoxazole competitively inhibit the production of fol ic acid by
· 35 " bacteria. Macrolides are not anti metabolites and do not affect bacterial folic acid pathways.
Foilc Acid
· 36 "

·37 "
· 38 "
D is not correct. 12% chose this.
Fluoroquinolones work by inhibiting DNA gyrase, wh ich bacteria need to replicate DNA.
· 39 " Selection for mutations in the gene for this enzyme creates resistance that renders it less
susceptible to inhibition. Although f1uoroquinolones are sometimes prescribed for "walking
· 40 "
pneumonia," they are not the most common choice.
· 41 " W~lking (~ctivity) Pneumon ia

· 42 "
E is not correct. 14% chose this.
· 43 "
Resistance to penicillin and other 13-lactam antibiotics is common ly achieved by bacteria l
·44 " production of 13-lactamases, which cleave the dru gs. Macrolides do not contain 13-lactam

Walking (activity) Pneumon ia

· 25 "
E is not correct. 14% chose this.
· 26 "

·27 "
· 28 "
Resistance to penicillin and other 13-lactam antibiotics is common ly achieved by bacteria l
production of 13-lactamases, which cleave the dru gs. Macrolides do not contain 13-lactam motifs.

· 29 "
Bottom II ne:
· 30 "
Macrolides such as azith romycin and erythromycin work by binding to the 23S rRNA of the

··3233 ""
• 31 " prokaryotic 50S ribosome, effectively blocking the translocation step of protein synthesis.
Methylation of the 235 rRNA in resistant bacteria diminishes the binding of macrolides.
Azithromy<:in Erythromycin

· 34 "
· 35 "
· 36 " I il1ifi'·'I.) for year: 2015
·37 "
· 38 "
FIII.S T AI O FA CTS

FAI5.p.I86 .1

· 39 " Mac;rolides Azithromyc;in. darithrQmycin. e ryt hromyci n.

· 40 " MWIAHISM Inhibit prote in synthes i ~ by blocking tran slocati on (Hmacroslides"); bind to the 23S rRNA of the
50S ribosomal subunit. Ract eriosMic.
· 41 " (lIN I(AlUSl: I\t )'pical pnelllnOllias (fo,·fycoplasmo. Chlamydia. Legioneila). STIs (Chlamydia). gram'posi)i"e cocc i
· 42 " (streptococcal il1fections in palients all erg ic 10 penicillin ). ;md B. pertussis

· 43 " IOII(IJY i\ IACRO; Gastrointeslilul ,\ Iotilit )' i ssue~. Arrhythmi;1causl-d by prolonged QT inle".. l. acule

·44 " C holeslatic hepalitis. Rash. eO sin ophilia. Inc reases SCHUll conce ntrali on of Ihl'OphylJines. ora l
anticoagul ants. Cbrithrom),cill and crrlhrom),ci n inhibit c)'tochrome P·-+ 50.
+ G ~ UI usmle rx com/ap l#qmax1 + iii :
Item 4S of 45
QI D 3084 i
~ • Mark <::J
Prcylo us
c:>
Ne '"
i.1
ta b~ ues
~'
Notes
r~".I
ca lcu lato r

I ~A r
.25 "
· 24 "

· 26 "
A child is broug ht to the physician for a check-up. Physical examination revea ls a
nevocellular nevus with dark coloration on his abdomen.

·27 "
· 28 "
The ce ll type responsible for this pigmentation is normally found in which of the fo ll owing skin
layers?
· 29 " 134415 : zheng
We value your feedback!
· 30 " A. Stratum corneum

8. Stratum gran ulosum

··3233 ""
• 31 "

C. Stratum lucidum

· 34 " D. Stratum spinosum


· 35 "
E. Stratum basalis
· 36 "

·37 "
· 38 "
· 39 "
The correct answer Is E. 54% chose this.
· 40 "
Nevus cells are derived from melanocytes, wh ich are melanin-producing cells located in the
· 41 " basal layer of the epidermis. They are distributed variably, depending on the skin color of the
· 42 " individual, interspersed between the columnar cells of the st ratum basal is. Within the
· 43 " melanocytes are specialized melan in-containing granules ca lled melanosomes, which are
transferred from the cytoplasm of the melanocytes to nearby keratinocytes. Melanoma is a
malignant tumor of I that manifest as moles with borders,
1 1

Nevus cells are derived from melanocytes, wh ich are melanin-producing cells located in the
basal layer of the epidermis. They are di stributed variably, depending on the skin color of the
· 25 "
individual, interspersed between the columnar cells of the st ratum basal is. Within the
· 26 " melanocytes are specialized melan in-containing granules ca lled melanosomes, which are
·27 "
· 28 "
transferred from the cytoplasm of the melanocytes to nearby keratinocytes. Melanoma is a
malignant tumor of melanocytes that may manifest as moles with changing borders, growth in
· 29 "
size, or change in color. The stratum basalis is also the most mitotically active of all the layers of
the skin, provid ing a constant supply of new keratinocytes to all the other layers.
· 30 " Epidermis Cyto~asm

··3233 ""
• 31 "
A is not correct. 8% chose this.
The strat um corneum is the topmost layer and consists primarily of dead cells that have lost all
their organelles and are completely keratinized. As the cells in this layer completely die, they
· 34 " lose the desmosomes that ordinarily connect them to each other in a process called
· 35 " desquamation.
Stratum corneum O'gilnelles Oesmosomes
· 36 "

·37 "
· 38 "
B is not correct. 16% chose this.
The cells of the stratum granulosum co ntain many basophili c granules, which further the
· 39 "
maturation of the keratinization process. No melanocytes are present in this layer.
Stratum granulosum
· 40 "
C is not correct. 6% chose this.
· 41 "
· 42 " The strat um lucidum is a thin layer that is present just below the strat um corneum.
Stratum corneum
· 43 "

·44 " D Is not correct. 16% chose this.


The cells of the stratum spinosum have a
+ G R< UI usmle rx com/apl#qmax 1 + ill :
Ite m 4S of 45
QID 3084 i
k • Ma rk <::]
Previous
t:>
"Ie'"
al
lab~ uu
~.
Notn
e-.:l
Cal(u l~tor

' 24 X C is not correct. 6% chose this.


· 25 X The stratum lucidum is a thin layer that is present just below the stratum corneum .
. 26 X Stratum (orneum
· 27 X D Is not correct. 16% chose this.
· 28 X The cells of the stratum spinosum have a prickly appearance. These cells are still developing
' 29 X and beginning the process of keratin synthesis. There are no melanocytes in this layer, although
. 30 X the cytoplasmic processes of the melanocytes found in the basalis layer may reach the stratum
· 31 X spinosum.
· 32 X
· 33 X Bottom line:
. 34 X Melanoma is a tumor of melanocytes, which are cells filled with melanosomes that typically
· 35 X reside in the stratum basalis (most basal layer) of the skin. Melanocytes are mostly in the skin,
but there are also some in the bowel and eye .
• 36 X
· 37 X '"
· 38 X
. 39 X
iii I jfi , ., 1.1 for year: 2015
. 40 X fIRST "1 0 f " CTS

· 41 X
FAtS, p . 436 .2
·42 X Epidermis tClyers S"in has 3 Ia)'ers: epiderm is. de rm is.
· 43 X subc ut ,mcous fat (hypod ermi s. subcuti s).
From surface to base:
· 44 X • Stf<li1Ull C orn cum ("cm!iu)

Two weeks after initiation of diuretic therapy to control essential hypertension, a 51-year-old
·2 X man presents to his physician for a check-up. Physica l examination reveals a blood pressure
· 3 X of 130/85 mm HQ. Laboratory tests show a blood pH of 7.48 , an arterial partial pressure of
·4 X ca rbon dioxide of 50 mm Hg , and a bica rbonate level of 41 mEq/ l.
·5 X
.6 X Which of the following etiologies is most consistent with this clinica l situation?
· 7 X 13441S: zhe ng
·8 X A. Acetazolam ide-induced metabolic alkalosis
· 9 X We value your feedback!
B. Acetazolam ide-induced respiratory alkalos is
' ,0 X
C. Hydrochlorothiazide-induced metabolic alkalosis
· 11 X

· 12 X O. Hydrochlorothiazide-induced respiratory alkalosis


· 13 X

" 4X
, '5X
, '6 X The correct answer is C. 65% chose this.
· 17 X Hydrochlorothiazide and furosemide cause metabolic alka losis. Note that the elevated PH
indicates an alkalosis, and the elevated bicarbonate and carbon dioxide indicate a metabolic
" 8X alkalosis with partial respiratory compensation. Both thiazides and loop diuretics inhibit sodium
uptake, and thus increase delivery of sodium distally to the late distal tubule and collecting
duct. An increased amount of sodium is taken into these distal cells from the luminal surface
and exchanged for potassium and protons to maintain electroneutrality. This creates a
condition of i (too little i in the blood), kaliuresis (urination of
+ G ~ UI usmle rx com/ap l#qmax1 + iii :
Item 1 of 46
QID 2346 i
~ • Mark <::J
Prcylous
c:>
Ne '"
i.1
tab~ ues
~'
Notes
r~".I
calculator

· 1
·2
•• Hydrochlorothiazide and furosemide cause metabolic alka losis. Note that the elevated PH
indicates an alkalosis, and the elevated bicarbonate and carbon dioxide indicate a metabolic
· 3
· 4
•• alkalosis with partial respiratory compensation. Both thiazides and loop diuretics inhibit sodium
uptake, and thus increase delivery of sodium distally to the late distal tubule and collecting

·6 •
duct. An increased amount of sodium is taken into these distal cells from the luminal surface
·5 and exchanged for potassium and protons to maintain electroneutrality. This creates a

•••
condition of hypokalemia (too little potassium in the blood), kaliuresis (urination of excess
· 7
potassi um ), and excess urinary aci d secretion. Th is loss of ac id promotes metabolic alka losis. In
addition, hypokalemia induces the movement of potassi um from its vast st orehouse within
· 8 tissues to the extracellular compartment in exchange for hydrogen ions. This movement of

·,0 •
· 9 potassium from tissues into blood and protons from the blood into tissues further promotes

.". . meta bolic alkalosis.


Hytlrochlorothiilzide Alkillos is Hypokil lemiil E~ril(ellulilr Hytlrogen Carbon Diox ide Sodium PoLlssium

· 12 • A is not correct, 25% chose this,


· 13 • Acetazolamide impa irs reuptake of bicarbonate and secretion of acid in the proxima l tubule.
The loss of bicarbonate in the urine leads to metabol ic acidosis .
· 14 •
Acetilzolam ide Kidney Tubules. Proximill
· 15 •
. ,6 . B is not correct. 6% chose this .
Th is patient 's laboratory values are consistent with metabolic alkalosis with respiratory
· 17 •
compensation. Acetazolam ide functions by impairing reuptake of bicarbonate and secreting
· 18 • acid within the proxima l tubule, thus you would expect to see metabolic acidosis with the use
of this agent.
Acetilzolilm ide Kidney Tubules. Proximill

D is not correct. 4% chose this.

•••
·2 Th is patient's laboratory values are cons istent with metabolic alkalosis with respiratory
· 3 compensation. Hydrochlorothiazid e causes a metabolic, not a respiratory, alkalosis.
Hytlrochlorothi<lzide ,o,1l«llosis
· 4
·5
· 6
•• Bottom Line:
· 7
· 8
•• Hydrochlorothiazide and furosem ide are diuretiC S capable of inducing metabolic alka losis.
Laboratory values consistent with metabolic alkalosis include elevated pH, elevated partial

• arterial carbon di oxide pressure, and elevated bicarbonate level.

".
· 9
Hyd.ochlo.ottlia.ide (".bon Dioxide
· ,0 •

.· 12 •
· 13 •
ul;fi"'''' for year: 2015
FIIU T .... 0 F"' CTS

· 14 •
FA15, p . 554 .3

.16.
· 15 •

· 17 •
Diuretics: electrolyte changes
Urine NaCi I wi lh all diurcl ics exeepl ace lazolam ide. Seru m NaC I may dec rease 3S 3 resili t
Urine K+ 1 wi th loop nud thiazide diuretics. SeTilm K+ ma), decrease as a res ult.
· 18 • Blood pH I (acidem ia): ca rbon ic anhydrase inhibilors: I HC O j - reabsorption. K+ spar in g: aldoslcro ne
bloc kad e pre,'cnls K+ secrel ion and 1-1+ secrcli on . Addit io nall )'. h)'perblcm ia lead s 10 K+ e ntering
all cells ("ia H' IK' exchanger) in exchange for H' ex ilin g cell s.
I (alkalcm i;I): loop di urcli cs and Ihi az ides eatlse Hlkal cm ia Ihrough se,"Cnll rIlechHnisms:
• Vol ulllc C1:m tT<l ction - , t' T II - 1 Na ' l l-I ' exchange ill rer - I HCO j - reabsorpt ion
("cOll traclio n alkal os is")
+ G ~ UI usmle rx com/ap l#qmax1 + iii :
Item 2 of 46
QID 2766 i
~ • Mark <::J
Prcylous
c:>
Ne '"
i.1
tab~ ues
~'
Notes
r~".I
calculator

""
· 1
A soccer player experiences a tibia l fracture resul tinq in a compression synd rome. He is no I ~A r
·2 longer able to dorsiflex the foot and a decreased dorsalis pedis pulse is appreciated.
· 3
· 4 "" Whic h nerve and compartment are most likely affected?

""
· 5
134415 ; zheng
· 6 A. Deep peroneal nerve, anterior compartment
· 7
· 8 "" 8. Deep peroneal nerve, posterior compartment
We value your feedback!

" C. Superficial femoral nerve, anterior compartment


· 9

· 10 " D. Superficial peroneal nerve, posterior compartment

··.1214"
13""
· 11 "
E. Tibial nerve, anterior compartment

F. Tibial nerve, posterior compartment

·15"
· 16 "

··18"
17" The correct answer Is A. 51% chose this.
A lacerated artery can bleed into a closed space, causing compartment syndrome. Symptoms of
compartment syndrome inc lude pain much greater than would be ex pected based on the
clinical exam and decreased blood flow at the site of injury. The deep peroneal nerve is located
in the anterior compartment and innervates the muscles that dorsiflex the foot. Also in the
anterior compartment is the anterior tibial artery , which gives rise to the dorsal artery of
I

A lacerated artery can bleed into a closed space, causing compartment syndrome. Symptoms of
compartment syndrome include pain much greater t han would be expected based on the
·2
· 3 "" cl inical exam and decreased blood flow at the site of injury. The deep peroneal nerve is located
in the anterior compartment and innervates the muscles that dorsiflex the foot. Also in the
· 4
" anterior compartment is the ant erior tibial artery , which gives rise to the dorsal artery of the

""
· 5 foot. A fracture of the t ibia can lacerat e this artery and cause hemorrhage that is contained in
· 6 the fascial compartment. Along with massive inflammation from the injury, acute compartment
syndrome can result. Treatment for any compartment syndrome is release of pressure by
· 7
· 8 "" surgica l incision. This must be done quickly to avoid ischemia and death of the tissue .
B is not correct. 12% chose this.

"
· 9
The deep peroneal nerve is found in the anterior compartment of the leg and is responsible for
· 10 " dorsiflexion. Contraction of the muscles in the posterior compartment of the leg plantar flexes
the foot. This vignette indicates a defect in dors iflexion .

··.14"
12 "
· 11 "
C is not correct. 6% chose this.
13" The superficial femoral nerve is found in t he lateral compartment of the leg and innervates the
peroneus longus and brevis muscles that evert and planta r flex the ankle.

··16"
15" D is not correct. 7% chose this.
The superficial peroneal nerve is found in the lateral compartment . These musc les are prima rily

··18"
17" responsible for eversion of the ankle, not dorsiflexion.
E Is not correct. 18% chose this.
The tibial nerve is found in the posterior compartment of the leg and innervates the muscles
primari ly responsible for plantar flexing the ank le. The posterior tibial artery courses through
the posterior compartment; its pulse can be palpated posterior to the media l malleolus.
+ G ~ UI usmle rx com/ap l#qmax1 + iii :
Item 2 of 46
QID 2766 i
~ • Mark <::J
Prcylous
c:>
Ne '"
i.1
tab~ ues
~'
Notes
r~".I
calculator

""
· 1
·2
E is not correct. 18% chose this.
The tibia l nerve is found in the posterior compartment of the leg and innervates the muscles
· 3
· 4 "" primari ly responsible for plantar flexing the ankle. The posterior tibial artery courses through
the posterior compartment; its pulse can be palpated posterior to the media l malleolus.

""
· S Ankle

· 6
F is not correct. 6% chose this.
· 7
· 8 "" The posterior compartment muscle group, innervated by the tibial nerve, is primarily
responsible for plantar flexion of the foot. Furthermore, the dorsal artery of the foot originates
from the anterior tibial artery; a decreased dorsalis pedis pu lse wou ld indicate compression of

"
· 9
this artery in the anterior compartment. A compartment syndrome in the posterior
· 10 " compartment may reveal a decreased posterior tibial pulse.
Compilrtment ~yndrome~

··.1214"
13""
· 11 "

Bottom Line :

·15"
· 16 "
A lacerated artery can bleed into a closed space, causing compartment syndrome.
Compartment syndrome is quit e painful and must be treated quickly to avoid death of tissue.
~yndromes

··18"
17"
Compilnment

Iii lif, , ., .. , for year: 2015


fl lUT ... ID f ... CTS

A woman who is 25 weeks' pregnant presents for her fi rst prenatal visit. She delivered her
•2
•3 "" first baby at home with no prenatal care. On ultrasound, the physician finds that the fetus
has asc ites, pleural effusion, and edema of the subcutaneous tissue.
· 4
"
""
· S The potential pathology that the clinician is concerned about is classified as which of the following
· 6
immune reactions?
· 7
· 8 "" A. Graft-versus-host disease
We value your feedback!
134415 : zheng

"
· 9 B. Type I hypersensitivity
· 10 "
C. Type II hypersensitivity

··.1214"
13""
· 11 "
O. Type III hypersensitivity

E. Type IV hypersensitivity

··16"
15"
··18"
17" The correct answer is C. 68% chose this.
PregrKIf\CY Ph.lgocytosi~ Ar.emiil Hydrops feLllis Pleuril
+ G ~ UI usmle rx com/ap l#qmax1 + iii :
Item 3 of 46
QID 2413 i
~ • Mark <::J
Prcylous
c:>
Ne '"
i.1
tab~ ues
~'
Notes
r~".I
calculator

· 1 " Pregnancy Phagocytosis Ar.emia Hydrops Fetali$ Pleura


Direct Coombs'
Step Indirect Coombs' Test
·2 " Hemolytic disorder Heart failure Hypersensitivity Tes t

· 3 " Sample from


pal~t
RaCs Sorum
· 4 " Reactant
added to
·5 " Anti-Ig antibody NOOTliII RBCs
patient
~m"
· 6 "
RBG clumping indicates thai
RSe clump indicates
· 7 " Interpretation the patient's serum contains
that the RBCs are
01 results anb-RaC antibody (e .g .. anli-
coated with Ig
· 8 " Rh lgG)

· 9 " The fetus has hydrops feta li s, which is defined as the collection of fl uid in two or more body
· 10 " compartments (subc utaneous tissue, abdomen, pericard ium, pleura). I n th is clinical scenario,
the cause is most likely hemolytic disease of the newborn. This disease can result when a

··.1214"
13""
· 11 "
mother is Rh-negative and th e fetus is Rh+. This disease is mediated by materna lly derived IgG
anti-Rh antibod ies developed in Rh-negative mothers t hat are directed at the Rh antigen present
on the fetal RBCs of a Rh+ fetus in a previous pregnancy. If the mother possesses the antibodies
developed from a previous exposure to an Rh+ fetus, t hey may cross the placenta (antibod ies of
·15"
· 16 "
the IgG isotype read ily cross the placenta) and coat the fetal RBCs of a Rh+ fetus if th e mother is
now pregnant with another Rh+ child. Antibody coating of t he RBCs leads to phagocytos is of

··18"
17" RBCs (via Fc receptors) and/or destruction of the RBCs by the com plement system and
potentially fatal anemia. The severe anemia can lead to heart failure and hydrops fetal is, wh ich
likely happened with this fetus. This antibody-med iated cytotoxic reaction is an example of a
type II hypersensitivity reaction. The result of an indi rect Coom bs' test on the mother'S blood
will be pos itive (ie, when normal RBCs are added to the mother's serum, her IgG against Rh wi ll
cause the RBCs to agg lutinate).
The differences between direct and indirect Coombs' tests are detailed in the ta ble.

I
Graft-versus-host disease is a potentially lethal side effect of bone marrow transplantation.

""
·2
Bone MarrowTranspjantation
· 3
B is not correct. 6% chose this.
· 4
" Type I hypersensitivity reactions are antibody-med iated but require anti gen binding to IgE,

""
·5
which is prebound to the surface of mast cells . Mast cell degranulati on then ensues . Examples
· 6
include anaphylaxis, asthma, hives, and loca l wheal and flare.
· 7
· 8 "" anaphylaxIs Asthma mast cell

D is not correct. 13% chose this.

"
· 9
Type III hypersens it ivity reactions are immune com plex-mediated. Examples include
· 10 " polyarteritis nodosa, glomerulonephrit is, rheumatoid arth ritis, and systemic lupus
erythematosus.

··.1214"
13""
· 11 "
Hypersens itillity PolyarteriUs Nodosa Rheumatoid Arthntis Lupus Erythematosus. System ic

E is not correct. 8% chose this.


Type IV hypersensitivity reacti ons are a group of T-ceil-mediated patholog ies. Examples inc lude

··16"
15" the tuberc ulin skin test, transplant rejecti on, and contact dermatitis.
Pur ir,ed Protein Derivative orTuberculin Graft Rejection Contact DermatItis

··18"
17" Bottom Li ne:
Erythroblastosis fetalis, or hemolytic disease of the newborn, is due to materna l IgG anti-Rh
anti bodies in a type II hypersensitivity reaction.
Hypersensitivity
+ G ~ UI usmle rx com/ap l#qmax1 + iii :
Item 3 of 46
QID 2413 i
~ • Mark <::J
Prcylous
c:>
Ne '"
i.1
tab~ ues
~'
Notes
r~".I
calculator

· 1 " Type III hypersensitivity reactions are immune complex-mediated. Examples include
·2 " polyarteritis nodosa, glomerulonephritis, rheumatoid arthritis, and systemic lupus
erythematosus.
· 3 "
Hypersensitivity Polyarteritis Nodo'Sa Rheumatoid ArthritiS lupus Erythematosus, System ic
· 4 "
E is not correct. 8% chose this.
·5 "
Type IV hypersensitivity reactions are a group of T-cell· mediated pathologies. Examples include
· 6 "
the tuberculin skin test, transplant rejection, and contact dermatitis.
· 7 " Pur ified Protein Deriv<ltive ofTuber,ulin Gr<lft Rejenion Contact Dermatitis
· 8 "

· 9 "
Bottom Line:
· 10 "
Erythroblastosis fetal is, or hemolytic disease of the newborn, is due to maternallgG anti-Rh j
··.14"
12 "
· 11 "
antibodies in a type II hypersensitivity reaction .

13" HypersenSitivity
'---------------'-'

·15"
· 16 "
I ill ;fi ,
FIII.ST AID FAC TS
., 1.1 for year: 2015
··18"
17" FA15, p, 211 .1
Hype r$Im$itivity type$
Type I t\n<lphyb ctic and ;ltopi c-r,,~e ~ nti ge n cross- First (type) JIU] F:lst (:"' J I)hrla~ is). Types [. II.
lin ks Ig E on presc nsiti;r;cd In~st cells and ~ "d 1[ [ arc a[1 ~"ti body mcdiated.
ba50phils. lriggcring iln nK'dia lc release of l est: skin test for sp('ci fic IgE.
\'ilSOilct i,·c ,lI nincs t h ~1 act at pos lc~pillil r}"
II I ~

When conducting a clinical trial on a new asthma drug, a nonblinded physician participating

""
•2 in the study notices benefits in patients who are taking the drug rather than the placebo.
•3 Knowing that the experimental drug is actually beneficial, he begins intentionally to assign
newly enrolled patients with more severe asthma to the experimental group rather than the
· 4
· 5 "" placebo group.

"""
•6
Which of the following types of bias is this?
•7
134415: zhe ng
•8 We value your feedback!
A. Late-look bias
· 9

· 10 "

· 11 "
" B. Length bias

C. Recall bias

···14"
1213""
"
D. Sampling bias

E. Selection bias
··16"
15"
··18"
17" The correct answer is E. 72% chose this.
Selection bias occurs when patients in a study are not randomly assigned to a treatment group.
This can occur because either the patients or the investigators select the group that an
individual patient wi ll enter. Selection bias can be avoided by randomization of participants to
treatment groups in a double-blind study.
+ G ~ UI usmle rx com/ap l#qmax1 + iii :
Item 4 of 46
QID 15SO i
~ • Mark <::J
Prcylous
c:>
Ne '"
i.1
tab~ ues
~'
Notes
r~".I
calculator

""
· 1
Selection bias occurs when patients in a study are not randomly assig ned to a treatment group.
·2 This can occur because either the patients or the investigators select the group that an
individual patient wi ll enter. Selection bias can be avoided by randomization of participants to
· 3

" treatment groups in a double-blind study.

""
· 4
·5 A is not correct. 4% chose this.
Late-look bias concerns information gathered at an inappropriate time in the study. It results in

"""
· 6
selection of patients with less severe disease because those with more severe disease died
· 7
before detection. An example would be enrolling patients with metastatic breast cancer in a
· 8 cl inical trial 6 months after diagnosis instead of immed iately.

"
· 9 Brea5t

· 10 " B is not correct. 2% chose this.


Length bias (and lead-time bias) occurs when the tempora l characteristics of a disease are not

··.1214"
13""
· 11 "
considered. An example would be a new cancer screening test that detects an incu rable form of
cancer 3 months before the patient becomes symptomatic. The patient's survival with the
disease wo ul d be "increased" by 3 months with the new test, but only because disease was
detected earlier and not necessarily because the screening was beneficial.
·15"
· 16 "
lead

C is not correct. 3% chose this.


··18"
17" Recall bias occurs when a patient's report of symptoms or past event s is selective, either
intentionally or not. An exam ple of when this is important wou ld be in a survey of elderly ad ults
with sh ingles when they are asked detai led questions about their childhood chickenpox
infections.
Elderly (pop<Jlauon group) Chickenpox

Reca ll bias occurs when a patient's report of symptoms or past events is selective, either

""
•2 intentionally or not. An example of when this is important wou ld be in a survey of eld erly adults
•3 with sh in gles when they are asked detailed questions about their childhood chickenpox
infections.
· 4
·5 "" Elderly (pop<Jlalion group) Chickenpox

D is not correct. 19% chose this.

"""
· 6
•7
Sampling bias occurs when a sa mple group is not representative of the population from which it
is taken. An example would be a study looking at obesity levels in a local town, where the
•8
surveys were distributed in the local fast food restaurants.
· 9

· 10 "

· 11 "
" Obe51ty

···14"
1213"" Bottom line:
Selection bias occurs when patients are not random ly assigned to treatment groups.

··16"
15" 141;1',1,,..1 for year: 2015
FtlST "10 FACTS

··18"
17" FA1S, p. 52.1
Bias and study errors
TTl'( O[f)N IIION EX.l.MPlES SIRAIEGYlO REOO(E BIAS
, Recruiting participant~
Selection bias
,Pad 9 2255 6010 . '

+ G R< UI usmle rx com/apl#qmax 1 + • :


Item 50f46
QID 3898 i
~ • M
ar
k <:l
F,e~lou$
C>-
Ne xt
Al
tab~ un
Cf '
"jolu
~~I
Calculator

· 1

•2
•• A 33-year-ol d woman was diagnosed with pneumocystis pneumonia. In addition to drugs to
treat her pneumonia, she began taking zidovudine, lamivud ine, and ritonavir. Although she
I~A I

•3
•4
•• in it ially responds to treatment, 1 year later, her physician considers adding enfuvirtide to her
reg imen.

·6 •
· S

•••
What is the mechanism of action of enfuvirtide?
· 7 134415: zheng
· 8

.,0 •
· 9 A. Blocking incorporation of the vi ral genome into the host cell genome
.
B. Blocking translation of the vira l messenger RNAs
· 11 •

· 12 • C. Interfering with posttranslational processing of viral proteins


· 13 •
" O. Preventing entry of viral particles into the target host cell
· 14 •
· 15 • E. Preventing synthes is of DNA from viral RNA
.,6 .
· 17 •
We value your feedback!
· 18 •

•••
Pneumocystis pneumonia is an AIDS-defining illness, and the patient is being treated with
·2 highly active antiretroviral therapy. HIV enters the host cell by binding its vira l envelope
· 3 glycoprotein (gp120) to the host ce ll 's CD4 glycoprotein. This interaction induces a
· 4 conformationa l change in gp120. The altered gp120 then binds the host cell's co-receptor,
· S
· 6
•• CXCR4 (on T-Iymphocytes) or CCRS (on T-Iymphocytes and macro phages). Another viral envelope
glycoprotein (gp41) is brought closer to the host cell, and it then mediates the fUSion of the viral
envelope with the host cell's lipid membrane. Entry inhibitors may act either by blocking the
· 7
· 8
•• interaction between gp120 and CD4 or by interfering with gp41-mediated fusion. Enfuvirtide is
an example of the latter.

·,0 •
enfuvirtide Antiretroviral Therapy, Highly AC\M!
· 9

. A is not correct. 16% chose this.


· 11 •
Integrase is a viral enzyme that integrates viral DNA into the host's own genome. This allows for
the transcription and translation of the viral genetic information. Integrase inhibitors are a
· 12 •
newer class of antiretroviral drug that inhibit the action of integrase. An example of this is
· 13 • ralteg ravir.
· 14 • Raltegravir

.16.
· 15 •

· 17 •
B is not correct. 11% chose this .
An additiona l target cu rrently under investigation as an HIV treatment is the vi ral protein tat
(encoded by the tat gene). One feature of the HIV life cycle not mentioned above is that when
the viral DNA is initially transcribed, the mRNA transcript undergoes internal nucleotide base-
· 18 •
pa iring. Th is hairpin structure prevents fu ll transcription of the viral DNA. The protein tat is one
of the few mRNA products that still get properly transcribed/translated, and it thus functions to
destabilize the mRNA hairpin, allowing full transcription. Overall, it is a positive-feedback
mechanism for HIV activation. A that cou ld block the function of tat would prevent
svr,th,>s;is of the of viral pre,te;ns.
,Pad 9 2255 6010 . '

+ G R< UI usmle rx com/apl#qmax 1 + • :

'.
~ <:l C>- Al Cf ' ~~I

........... , ... ......


Item 50f46 • M k
QID 3898 i ar F,e~lou$ Ne xt tab~ un "jolu Calculator
,. -, ......... .
.3 '2
.4. . (encoded by the tat gene). One feature of the HIV life cycle not mentioned above is that when
the viral DNA is initially transcribed, the mRNA transcript undergoes internal nucleotide base-
pairing. This hairpin structure prevents fu ll transcription of the viral DNA. The protein tat is one

..''8
567..
of the few mRNA products that still get properly transcribed/translated, and it thus functions to
destabilize the mRNA hairpin, allowing full transcription. Overall, it is a positive-feedback
mechanism for HIV activation. A drug that cou ld block the function oftat would prevent

'9.
. synthesis of the majority of viral proteins .
C is not correct. 14% chose this .
This is the mechanism of action of protease inhibitors such as saquinavir. Protease is an HIV-
encoded enzyme responsible for cleaving the initial pol and gog polypeptides into their final
.,0 . products. (Note: The env gene product also undergoes posttranslational Cleavage. A host-
encoded enzyme mediates this cleavage, however, not HIV protease.)
· 11 •
SaqulrKlvlr
· 12 •
E is not correct. 11% chose this.
· 13 •
Because the original genetic information from the HIV virus is in the form of RNA, reverse
· 14 •
transcriptase is needed to convert the si ngle-stra nded RNA into double-stranded DNA. There
' 15 • are two classes of reverse transcriptase i nhibitors. Nucleoside analogue reverse transcriptase
,'6 . inhibitors, such as zidovudine, are incorporated into the DNA strand and prevent elongation. On
· 17 •
the other hand, nonnucleoside reverse transcriptase inhibitors, such as nevirapine, bind the
enzyme directly to block the cata lytic site.
· 18 •
l idovudine Nevirapjne

are two classes of reverse transcriptase i . Nucleoside analogue reverse transcriptase

•••
·2 inhibitors, such as zidovudine, are incorporated into the DNA strand and prevent elongation. On
the other hand, nonnucleoside reverse transcriptase inhibitors, such as nevirapine, bind the
· 3
enzyme directly to block the catalytic site.
· 4

••
lidovudine Nevirapine
·5
•6

· 7
· 8
•• Bottom Line:
Highly active antiretroviral therapy targets HIV through multiple mechanisms. Entry inhibitors,

"0.•
such as enfuvirtide, act by blocking interaction between viral gp120 and host (04, or by
· 9
interfering with vira l gp41-mediated fusion. Other drugs that are used include integrase
inhibitors, nucleoside analogue reverse transcriptase inhibitors, nonnucleoside reverse
transcriptase inhibitors, protease inhibitors, and inhibitors of messenger RNA trans lation . [
· 11 •
enflNirtide AntJretroviral Therapy. Highly Active
· 12 •
· 13 •
· 14 •
141 jfj , .j 1.1 for year: 2015
.16.
' 15 •

· 17 •
FIRS T A I D FACTS

FA15.p.I68.1
HIV
· 18 •
Diploid genome (2 moleel1les of RNA).
T he 3 strneiur;.1 genes (protein coded for):
• em'(gpI20Jlldgp41 ):
• Formed from clcJ\';Igc of gpl60 10 form
cn\"elopc gl)'coprolcins.
I I I
,Pad 9 2255 6010 . '

+ G R< UI usmle rx com/apl#qmax 1 + • :


Item 60 f46
QID 3229 i
~ • M
ar
k <:l
F,e~lou$
C>-
Ne xt
Al
ta b~ un
Cf '
"jolu
~~I
Calcu lator

· 1

•2
•• A 13-year-old gi rl recently ret urned from spending a day in the woods with the Girl Scouts.
Upon return ing she started to com plain of a headache and ch ills. After 3 days of these
•3
•4
•• constitutional symptoms, she developed a ra sh that began on her palms and so les, but
spread inward to her trunk. Her worsening condition led her parents to take her to the emergency

·6 •
department, where a blood test reveals anti bodies that react with the Proteus antigen.
·5

•7
•8
••• This patient is most likely infected with which of the following?

We va lue your feedback!


134415 : zheng

.,0 •
A. Borrelia burgdorjeri
· 9
. B. Coxiella burnetti
· 11 •
C. Coxsackievi rus A
· 12 •
· 13 • " D. Rickettsia rickettsii
· 14 •
E. Rickettsia typhl
· 15 •
.,6 . F. Treponema pallidum
· 17 •
· 18 •

The correct answer is O. 71% chose this.


Th is patient most likely has Rocky Mountain spotted fever, as indicated by the rash on
pa lm s and soles and the inward, "centripetal " pattern of

•••
Th is patient most likely has Rocky Mountain spotted fever, as indicated by the rash on her
·2
pa lms and soles and the inward, "centripetal " pattern of spread. Other supporting evidence is
· 3 the accompanying headache and fever, and a positive Weil-Felix reaction, which is a cross-
· 4 reaction of certain anti rickettsial antibodies with the Proteus antigen. Rocky Mountain spotted
·5
· 6
•• fever is caused by Rickettsia rickettsii and is endem ic to the east coast of the United States. It is
transmitted by the Dermacentor tick; thus the patient probably became infected during her
recent outdoor excursion . Treatment is with a cou rse of doxycycline or, if necessary,
· 7
· 8
•• chloramphenicol.
Headache Doxycycl ine Chloramphenicol Rocky Mountain SlK'ned Fever

· 9

·,0 . • A is not correct. 9% chose this.


Borrelia burgdorferi causes Lyme disease. The rash of Lyme disease is typica lly a bUll's-eye type
rash known as erythema chronicum migrans, and it ha s a negative Wei I-Felix react ion.
· 11 •
Treatment is with doxycycline.
· 12 •
Doxycycline Lyme Disease VacclI)e Eye Erythema Chronicum Migrans
· 13 •
· 14 •
B is not correct. 4% chose this.

.16.
· 15 •

· 17 •
Coxiella burnetti is a rickettsial organism. It is highly infectious, as it is transmitted by
aerosolized endospores. It causes Q fever, wh ich is characterized by high fever, headache,
shaking, malaise, and mya lgia. Q fever can sometimes cause a life-threatening endocarditis,
especially in patients with abnormal heart valves. It is not associated with rash or a positive
· 18 • WeB-Felix reaction.
Headache Q Fever

C is not correct. 7% chose this.


Coxsackie A is an RNA vi rus that causes hand, foot, and mouth disease, which can also
with a rash on the and in add iti on to oral and I lesions.
,..·23.
,Pad 9 2255 6010 . '

+ G R< UI usmle rx com/apl#qmax 1 + • :


Item 60 f46 ~ • M k <:l C>- Al Cf ' ~~I

.
QID 3229 i
. ar
. F,e~lou$
. Ne xt ta b~ un "jolu Calcu lator

Coxsackie A is an RNA virus that causes hand, foot, and mouth disease, which can also present

.4
.
·5
.
with a rash on the palms and soles, in addition to oral and occasionally genital lesions .
However, the positive Wei I-Felix reaction in th is case and recent history of a camping trip point
to Rickettsia rickettsiae as a more likely causative organism in this case.

·6
.
.7
.8.
Hand, ~oot and Mouth Disease

E is not correct. 6% chose this .


Rickettsia typhi causes endemic typhus and is transmitted by fleas. This disease is often spread

·9.
.,0 .
· 11 •
in refugee camps and war zones, where rodents can spread the fleas that ca rry this disease. The
rash of typhus is centrifugal; it spreads outward, not inward, as in this patient.
TYPHUS

F is not correct. 3% chose this.


Treponema pafJidum is the spirochete that causes syphilis, a sexually transmitted disease.
· 12 •
Although secondary syphil is can also present with a rash on the palms and soles, this patient
· 13 • has no history of a sexual encounter that would put her at risk for this disease. Social history
· 14 • suggests a tick-borne disease.
Tick·Borne Diseases
· 15 •
.,6 .
· 17 • Bottom li ne:
· 18 •
Patients complaining of headache and centripeta l rash after an outdoor trip should receive a
work-up for Rocky Mountain spotted fe ver.
Headache Rocky Mountain Spotted Fever

•••
Treponema pafJidum is the spirochete that causes syphilis, a sexually transmitted disease.
·2
Although secondary syphilis can also present with a rash on the palms and soles, this patient
· 3 has no history of a sexual encounter that would put her at risk for this disease. Social history
· 4 suggests a tick· borne disease.
·5
· 6
•• Tick·Borne Diseases

· 7
· 8
•• Bottom line:
Patients complaining of headache and centripeta l rash after an outdoor trip should receive a
· 9

·,0 . • work-up for Rocky Mountain spotted fever.


Headache Rocky Mountain Spotted Fever

· 11 •

· 12 •
· 13 •
iii I jfj , .j 1.1 for year: 2015
flll. S T AI O fA CTS
· 14 •

.16.
· 15 •

· 17 •
Rickettsh'll disenes
,lind vector-borne
illness
Tre<llmcnt for ;.11, dox),c)'c1;nc.
FAI 5.p.143.1

RASH (I)hIMON
· 18 •
Rocky Mountain Ricketlsill rid el/sii. veclor is ti ck. Dcspite its C I3ssic tri;,d- he<ld ache. boer. rash ('<lsc ulit is)
spotted fever name, di sease occ urs primar ily in th e South P~l ms 'IIld ~olcs rOlsh is seen in Co~s"ck i l'\' irus
Atl<l ntic st ~ tes. cspccially North Carolin a. A infITt ion (hand. foot. and mouth disc~ se).
Rash t)'picall r starts ~t wri sts r.J and ~nklcs ~nd Roc k)' ~ l ounlain spotted fcvcr. and 2° Syphilis
thcn spreads to trunk , 1I.1 Im5, a nd soles. (yo u drh'c C t\ RS usin g rour Il.lllms lllHl .>olcs).
,Pad 9 2256 6010 . '

+ G R< UI usmle rx com/ap l#qmax 1 + • :


Item 70 f46
QID 1984 i
~ • M
ar
k <:l
F,e~lou$
C>-
Ne xt
Al
ta b~ un
Cf '
"jolu
~~I
Calcu lator

· 1

•2
•• A 46-year-ol d female patient has recently undergone all ograft kidney transplant surgery. She
is now on an immunosuppressive drug regimen to help prevent rejection of the donor
I ~A I

•3
•4
•• kidney.

·6 •
·5 Which of the following immunosuppressive agents modulates the level of interleukin-2 secretio n?

•7
•• A. Azathioprine
We value your feedback!
134415 : zheng

•8

.,0 •
· 9
•. 8. Cyclophosphamide

C. Methotrexate
· 11 •
D. Sirolimus
· 12 •
· 13 • " E. Tacrolimus
· 14 •
· 15 •
. ,6 .
The correct answer Is E. 52% chose this.
· 17 •
Tacrolimu5 binds to FK-binding protein on a nuclear level, thereby inhibiting secretion of
· 18 •
interleukin-2 (Il-2) and ot her cytokines . This blocks the differentiation and activation of T
lymphocytes, which normally function to modulate the immune response. Tacrol imus is similar
in its effects to cyclosporine and when using it in kidney transplantation, it has been shown to
have fewer adverse effects than sirolimus.

•••
·2 Tacrolimus binds to FK-bindi ng protein on a nuclear level, thereby inhibiting secretion of
· 3 interleukin-2 (Il -2) and other cytokines. This blocks the differentiation and activation of T
lymphocytes, which normally function to modulate the immune response. Tacrol imus is simila r
· 4

••
in its effects to cyclosporine and when using it in kidney transplantation, it has been shown to
·5 have fewer adverse effects than sirolimus.
· 6 Immune response C~closporine Sirolimus Tauolimus I(jdne~Transplantation

•••
· 7
A is not correct. 8% chose this.
· 8 Azathioprine is an immunosuppressive agent that acts as an antagonist to purine metabolism,
· 9 and therefore cell proliferation.
·,0 . Azathioprine Metabolism

· 11 • B is not correct. 14% chose this.


· 12 • Cyclophosphamide is an alkylating agent that exerts its effects on cell division by causing cross-
· 13 • linkages in DNA. It is not specific to any particula r phase of the cell cycle.
Cydophosph<lm ide
· 14 •

.16.
· 15 •

· 17 •
C is not correct. 4% chose this .
Methotrexate is an antimetabolite that acts as a folic acid antagonist, preventing dihydrofolate
reductase from making tetrahydrofolate (TH4). Because DNA cannot be made without TH4,
methotrexate is an S phase-spec ific antimetabolite.
· 18 • Folic Acid

o is not correct. 22% chose this .


Sirolimus acts at the mammalian target of rapamycin (MTOR) receptor . like tacrolimus, it binds
to FK-binding protein, but then acts by blocking signal transduction from the Il-2
,Pad 9 2256 6010 . '

..3·2,..
+
Item 70 f46
QID 1984
G
i
R<
~ •
UI usmle rx com/ap l#qmax 1

M
ar
k <:l
F,e~lou$

D Is not correct. 22% chose this.


C>-
Ne xt
Al
ta b~ un
Cf '
"jolu
~~I
Calcu lator
+ • :

.4 .
·5..
Si rolimus acts at the mammalian target of rapamycin (MTOR) receptor . Like tacrolimu5, it bi nds
to FK-binding protein, but then acts by blocking signal transduction from th e IL-2 receptor. The
end result is to inhibit T- and B-Iymphocyte prol iferation . There is no effect on the level of IL-2

·6
.·98..
. 7 •
secretion.
Sirol imus Signal Transduction Tacrolimus Lymphocyte

Bottom II ne:

.,0 . Tacrolimu5, an immunosuppressive agent, binds to FK-binding protein to inhibit the secretion
ofIL-2.
· 11 • Tacrolimus

· 12 •
· 13 •
· 14 • Iii lif, , ., I,' for year: 2015
FIRST AI D FACTS
· 15 •
. ,6 . FAt 5, p. 218.'
· 17 • Immunosuppre ssants Agen ts that block lymphoc)'tc ad il'ation and prolifeflltion. Rcd uce acute Ifllll.lplant rejcr tion by
supprcssill g ed lu b r illllllu ui ly. FrC'llleuI I), eo,ll],in..,d 10 achievc gre:lle r ..,ffieacy " ill, I to.~icil y.
· 18 •
C hron ic supprcssion t ris k of infedion and mali gnancy.
DRlJG MECHIJHlM USl IOXI(lTT NOlES
Cyclosporine TrJ llspl;",j r..,jedion
proph),l'lxis. psor iasis.
rl lCllillatoid arthritis.

•••
Drug A is an agonist. Drug X is a drug that will bind to drug A's receptor but will not activate
·2 it. Drug X cannot be counteracted by increasing the dose of drug A.
· 3
· 4 Drug X is most likely which of the fo ll owing?
·5
· 6
•• A. Chemical antagonist
We value your feedback!
134415: zheng

· 7
· 8
•• 8. Competitive antagoni st

·,0 •
· 9 C. I rreversible antagoni st
. D. Partial agonist
· 11 •

· 12 • E. Phys iologic antagonist


· 13 •
· 14 •

.16.
· 15 •

· 17 •
The correct answer is C. 70% chose this .
A pharmacologic antagon ist is a drug that binds to a specific receptor but does not activate it.
An irreversible antagonist is a pharmacolog ic antagonist that cannot be overcome with
· 18 •
increasing concentrations of agonist.
A is not correct. 6% chose this.
A chemical antagonist wi ll bind directly to the agonist, preventing interaction with its
removing it.
,Pad 9 2256 6010 . '

+ G R< UI usmle rx com/ap l#qmax 1 + • :


Item 80 f46
QID 1930 i
~ • M
ar
k <:l
F,e~lou$
C>-
Ne xt
Al
ta b~ un
Cf '
"jolu
~~I
Calcu lator

· 1

•2
•• A pharmacologic antagonist is a drug that binds to a specific receptor but does not activate it.
An irreversible antagonist is a pharmacologic antagonist that ca nnot be overcome with
•3
•4
•• increasing concentrations of agonist.
A Is not correct. 6% chose this.

·6 •
A chemical antagonist wi ll bind directly to the agonist, preventing interaction with its target or
·5

•••
rem oving it .
B is not correct. 12% chose this.
•7
A competitive antagonist is a pharmacologic antagonist that can be overcome with increasing
•8 concentration of agonist.

. ,0 •
· 9 o is not correct. 6% chose this .
. When compared with a full agonist, a partial agonist produces a reduced effect upon binding its
· 11 • receptor. It may also decrease, increase, or have the same potency as the agonist.
· 12 • E is not correct. 6% chose this.
· 13 • A physiologic antagonist is a drug that counteracts the effects of the agonist by binding to a
· 14 • different receptor and producing opposing effects.
· 15 •
. ,6 . Bottom li ne:
· 17 • An irreversible antagonist is a pharmacologic antagonist that cannot be overcome with
· 18 • increasing concentrations of agonist.

Ul;f;,·,.·1 for year: 2015


fIRST ... ID f ... C TS

D Is not correct. 6% chose this.

•••
•2 When compared with a full agonist, a partial agonist produces a reduced effect upon binding its
•3 receptor. It may also decrease, increase, or have the same potency as the agonist.
•4 E is not correct. 6% chose this.
·5
•6
•• A physiologic antagonist is a drug that counteracts the effects of the agonist by binding to a
different receptor and producing opposing effects.

•7
•8
•• Bottom li ne:

·,0 •
· 9 An irreversible antagonist is a pha rmacologic antagonist that cannot be overcome with
. increasing concentrations of agonist.

· 11 •

· 12 •
· 13 •
Ul;fi""" foryear: 2015
fIRST "'ID F... CTS

· 14 •

.16.
· 15 •

· 17 •
Receptor binding
FA15, p. 2461

.00 r --- .. -- .. -- .. -- .. --.-- ,,- --

1.
· 18 •
t
,i · t----··-----I=rf--:~ , t --·------\'--7'f-----··---

J ~
,Pad 9 2256 6010 . '

+ G R< UI usmle rx com/ap l#qmax 1 + • :


Item 90 f46
QID 2950 i
~ • M
ar
k <:l
F,e~lou$
C>-
Ne xt
Al
ta b~ un
Cf '
"jolu
~~I
Calcu lator

· 1

•2
•• A 65-year old man with a 20-year history o f type 2 diabetes presents to his physician for
reqular follow- up. He has no compla ints. Neurologic examination is significant for decreased
I ~A I

•3
•4
•• vibrat ory sense in the feet bilaterally.

·6 •
· S Which of the following is most likely defective in this patient?

•••
134415 : zheng
We valu e your feedback!
· 7 A. Goigi tendons
· 8

.,0 •
8. Meissner's corpuscles
· 9
. C. Nociceptors
· 11 •
" D. Pacin ian corpuscles
· 12 •
· 13 •
E. Thermoreceptors
· 14 •
· 15 •
. ,6 .
The correct answer Is D. 63% chose this.
· 17 •
Pacinian corpuscles are large, ovoid receptors ' -2 mm long byO.l-D.7 mm in diameter. They are
· 18 •
found primarily in deeper layers of the skin at joint capsules, serous membranes, and
mesenteries. These receptors are mainly involved in perception of pressure, coarse touch,
vibration, and tension. These receptors are often damaged in diabetic neuropathy. Several
mechan isms are involved in the pathophysiology of this condition. However, micrc)vas()JI,
disease appears to be the most prominent contributor.

•••
Pacinian corpuscles are large, ovoid receptors 1-2 mm long byO.1-0.7 mm in diameter. They are
·2
found primarily in deeper layers of the skin at joint capsules, serous membranes, and
· 3 mesenteries. These receptors are mainly involved in perception of pressure, coarse tou ch,
· 4 vibration, and tension. These receptors are often damaged in diabetic neuropathy. Several
· S
· 6
•• mechan isms are involved in the pathophysiology of this condition. However, microvascular
disease appears to be the most prominent contributor.

••
Pacinian Corpuscles Perception Touch sensation Diabetic Neuropathies
· 7
A is not correct. 4% chose this.
· 8
· 9

·,0 . • Golgi tendon organs and muscle spindles are encapsulated mechanoreceptors involved in
proprioception, not vibration.
Neurotendinous Spnd les Muscle SpIndles

· 11 •
B is not correct. 26% chose this.
· 12 •
Meissner's corpuscles are small, also encapsulated receptors found in the derma l papi llae of th e
· 13 • glabrous (hairless) skin. They are largely responsible for light discriminatory touch, not
· 14 • vibration.

.16.
· 15 •

· 17 •
Touch sensation

C is not correct. 6% chose this .


Nociceptors are receptors that are sensitive to pain (not vibration) caused by mechanical
· 18 •
stresses, extreme temperatures, and cytokines such as bradykinin and histam ine.
Bradykinin Assay Hislilmine

E is not correct. 1% chose this.


Thermoreceptors are naked endings of sma ll nonmyelinated nerve fibers that respond to
tem not vibration.
,Pad 9 2256 6010 . '

..3·2,..
+
Item 90 f46
QID 2950
G
i
R<
~ •
UI usmle rx com/ap l#qmax 1

M
ar
k

C is not correct. 6% chose this.


<:l
F,e~lou$
C>-
Ne xt
Al
ta b~ un
Cf '
"jolu
~~I
Calcu lator
+ • :

.·5
4.. Nocieeptors are receptors that are sensitive to pain (not vibration) caused by mechanical
stresses, extreme temperatures, and cytokines such as bradykinin and histamine .
Bradykin in Assay Histamine

·6
.7
.·98... E is not correct. 1% chose this.
Thermoreceptors are naked endings of sma ll nonmyelinated nerve fibers that respond to
temperature change, not vibration .

.,0 . Bottom Line:

· 11 • Mechanoreceptors in the skin are specialized for different stimuli: Pacinian corpuscles for high-
frequency vibration, Meissner's corpuscles for low-frequency vibration, and Ruffini end organ
· 12 •
and Merkel's disks for sustained pressure.
· 13 • P~cini~n Corpuscles

· 14 •
· 15 •
.,6 . ul;fii,' .. , for year: 2015
FIII.5T ""0 F"'CTS
· 17 •
· 18 • FA15, p . 452 .2
Sensory receptors
A!C£P1IJo11ITP1 D!YAIPTlOil tCKATKlN '>IN'>lS
Free nerve endings All S~iT1. cpidcTmi~> SOllle P~in. temperature

•••
A 1S-year-old girl comes to the clin ic complaining of lower abdominal pain that worsens
· 2 du ring coitus. She also reports new onset of vag inal discharge, proctitis, fevers, and chills.
·3 She uses oral contraception and has had three sexual partners. Her most recent partner is
· 4 present and complains of penile discharge. Cell lines are grown with cultures from her vagina, as
· 5
· 6
•• shown in the image.

· 7
· 8
••
·,0 •
· 9

.
· 11 •
· 12 •
. ,3 .
· 14 •
· 15 •
. ,6 .
· 17 • Image courtesy of CDC/Dr. E. Arum and Dr. N. Jacobs
· 18 •

Which of the following is a possible complication of th is pathogen?

We value your feedback!


A. Cervical adenocarcinoma
,Pad "" 2257 5910 _ '

+ G R< UI usmle rx com/apl#qmax 1 + • :


Ite m 10of46
QID 3967 i
~ • M k
ar
<:l
F,e~lou$
C>-
Ne xt
Al
tab~ un
Cf '
"jolu
~~I
Calculator

· 1

•2
•• Whic h of the following is a possible complication of t his pathogen?
•3
•4
•• A. Cervical adenocarcinoma
We value your feedback!
134415 : zheng

·6 •
·5

•••
8. Ectopic pregnancy

•7
C. I nguinal lymphadenopathy

•8 D. Menorrhagia

.,0 •
· 9
. E. Recurrent cystit is

· 11 •

· 12 •
· 13 •
The correct answer is B. 62% chose this.
· 14 •
Th is gi rl has signs and symptoms of pelvic inflammatory disease (PID), which is an ascending
· 15 •
infection affecting t he uterus and fallopian tubes, with or without involvement of the ova ries .
. ,6 . PID can leave scars and adhesions in the affected tissues, which may lead to chronic pelvic pain,
· 17 • infertility, ectopic pregnancy, and Fitz-Hugh-Curtis syndrome; these symptoms involve either
direct extension of infected material from the cul-de-sac through the peritoneum and/or
· 18 •
lymphatics or an immunologically mediated mechan ism leading to perihepatitis. It is frequently
caused by Chlamydia trachomatis, the most common genital infection in the world. More than
75% of people are asymptomatic. Chlamydia is an obligate intracellula r parasite that ex ists in
two forms: the elementary body (EB) and the reticulate body (RB). The EB is metabolically
inactive, as it lacks the abili its own adenosine tri it must be

caused by Chlamydia trachomatis, the most common genital in the world. More

•••
·2 75% of people are asymptomatic. Chlamydia is an obligate intracellular parasite that exists in
two forms : the elementary body (EB) and the reticulate body (RB). The EB is metabolically
· 3
inactive, as it lacks the ability to produce its own adenosine triphosphate; it must be
· 4 endocytosed by host cells. Once inside the endosome, it becomes the RB and uses the host's
·5
· 6
•• metabolic energy to grow and replicate. Although these bacteria are classified as gram-
negative, they are extremely difficult to sta in with traditional techniques, as they have a unique
· 7
· 8
•• outer cell wall and are primari ly located within host cells. C. trachomatis can be detected
through direct fluorescent antibody, enzyme immunoassay, or polymerase cha in reaction
testing of a sample from a cervical swab. The micrograph in the stem shows inclusion bodies

·,0 •
· 9 (the brown spots) suggestive of chlamydia l infections. Overall, the differential for PID should
. include ectopic pregnancy, appendicitis, ruptured ovarian cyst, and septic abortion.
Infertility Polymerase Chain Reacuon Per..lc Innammatory Disease Uterus Fallopian Tubes ChroniC per..IC pain of female Ectopic Pregnancy
· 11 •
FiU·Hugh-Curtls syndrome Adenosine Enzyme Immunoassay ChlalTl'jdia Infections OVarian Cysts
· 12 •
· 13 • A is not correct. 10% chose this.
· 14 • Cervical adenocarcinoma is not a complication of PIO. This neoplasm is associated with
infection by human papillomavirus types 16, 18, and 31.

.16.
· 15 •

· 17 •
CelVical Adenocarcinoma

C Is not correct. 14% chose this.


Inguinal lymphadenopathy is not a complication of PID. Inguinal lymphadenopathy is seen in
· 18 •
lymphogranu loma venereum, which is caused by C. trachomatis serovars Ll, L2, and L3.
L)IrTlphogranuloma Venereum

o is not correct. 5% chose this.


Menorrhagia describes heavy and prolonged menstruation.
,Pad "" 2257 59 10 _ '

..3·2,..
+
Ite m 10of46
QID 3967
G
i
R<
~
UI usmle rx com/apl#qmax 1

• M k
ar
<:l
F,e~lou$
C>-
Ne xt
Al
tab~ un
Cf '
"jolu

Menorrhagia describes heavy and prolonged menstruation. It can be caused by a number of


~~I
Calculator
+ • :

.·54.. underlying disorders, incl uding clotting abnormalities and uterine fibroids. It is not a
complication of PID .
Menor rhagIa Menstruation Uterine Fibroids

·6
.7
.8. . E is not correct. 9% chose this.
Recurrent cystitis, infection of t he bladder, is not a complication of PID. Recurrent cystitis is
defined as at least two infections in the past 6 months or three infections within the past yea r.
Factors that may predispose to recurrent cystitis include fi rst infection at a young age, diabetes,
incomplete bladder emptying, kidney stones, and multiple sex partners.
· 9 •
. ,0 .
Bottom li ne:
· 11 •

· 12 • PID is an ascending infection of the reproductive tract most commonly caused by C.


trachomatis. Some long-term complications associated with pro include chronic pelvic pain,
· 13 •
infertility, and ectopic pregnancy.
· 14 • Infertility Chronic pe~ic pa in of female EctopIC Pregnancy
· 15 •
. ,6 .
· 17 • I iii I jfj
fIRST AI D fACTS
, .j I.) for year: 2015
· 18 •

FA15.p.144 .1
Chlamydiae Chlamy<lia e <:;II1nol male th e ir own ,\T P T hey ChI(l"'Ys = doal (illjra<:ell nlar).
are obli gate illtracellular organisms that came Ch/(ll1lydophild psi/taci- nolable for an avian
llluco5.11 infections. 2 fonm;

•••
A 38-year-old man comes to the emergency department complaining of cyc lic fevers and
·2 headaches. The fevers began about 1 week after he returned from a trip to Africa several
· 3 weeks ago. Physical exam ination revea ls hepatosplenomegaly. Imaging of the brain shows
· 4 signs of significant cerebral involvement.
·5
· 6
•• Which of the following pa rasites most likely caused this patient's symptoms?
· 7
· 8
•• A. Plasmodium /alciparum
We value your feedback!
134415: zheng

· 9

·,0 . • B. Plasmodium knowlesi

C. Plasmodium malariae
' 11 .

· 12 • O. Plasmodium ovale
· 13 •
E. Plasmodium vivax
· 14 •

.16.
· 15 •

· 17 •
The correct answer is A. 64% chose this.
· 18 •
Th is patient has symptoms that are suggestive of ma laria given his recent travel history. There
are four members of the Plasmodium genus of protozoa that common ly infect humans. All are
transmitted through the female Anophel es mosquito. Of these four species, P. !alciparum is
known for its cerebral involvement (cerebral ma laria), which can lead to coma and death.
,Pad "" 2257 5910 _ '

+ G R< UI usmle rx com/apl#qmax 1 + • :


Ite m 11 of46
QID 2094 i
~ • M k
ar
<:l
F,e~lou$
C>-
Ne xt
Al
tab~ un
Cf '
"jolu
~~I
Calcu lator

· 1

•2
•• This patient has symptoms that are suggestive of ma laria given his recent travel history. There
are four members of the Plasmodium genus of protozoa that common ly infect humans. All are
•3
•4
•• transmitted through t he female Anophel es mosq uito. Of these four species, P. /afciparum is
known for its cerebral involvement (cerebral ma laria), which can lead to coma and death.

·6 •
Malaria Malaria, Cerebral

·5

•••
B is not correct. 5% chose this.
Plasmodium knowles; is a simian ma laria parasite that primarily infects macaques, although it
•7 has been reported to infect humans in southeast Asia. There are reports of cerebra l involvement
•8 in monkeys, and of isolated fatal human cases.

.,0 •
· 9 C is not correct. 12% chose this.
. Plasmodium malariae infection causes a 72-hour cyclic fever. It does not cause cerebral malaria.
Pel Ebsteirl fever M~I~ri~, Cerebr~ 1
' 11 .

· 12 • o is not correct. 9% chose this.


· 13 • Plasmodium ovale infection causes a 48-hour cycl ic fever. A unique feature of P. vivax and P. ovale
· 14 • organ isms is that they can form hypnozoites that can remain dormant in the liver for long
periods and cause disease later in the infected individual 's life. However, P. ovale does not cause
· 15 •
cerebral malaria .
. ,6 . Pel Ebsteirl fever M~I~ri~, Cerebr~ 1

· 17 •
E Is not correct. 10% chose this.
· 18 •
Plasmodium vivax infection causes a 48-hour cyclic fever. A unique feature of both P. vivax and P.
ovale organ isms is that they can form hypnozoites that can remain dormant in the liver for long
periods and cause disease months to years later. However, P. vivax does not cause cerebral
ma laria.

•••
Plasmodium vivax infection causes a 48-hour cyclic fever. A unique feature of both P. vivax and P.
·2
ovale organ isms is that they can form hypnozoites that can remain dormant in the liver for long
· 3 periods and cause disease months to years later. However, P. vivax does not cause cerebral
· 4 ma laria .
·5
· 6
•• Pel Ebsteirl fever M~I~ria, Cerebra l

· 7
· 8
•• Bottom Li ne:
Of the Plasmodium species that cause malaria in humans, only P. !alciparum has cerebral

·,0 •
· 9 involvement.
. Mal~r i~

' 11 .

· 12 •
· 13 •
hl;fi.·,.·1 for year: 2015
FaST "" D F... cTS
· 14 •

.16.
· 15 •

· 17 •
Protozoa- Hematologic infection5
OR<iANISM
Plasmodium
[)&AS( T~A~SMISSION

Ma laria - fcwr. hC3(bchc, 3ncmi3, l\ uoplle1e$ mO$quito


tJlA6NOSIS
Blood smcar'
TR(ATM(NT
FAI5.p . 151.1

Chloroqui nc (for
· 18 • P. vivox/ovole SpICllDrllcga l)' trophozoitc ring scnsitivc spccics),
P. foldpalum p ,·;l't/xlol't/le-48· hr cp:: lc (terti;",; fOfm wilhiu which blocks
P. rnaloliae incl ndcs fcvcr on tirst ,br ;",d RBC fl. sc hizont Plas modium hcmc
third dar. Ihus fC"cls arc actuall r polrmcrJsc; if
48 hr apart); dormant for111 rcsistant. usc
(
,Pad "" 2257 5910 _ '

+ G R< UI usmle rx com/apl#qmax 1 + • :


Ite m 12of46
QID 3892 i
~ • M k
ar
<:l
F,e~lou$
C>-
Ne xt
Al
tab~ un
Cf '
"jolu
~~I
Calculator

· 1

•2
•• A microbiology laboratory is investigating the possibility of using enzymes as a novel class of I ~A I
antibacterial. These enzymes cannot traverse the double-layer lipid membrane. I nstead, they
•3
•4
•• bind directly to the surface pepUdoglycans and hydrolyze bonds within them.

·6 •
·5 Which of the following organisms would be a logical target for this type of drug?

•••
134415 : zheng
We value your feedback!
•7 A. Brucella abortis
•8

.,0 •
8. Chlamydia trachoma
· 9
. C. Escherichia coli
· 11 •
D. Staphylococcus aureus
· 12 •
· 13 •
E. Ureaplasma urealyticum
· 14 •
· 15 •
. ,6 .
The correct answer Is D. 65% chose this.
· 17 •
Knowledge of these enzymes is not required to answer the question. It is simply testing for a
· 18 •
basic understanding of bacterial cell structure . The question states that the enzymes are able to
bind surface peptidoglycans directly from the extracellular space. Peptidoglycans make up
bacterial cell walls. In other words, the question is merely asking: "In what type of bacteria is
the cell wa ll the outermost structure?" The answer is the gram-positive bacteria, which
thick I cell wall. Of the answer choices, the only gram-positive bacterium is

i i i a

•••
basic i The question states that the enzymes are able to
·2
bind surface peptidoglycans directly from the extracellular space. Peptidoglycans make up
· 3 bacterial cell walls. In other words, the question is merely asking: "In what type of bacteria is
· 4 the cell wa ll the outermost structure?" The answer is the gram-positive bacteria, which have a
·5
· 6
•• thick peptidoglycan cell wall. Of the answer choices, the only gram-positive bacterium is
Staphylococcus aureus.

· 7
· 8
•• extr~(ellul~r

A is not correct. 4% chose this .

·,0 •
Brucella abortis is a gram-negative bacterium.
· 9

. B is not correct. 9% chose this.


Chlamydia trachoma is an intracellular pathogen, and wou ld be inaccessible to enzymes in the
· 11 •
extracellular space.
· 12 • Trachoma extracellular Space
· 13 •
C is not correct. 16% chose this.
· 14 •
Escherichia coli is a gram-negative organism. Thus it has an outer membrane surrounding a thin

.16.
· 15 •

· 17 •
peptidoglycan layer. The cell wall would not be directly accessible to an enzyme in the
extracellular space.
extracellular Space

· 18 • E is not correct. 6% chose thi s.


Ureaplasma urealyticum lacks a cell wall entirely.
,Pad "" 2258 5910 _ '

+ G R< UI usmle rx com/apl#qmax 1 + • :


QID 3892 i ar F,e~lou$
is>
Ne xt
Al
tab~ un
Cf '
"jolu
~~I
Calculator

· 1

•2
•• Escherichia coli is a gram-negative organism. Thus it has an outer mem brane surrounding a thin
peptidoglycan layer. The cell wall would not be directly accessible to an enzyme in the
•3
•4
•• ext race llular space.
Extracellular Space

·6 •
·5 E Is not correct. 6% chose this.

•••
Ureaplasma ureafyticum lacks a cell wall entirely.

•7
•8 Bottom line:

.,0 •
· 9 For a hypothetical antibiotic that hydrolyzes surface peptidoglycans, gram-positive bacteria
. would be most susceptible since their outermost structu re is a thick peptidoglycan cell wal l.
· 11 •
Intracellular bacteria, bacteria without cell wa lls, or gram-negative bacteria, whose thin
pept idoglycan cell wall is enclosed by an outer lipopolysacchari de membrane, woul d be less
· 12 •
susceptible to this mechanism of action.
· 13 •
· 14 •

· 15 • 141;1', • .,..1 for year: 2015


.,6 . FUUT "10 FACTS

· 17 •
FA1S, p. 116.2
· 18 • Cell wa lls
Com rnont obol; ~ Uniqueto
grlm-":'9"ti"

.6
.7 .. A nondiabetic man consumes a carbohydrate-rich meal. His fasting blood glucose is 80
mg/dL, and his postprandial blood glucose rises to 160 mg/dL.

··98..
·,0 .
What enzymatic characteristic enables this patient to maintain norma l postprandial glucose levels
despite ingesting a high ca rbohydrate load?
13441S ; zheng
We value your feedback!
· 11 • A. The high Km of hexokinase
· 12 •
8. The high Vmax of glucokinase
· 13 •

' 14 . C. The high Vmax of hexokinase

.16.
· 15 •

· 17 •
D. The long half-life of glucokinase

E. The low Km of glucokinase


· 18 •
. ,9 •
. 20 .
· 21 •
The correct answer is 8. 52% chose this.
After glucose is ingested in the form of a carbohydrate meal, it can only be utilized by cells once
it is phosphorylated to glucose-6-phosphate. The phosphorylation of glucose traps glucose
within cells, effectively removing it from the circulation and thereby lowering blood glucose
levels. The body uses two similar enzymes to phosphorylate glucose: hexokinase and
glucokinase. Hexokinase is an enzyme that functions in most tissues; glucokinase
exclusively in hepatocytes and pancreatic islet cells.
,Pad "" 2259 59« _ .

+ G R< UI usmle rx c om/ap l#q max1 + III :


Item ' 13 of 46
i
L • Mark --<l t:> _!
la b~
~. C';'\I
Cakul~ tor

.6
.7..
QID 1S41 Pre Ylous Next ues Notes

y p yt p

."0.
Compared with glucokinase, hexokinase has a greater affinity for glucose (a lower Kml but a
lesser enzymatic rate (a lower Vmaxl. Therefore, hexokinase can effectively bind small amounts

'8.
.,
of glucose and enhance phos phorylation, but the reaction rate is slow and the enzyme reaches
zero kinetics at a relatively low concentration of substrate. In contrast, glucokinase has a lower
affin ity for glucose (a high Kml but a greater enzymatic rate (a greater Vmaxl. Glucokinase,

.".
therefore, does not significantly function until there is a high concentration of glucose, at wh ich
time it can rapidly phosphorylate a large amount of substrate. Glucokinase therefore functions
in the hepatocyte and in the pancreatic islet cell as a glucose "se nsor," in that it is activated in
· 12 • the presence of relative hyperglycemia. It is glucokinase's high Vmax (and high Km) that enables
· 13 • the body to rapidly restore normal blood glucose levels after a high carbohydrate load.

"4 .
.16.
· 15 •
Phosphorylation Hepatocyte Hyperglycemia Glucose

A is not correct. 12% chose this .


Hexokinase has a relatively low Michaelis-Menten constant (Km) compared with glucokinase.
· 17 • C is not correct. 17% chose this.
· 18 • Compared with glucokinase, hexokinase has a low Vmax. At higher glucose levels, for example,
,'9 . after a ca rbohydrate-rich meal, hexokinase is overwhelmed (low maximum reaction rate), but
sufficient substrate is available for glucokinase to process the excess glucose load and restore
.20 . glucose homeostasis.
· 21 • Glucose

D is not correct. 3% chose this.


The half-life of glucokinase (ie, its enzymatic sta bility) is not typically discussed in its relation to
hexokinase or to glucose homeostasis.
Glucose

..67..
not correct. i

'8.
. , . The half-life of glucokinase (ie, its enzymatic sta bility) is not typi ca lly discussed in its relation to
hexokinase or to glucose homeostasis .
Glucose

E is not correct. 16% chose this .

".
" 0 •

.· 12 •
· 13 •

' 14 .
Glucokinase has a relatively high Michaelis-Menten constant (Km) compared with hexokinase.

Bottom Line:
Glucokinase has a low affin ity for glucose but a high capac ity for activity; it is most active
when glucose levels are high, helping the body handle large increases in blood glucose, for
example, after a ca rbohydrate-rich meal.
.16.
· 15 •
Glucose

.".
· 17 •
· 18 •
ul;fi"'''' for year: 2015
FaST ""0 F"'CTS

.20 .
FA15. p . 98.4
· 21 •
Hexokinase vs. Ph osphoTylalion of glu cose 10 yield glucose.6.phosphale sen'e! as Ihe lsi slCP of glycolysis (al so
glucokinase scrves as thc 1st step of glycogen synthesis in the li\"e r). Rcaclion is cat alyzed b)' eithcr hexokinase
ur gluco kin ;lse. depending un the ti$Slle. AI lu", glucose cunCC llt r;llions. hnokill;lse seq ll es ters
glu cose ill th e lissue. t\t high glucose cOllcentr;JtiollS. excess gilicose is stored in th e li'·er.
Hexokinase Glucokinase
Loca tion ~ I ost
II Slue$ except I"cr Li,·c r. Pcells of pallcre3S A. 24 Sis
and ) IIlcre.l llc cells 539 Sis

8
l ock
0
End Bloc k
,Pad "" 2259 59« _ .

+ G R< UI usmle rx c om/ap l#q max1 + III :


Item ' 14of46 L • Mark --<l t:> _! ~. C';'\I
i la b~ Cakul~ tor

.6
.7..
QID 2406 Pre Ylous Next ues Notes

IAA[

."0.
A 24-year-old woman patient presents to the physIcian with fatigue. PhYSical examination
reveals splenomegaly and signs of anemia. A peripheral blood smear demonstrates the
presence of spherocytes. The patient says that she recently fin ished a course of penicillin for
' 8 • treatment of otitis media.
. ,

.".
In the etiology of drug-induced hemolytic anemia, penicillin is believed to act as which of the
following?

· 12 • 134415 : zheng
We value your feedback!
A. Adjuvant
· 13 •

"4 .
.16.
· 15 •
B. Antibody

C. CD4+ helper epitopes

· 17 • D. Hapten
· 18 •
E. Soluble antigen
, '9 •
. 20 .
· 21 •

The correct answer is D. 70% chose this.


This is a case of penicillin-induced autoimmune hemolytic anemia . Penicillin is believed to act as
a hapten. On its own, a hapten molecule cannot elicit an immune response. However, when a
hapten molecule binds to a protein, the hapten-protei n conjugate is capable of i i
immune response. In this case, penicillin binds to proteins on the surface of

.6
.7..
."0.
This is a case of penicillin-induced autoimmune hemolytic anem ia. Penicillin is believed to act as
a hapten. On its own, a hapten molecule cannot elicit an immune res ponse. However, when a

'8.
.,
hapten molecule binds to a protein, the hapten-protein conjugate is capable of eliciting an
immune response. In this case, penicillin binds to proteins on the surface of RBCs, eliciting an
immune response and autoimmune destruction of the RBCs.lt also explains the patient's

.".
anemia.
Immune response Anem i ~ Autoimmune hemolytic ~nemi~

A is not correct. 9% chose this.


· 12 •
Penicillin is not acting as an adjuvant (nonspecific immune activator), but rather as a hapten.
· 13 •
B is not correct. 6% chose this.
' 14 .
Penicillin is not an antibody.

.16.
· 15 •
C is not correct. 5% chose this .

· 17 •
· 18 •

."
. 20 .
· 21 •
. Penicillin does not contribute (04+ helper epitopes.
E Is not correct. 10% chose this.
Pen icillin acts as a hapten, not a soluble antigen .

Bottom Line:
Penicillin is bel ieved to act as a hapten, a molecule that cannot elicit an immune response on
its own but can do so whe n protein is bound to it.
Immune response
,Pad "" 2259 59 « _ .

+ G R< UI usmle rx c om/ap l#q max1 + III :


Item ' 14of46 L • Mark --<l t:> _! ~. C';'\I
i la b~ Cakul~ tor

..67.. .
QID 2406 Pre Ylous Next ues Notes
...

."0.
Penicil lin is not acting as an adjuvant (nonspecific immune activator), but rather as a hapten .
e is not correct. 6% chose this .

.'8, . Penicillin is not an antibody.


C is not correct. 5% chose this .

".
.· 12 •
· 13 •
Penicillin does not contribute CD4+ helper epitopes.
E is not correct. 10% chose this .
Penicillin acts as a hapten, not a soluble antigen.

"4 .
.16.
· 15 •
Bottom line:
Penicillin is believed to act as a hapten, a molecule that cannot elicit an immune response on
its own but can do so when protein is bound to it .
Immune response
· 17 •
· 18 •

,'9. ul;f.,.·,.·1 for year: 2015


.20 . FI R ST ""D FA cts

· 21 •
FA15, p. 211 .1

Hype rse nsitivity types


Typ e I Anaphylactic and atopic-frce anti ge n cross-- First (type) and Fast (a nal'hylax is). Types I. [I.
lin ks [gE on prescnsitizcd m;l$t cells ami 3,,{1 II I are all 3nt ihotly mediated
hasoph ils. trigge ring illlnlC(Ji;. te rcle~sc of Test: skin test for specific IgE
,·as(};.cti\'e ;' IIlines th;.t act ;.1 pOIIl';'pil!;',y

.6
.7..
."0.
Blood and tissue samples obtained from various sites have been m islabeled in the
laboratory, and you must identify the type of cell in one sample. The cell stru cture was

'8.
. ,
disturbed such that the architecture observed on microscopy revea ls no clues; however, the
cells are rich in smooth endopla smic reticu lum. Add iti onally, a new molecular study reveal s that
this cell type is capable of reverting from the Go to G, phase of the cell cycle in vivo .

.".
· 12 •
· 13 •
The sample was most likely made up of whi ch cell type?

A. Adrenal cortical cell s


We value your feedback!
134415: zheng

' 14 . B. Gonadal cells

.16.
· 15 •

.
C. Hepatocyte

· 17 • D. Lymphocyte
· 18 •
E. Neuron
."
. 20 .
· 21 •

The correct answer is C. 67% chose this.


The function of smooth endoplasmic reticulum (SER) is primarily in steroid biosynthesis,
membrane synthesis and repair, and detoxification. Cell s that actively detoxify metabolic
byproducts, such as hepatocytes, are therefore rich in SER. Other cells with abundant
include steroid hormone-producing cells of the adrenal glands and gonads. The
,Pad "" 2259 59« _ .

+ G R< UI usmle rx c om/ap l#q max1 + III :


Item ' 1Sof46 L • Mark --<l t:> _! ~. C';'\I
i la b~ Cakul~ tor

.6
.7..
QID 3OS3 Pre Ylous Next ues Notes

p p y y

."0.
membrane synthesis and repair, and detoxification. Cells that actively detoxify metabolic
byproducts, such as hepatocytes, are therefore rich in SER. Other cell s with abundant SER

'8.
.,
include steroid hormone-producing cells of the adrenal glands and gonads. The ability to revert
from Go to G, is characteristic of stable (ie, qu iescent) cell types, which include hepatocytes and
lymphocytes. Knowledge that this cell line is stable and also rich in SER indicates it is most likely

.".
a hepatocyte.
Detoxification procedure Hepatocyte Smooth Endoplasmic Reticu lum Adrenal Glands

· 12 • A is not correct. 12% chose this.


· 13 • Adrenal cortical cells function in the production of steroid hormone and therefore are rich in

"4 .
.16.
· 15 •
SER. They are not, however, stable (quiescent) cells.
B is not correct. 12% chose this.
Gonadal cells, like adrenal cortical cells, are rich in SER but do not match the cell type
(stable/quiescent) desc ribed in the question stem, making this answer less likely than
· 17 • hepatocytes.
· 18 • Gonadal structure

,'9 . D is not correct. 5% chose this .


. 20 .
Lymphocytes are indeed stable (qu iescent) cells capable of reverting from Go to G,.
· 21 • Lymphocytes (spec ifica lly antibody-secreting plasma cells), however, are rich in rough
endoplasmic reticulum used in protein synthes is rather than SER, as given in the question stem .
Plasma (ells Rough endoplasmic reticulum

E Is not correct. 4% chose this.


Because neurons have a greater demand for
do not contain abundant

..67..
i

."0.
Lymphocytes (spec ifica lly antibody-secreting plasma cells), however, are rich in rough
endoplasmic reticulum used in protein synthes is rather than SER, as given in the question stem .

'8.
. ,
Plasma (ells Rough endoplasmic reticulum

E Is not correct. 4% chose this .

.".
Because neurons have a greater demand for peptide neurotransmitter production than for lipid
synthesis, they do not contain abundant SER. Instead, they contain large aggregations of rough
endoplasmic reticulum (RER), the site of synthesis of secretory proteins, thereby fac ilitating their
· 12 • greater demand for peptide synthes is. These aggregates of RER in a neuron are called Nissl
· 13 •
bodies.

"4 .
.16.
· 15 •
Neurons Rough endoplasmic reticulum

Bottom line:

.".
· 17 • Hepatocytes, adrenal celis, and gonadal cells are rich in SER, which is responsible for steroid
· 18 • biosynthesis, membrane synthesis, and detoxification. Hepatocytes and lymphocytes are
stable (quiescent) cell s, meaning that when stimulated they are capable of entering the G,
phase of the cell cycle from Go.
. 20 . Gonada l structure Detoxification procedure
· 21 •

h iiI'., '
FIlST ".0 F"CTS
,iI .1 for year: 2015
,Pad "" 2259 59« _ .

+ G R< UI usmle rx c om/ap l#q max1 + III :


Item' 16of 46 L • Mark --<l t:> _! ~. C';'\I
i la b~ Cakul~tor

.6
.7..
QID 18S6 PreYlous Next ues Notes

IAA[

."0.
A newborn boy IS brought to the ped iatriCian for evaluation of an extensive skin rash.
Physical examination shows that this child has microcephaly, hearing loss, and a petechial

'8.
. ,
skin rash . The abdomina l examination reveal s hepatosplenomegaly. Questioning of the
infant's mother reveal s that she had "the flu" early in her pregnancy. A tissue sample from the
infant is sent for culture, which confirms the diagnosis .

.".
· 12 •
· 13 •
Which of the following is the correct diagnosis?

" A. Congenita lly acquired cytomega lovirus


134415 : zheng

"4.
.16.
· 15 •
B. Congenita lly acqu ired Epstein·Barr virus

C. Congenita lly acquired herpes simplex virus


[ We value your feedback!

· 17 • D. Congenita lly acquired HIV


· 18 •
E. Congenita lly acquired syphilis
L ...J

, '9 •
. 20 .
· 21 •

The correct answer is A. 65% chose this.


This is a case of congenitally acquired cytomegalovirus (CMV) infection. Fetuses exposed to CMV
during the first trimester may experience intrauterine growth retardation and be afflicted with
central nervous system damage, with hearing and sight impairments. Mental retardation
occur along with microcephaly.

.6
.7..
i a case i i i

."0.
during the first trimester may experience intrauterine growth retardation and be afflicted with
central nervous system damage, with hearing and sight impairments. Mental retardation will

'8.
.,
occur along with microcephaly.
Microceph31y FeLli Growth ReLlrdation MenLlI Rewrd<ltion

".
B is not correct. 13% chose this.
Epstein-Barr vi rus, the cause of infectious mononucleosis, is a rare cause of congenital defects .
These defects include cataracts, hypotonia, cryptorch idism, and micrognath ia,
.· 12 • Muscle hypotonia Cryptorch idism Infectious Mononucleosis Bilateral cataracts (disorder)

· 13 •
C is not correct. 9% chose this.
' 14 .
Herpes simplex virus can result in a variety of congenita l defects, abortions, and neonatal

.16.
· 15 • encephalitis. It can be associated with skin rashes and microcephaly but is not commonly
associated with hearing loss,

".
Herpes Slm~ex Infections Er.c:epha lnis MlCfoceph<lfy
· 17 •
· 18 • o is not correct. 3% chose this .
Congenital HIV results in neonatal AIDS .
.. 20 . E is not correct. 10% chose this.
· 21 • Congenital syphilis can result in cran ial nerve VIII deafness, mulberry molars, saber shins,
saddle nose, and Hutchinson's teeth,
Syphilis, CongeniLlI ACOI.I5tic Nerve Deafness Moon's molar teeth Saber shin
,Pad "" 2259 59« _ .

+ G R< UI usmle rx c om/ap l#q max1 + III :


Item' 16of 46 L • Mark --<l t:> _! ~. C';'\I
i la b~ Cakul~tor

..67..
QID 18S6 PreYlous Next ues Notes

."0.
D is not correct. 3% chose this .
Congenital HIV results in neonatal AIDS .

'8.
.,
E is not correct. 10% chose this.
Congenital syphilis can result in cranial nerve VIII deafness, mulberry molars, saber shins,

.".
saddle nose, and Hutchinson's teeth.
Syphilis, Congenital Acoustic Nerve Deafness Moon's molar teeth Saber shin

· 12 •
Bottom li ne:
· 13 •

"4.
.16.
· 15 •
Congenital (MV manifests as petechia l rash, hepatosplenomega ly, and microcephaly.
Intracranial calcifications are seen on CT of the head.
Cotl9f!nital cytomegalovirus infection Microcephaly

· 17 •
· 18 • iii I ifi , .j 1.1 for year: 2015
,'9 . fIRST AID F.... CTS

.20 . FA15,p.175 .1
· 21 • ToRCHeS infection5 Microbes that ma)' pass (rom mother to (etm, Transmission is tran splacc nt;,1 in most cascs. or via
deli,'er), (especiall)' 1-15V_2). Nonsp~,<:ific signs comillon to ma n)' To RC llc5 ir&'Chons include
hepatosplenomegal)'. jaundice. thromboc)'topenia. and growth retardation.
Other important infcrtious agents include Slreplococc!JS <lg<li<lC/i<le (group B streptococci). E. coli.
and Usleri<l mOllocr logelles-a ll ca uscs of mcningitis in neonates. Paf\'o\'irns Bl9 ca uscs hydrops
fctali s.

· 17 • A 3-year-old girl presents to a pediatrician with restricted joint movement. The physician also I'" AI
notes that she has coarse facial features and skeletal abnormalities. She is diagnosed with a A
· 18 •
rare condition and is enrolled in a resea rch study, where a sma ll tissue sample is taken for
'19.
.20 .
microscopic examination. On patholog ic examination of the specimen, inclusion bodies are seen
in the cytoplasm of individual cells. These inclusion bodies stain positive for lysosomal proteins on
immunohistochemistry. On receiving the resu lts of the tissue biopsy, the physician explains to the
· 21 •
patient's mother that her daughter has a disease resulting from the aberrant intercellular ta rgeting
· 22 •
of proteins.
· 23 •

' 24 .
Which of the following biochem ical steps is lacking in patients with this disease?
.25 .
.26.
· 27 •
134415: zhen9

A. Administration of a peptidase that localizes to the trans Golgi


..3029..
' 28 •

B. Failure of targeting proteins from endocytosis to the nucleus

' 31 .
C. Phosphorylation of glucose residues at the trans Golg i
· 32 • O. Phosphorylation of mannose residues at the cis Golgi
· 33 •
E. Phosphorylation of mannose residues in the endoplasmic reticulum
.34 .
' 35 . F. Synthesis of proteins in rough endoplasm ic reticulum
,Pad "" 2301 59« _ .

+ G R< UI usmle rx com/ap l#qmaxl + III :


Item ' 17 of 46
QID 3791 i
L • Mark --<l
Pre Ylous
t:>
Next
_!
la b~ ues
~.
Notes
C';'\I
Cakul~ tor

X
· 17 I-cell disease is a syndrome in w hich a patient is unable to phosphorylate mannose resid ues on
enzymes destined for Iysosomes wit hin cis Goigi of cells. Given that the trans Goigi have a
" 8X phosphorylated mannose receptor that specifies enzymes as lysosomal, these
".X non phosphorylated enzymes are misd irected as secretory products from the cell into t he
. 20 X plasma, leading to the clin ical syndrome described here. Th is leads to t he presence of
· 21 X characteristic incl usion bodies in the cytoplasm of celis, whi ch are the result of lysosomal
proteins acting on the contents of the cytoplasm.
· 22 X M~nr.o!ie lysosomes Goigi App3,;,tus Cytopl;,sm
· 23 X
A is not correct. 4% chose this.
' 24 X
Given that the problem is one of phosphorylation in the cis Golgi, a peptidase ta rget ed to the
· 2S X trans Golgi wou ld not effectively treat t he underlying pathogenesis of this disease nor would it
. 26 X localize to the correct area of the secretory pathway.
· 27 X Ph05phoryl;,tion Goigi App<I,,,tus P;,\hogenesis

' 28 X B is not correct. 4% chose this .


. 2. X Failure of t argeting proteins from endocytosis to the nucleus would lead to a problem of
. 30 X importing proteins, not the packaging and targeting of proteins synthesized within the cel l. r-
' 31 X cell disease results from problems targeting proteins synthesized with in the cell to Iysosomes.
lysosomes
· 32 X
· 33 X C is not correct. 9% chose this.
. 34 X Phosphorylation of mannose (not glucose) residues on proteins, which targets them to the
Iysosomes, takes place on the cis (not trans) Golgi.
' 3S X Ph05phorylation Mannose Lysosomes Goigi App<lr"tus Gluo::ose

I I
Iysosomes, takes place on the cis (not trans) Golgi.
" 8X Ph05phorylation Mannose Lysosomes Goigi App<lr"tus Gluo::ose

".X E is not correct . 20% chose this .


. 20 X Phosphorylation of mannose residues on proteins, which targets them to the Iysosomes, takes
· 21 X place in the Golg i appa ratus. It does not take place in the endoplasmic reticulum.
Ph05phorylation Mannose Lysosomes Goigi App<lratus Endoplasmic RetICulum
· 22 X
· 23 X F is not correct. 2% chose this.
' 24 X The synthesis of proteins on rough endoplasmic reticu lum plays no role in the pathogenesis of
· 2S X I-cell disease.
Pall'logenes is Rough endoplasmic reticulum
. 26 X
· 27 X 1'1
' 28 X Bottom Line:
.29 X I·cell disease affects protein tagging, preventing phosphorylation of mannose at the cis Golgi
. 30 X and interfering with lysosomal storage. Restricted j oint movement is a symptom .
Phosphorylation M"nnose Goigi App<lr"tus Joint mobi lization
' 31 X
· 32 X
· 33 X
. 34 X
ul;fi.·,.·1 for year: 2015
FIIUT "10 F"CTS

' 3S X
,Pad"" 2301 59« _ .

+ G R< UI usmle rx com/ap l#qmaxl + III :


Item' 18of 46 Mark --<l t:> _! ~. C';'\I

.
L •

QID 1361 i PreYlous Next la b~ ues Notes Cakul~tor

· 17 • A 37-year-old woman requests diagnostic testing before her frozen embryos are implanted. IAA[
·,8 . The physician requests a work-up for cystic fibrosis, as both the patient and her husband are
., carriers. The laboratory isolates some of the DNA from each 8- to 10-cell embryo prior to
implantation and proceeds to run tests to determine the genotype of the embryos .
. 20 .
· 21 • Which of the following laboratory procedures is the first step performed in determining the
· 22 • genotype of the embryos?
· 23 • -------------,i
We va lue your feedback!
134415 : zheng
·24 . A. Enzyme-linked immunosorbent assay
.25. B. Ligase chain reaction
. 26 .
· 27 •

·28 .
. 2• •
C. Northern blot

D. Polymerase chain reaction


LDDil J
. 30 . E. Southern blot
· 31 .
· 32 •
. 33 .
. 34 . The correct answer is D. 57% chose this .
To determine the genotype of the embryos, the genetic material must first be isolated and
· 35. amplified. Polymerase chain reaction (peR) is a laboratory procedure used to exponentia lly
amplify DNA. Based on the primer sequences chosen, it results in the production of
copies of a desired fragment of DNA. peR may be the initial step of a variety of diilarlostic

.
· 17 •
·,8 .
.,
. 20 .
· 21 •
To determine the genotype of the embryos, the genetic material must first be isolated and
amplified. Polymerase chain reaction (peR) is a laboratory procedure used to exponentia lly
amplify DNA. Based on the primer sequences chosen, it results in the production of many
copies of a desired fragment of DNA. peR may be the initia l step of a variety of diagnostic
techniques, including Southern blot. Once the DNA is amplified, other laboratory techniques,
such as gene sequencing, may then be implemented. The main advantage of peR is speed. A
standard peR protocol typically takes only a few hours to complete, as opposed to any of the
· 22 •
"blots," which take multiple days and may even require overnight steps.
· 23 •
Po~merase Chain Reaction
·24 . A is not correct. 5% chose this .
. 25 .

.
Enzyme-linked immunosorbent assay (ELISA) is a test of antigen-antibody hybridization used to
. 26 . detect an antigenic match in a patient's blood sample. This test is used when trying to detect a
· 27 • specific antigen or antibody in a patient's sample, such as the HIV antibody. ELISA does not
·28 . yield any information about an organism's DNA sequence.
.,. Enzyme-Linked lmmunosorbent Assay

. 30 . B is not correct. 4% chose this.


· 31 . Ligase chain reaction is a technique performed on mutant alleles for the purpose of detecting
· 32 •
single-point mutations.

. 33 . C is not correct. 6% chose this .


.34 . A Northern blot is used to detect RNA expression through RNA-DNA hybrid ization. It can only
detect genes that are actively being transcribed, rather than the entire DNA sequence.
· 35.
E is not correct. 28% chose this.
A Southern blot is used to detect DNA via DNA-DNA hybridization with labeled DNA
,Pad "" 2301 59« _ .

+ G R< UI usmle rx com/ap l#qmaxl + III :


Item' 18of 46 Mark --<l t:> _! ~. C';'\I

.
L •

QID 1361 i PreYlous Next la b~ ues Notes Cakul~tor

· 17 •
C is not correct. 6% chose this.

·,8 . A Northern blot is used to detect RNA expression through RNA-DNA hybridization. It can only
., detect genes that are actively being transcribed, rather than the entire DNA sequence .
E is not correct. 28% chose this .
. 20 .
A Southern blot is used to detect DNA via DNA-DNA hybridization with labeled DNA probes. It
· 21 •
can be used to characterize a segment of DNA, but it can be performed only after amplified DNA
· 22 • has been obtained (via peR).
· 23 •

·24 . Bottom Line :


.25 . Amplification of DNA for genotyping is achieved with peR .
. 26 .
· 27 •

·28 . Iii lif, , ., I.) for year: 2015


~III.ST "' 10 FA CTS
. 2• •
.30 . FA15, p . 71.4
· 31 . Polymerase chain "'lo1ccula r hiology lahomtory procedure used to amplify 3 desired fragment of DNA. Useful as a
· 32 • reaction diagnostic tool (e.g .. neona tlll H IV. herpes encephali ti s).

.33 . Steps:
I. Denat urat ion-DNA is denatured hy hc~ting to generate 2 separate st rands.
. 34 . 2. Annealing-during cooling, exct'Ss prcmade DNA primers almeal to a specific sequence on
each st«",d to he amplified
· 35 . Elong,Jtion-he;tt-st"bl e DNA polymerase replic;ttes the DNA sc<lnence following eac h
prnner.
Th ese steps <Ire repeated mull iple ti'ncs for DNA scquellCe amplification.

.,.
. 20 .
· 21 •
A 68-year-old man presents to his primary care physician with complaints of nocturia,
difficulty urinating. and occasional urine dribblinq. Past medical history is significant for a
20-year history of diabetes mellitus and hypertension . Laboratory results from 2 weeks ago
show hemog lobin Al e (HbAld of9.7%. The physician counsels the patient on diabetes
manaqement and suspects that the patient suffers from neurogeniC bladder, a complication of
·22 •
diabetes .
· 23 •

·24 . Which of the following is an appropriate treatment for his condition?


.25 .
.26 . We value your feedback!
134415: zheng

.,..
A. Benztropine
· 27 •

·28 . B. Bethanechol

C. Carbachol
.30 .
· 31 . D. Phenylephrine
·32 • E. Propranolol
. 33 •
. 34 .
· 35 .
The correct answe r i s B. 62% chose thi s.
Bethanechol is a cholinomimetic that se lectively stimulates muscarinic receptors without
having any effect on nicotinic receptors. Unlike acetylcholine, it is not hydrolyzed by
acetylcholinesterase and therefore has a longer duration of action. It is used to treat
,Pad "" 2301 59« _ .

+ G R< UI usmle rx com/ap l#qmaxl + III :


Item'190f46 L • Mark --<l t:> _! ~. C';'\I
QID 4010 i Pre Ylous Next la b~ ues Notes Cakul~ tor

y p
having any effect on nicotinic receptors. Unlike acetylcholine, it is not hydrolyzed by
' 19 "
acetylcholinesterase and therefore has a longer duration of action. It is used to treat
· 20 "
.21 " postoperative or neurogenic ileus, as well as urinary retention caused by genera l anesthetic or
diabetic neuropathy of the bladder.

··2223 ""
6eth,IIle(hol Acetylcholinesterase Ileus Urirl3ry Retention General Anesthesia Acetylcholir.e Diabeti( NeurOp<llt1ies

A is not correct. 15% chose this.


· 24 " Benztropine is an antagonist of muscarinic cholinergic receptors located in the central nervous
· 25 " system. It is used to treat the tremors associated with Parkinson's disease. It is not indicated
· 26 " for the treatment of neurogenic bladder.

·27 "
· 28 "
6enztropir.e Neurogenic Urirl(l'Y

C is not correct. 9% chose this.


BI~dder

· 29 " Carbachol is a direct cholinomimetic that activates the ciliary muscles of the eye. It is most
commonly used for treatment of open-angle glaucoma, and is administered as eye drops. It is
· 30 "
not indicated for the treatment of neurogenic bladder.
· 31 " C~rb.lctJoj Entire cilia'Y mUSCle Eye GI~ucom~. Open-Ang le Neurogenic Urirl(l'Y BI~dder

·32 " o Is not correct. 10% chose this.


·33 "
· 34 "
Phenylephrine is an Q, -adrenergic receptor agonist used to treat nasa l decongestion, dilate
pupils, or induce vasoconstriction in the setting of hypotension and shock. It is not indicated for
· 35 " the treatment of neurogenic bladder.
Phenylephrir.e Hypotension Neurogenic Urirl(l'Y BI~dder

E is not correct. 4% chose this.


Pr<,plrar10lol is a nonselective !3-adrenergic receptor anta'lo,ni 5,t
I i i I

C~rb.lchol Entire cili~'Y muscle Eye Glaucom.l. Open-Ang le Neurogenic Urirl(l'Y Bladder
' 19 "
· 20 " o is not correct. 10% chose this .
.21 " Phenylephrine is an Q,-adrenergic receptor agon ist used to treat nasa l decongestion, dilate

··2223 ""
pupils, or induce vasoconstriction in the setting of hypotension and shock. It is not indicated for
the treatment of neurogenic bladder.
Phenylephnr.e HypotenSion Neurogenic Urirl(l'Y Bladder
· 24 "
· 25 "
E is not correct. 4% chose this.
· 26 "
Propranolol is a nonselective !3-adrenergic receptor antagonist used to treat hypertension,

·27 "
· 28 "
angina, myocardial infarction, supraventricular taChycardia, and congestive heart failure. It is
not indicated for the treatment of neurogenic bladder.
Propr~nolol Myoc~rdlal Infarction Supr~ventricul~r tachy<:~rdla Congesti~e heart failure Neurogenic Unrl(l'Y Bladder

· 29 " I~
· 30 "
Bottom line:
· 31 "

·32 " Bethanechol is a cholinomimetic that stimu lates muscarinic receptors, but has no effect on
nicotinic receptors. It is commonly used for postoperative ileus and neurogenic bladder.
·33 "
· 34 "
Bethar.echol Postoperative ileus Neurogenic Urirl(l'Y Bladder

· 35 "

I ill jfj , .j I.' for year: 2015


FIII. S T A I D FA CTS
,Pad "" 2301 59« _ .

+ G R< UI usmle rx com/ap l#qmaxl + III :


Item' 20 of 46
QID 3068 i
L • Mark --<l
Pre Ylous
t:>
Next
_!
la b~ ues
~.
Notes
C';'\I
Cakul~ tor

· 19 "
A 3·vear-old QlriiS brouqht to the pediatriCian because of a "barking" cough, fever, and IAA[
cervical Ivmphadenopathy . Results of throat culture are negative and she is diagnosed with
· 20 " vira l croup .
• 21 "

··2223 ""
Which of the following regions of the lymph node is most likely affected by this patient's viral
infection?
· 24 " 13441 5 : zheng
We value your feedback!
· 25 " A. Efferent lymphatic
· 26 " B. Follicle
·27 "
· 28 "
C. Marginal zone

· 29 " D. Medulla
· 30 "
E. Paracortex

··3233 ""
· 31 "

· 34 "
The correct answer is E. 66% chose th is.
· 35 "
The paracortex of the lymph node is the region of cortex between the follicles of the cortex and
the medulla. It contains T lymphocytes that become activated during a cell-mediated immune
response, as in a vira l infection such as croup (commonly caused by parainfluenza virus). This
area expands as the T lymphocytes activate and proliferate, resulting in adenopathy.
none of the other regions listed is specifically involved in the immune response to a i

paracortex i cortex cortex


· 19 " the medulla. It contains T lymphocytes that become activated during a cell-mediated immune
response, as in a vira l infection such as croup (commonly caused by parainfluenza virus). This
· 20 "
area expands as the T lymphocytes activate and proliferate, resulting in adenopathy. Note that
• 21 " none of the other regions listed is specifically involved in the immune response to a virus.

··2223 ""
Immune response Croup Parainfluenza

A is not correct. 3% chose this .


· 24 " Lymph enters the lymph node through afferent lymphati cs, which pierce the capsule, drain into
· 25 " the subcapsu lar space, pass through the lymph node, and recollect in the hi lum of the lymph
· 26 " node. In the hilum, lymph exits through efferent lymphatics, eventually reaching the thoracic

·27 "
· 28 "
duct before rej oining the bloodstream.
B is not correct. 14% chose this.
The follicles are within the cortex of the lymph node and house B lymphocytes. As T
· 29 "
lymphocytes are largely responsible for the host response to vira l infections, one would not
· 30 " expect to see greatly expanded follic les i n this patient.
Immune response

··3233 ""
· 31 "
C is not correct. 10% chose this.
The marginal zone is a region in the spleen between the red pulp and the white pulp that
· 34 " contains macrophages and antigen -presenting cells .
Spleen
· 35 "
D is not correct. 8% chose this.
The medu lla consists of medullary cords and sinuses . Within the cords are lymphocytes and
plasma cells, and within the sinuses are macrophages.
,Pad "" 2301 59« _ .

+ G R< UI usmle rx com/ap l#qmaxl + III :


Item' 20 of 46
QID 3068 i
L • Mark --<l
Pre Ylous
t:>
Next
_!
la b~ ues
~.
Notes
C';'\I
Cakul~ tor

Spleen
· 19 "
· 20 " D is not correct. 8% chose this.
The medulla consists of medullary cords and sinuses. Within the cords are lymphocytes and
• 21 "

··2223 ""
plasma celis, and within the sinuses are macrophages.
Plasma Cells

· 24 "
· 25 " Bottom II ne:
· 26 " Vi ral infection elicits a cell-mediated immune response in wh ich T lymphocytes are activated
·27 "
· 28 "
and proliferate. T lymphocytes are housed in the paracortex of lymph nodes.
Immur.e response lymph nodes

· 29 "
· 30 "
iii! if.., , ., I.) for year: 2015
FIII.ST AI D FAC TS

··3233 ""
· 31 "

FA15,p . 1961

Lymph node t\ 20 lymphoid organ that has nmny affcrenls. I or more cffercnts. Encap>lilalcd. wilh trabeculae.
· 34 " FUllctions are nonspecific fihration by macrophages. stordge of B and T cells. and immune
· 35 " response activation.
Fo ll icle Site of B-ceillocalization and
proliferation. In Ollter cortex. 10 follicles
:lre dense :lnd dormallt. 20 follicles h:l\'!'
pale central germinal centers ami are

A loss-of-function mutation in a particular tumor suppressor gene mapped to chromosome


· 19 " 17 is believed to give rise to a number of neoplasms in the lung, colon, and breast. This
· 20 " t umor suppressor factor inhibits cell-cycle progression and promotes DNA repair and
apoptosis in response to a number of stimul i, including the presence of DNA damage secon dary to

··2223 ""
• 21 " ultraviolet radiation, chemica ls, or oxi dative stress.

A defect in this tumor suppressor gene also leads t o which of the following conditions?
· 24 "
· 25 " 134415: zheng
We value your feedback!
A. Bi lateral renal cell carci noma
· 26 "

·27 "
· 28 "
B. Familial adenomatous polyposis

" C. Li-Fraumeni syndrome


· 29 "
· 30 " D. Multiple endocrin e neoplasia IIa

E. Retinoblastoma
·32 "
· 31 "

·33 "
· 34 "
· 35 " The correct answer is C. 58% chose this.
The tumor suppresso r gene described in this question is p53 . Germline mutation of the p53
gene leads to Li-Fraumeni syndrome, an autosomal dominant hereditary disorder t hat
predisposes patients to a number of neoplasms in the breast, brain, soft tissue, bone, a
ad renal cortex. The diagnostic criteria for Li-Fraumeni syndrome are (1) age <45 years,
,Pad "" 2301 59« _ .

+ G R< UI usmle rx com/ap l#qmaxl + III :


Item' 21 of46 L • Mark --<l t:> _! ~. C';'\I
QID 3942 i Pre Ylous Next la b~ ues Notes Cakul~ tor

pp 9 q P P
· 19 " gene leads to Li -Fraumeni syndrome, an autosomal dominant hereditary disorder that
predisposes patients to a number of neoplasms in the breast, bra in, soft tissue, bone, and
· 20 "
adrenal cortex . The diagnostic criteria for Li-Fraumeni syndrome are (1) age <45 years, (2) a first-
degree relative diagnosed with any cancer at age <45 years, and (3) another first-degree or

··2223 ""
• 21 "
second-degree relative diagnosed with any cancer at age <45 yea rs or with a sarcoma at any
age.
Mutation Sarcomil Li·Frilumen i Syndrome Breilst 50ft tissue AdrerKIl Cortex
· 24 "
· 25 " A is not correct. 8% chose this .
· 26 "
Bilateral renal cell carcinoma (aka von Hi ppel-Lindau syndrome) is an autosomal dominant

·27 "
· 28 "
hereditary disease cha racterized by cafe au lait spots, angiomatosis, hemangioblastoma,
pheochromocytoma, rena l cell carcinoma, and pancreatic cysts. It is ca used by loss of fun ction
in the tumor suppressor gene VHL located on chromosome 3.
· 29 " Hf!mangioblastoma Rf!MI Cf!1I Carcinoma Von Hippel·Undau Syndromf!

· 30 " B Is not correct. 11% chose this.


Familial adenomatous polyposis is an autosomal dominant hereditary condition t hat leads to
·32 "
· 31 "
development of hundreds to t housands of colonic polyps that ultimately develop into colorectal

·33 "
· 34 "
cancer by a relatively early age without prophylactic colectomy. Th is disease is caused by a loss
of function in t he APe gene, a tumor suppressor gene located on chromosome 5.
Colf!ctomy Adf!nomatous f>olyposis Coli Chromosomf!s, Human, Pair S
· 35 "
D is not correct. 7% chose this.
Multi ple endocrine neoplasia type Ira (aka Sipple's synd rome) is characterized by the triad of
pa rathyroi d hyperplasia, medu llary thyroid carcinoma, and pheochromocytoma. It is caused
a gain of function in the RET gene, a that encodes a tyrosine kinase

I
ext racellular signaling. The gene is locat ed on Ch ,eO rnO'OITl'
Tyrosine Extracf!lIular MLllllplf! Endocrinf! Nf!oplasla Parathyroid hyperplasIa

E is not correct. 16% chose this.


Retinoblastoma is a rapidly progressive malignant neoplasm of the retina occ urring in both a

·23 "
· 24 "
sporadic and a heritable form . It is caused by loss of function in the retinoblastoma gene (Rb), a
tumor suppressor gene located on chromosome 13. Like p53, Rb can inhibit cell-cycle
progression; however, it does not promote DNA repair or apoptosis and is not associated with
· 25 " lung, colon, and breast cancers.
Rf!tinoblastoma Apoptosis DNA Rf!p<lir Brf!ilst
· 26 "

·27 "
· 28 " Bottom Line:
r

· 29 " The tumor suppressor gene p53 is located on chromosome 17. DNA damage secondary to
· 30 " chemical insult, oxidative stress, or ultraviolet radiation will activate p53, leadi ng to arrest of
cell-cycle progression, DNA repair, and apoptosis. l oss-of-fun ctio n mut ati ons in p531ead to
· 31 "

·32 " neoplasms of t he lung, breast, and colon, as well as a condition known as Li-Fraumeni
syndrome.
·33 "
· 34 "
Apoptosis If!ad DNA Damagf! DNA Rf!p<lir lLlng Nf!oplilsms Brf!ilst U·Frilumeni Syndrome

· 35 "
· 36 " iii lif, , ., 1.1 for year: 2015
Fl"-ST AID FAC TS
,Pad 9 2302 59« _ '

+ G R< UI usmle rx c om/ap l#qmax1 + III :


Item' 22 of 46
QID 1315 i
L • Mark --<l
Pre Ylous
t:>
Next
_!
la b~ ues
~.
Notes
C';'\I
Cakul~ tor

· 19 "
A 34-year-ol d man VISits hiS pnmary care physIcia n because he has been "feeling diZZY" for IAA[
the past week. On further questioning, he reports feeling lightheaded whe n he gets out of
· 20 " bed in the morning. This morning, when he stood up from his desk, he felt faint and
subsequently "blacked out." He is in good hea lth otherwise. On reviewi ng his medications, he
• 21 "

··2223 ""
denies taking any prescription or over-the-counter med ications, but does use some herbs and
supplements, incl uding a medicinal tea he recently started drinking.

· 24 "
This tea may be causi ng the patient's symptoms: Which of the foll owing mecha nisms of action
· 25 " coul d explai n this?
· 26 "

·27 "
· 28 "
134415; zheng

A. Activation of Q1-receptors leading to decreased generati on of cAMP and protein


· 29 "
kinase A
· 30 "
8. Activation of ~2-receptors lead ing to an increase in phospholipase C and
in creased calcium release

··3233 ""
· 31 "
C. Blockade of 01-receptors leadi ng to a decrease in phospholipase C and
decreased ca lcium release
· 34 " D. Blockade of 01-receptors leadi ng to increased generation of cAMP and protei n
· 35 " kinase A
E. Blockade of ~2·receptors leading to decreased generatio n of cAMP and protein
kinase A

· 19 "
This patient is experienc ing orthostatic hypotension, defined as a decrease in systolic blood
pressure >20 mm Hg from supine to standing. On standing, compensatory mec hanisms
· 20 " norma lly increase vasc ular tone to ma int ain bra in perfusion . "Blacking out" is the patient's
• 21 " experience when these mec hanisms fail. Given that he is a young and otherwise healthy person,

··2223 ""
it is likely that t his is secondary to one of his medications as opposed to an underlying
cardiop ulmonary or neurologic problem. You need t o review two concepts to answer this
questio n: Which receptors med iate vasc ul ar smooth muscle tone, and whic h G-protein second
· 24 " messenger pathways are signaled by these receptors?
· 25 " Perfusion (procedure) Hypotension

· 26 "
A is not correct. 7% chose this.
·27 "
· 28 "
Activation of 02 (not 01) woul d lead to decreased generation of cAMP and protein kinase A
throug h t he Gi seco nd messenger pathway.
· 29 " B Is not correct. 6% chose this.
· 30 " ~2- Receptors cause vasodilation on activation. However, they work t hro ugh t he Gs pat hway, not
Gq as described here.

·32 "
· 31 "
o is not correct. 12% chose this.
·33 "
· 34 "
Th is statement summarizes the signal ing involved in Gs and Gi pathways. 01 Works through Gq,
not Gs or Gi - only Gq causes the release of calcium, w hereas Gs and Gi work through adenyl at e
cyclase intermed iates ..
· 35 " C"kium

E is not correct. 12% chose this.


Th is correctly describes the molecular events of inhibiting the Gs pathway via ~2-b l ockade.
However, since ~2 signal ing causes vasodilation, blockade of this woul d result in
,Pad 9 2302 59 « _ '

+ G R< UI usmle rx c om/ap l#qmax1 + III :


Item' 22 of 46
QID 1315 i
L • Mark --<l
Pre Ylous
t:>
Next
_!
la b~ ues
~.
Notes
C';'\I
Cakul~ tor

Th is correctly descnbes the molecular events of Inhlbltmg the Gs pathway via f32-blockade .
· 19 " However, since 132 signal ing causes vasodilation, blockade of this wou ld result in
· 20 " vasoconstriction and would not explain the patient's symptoms.

• 21 "

··2223 ""
Bottom line:
a1 Signaling causes increased vascular tone, which is opposed by vasodilation through 132
· 24 " signal ing. Q1 Works through the Gq second messenger system (involving PIP2, protein kinase
· 25 " C, and calcium).
Ca lcium
· 26 "

·27 "
· 28 "
141 ;fi , ., I.) for year: 2015
· 29 " fIRST AID FACTS

· 30 "
FA15, p. 248.1

··3233 ""
· 31 " G-protein- linked 2nd messengers
R(((PTo. G·PR01[JIj(lASS MAJOR rUNCIiONS
Sympathetic
· 34 " r vascular smooth muscle contraction. I pupillary dilator muscle
contraction (mydriasi s). I intcstinal3nd bladder sph incter Ilmsele
· 35 "
contraction
1 S),1ll1);.thctic olltAow. 1 insulin release, l lipolrsis. I pbtelct aggregation.
1 aqILe(n]$ humor production

A 60-year-old woman has an autoimmune, vesiculobullous disease that manifests as tense


· 19 " bu llae, filled with clear fluid, on normal skin. The bullae are about 2 cm in diameter, do not
· 20 " rupture eas ily, and have healed without scarring ,

• 21 "

··2223 ""
· 24 "
· 25 "
· 26 "

·27 "
· 28 "
· 29 "
· 30 "

·32 "
· 31 "

·33 "
· 34 "
E
,.l-,
· 35 "

This patient likely has antibodies that react with antigen to which of the following junction types
shown in the image?
,Pad 9 2302 59« _ '

+ G R< UI usmle rx c om/ap l#qmax1 + III :


Item' 23 of 46
QID 2557 i
L • Mark --<l
PreYlous
t:>
Next
_!
la b~ ues
~.
Notes
C';'\I
Cakul~tor

' 19 " Th is patient has bullous pemphigoid, which is differentiated from pemphigus vulgaris by the
, 20 "
absence of the Nikolsky sign (ie, bu llae do not rupture easily) and healing wit hout scarring.
Bullous pemphigoid is caused by antibodies directed against antigens present in
, 21 " hem idesmosomes (structure E). Hemidesmosomes anchor cells t o the basement membrane.
,22 " Integrin is a key structu ral component of hemidesmosomes.
Hemidfosmosomes Basemen! membrane Bul lous pempt1igo id N,kolsky sign
'23 "
, 24 " A is not correct. 6% chose this.
, 25 " The zonula occ ludens are also called tight junctions. Tight junctions prevent leakage across the
, 26 " intracellular space. They are often found toward the luminal side of epithelial cells, where they
prevent luminal contents from penetrating into the tissue. Tight junctions form a band around
, 27 "
the entire cell.
' 28 " TightJunctions Epithelia l Cel ls
, 29 "
B is not correct. 12% chose this.
, 30 "
The zonula adherens is made up of actin filaments and E-cadherin and functions to connect the
, 31 " cytoskeleta l elements of neighboring cel ls. The zonula adherens is below the zonu la occludens
'32 " and allows groups of cells to create one functiona l unit by forming a band around each cel l.
Zonula Adherens
, 33 "
, 34 " C is not correct. 14% chose this.
, 35 " The macula adherens is also known as the desmosome, or the spot-adhering junction.
Structura lly it is similar to the zonula adherens, but it also contains keratin and desmoplakin
(desmoglein). The macula adherens joins small, discrete sections of neighboring cells. The
in pemphigus vulgaris, another autoimmune blistering disorder characterized
",oem" '.'1 bullae that ruptu re easily (positive Nikolsky and leave sha llow erosions

i i i
with dried serum and crust, is directed against desmog lein 3.
' 19 " Zonula Adherens Pemphigus N'kolsky sign
, 20 "
D is not correct. 10% chose this.
, 21 "
Gap junctions allow cells to communicate with one another. They contain connexins and have a
,22 "
central channel that allows electrical signals, chemical signals, and nutrients to pass .
'23 " Gap Junctions

, 24 "
, 25 "
Bottom li ne:
, 26 "
Bullous pemphigoid is caused by antibodies directed against antigens present in
, 27 "
' 28 "
, 29 "
hemidesmosomes, which anchor cells to the basement membrane.
Hem idesmosomes Basement membrane Bul lous pemphigoid
f
, 30 "

, 31 " 141;1','·'1.' for year: 2015


~IRST "'10 ~"'CTS
'32 "
, 33 " FA15,p, 437,1
, 34 " Epitheli;1II (ell jun(tions
, 35 "
,Pad 9 2303 58« _ '

+ G R< UI usmle rx c om/ap l#qmax1 + III :


Item ' 24 of 46
QID 1579 i
L • Mark --<l
Pre Ylous
t:>
Next
_!
la b~ ues
~.
Notes
C';'\I
Cakul~ tor

· 19 "
A young boy has a metabolic disorder that sometimes causes him to go Into a ketoacldotlC IAA[
state. After performing a few tests, the physician discovers that the child is constitutively
· 20 " deficient in insulin as well as its precursor proinsulin.
• 21 "

··2223 ""
Which of the following enzymes is produced in a precursor form and is proteolytically cleaved to
form the active enzyme?
· 24 " 13441 5 : zheng
We va lue your feedback!
· 25 " A. Aldosterone
· 26 " B. Chymotrypsin
·27 "
· 28 "
C. Glycogen synthase

· 29 " D. Hexokinase
· 30 "
E. Progesterone

··3233 ""
· 31 "

· 34 "
The correct answer is B. 80% chose this.
· 35 "
The child described has type 1 diabetes mellitus due to a defect in insulin production. Proinsu lin
is synthesized in the endoplasmic reticu l um of (3 cells in the pancreas . It is packaged into
vesicles and transported to the Golgi apparatus, where it is cleaved by specific prot eases to
form the active hormone insulin. Chymotrypsinogen is secreted as a zymogen as well,
proteolytically cleaved by trypsin to produce the active enzyme chymotrypsin. (h',motrvP'

form the active hormone insulin. i is secreted as a zymogen as well, and is


· 19 " proteolytically cleaved by trypsin to produce the active enzyme chymotrypsin. Chymotrypsin
breaks down protein by cleaving peptide bonds in the small intestine.
· 20 "
Assay of Proinsul in TRYPSIN CHYMOTRYPSIN Diabetes Mellitus, Insulin-Dependent Insulin Endoplasmic Reticulum Pancreas Goigi Apparatus
• 21 "

··2223 ""
Entire sma ll intestine

A is not correct. 3% chose this.


· 24 " Aldosterone is a steroid hormone that is regu lated by upregulation or down regulation of its
corresponding DNA.
· 25 "
C is not correct. 7% chose this.
· 26 "

·27 "
· 28 "
Glycogen synthase is phosphorylated and is inhibited or dephosphorylated and activated.
Glycogen synthase is a key enzyme in glycogenesis.
D is not correct. 7% chose this.
· 29 "
Hexokinase is feedback-inhibited by its end product, glucose-6-phosphate . It is an important
· 30 " enzyme in glycolysis.
Glucose

··3233 ""
· 31 "
E Is not correct. 3% chose this.
Progesterone is a steroid hormone that is regulated by upregulation or downregulation of its
· 34 " corresponding DNA.
Progesterone
· 35 "

Bottom Li ne:
Chymotrypsin and insulin are examples of proteins that are formed as Inactive precurso

l ock End Block


,Pad 9 2303 58« _ '

+ G R< UI usmle rx c om/ap l#qmax1 + III :


Item' 24 of 46
QID 1579 i
L • Mark --<l
PreYlous
t:>
Next la b~
_! ues
~.
Notes
C';'\I
Cakul~tor

Hexokinase IS feedback-inhibited by Its end product, gl ucose-6-phosphate. It IS an Important


· 19 " enzyme in glycolysis.
· 20 " Glucose

• 21 " E Is not correct. 3% chose this.

··2223 ""
Progesterone is a steroid hormone that is regulated by upregul ation or downregulation of its
correspond ing DNA.
Pr~stE'rone
· 24 "
· 25 "
· 26 " Bottom Line:
·27 "
· 28 "
Chymotrypsin and insulin are examples of proteins that are formed as inactive precursors first
before bei ng proteolytica lly cleaved into their active forms.
CHYMOTRYPSIN Insu lin
· 29 "
· 30 "

·32 "
· 31 "
ul;f.,.·,.·1 for year: 2015
·33 "
· 34 "
FIRST ""D FActs

FA1S, p. 335.1

· 35 " Type 1 VS. type 2 diabetes mellitus


Variable Type 1 Type 2
,' DlF((f t\l1to im ll1lm c dcstruct ion off3 cel ls I rcsis tallcc to inslllin, prog rcssive pallcreati c
~-(:e ll failure

An HIV-positive, 37-year-old ma n presents to his physician because of worsen ing


· 19 " odynophagi a and oral t hru sh. His (04+ cell count is 250/~l. He is prescribed a course of
· 20 " fl uconazole and beg un on hig hly active antiretrovi ral therapy. Over the next 3 months, he
experiences intermittent nausea and diarrhea, and he notices a redistribution of fat on his body,
• 21 "

··2223 ""
predominately around his face.

Whic h of the following agent s is most likely ca using this patient's red istrib ut ion of fat?
· 24 "
· 25 " 13441S: zheng
A. Fl uconazole
· 26 "

·27 "
· 28 "
B. Nonnucleoside reverse tra nscriptase in hibitor
[ We value your feedback!

C. Nucleoside reverse transcri ptase in hibitor


· 29 "
· 30 " D. Nucleotide reverse transcriptase in hibitor

" E. Protease inhibitor


·32 "
· 31 "

·33 "
· 34 "
· 35 " The correct answer is E. 51% chose this.
Protease inhib itors, such as lopinavir-ritonavir (Karetra), amprenavir, nelfinavir, indinavir, and
saquinavir, can ca use gastrointestinal intolerance and lipOdystrophy (fat redistribution). Patie nts
experience lipoatrophy perip herally and accumulation of fat within the abdomen, breasts
wome n), and along t he cervical vertebrae, called a cervicodorsa l fat pad. Pati ents may I
,Pad 9 2303 58« _ '

+ G R< UI usmle rx c om/ap l#qmax1 + III :


Item ' 2S of 46
QID 1741 i
L • Mark --<l
Pre Ylous
t:>
Next
_!
la b~ ues
~.
Notes
C';'\I
Cakul~ tor

Protease Inhibitors, such as IOplnaVlr-ntonaVlr (Kaletra), amprenavlr, nelfinavlr, Indlnavlr, and


· 19 " saquinavir, can cause gastrointestinal intolerance and lipodystrophy (fat redistribution). Patients
· 20 " experience lipoatrophy peripherally and accumulation of fat within the abdomen, breasts (for
women), and along the cervical vertebrae, called a cervicodorsal fat pad. Patients may also
• 21 "

··2223 ""
develop hyperlipidemia. Lopinavir-ritonavir is one of the main combinations forming the basis
of highly active anti ret roviral therapy. Two nucleoside reverse transcriptase inhibitors are added
to form the backbone.
· 24 " Amprenavir Nelr.navir Indinavir Saqu inavir Lipodystrophy Hyperlipidemia Lipoatrophy Fat pad Antlretroviral Therapy. Highly Active

· 25 " A is not correct. 13% chose this.


· 26 " Fluconazole toxicity can manifest as nausea, vomiting, abdominal pain, diarrhea. In severe
·27 "
· 28 "
cases, hepatotoxicty and hypokalemia can be an issue Fluconazole is not associated with
lipodystrophy.
Fluconazole Nausea Vom iting Diarrhea Hypokalemia Lipodystrophy Abdom inal Pain
· 29 "
· 30 "
B is not correct. 10% chose this.
Nonnucleoside reverse transcriptase inhibitors such as nevi rapine, delavirdine, and efavirenz

·32 "
· 31 " cause a variety of adverse effects such as rash and hepatitis, but they do not cause
lipodystrophy.
·33 "
· 34 "
C
Nevirapine efav.reru Lipodystrophy Hepatitis

is not correct. 16% chose this.


· 35 "
Older drugs in the nucleoside reverse transcriptase inhibitor class were reported to cause
lipodystrophy in rare instances. However, the more modern nucleoside reverse transcriptase
inhibitors (including stavudine, embtricitabine, abacavir, and lamivudine) used today are known
for causing peripheral neuropathies, gastrointestinal intolerance, and pancreatitis.

Older drugs in the nucleoside reverse transcriptase inhibitor class were reported to cause
· 19 " lipodystrophy in rare instances. However, the more modern nucleoside reverse transcriptase
· 20 " inhibitors (including stavudine, embtricitabine, abacavir, and lamivudine) used today are known
for causing peripheral neuropathies, gastrointestinal intolerance, and pancreatitis.
• 21 "

··2223 ""
Lipodystrophy Stavudine abacavir Lamivudine

D Is not correct. 10% chose this.


· 24 " The nucleotide reverse transcriptase inhi bitor tenofovir is usually well tolerated. It can lead to
some gastrointestinal disturbance and is therefore recommended to be taken with meals.
· 25 "
Tenofovir
· 26 "

·27 "
· 28 "
Bottom Li ne:
Protease inhibitors can cause gastrointestinal intolerance and lipodystrophy.
· 29 "
· 30 "
Lipodystrophy u
·32 "
· 31 "

hli1',"'''' for year: 2015


·33 "
· 34 "
fIRS T ... 10 f ... C TS

FA1S. p. 1~ . 2
· 35 "
HIVtherapy Ilig!.l)' aet ivc ,m tiret rO\';ra\ th er'lpy ( I-I AART): often initi ated at the time of IIIV diagnO$is.
Stron g('!) t indi e~t ion fot paticn ts presc •• tin g with AllJS·definin g il lness. low ClJ4+ cd l counts
« 500 ce ll slmm >). or hi gh \'ir;.l luac!, Regimen consists of :; drugs tu pre\'ent rcsist,mce:
2 NRTl s auJ I of the followi ng: NN RTI or proteasc iuh ibit or or in1cgrasc inhi bitor.
,Pad 9 2304 58« _ '

+ G R< UI usmle rx c om/ap l#q max1 + III :


Item' 26 of 46
QID 2495 i
L • Mark --<l
PreYlous
t:>
Next
_!
la b~ ues
~.
Notes
C';'\I
Cakul~tor

Following the discovery of a SUSpICIOUS abandoned package on the subway, a number of


· 19 " passengers present to the emergency department with abdominal cramps, vomiting,
· 20 " shortness of breath, and generalized weakness. Physical exam ination of these patients
reveals excessive perspiration, bilateral wheezes, bradycardia, and miosis.
• 21 "

··2223 ""
· 24 "
Which of the following is the most appropriate treatment for these patients?
134415 : zheng
We value your feedback!
· 25 " A. Echothiophate
· 26 " B. Hexamethonium
·27 "
· 28 "
C. Pancuronium

· 29 " D. Pralidox ime


· 30 "
E. Pyridostigmine

··3233 ""
· 31 "

· 34 "
The correct answ er i s D. 58% chose this.
· 35 "
Th is scenario is commonly observed in cases of organophosphate poisoning.
Organophosphates lead to phosphorylation and deactivation of acetylcholinesterase, causing
an increase in acetylcholine levels and their associated cholinergic effects. The of
organophosphate poisoning can be remembered by the mnemonic DUM
Urination, Miosis, Brochospasm, Bradycardia, Excitation, Lacrimation, Sw'eating,

I I I I
Organophosphates lead to phosphorylation and of acetylcholinesterase, causing
an increase in acetylcholine levels and their associated cholinergic effects. The symptoms of
organophosphate poisoning can be remembered by the mnemonic DUMBBELSS: Dia rrhea,
Urination, Miosis, Brochospasm, Bradycardia, Excitation, Lacrimation, Sweating, and Salivation.
Pralidoxime dephosphorylates the acetylchol inesterase and reactivates it, primarily in the

·23 "
· 24 "
neuromuscular junction. Atropine, a muscarinic receptor antagon ist, is commonly added for
symptomatic relief of salivation, cramping, sweating, and wheezing.
lead Phosphorylation Acetylcholinesterase pralidoxime Neuromuscular junction Atro~ne Orgaoophosphilte poisoning Acetylcholine
· 25 "
A is not correct. 6% chose this .
· 26 "
Echothiophate is an acetylcholinesterase inhibitor; it wou ld further increase acetylcholine levels
·27 "
· 28 "
in the neuromuscu lar junction and worsen the symptoms.
Echothiophilte Acetylcholinesterase Neuromuscular Junction Acetylcholll)e

· 29 " B is not correct. 6% chose this.


· 30 " Hexamethonium is a nicotinic receptor antagonist and would be inappropriate, as it does not
interact with muscarinic receptors .

·32 "
· 31 "
C is not correct. 9% chose th i s.

·33 "
· 34 "
Pancuronium is a long-lasting nicotinic receptor antagonist. It does not act at the muscarinic
receptors responsible for the patient's sweating, salivation, cramping, and wheezing.
· 35 " E is not correct . 21 % chose this.
Pyridostigmine is an acetylcholinesterase inhibitor; it would further increase acetylcholine levels
in the neuromuscu lar junction and worsen the symptoms.
Pyfidostlgmine Acetylcholinesterase Neuromuscular junction Acetylcholine
,Pad 9 2304 58« _ '

+ G R< UI usmle rx c om/ap l#q max1 + III :


Item' 26 of 46
QID 2495 i
L • Mark --<l
Pre Ylous
t:>
Next
_!
la b~ ues
~.
Notes
C';'\I
Cakul~ tor

p
· 19 " C is not correct. 9% chose this.
· 20 " Pancuronium is a long-lasting nicotinic receptor antagonist. It does not act at the muscarinic
receptors responsible for the patient's sweating. sal ivation, cramping, and wheezing.
• 21 "

··2223 ""
E Is not correct. 21% chose this.
Pyridostigmine is an acetylcholinesterase inhibitor; it would further increase acetylcholine levels
in the neuromuscu lar junction and worsen the symptoms.
· 24 "
Py.-idostigm ine Acetylcholinester~se NeuromV5cul", Junction Acetylchol ine
· 25 "
· 26 "

·27 "
· 28 "
Bottom Line :
Organophosphate poisoning produces such symptoms as abdominal cramps, vomiting,
· 29 "
shortness of breath, generalized wea kness, excessive perspirati on, bi lateral wheezes, and I
bradycardia. Pralidox ime reverses the toxic process.
· 30 " Vomiting 8'<tdy<:<trdi<t pr<tlido~ilTl{! Qrg<tr.opl\osphate poisoning Abdom inal (,<tmps We<tkness

··3233 ""
· 31 "

141;1','·".1 for year: 2015


~III.ST A,IO FA,CTS
· 34 "
· 35 "
FA15. p. 250 .1

Cholinomimetic i!lgent$
DRUG (LiNICALAPPLKAIIDJjS AmON

Researchers are investigating the relationship between cell phone use and brain cancer. Of
· 19 " 50 brain cancer patients, 30 admitted to using a cell phone for 10 years or more. Of 400
· 20 " healthy participants in the study, 250 were found to have used a cell phone for 10 years or
more.
• 21 "

··2223 ""
· 24 "
Which of the following is an appropriate conclusion to draw from this st udy regarding cell phone
use and brain cancer?

· 25 " 134415 : zheng


A. The attributable risk of cell phone use is 1:40
· 26 "

·27 "
· 28 "
B. The relative risk of cell phone use is 3:8
We value your feedback!

C. The relative risk of cell phone use is 8:3


· 29 "
· 30 " D. The odds ratio for ce ll phone use is 9:10

E. The odds ratio for ce ll phone use is 10:9


·32 "
· 31 "

·33 "
· 34 "
· 35 " The correct answer is 0 . 48% chose this.
The clinical study described in the question stem is a case-control study. In case-control studies,
a group of participants wit h a disease (the "cases") is compa red to another group without the
disease (the "co ntrols"). Researchers then com pare the odds of one or more ex posures in
case group to those in the cont rol group, reporting th e results as odds ratios. In the
,Pad 9 2304 58« _ '

+ G R< UI usmle rx c om/ap l#q max1 + III :


Item' 27 of 46
QID 5183 i
L • Mark --<l
PreYlous
t:>
Next
_!
la b~ ues
~.
Notes
C';'\I
Cakul~tor

y q y
a group of participants with a disease (the "cases") is compared to another group without the
' 19 "
disease (the "controls"). Researchers then compare the odds of one or more exposures in the
, 20 " case group to those in the control group, reporting the results as odds ratios. In the scenario
, 21 " above, one would calculate the odds ratio as: OR = (odds in disease group) I (odds in control
,22 "
group) = [30 I (50 - 30)] I [250 I (400 - 250)] =9/ 10. Note that this calculation isn't actually
necessary to answer the question if one recognizes that the correct answer will be an odds ratio
, 23 " of <1.
, 24 "
A is not correct. 13% chose this.
, 25 "
Attributable risk is defined as the difference in rate of a condition between an exposed
, 26 " popu lation and an unexposed population. Attributable risk is not an appropriate statistic for
, 27 " use in the case-control study described above.
' 28 " B is not correct. 16% chose this.
, 29 " Relative risk is a statistic reported in cohort st udies. Relative risk is the risk of an event (or of
, 30 "
developing a disease) relative to exposu re.
C is not correct. 14% chose this.
, 31 "
Relative risk is a statistic reported in cohort studies.
'32 "
E is not correct. 9% chose this.
, 33 "
This is the inverse of the correct odds ratio.
, 34 "
, 35 "
Bottom li ne:
Case-control stud ies evaluate the odds of exposure to one or more risk factors in a
people with a disease against the corresponding odds in a group without the
>1 association between status and de'vellopm,mt old;,;eaSE

' 19 " C is not correct. 14% chose this.


, 20 " Relative risk is a statistic reported in cohort studies.
, 21 " E is not correct. 9% chose this.
,22 " This is the inverse of the correct odds ratio.
, 23 "
, 24 " Bottom li ne:
, 25 "
Case-control stud ies evaluate the odds of exposure to one or more risk factors in a group of
, 26 " people with a disease against the corresponding odds in a group without the di sease. An OR
, 27 " >1 suggests a positive association between exposure status and development of disease.

' 28 "
, 29 "
, 30 " Ul;fi'-
~IRST
" 10
'I.' for year: 2015
~"' CTS

, 31 "
fA15. p . 48.1
'32 "
, 33 "
, 34 "
, 35 "
Observational studies
SIUDrlYPf.
Cross-sectional study
.".
Collects data from a group of people to assess
frequency of disease (and rebted risk factors) at
IUASlIAlSJUAMPLE
Disease pre\<llence.
Can show rist fac tor association with disease. but
a particular point ill timc. does nol cstablish c3nsality.
Ash, "Wlud is happeningr
Case-control study Odds rat io (OR).
,Pad 9 2304 58Y0 _ '

+ G R< UI usmle rx com/ap l#Qmax1 + • :


Item ' 28 of 46
QID 4028
L

J.
• Mark -<l
PreYlDu,
t>-
NeKt
all
lab"" ues
~.
Notes
~'t
c .. ku l~tor

A 73-year-old woman IS discharged after a complicated hospita l cou rse. She was initia lly
" 9X admitted for a colon resection after a (olonoscopy discovered a ca ncerous growth. After the
' 20 X surgery, she experienced a lot of pain and had difficulty moving out of bed due to her level of
." X sedation from pain control medications. On day 7 post-s urgery, the nurse found her short of
· 22 X breath and complaini ng of pain in her right leg. A Doppler study revealed a clot in her leg and a
diagnosis of pulmonary embolism secondary to deep vein thrombosis (DVT) was made. Several
· 23 X days after discharge, the patient ret urns t o the emergency department after noti cing gross blood
' 24 X in her stool. She is co ncerned that th is may be related to her recent surgery. An astute physician
· 25 X reviews her list of medications and suspects that her current condition may be a result of a drug
interaction .
. 2. X
." X Which of the following medications is she likely taking?
' 28 X
. 29 X We value your feedback!
13441S : zheng
A. Carbamazepine
. 30 X
' 3' X B. Erythromycin
X
l
'32
C. Griseofulvin
· 33 X
· 34 X D. Phenytoin
· 35 X E. Rifampin
· 3. X

I I

" 9X Given her history of DVT, this patient is likely taking warfarin as prophylaxis. The use of warfarin
after colon resection surgery is controvers ial, but t he benefits often outweigh the risks in a
. 20 X patient who previously developed a pulmonary embolism. Additio nally, t he patient's current
." X symptom of blood in her stool is possibly a symptom of warfarin toxicity. Erythromycin is a
·22 X cytochrome P450 (CYP450) inhibit or that ca n resu lt in increased levels of medicati ons
· 23 X meta bolized by the CYP450 enzyme system. Other CYP450 inhibitors include Acute alco hol
abuse, Gemfibrozil, Ciprofl oxacin, I soniazid, Grapefruit juice, Quinidine, Amiodarone,
' 24 X Ketoconazole, Mac rolides, Sulfonamides, Ci meti dine, and Ritonavir; remember t he mnemonic
· 25 X itA cute Ge ntleman Cipped I ced Grapefruit juice Quickly And Kept Munching on Soft
. 2. X Cinnammon Rolls."
ErythromYCin Pu lmonary Embolism Alcohol abuse grapefruit allergenIC extract
." X
' 28 X A is not correct. 11% chose this •
. 29 X Ca rbamazepine is a CYP450 inducer that can result in decreased levels of drugs metabolized by
. 30 X the CYP450 system. This anticonvulsa nt can also be used for treatment of mood stabilization in
man ia, and for trigeminal neuralgia.
' 3' X Carbamazepine Trigem inal NeuralgIa
'32 X
C is not correct. 15% chose this.
· 33 X
Griseofulvin is a CYP450 inducer that can result in decreased levels of drugs met abolized by the
· 34 X CYP450 system. This drug is used for treatment of dermatophytoses by inhibiting fungal
· 35 X mitosis. This drug is usually t aken systemically and may be needed for several months before
the fungal infection is eradicated. Because of its adverse effects and need for prolonged
adm inistration, this drug is rarely prescribed.
MItosis
,Pad 9 2304 58« _ '

+ G R< UI usmle rx c om/ap l#q max1 + III :


Item' 28 of 46
QID 4028 i
L • Mark --<l
PreYlous
t:>
Next la b~
_! ues
~.
Notes
C';'\I
Cakul~tor

D is not correct. 13% chose thIs.


· 19 "
Phenytoin is a cytochrome P450 inducer that can result in decreased levels of drugs metabolized
· 20 " by the CYP450 system . Phenytoin is a first-line medication for patients with epilepsy, especially
tonic-clon ic and partial seizures. Toxicity results in gingival hyperplasia, folate deficiency, and
• 21 "

··2223 ""
central nervous system depression. In newborns with mothers taking this medication, it may
cause fetal hydantoin syndrome, characterized by mental and growth deficiencies, and
congenita l malformations.
· 24 " Ph<:onytoin Epilepsy Gingival Hyperplasia FQlale ~tal hydar'ltOir'l sylldrome
· 25 "
E is not correct. 12% chose this.
· 26 "
Rifampin is a CYP450 inducer that can result in decreased levels of drugs metabolized by the
·27 "
· 28 "
CYP450 system, This drug is often used to treat tuberculosis by suppressing RNA synthesis. One
of the major adverse effects of this drug is to turn tears, sweat. and urine orange. Patients
should be informed of this pOSSibility before starting them on the drug, to prevent alarm.
· 29 "
lUfampir'l Tuberculosis
· 30 "

·32 "
· 31 "
Bottom II ne:

·33 "
· 34 "
CYP450 inhibitors often interact with warfarin and oral contraceptive pills, increasing the
concentrations of these medications in the bloodstream, The mnemonic "A cute Ge ntleman
Cipped I ced Grapefruit juice Quickly And Kept Munching on Soft Cinammon Rolls" wi ll help
· 35 "
you remember the CYP450 inhibitors (Acute alcohol abuse, Gemfibrozil, Ciprofioxacin,
I soniazid, Grapefruit juice, Quinidine, Amiodarone, Ketoconazole, Macrolides, Sulfonamides,
Cimetidine, and Ritonavi r).
grapefruit allerger'lic extract Alcohol abuse

should be informed of this possibility before starting them on the drug, to prevent alarm.
· 19 " f(jfampin Tuberculosis
· 20 "

• 21 "

··2223 ""
Bottom Line:
CYP450 inhibitors often interact with warfarin and oral contraceptive pills, increasing the
concentrations of these medications in the bloodstream, The mnemonic "A cute Ge ntleman
· 24 " Cipped I ced Grapefruit juice Quickly And Kept Munching on Soft Cinammon Rolls" will help
· 25 " you remember the CYP450 inhibitors (Acute alcohol abuse, Gemfibrozil, Ciprofloxacin,
· 26 " I soniazid, Grapefruit juice, Quinidine, Amiodarone, Ketoconazole, Macrolides, Sulfonamides,

·27 "
· 28 "
Cimetidine, and Ritonavir).
grapefruit allergenic extract Alcohol abuse

· 29 "
· 30 " I ill ;fi
fIRST AID fAC TS
, .jI.) for year: 2015

··3233 ""
· 31 "

FAtS, p. 260.3
Cytoch rome P-4S0 Inducers (+) Substrates Inh ibitors H
· 34 " interactio n s (select ed) Chronic alco hol nse ,\ nl i.cpilcplies Acule alcohol abuse
· 35 " St. John's worl T heoph )'lline Ritonal'ir
I'hell )'toin Warfarin Amiooarone
Phcllobarbilal DCl's C imcti<i inc
Ketocon azole
,Pad 9 23 0 5 58Y0 _ '

+ G R< UI usmle rx com/ap l#Qmax1 + • :


Item ' 29 of 46
QID 3351
L

J.
• Mark -<l
PreYlDu,
t>-
NeKt
all
lab"" ues
~.
Notes
~'t
c .. ku l~tor

A male Infant was born With an abnormally larg e craniu m, small law, an d low· set ears . An
" 9X image of the patient is shown. The child subseque ntly dies at 9 months of age .
' 20 X
." X
· 22 X
· 23 X
' 24 X
· 25 X
. 2. X
." X
' 28 X
. 29 X
. 30 X
' 3' X
'32 X
· 33 X
· 34 X
· 35 X
· 3. X

" 9X
. 20 X A. 46 XV, deletion at Sp
We value your feedback! I 134415 : zheng

." X

DDil j
·22 X B. 46 XV, microdeletion at 22q11
· 23 X C. 47 XV, trisomy 13
' 24 X
D. 47 XV, trisomy 18
· 25 X
. 2. X E. 47 XV, trisomy 21
." X
' 28 X
. 29 X
. 30 X The correct answer is O. 59% chose this .
' 3' X This infant was born with trisomy 18, also known as Edwards' syndrome. Infants born with t his
disorder exhibit severe intellectua l disability, rocker-bottom feet, low-set ears, micrognathia
'32 X
(small j aw), congenita l heart disease, clenched hands with overlapping fingers (as shown in the
· 33 X image), and a prominent occi put. These babies rarely survive past th e age of 1 year.
· 34 X Complete tr isomy 18 syndrome Disability Vertical talus, congen ital Low set ears Jaw Congenital heart disease

· 35 X A is not correct. 5% chose this.


A Sp deletion is cha racteristic of cri-du-c hat syndrome, which causes microcephaly, severe
intell ectual disability, hig h-pitched crying, epicanthal folds, and cardiac abnorma lities. Persons
with this disorder genera lly have a norma l lifespan.
,Pad 9 2305 58« _ '

+ G R< UI usmle rx c om/ap l#q max1 + III :


Item' 29 of 46
QID 3351 i
L • Mark --<l
Pre Ylous
t:>
Next
_!
la b~ ues
~.
Notes
C';'\I
Cakul~ tor
~ . : I.' •
disorder exhibit severe intellectual disability, rocker-bottom feet, low-set ears, micrognathia
· 19 "
(small j aw), congenita l heart disease, clenched hands with overlapping fingers (as shown in the
· 20 " image), and a prominent OCCiput. These babies rarely survive past the age of 1 year.
Complete tr isomy 18 syndrome Disability Vertical talus, congen ital low set ears Jaw Congenital hear! d,sease
• 21 "

··2223 ""
· 24 "
A is not correct. 5% chose this.
A Sp deletion is cha racteristic of cri-du-chat syndrome, which causes microcephaly, severe
intell ectual disability, high-pitched crying, epicanthal folds, and cardiac abnormalities. Persons
· 25 " with this disorder genera lly have a norma l lifespan.
Microcephaly Disability
· 26 "

·27 "
· 28 "
B is not correct. 7% chose this.
The 22q11 synd rome, also known as DiGeorge's syndrome, results from the abnormal
development of the third and fourth pharyngeal pouches. The principle defects can be
· 29 "
remembered by the mnemonic CATCH·22, and include Cleft palate, Abnorma l facies, Thymic
· 30 " aplasia, Cardiac defects, Hypocalcemia, and deletion of chromosome 22. Those born without
fatal heart defects usually survive to adulthood.

·32 "
· 31 "
ChrOm05omes. HUmiln. Pair 22 Congen ital Hean Defects

·33 "
· 34 "
C is not correct. 25% chose this.
Patau's syndrome, or trisomy 13, is a rare cause of severe intellectual disability. Infants present
· 35 " with congenital anoma lies such as microophthalmia, microcephaly, cleft lip/palate, abnormal
forebrain structu res, polydactyly, and congenital heart disease. These infants rarely survive past
the age of 1 year.
Microcephaly Polydactyly Disabil ity Congenital musculoskeleta l anomillies Cleft Lip Congenital hean disease

Patau's syndrome, or trisomy 13, is a rare cause of severe intellectual disability. Infants present
" 9 "
with congenital anomalies such as microophthalmia, microcephaly, cleft lip/palate, abnormal
· 20 " forebrain structures, polydactyly, and congenital heart disease. These infants rarely survive past
the age of 1 year.

··2223 ""
• 21 " Microcephaly Polydactyly Disabil ity Congenital mustulOSkeletal anomalies Cleft Lip Congenital hean disease

E is not correct. 4% chose this.


Down syndrome, or trisomy 21, is the most common chromosomal disorder and the leading
· 24 "
cause of congenita l intellectual disability. Children born with trisomy 21 may present with
· 25 " intellectual di sability, flat facial profiles, prominent epicanthal folds, simian palmar crease,
· 26 " duodenal atresia, or heart defects. The average age of death for a person with Down syndrome

·27 "
· 28 "
is 50-55 years.
Down Syndrome Disability Duodenal atresia CongenItal Hean Defects

· 29 "
· 30 " Bottom line:

· 31 " Abnormally large cranium, sma ll jaw, low-set ea rs, and clenched hands with overlapping
·32 " fingers are found in trisomy 18 (Edwards' syndrome).

·33 "
· 34 "
Jaw low set ears Com~ete trosomy 18 syndrome

· 35 "
· 36 "
iiI! ifi I ,' 1.1 for year: 2015
flll.ST "1 0 f"CTS
,Pad 9 2305 58« _ '

+ G R< UI usmle rx c om/ap l#q max1 + III :


Item' 30 of 46
QID 3104 i
L • Mark --<l
Pre Ylous
t:>
Next la b~
_! ues
~.
Notes
C';'\I
Cakul~ tor

A 47-year-ol d man presents to hiS phys Icia n com plaining of a painless "bulge" In hiS grom.
· 19 " He says the bulge first appeared several months ago and has remained pain less. It appears
· 20 " with stra ining, such as lifting heavy boxes at work, and spontaneously reduces; he has never
needed to manually reduce it. Physical examination demonstrates a swelling that appears above
• 21 "

··2223 ""
the inguinal ligament with straining and spontaneously reduces . The patient is diagnosed with a
hernia.

· 24 "
In which of the fo ll owing areas does the hernia most li kely protrude in relatio n to normal anatomic
· 25 " landmarks?
· 26 "

·27 "
· 28 "
A. Below and lateral to the pubic tubercle
13441S; zheng

We value your feedback!


8. Lateral to t he inferior epigastric artery and vein
· 29 "
· 30 " C. Medial to the femoral vein

·32 "
· 31 " D. Media l to the inferior epigastric artery and vein

·33 "
· 34 "
E. Media l to the rectus abdominis muscle

· 35 "

The correct answer is D. 61% chose this.


Th is man has the classic presentation of a direct inguinal hernia : He has an easily
mass that protrudes with effort and disappears at rest, and he has a job that reclui lres

Th is man has the classic presentation of a direct inguinal hernia : He has an easily reducib le
· 19 " mass that protrudes with effort and disappears at rest, and he has a job that requires heavy
lifting. Additionally, indirect ing uinal hernias are more common as congenital defects. Direct
· 20 "
ingui nal hernias are protrus ions of the peritoneum thro ugh Hesselbach's triangle, which is
• 21 " bounded by the inferior epigastric artery lat eral ly, the lat eral border of the rect us abdominis

··2223 ""
muscle medially, and the inguinal ligament inferiorly. Consequently, it protrudes med ial to the
inferior epigastric artery and vein. Remember, "MDs don't LIe" (M edial- Direct and Lateral-
I ndirect).
· 24 "
InguirJ<l1 Herniil. Direct Rectus Abdominis InguirJ<lllig<lment
· 25 "
A is not correct. 4% chose this.
· 26 "

·27 "
· 28 "
Femoral hernias protrude below and lateral to the pubic tubercle. Femoral hernias are more
common in women and are dangero us because they are prone to strangulation.
Hernlil. Femorill
· 29 "
B is not correct. 25% chose this.
· 30 "
An indirect inguinal hernia enters the interna l ing uinal ring and goes through the external
ingu inal ring. Thus a hernia that involves passage of peritoneal contents into the scrotum is
·32 "
· 31 "
usually an indirect hernia. However, indi r ect hern ias may present simi larly as abdominal bulges
·33 "
· 34 "
with no scrotal mass. Indirect hernias are not usually easily reducible on presentatio n and are
more common in young ma les, es pecially infants. Altho ugh it can be difficult to differentiate
between direct and ind irect inguinal hernias based on presentation, both require surgical
· 35 " management if symptomatic, so it is not necessary to make this distinction in the cli nic.
Indirect hernias protrude lateral to t he inferior epigastric artery.
Herniil Scrotum
,Pad 9 2305 58« _ '

+ G R< UI usmle rx c om/ap l#q max1 + III :


Item' 30 of 46
QID 3104 i
L • Mark --<l
Pre Ylous
t:>
Next
_!
la b~ ues
~.
Notes
C';'\I
Cakul~ tor

· 19 " Femoral hernias protrude medial to the femoral vein. They occu r more frequently in females.
Hernia, Femoral
· 20 "
E Is not correct. 3% chose this.
• 21 "

··2223 ""
There are no hernias that protrude medial to the rectus abdominis muscle.
Rectus Abdominis

· 24 "
· 25 " Bottom Line:
· 26 " Direct inguinal hernias protrude media l to the inferior epigastric artery and vein within
·27 "
· 28 "
Hesselbach'$ triangle. Indirect inguina l hernias protrude lateral to the inferior epigastric artery
and vein.
InguiJlilI Hem;... Dire't
· 29 "
· 30 "

h!;fj,·, ... for year:


·32 "
· 31 "
fI RS T ""D F... CTS
2015

·33 "
· 34 "
FA15. p. 352.1

Hernias A prOlrtlsioll of peritonellm through 31l opening, lI Slmll )' a site of \\"eaknt'S~"
· 35 "
Dia phragmatic he rnia Ahdominal ~truct mes cllter the thorJx; Sliding hiatal hernia is most C()rllm()11.
ma), occ ur due to congcnital defect ()f C;lstrocsophageal junction is displaced
H

plellroperitone;llmcml>T<mC, ur ;lS;1result uf upw;lT<l; ""huurglass stom;lch .


traum a. Commonl), OCC llrs ou left si\le due to Pa raesophageal hernia -g,lStroesopll;lgea l
rel3 ti\'c prutcctiou of ri ght hCllIidiaphragm hl' 1'11111111S prutTlldcs

A 6·month·old child develops failure to thrive, developmental delay, and recurrent


· 19 " respiratory tract infections . The child is diagnosed with an autosomal recessive disease
· 20 " ca used by a deficiency in N·acetylglucosamine phosphotransferase, which phosphorylates
mannose reSidues "
• 21 "

··2223 ""
· 24 "
What happens to enzymes produced by th is child's cells?
13441S : zheng
· 25 " A. Destruction by the proteasome
· 26 " B. Exocytosis
·27 "
· 28 "
C. Expression at the cell surface

· 29 " D. Retrograde transport to the end oplasmic reticulum


· 30 "
E. Targeting to the lysosome
• 31 "

··3233 ""
· 34 "
We value your feedback!

· 35 "
,Pad 9 2305 58« _ '

+ G R< UI usmle rx c om/ap l#q max1 + III :


Item '31of46 L • Mark --<l t:> _! ~. C';'\I
QID 4430 i Pre Ylous Next la b~ ues Notes Cakul~ tor

· 19 " Th is child has I-cell disease. Failure to phosphorylate mannose residues to mannose-6-
· 20 " phosphate results in eXQcytosis of enzymes that should be targeted to the lysosome. This
results in many intracellular inclusions that are not properly degraded by lysosomal enzymes.
• 21 "

··2223 ""
Mannose

A is not correct. 15% chose th is.


Proteins that undergo ubiquitination are targeted for proteasomal destruction.
· 24 "
C is not correct. 9% chose this.
· 25 "
N-terminal signal sequences and internal membrane-anchor sequences direct proteins for
· 26 "
membrane insertion.
·27 "
· 28 "
D Is not correct. 12% chose this.
Retrograde transport to the endoplasmic reticulum is mediated by coat protein complex I
· 29 " vesicular trafficking protein .
· 30 " Endoplasmic Reticulum

• 31 " E Is not correct. 20% chose this.


·32 " If N-acetylglucosamine phosphotransferase was not deficient, enzymes wou ld be properly
·33 "
· 34 "
targeted to the lysosome.

· 35 " Bottom Line:


I-cell disease results from failure to phosphorylate mannose residues and default exocytosis of
the protein product, rather than proper targeting to the lysosome.

Endoplasmic Reticulum

E is not corre ct . 20% chose this.


If N-acetylglucosamine phosphotransferase was not deficient, enzymes wou ld be properly
targeted to the lysosome.
·23 "
· 24 "
Bottom Line:
· 25 "
I-cell disease results from failure to phosphorylate mannose residues and default exocytosis of
· 26 "
the protein product, rather than proper targeting to the lysosome.
·27 "
· 28 "
Mannese

· 29 "
· 30 " I ill jfj , .j I.' for year: 2015
FIII.S T AI D FACTS
• 31 "

··3233 ""
FAtS.p_ 73. '
Cell trafficking Goigi is the distribntion cen ter for protei ns and lipids from the Fo R to the vesicles alld plasma
n1(:lnbr;II1C. Modifies N-oligos;lCcharidcs on ;Ispa ragine. Adds O-oligosacch~rides on serine and
· 34 "
thrt"Oninc. Adds mannose-6-phosphate to proteins for trafficking to Iysosomes
· 35 " Endosollies arc sorting centers for material from outside the cell or from thl'" Golgi. sl'"nding it to
I)'sosomes for dl'"struclion or bac k to the mellibranl'"/Goigi for further usc

I-ce ll disease (inclusion cell disease)- inherited lysosomal storage disorder: defecl in
N-'ICl'"I)'lglucosamin)'I-I-phospholransfl'"rdsc - failurl'" of the Golgi 10 1 ,I, I 1II:lllnosc
residues (i.e .. I mallilose-(,-phosphate) Oil d·
t dI I tI i
,Pad 9 2305 58« _ '

+ G R< UI usmle rx c om/ap l#q max1 + III :


Item' 32 of 46
QID 1934 i
L • Mark --<l
Pre Ylous
t:>
Next
_!
la b~ ues
~.
Notes
C';'\I
Cakul~ tor

· 19 "
The plasma conce ntration of drug Y IS 50 mg/L, and It IS eliminated at a rate of 2 mg/mln. IAA[
· 20 "
Which of the following is the cleara nce rat e of drug Y?
• 21 "

··2223 ""
134415 : zhe ng
We value your feedback!
X A. 0.04 Urnin

DDil
· 24 " B. 4 Umin
· 25 "
c. 10 Urnin
· 26 "

·27 "
· 28 "
0 .400 Urnin
~

· 29 "
· 30 "
The correct answer is A. 73% chose this.

··3233 ""
· 31 " Clearance is ca lculat ed as: (rate of drug elimination I plasma drug co ncentration) = (2 mg/min) I
(50 mg/l) =0.04 U rn in. If you forget t he exact form ula for ca lculati ng clearance, yo u ca n use unit
analysis to work backward. You may reca ll that cl earance is typica lly expressed in volume per
· 34 " time (Umi n in this exam ple). Concentrati ons are typically expressed in mass per vol ume (mg/L
· 35 " in this example) and elimi nation rates can be expressed as mass per time (mg/min in this
example.) Using a bit of scrap paper you can jot down the fo ll owi ng formu la: (volume / t ime) =
(mass / t ime)? (mass / volume), where ? is the unknown function. Sim ple algebra and
substitutio n of th e appropriate variables. will revea l that ? is the division function and t hat
clearance = (el imination rat e) / (plasma co nce ntratio n). Beware that some questions may
extraneo us or variables or answer cho ices w ith units that are either in t he

I
example.) i a bit of sc rap paper you can jot down the fol lo,vinq
(mass / time)? (mass / volume), where ? is the unknown function. Sim ple algebra and
substitutio n of t he appropriate variables will revea l that ? is the division function and t hat
cl earance = (elimination rate) / (plasma co nce ntratio n). Beware that so me questions may incl ude
extraneo us or dist racting variables or answer choices with units that are either in t he wrong

·23 "
· 24 "
scale (ie, m L instead of L) or wrong type entirely (ie, mmol instead of mg). Stude nts shoul d be
aware genera lly of the definiti on of clea rance to avoid making these mistakes.
B is not correct. 13% chose this.
· 25 "
Ca lculation error.
· 26 "
e is not correct. 12% chose this.
·27 "
· 28 "
Ca lculati on error.
D is not correct. 2% chose this.
· 29 "
Ca lculation error.
· 30 "

·32 "
· 31 " Bottom line:

·33 "
· 34 "
Clearance = (rate of drug elimination) / (plasma drug concentrati on).

· 35 "
ul;fi.·,.·1
FaST -,.0 F-'CTS
for year: 2015
,Pad 9 2305 58« _ '

+ G R< UI usmle rx c om/ap l#q max1 + III :


Item ' 33 of 46
QID 3177 i
L • Mark --<l
Pre Ylous
t:>
Next
_!
la b~ ues
~.
Notes
C';'\I
Cakul~ tor

· 19 "
Many tissues In the body, particula rly smooth and cardiac muscle, use a cell structure IAA[
constructed of proteins called connexins, illustrated in the image. These structures allow
· 20 " rapid communication of internal chemical and electrical states between celis, enabling
physiological functions such as parturition and the cellu lar response to environmenta l stress .
• 21 "

··2223 ""
· 24 "
· 25 "
· 26 "

·27 "
· 28 "
· 29 "
· 30 "

·32 "
· 31 "

·33 "
· 34 "
E

· 35 "
,..l.,

Which of the following labels corresponds to this cell structure?

· 19 " o corresponds to a gap junction, the cel l structure described in the stem. Each gap junction is
like a tiny pore between two cells. They are composed of connexins, which combine to form
· 20 "
connexons. Two connexons combine to form the gap junction. Gap junctions are found in nearly
• 21 " every cell in the body, with some except ions being skeletal muscle, RBCs, and lymphocytes. Gap

··2223 ""
junctions are particularly important in the myocardium because they allow synChronous
depolarization of the cardiac myocytes.
GapJuflCtions Myocardium Skeleta l muscle structure Myocytes. Card iac
· 24 "
· 25 " A is not correct. 6% chose this.
· 26 " A corresponds to the zonula occludens, also known as a tight junction. This structure prevents

·27 "
· 28 "
diffusion across intracellular spaces. The distal convoluted tubule of the nephron is a prime
example of the effect of tight junctions, where diffusion is limited.
TightJunclions

· 29 "
B is not correct. 14% chose this.
· 30 "
B corresponds to the zonula adherens, also known as an intermediate junction. These are
composed of cadherins and actin filaments and act as an additional, but mutable, barrier just
·32 "
· 31 "
below the zonula occlud ens.

·33 "
· 34 "
Zonula Adherens

C is not correct. 10% chose this.


· 35 " C corresponds to the macula adherens, also known as a desmosome. These are small, discrete
sites of attachment between cells, composed of keratin filaments and desmoplakin. Pemphigus
vulgaris is an autoimmune condition cha racterized by autoantibodies to desmosomes, resulting
in blistering of the epithelium.
Oesmosomes Epithe lium Pemptl igus
,Pad 9 2306 58« _ '

+ G R< UI usmle rx c om/ap l#q max1 + III :


Item ' 33 of 46
QID 3177 i
L • Mark --<l
Pre Ylous
t:>
Next
_!
la b~ ues
~.
Notes
C';'\I
Cakul~ tor

C corresponds to the macula adherens, also known as a desmosome. These are small, discrete
· 19 " sites of attachment between cells, composed of keratin fi laments and desmoplakin. Pemphigus
· 20 " vulgaris is an autoimmune condition characterized by autoantibodies to desmosomes, resulting
in blistering of the epithelium.
• 21 "

··2223 ""
OemlOsomes Eplthe lH.!m Pemph igus

E is not correct. 3% chose this.


E corresponds to the hemidesmosome. This structure, comprised in part of integrins, connects
· 24 "
celis to the underlying extracellular matrix. Bu llous pemphigoid is an autoimmune condition
· 25 " characterized by autoantibod ies to hemidesmosomes, resulting in blisters below the skin that
· 26 " may also include mucus membranes.
·27 "
· 28 "
extr~(ellul~r Hemidesmosomes Bullous pemphigoid

· 29 " Bottom Line:


· 30 "
Gap junctions are composed of connexons and allow for rapid (and therefore almost
synchronous) depolarization of cardiac myocytes in heart beats.

··3233 ""
· 31 "
G~pJunc(lOJlS Myocytes. Card i ~(

· 34 "
· 35 " Ul;f;,·,.·1 for year: 2015
fIRST ... ID f ... CTS

FA15,p . 437 .1

Epithelial cell junctions

· 26 " A 22-year-old man presents to the emergency room compla ining of a spreading rash after

·27 "
· 28 "
several days of high fevers, headaches (9 of 10 in severity), and myalgia. He recently returned
from a Peace Corps trip to Rwanda. He first noticed the rash on his chest, but it has now
spread to hi s arms and legs. Hi s temperature is 38°C (100.4°F), pulse is 88/min, respiratory rate is
· 29 " 20/min, and blood pressure is 125/87 mm Hg. Physical examination reveals erythematous
· 30 "
maculopapular eruptions on his chest, forearms, and thighs. A Wei I-Felix reaction is performed and
is positive for OX -19 , but negative for OX-2 and OX-K .
• 31 "

··3233 ""
· 34 "
Which of the following is the most appropriate treatment?

We va lue your feedback!


134415 ; zheng
A. Ceftriaxone
· 35 "
· 36 " B. Doxycycline
·37 "
· 38 "
c. Nothing; the cause is a viru s

· 39 " D. Penicillin
· 40 " E. Vancomycin
• 41 "
· 42 "
· 43 "

·44 " The correct answer is B. 58% chose this.


Th is patient presents with the classic triad of rickettsial illness (headache, fever, and
caused in this case by the gram-negative organism Rickettsia prowazekii (epidemic
,Pad 9 2306 58« _ '

+ G R< UI usmle rx c om/ap l#q max1 + III :


Item ' 34 of 46
QID 203 1 i
L • Mark --<l
Pre Ylous
t:>
Next
_!
la b~ ues
~.
Notes
C';'\I
Cakul~ tor

Th is patient presents With the classIc tnad of ri ckettsi al Iii ness (headache, fever, and rash)
· 26 " caused in this case by the gram-negative organism Rickettsia prowQzekii (epidemic typhus). The

··2728 ""
rash of typhus differs from that of Roc ky Mountain spotted fever because it starts on the trunk
and moves outward, ratherthan starti nq on the extremities and moving inward. Nevertheless,

·29"
· 30 "
the treatment of choice for all ric kettsia l illnesses is doxycycline .
Headache TYPHUS Doxycycline Typhus. EpIdemic Louse·Bome Rocky Mounta in Spotted Fever

A is not correct. 11% chose this.

··3233 ""
• 31 " Ceftriaxone is a t hird-generation cephalosporin used most commonly to treat Neisseria
gonorrhoeae infecti on.
Ceftrl<lKOne
· 34 "
C is not correct. 19% chose this.
· 35 "
The Wei l-Fel ix reaction confirms the bact erial eti ology, and antibiotic treatment sho uld be
· 36 " initiated early.
·37 "
· 38 "
D is not correct. 8% chose this.
Penicil lin, an inhibitor of cell wall synthesis, is the treatment of choice for syph ilis, caused by the
· 39 " spirochete Treponema palfidum.
· 40 " E is not correct. 4% chose this.

• 41 "
Vancomycin works by inhibiting cell wall synthesis, but is effective only aga inst gram-positive
· 42 "
organ isms.
· 43 "

·44 " Bottom line:


The rash of Rickettsia typhus, or endemic typhus, characteristica lly begins centra lly and
out; it is best treated with tetracyclines .

· 26 " E Is not correct. 4% chose this.


·27 "
· 28 "
Vancomycin works by inhibiting cell wall synthesis, but is effective only aga inst gram-positive
organ isms.
· 29 "
· 30 " Bottom line:
The rash of Rickettsia typhus, or endemic typhus, characteristically begins centra lly and spreads

··3233 ""
• 31 "
out; it is best treated with tetracyclines.
TYPHUS

· 34 "

· 35 "
· 36 "
ul;f.,.·,.·1
cts
for year: 2015

·37 "
fiRST ... ID f ...

· 38 " FA1S, p . 185.'


Tetracyclines Tctracycline. dOK),e),elinc, mi llocycline.
· 39 "
M((HAHISoII Bacteriostatic; bind to 30S and prevent attachment of aminoacyl-t RNA; limited CNS penetration.
· 40 " [)Qx),c),clinc is fccall )' climin atcd 3nd can be uscd in paticnts with rcnal fa ilu re. Do not take
tetrac),clines with milk (Ca !~), antacids (Ca! ' or i·... lg2+), or iron-runtaining preparations hecamc
• 41 "
divalent c~"ioll s inhibit dmgs' absorption inthc gilt.
· 42 "
UlN lo,LUSf. Borre/ia hllrgdorferi. M. pllewIJouille. Dmgs' abi li ty to aCCUllnil;ltc intracellularl), makcs thcm ,'cry
· 43 " cfft'C ti"e agJillst /{ickellsill ;lIld CJ.lalltrdia. Also uscd to treat acne.

·44 " 10liCII1 C l distress, discoloration of teeth and inhibitiOiI of bone growth in children. photoS('nsiti\"itr-
COlltraimlicatcd illl'rcgllallC)".
MWWIISMOf Rl~ITAJI([ I nptakc or I efflux out ofhacterial cells by plasmid-encoded tr;lIlsport pumps.
,Pad 9 2306 58« _ '

+ G R< UI usmle rx com/ap l#qmax1 + III :


Item ' 3S of 46
QID 1933 i
L • Mark --<l
Pre Ylous
t:>
Next
_!
la b~ ues
~.
Notes
C';'\I
Cakul~ tor

· 26 " Drug Y has a bioava ilability of 40% and a vol ume of distribution of 10 L.

··2728 "" What is the correct loading dose required to obt ain a plasma concentration of 20 mg/L?

·29"
· 30 " A. 8mg
We value your feedback!
13441 5 : zhe ng

B. 20 mg

··3233 "" DDil


• 31 "

c. 80 mg
~ ~
· 34 " D.SOOmg
· 35 "
· 36 "

·37 "
· 38 "
" E. 800 mg

· 39 "
The correct answer is D. 60% chose this.
· 40 " The formula for calc ulating loading dose (LD) is: l D = (desired plasma conce ntration) x (volume
• 41 " of distribution I bioavailability) = (20 mg/l) x (10 I I 0.40) =SOD mg.
· 42 " A is not correct. 5% chose this.
· 43 " Ca lculation error.
·44 " e is not correct. 6% chose this.

answer I s.
The formula for calc ulating loading dose (LO) is: l D = (desired pla sma concentration) x (volume
of distribution I bioavailability) = (20 mg/l ) x (10 I I 0.40) = 500 mg.
A is not correct. 5% chose this.
· 29 " Ca lculation error.
· 30 " B is not correct. 6% chose this.
Ca lculation error.

··3233 ""
• 31 "
e is not correct. 23% chose this.
Ca lculation error.
· 34 " E is not correct. 6% chose this.
· 35 " Ca lculation error.
· 36 "

·37 "
· 38 "
Bottom li ne:
lD = (desired plasma concent ration) x (volume of distribution I bioavailabi lity).
· 39 "
· 40 "
• 41 " 141 ;fi
fiRS T A I D fACTS
, .jI.) for year: 2015
· 42 "
· 43 "

·44 " Dosage calculations C x Vd


Loading dose == ~
FA15 , p. 243.2

III ren al or li\'er d isease. Ul ai"ten:lllce dose I and


lo.1ding dose is Il sll" lly nnch ange,l.
T ime to ste .. dy sM e depe nds pri"''',il), on
,Pad 9 2306 58Y0 _ '

+ G R< UI usmle rx com/ap l#Qmax1 + • :


Item" 36 of 46
QID 1956
L

J.
• Mark -<l
Pre YlDu,
t>-
NeKt
all
la b"" ues
~.
Not es
~'t
c .. k ul~ tor

' 26 . A woman comes to the clinic complaining of vag inal itching and burning. She says she feels
as if she has the flu and has had intermittent fevers and muscle aches over the past few
IAAI
· 27 •
days. Vaqinal examination reveals the lesions seen in the imaqe, and treatment is started .
' 28 . The pati ent subsequen t ly deve lops elevated leve ls of blood urea nitrogen and creatin ine in
· 29 • addit ion to a t rem or and m ental statu:s~~~=.......~",,,.....
' 30 .

' 31 •
· 32 •
· 33 •

' 34 •
. 35 •
. 36 .
· 37 •

' 38 .
' 39 .
' 40 •
. 41.
' 42 •
. 43 .

· 44 • Which of the following agents did this patient most likely receive?

We value your feedback!

i i i i i
the woman in this Vignette, patients with genital herpes will often complain of pain, itching,
dysuria, and vaginal or urethra l discharge. Moreover, a primary episode is usually accompanied
by fever, malaise, and mya lgias. The treatment for genital herpes is acyclovir, usually given
orally. Acyclovir is qenerally well tolerated, but a few serious adverse effects may occur:
' 29 . neu rotoxicity (confusion, tremor, delirium, seizures) and rena l toxicity (interstitial nephritis,
' 30 . obstru ctive nephropathy from crystalluria).
' 31 • Herpes Simplex Infections Confusion Delirium Seizures Crystal luri" GenItal Herpes

· 32 • B is not correct. 10% chose this.


· 33 • Fluconazole is an antifungal agent used to treat candidiasis and cryptococcal meningitis. It is
' 34 . also used as prophylaxis against funga l infections in immunocompromised patients. A patient
with vu lvovaginal candidiasis will typically have deep-red labia with sha llow ulcerations and
. 35 .
satellite lesions. Thi ck white vaginal discharge may also be present. Neither of these findings is
.36 . seen in the image. Adverse effects of fluconazole incl ud e gast rointest inal upset, alopecia ,
· 37 • increased levels of liver enzymes, and, occasionally, neurotoxicity.
' 38 . Auconazole CandidiasIs Alopecia Meningitis. Cryptococcal CandidiasIs. Vulvovaginal

' 39 . C is not correct. 12% chose this.


' 40 . Metronidazole is used to treat protozoal and anaerobic bacteria l infections, including
.41 . Trichomonas vag/nails and bacteria l vaginosis . Neither infection is accompanied by systemic
symptoms or demarcated vag inal lesions like the ones seen in the image. Adverse effects of
' 42 .
metronidazole include a disulfiram-like reaction with alcohol ingestion, gastrointestinal
. 43 . di stress, headache, and a metallic taste .
' 44 . Metronidazole Trichomonaslnfeetions Baeten,,1 Vag inosis Disulrlram Headache
,Pad 9 2306 58« _ '

+ G R< UI usmle rx c om/ap l#q max1 + III :


Item' 36 of 46
QID 1956 i
L • Mark --<l
PreYlous
t:>
Next
_!
la b~ ues
~.
Notes
C';'\I
Cakul~tor

Metronlda~ole Trichomonas InfectIOns Bacterial Vag inas,s Q'$ulroram H~adadle

· 26 "

··2728 ""
D Is not correct. 12% chose this.
Penicillin is the treatment of choice for syphilis. Unlike the lesions in the image, a syphilitic

·29"
· 30 "
chancre, indicative of primary syphilis, is usually a solitary, pa inless, , - to 2-cm papu le with an
indurated base . Primary infection is not accompanied by systemic symptoms. Adverse effects of
penicillin include hypersensitivity reactions and hem olytic anemia .
Primary s)'pf1i1is HypersensitIVity

··3233 ""
• 31 "
E Is not correct. 6% chose this.
Ribavirin is an antiviral agent used to treat chronic hepatitis C or respiratory syncytial virus
· 34 " infections. The oral form has been associated with hemolytic anemia and headache and the
aerosolized form with bronchospasm, rash, and conjunctival irritation.
· 35 "
IUba~irin Headache Hepatitis C. Chronic RespIratory Sync~al Virus Infections
· 36 "

·37 "
· 38 "
Bottom II ne:
· 39 " Genitial herpes is treated with acyclovir. which is generally well-tolerated but can sometimes
cause neurotoxicity and nephrotoxicity.
· 40 "
• 41 "
· 42 "
· 43 "
ul;fi"'''' for year: 2015
FIII.ST ... ID F"'CTS

·44 "

· 26 " A 13-month-old girl is brought to the pediatrician because of fever and a sore throat. Her

·27 "
· 28 "
mother confides that the child has always been healthy and has never needed any
immunizations. However, over the past several days. the child is less active and refusing to
eat. Today her neck appears swollen. Examination reveal s a grayish membrane-like material in the
· 29 " posterior pharynx. The pathogen most likely responsible for this chi ld's illness is normally not
· 30 "
virulent unless it undergoes a certain process.

··3233 ""
• 31 "
Which of the following organisms can increase its pathogenicity by the same process?
134415 : zhe ng
We value your feedback!
· 34 " A. Coagulase-negative staphylococci

· 35 " B. Coagulase-positive staphylococci


· 36 "

·37 "
· 38 "
C. Gram-negative diplococci

D. Group A streptococc i
· 39 "
E. Group B streptococci
· 40 "
• 41 "
· 42 "
· 43 "
The correct answer is O. 38% chose this.
·44 " The question describes infection by Corynebacterium diphtherlae. This child is oa lrticul ,,,lvat risk
because she has not received the appropriate immunization (the d i~)htheria-pE"ttJss.is - tet.a.
vaccine). Classically, C. diphtheriae infection produces a sore
,Pad 9 2306 58« _ '

+ G R< UI usmle rx c om/ap l#q max1 + III :


Item' 37 of 46
QID 3907 i
L • Mark --<l
Pre Ylous
t:>
Next
_!
la b~ ues
~.
Notes
C';'\I
Cakul~ tor

· 26 " The correct answer is D. 38% chose this.

··2728 "" The question describes infection by Corynebacterium diphtheriae. This child is particularly at risk
because she has not received the appropriate immunization (the diphtheria-pertussis-tetanus

·29 "
· 30 "
vaccine). Classically. C diphtherioe infection produces a sore throat. difficulty swallowing,
swollen neck, and fever. Other sequelae include myocarditis and peripheral nerve palsies.
Physical examination may reveal the presence of a grayish pseudomembrane in the pharynx. C.
diphtheriae's virulence factor, a 2-subunit exotoxin responsible for these symptoms, is derived

··3233 ""
• 31 " from a temperate bacteriophage. Bacteriophages are viruses that only infect bacteria. Once a
bacteriophage infects a host, it may either replicate immediately and lyse the bacterium (a lytic
bacteriophage), or incorporate its DNA into the bacterial genome and enter a proviral latent
· 34 " state (a temperate bacteriophage). Certain temperate bacteriophages carry genes that
· 35 "
transform the phenotype of the host once they beco me incorporated. This is known as
lysogenic conversion. Only strains of C. diphtherlae that have been lysogenized by a phage
· 36 "
encoding an exotoxin are capable of causing diphtheria. Other examples of lysogenic
·37 "
· 38 "
conversion include strains of Streptococcus pyogenes, the group A streptococcal species that
causes scarlet fever, and strains of Clostridium botulinum, the gram-positive bacillus that causes
botulism.
· 39 "
Oipfltheria MyocardItis Pseudomembrane Phenotype determination Botulism Pertussis Sore Throat brand of Phenol Entire peripheral neNe
· 40 "
Entire pharynx Scar let Fe\lf!r
• 41 "
· 42 " A is not correct. 10% chose this.
· 43 "
Coagulase-negative staphylococci include Staphylococcus epidermidis and Staphylococcus

·44 " saprophyticus. S. epidermidis is often involved in nosocomial infections of prosthetic devices
Uoints, heart va lves, and intravenous lines). S. saprophyticus is a common cause of urinary tract
infection in young, sexually-active women. It is resistant to novobiocin, as opposed to S.
epidermidis, wh ich is novobiocin-sensitive. These bacteria do not undergo lysogenic

i
Coagulase-negative staphylococci include Staphylococcus epidermldis and Staphylococcus
saprophyticus. S. epidermidis is often involved in nosocomial infections of prosthetic devices
Uoints, heart va lves, and intravenous lines). S. saprophyticus is a common cause of urinary tract
infection in young, sex ually-active women. It is resistant to novobiocin, as opposed to S.
· 29 "
epldermldis, which is novobiocin-sensitive. These bacteria do not undergo lysogenic conversion.
· 30 " Novob;ocm Urinary tract InfectIon

··3233 ""
• 31 " B is not correct. 16% chose this.
The only gram-positive staphylococcal species that is also coagulase-positive is Staphylococcus
aureus. Direct S. aureus inoculation causes pneumonia, osteomyelitis, acute bacterial
· 34 " endocarditis, sk in infection, and bacteremia/sepsis. Ingestion of preformed exotoxin causes
· 35 " gastroenteritis. S. aureus's exotoxin is also responsible for toxic shock synd rome and sca lded
skin syndrome. S. aureus does not undergo lysogenic conversion.
· 36 "

·37 "
· 38 "
PneumonIa Baneremia GastroenteritIS Skm Olseases,lnfecti0V5 Staphylococca l Scalded SkIn Syndrome

C is not correct. 21% chose this.


Gram-negative diplococci include Neisseria meningitldis and N. gonorrhoeae. N. menlngltldis
· 39 "
causes meningitis, Waterhouse-Friderichsen syndrome, and meningococcemia. N. gonorrhoeae
· 40 " causes urethritis, cervicitis, and pelvic inflammatory disease. These organisms are not known to
• 41 "
undergo lysogenic conversion.
· 42 " Meningitis Meningococcemia Waterhouse·Fndenchsen Syndrome. Meningococcal Urethri~s Uterine ceNic itis and endocervicitis

· 43 " Pelvic lnnammatory Disease

·44 " E is not correct. 15% chose this.


Group B streptococci (GBS) include Streptococcus agalactiae. These are gram·positive, ~­
hemolytic, catalase-negative cocci. GBS are among the most common causes of neonatal
,Pad 9 2307 58« _ '

+ G R< UI usmle rx c om/ap l#qmax1 + III :


Item' 37 of 46
QID 3907 i
L • Mark --<l
Pre Ylous
t:>
Next
_!
la b~ ues
~.
Notes
C';'\I
Cakul~ tor

· 26 " E is not correct. 15% chose this.

··2728 ""
Group B streptococci (GBS) include Streptococcus ago/aetiae. These are gram-positive, [!-
hemolytic, catalase-negative cocci. GBS are among the most common causes of neonatal

·29"
· 30 "
meningitis, neonatal pneumonia, and neonatal sepsis. The pathogenesis of GBS does not
depend on a temperate phage to increase its pathogenicity.
CATALASE Pathogenesis Sepsis of the newbofn

··3233 ""
• 31 "

Bottom li ne:

· 34 " The pathogenesis of certain bacteria is dependent on the expression of a temperate


bacteriophage exotoxin, a process known as lysogenic conversion. Examples include
· 35 " diphtheria (C diphtheriae), scarlet fever (S. pyogenes), botulism (C botulinum), and cholera (Vibrio
· 36 " cholerae).
·37 "
· 38 "
Pathogenesis DIphtheria Botul ism Scarlet Fever Cholera

· 39 "
· 40 " 141 ;fi
fiRST AID fACTS
, .j1.1 for year: 2015
• 41 "
· 42 " FA1S, p, 124,1

· 43 " Bugs with eKotoKins

·44 " 8,l(n~ IA

Inhibit protein syn thesis


Corynebalterium
TOXIN

Diphtheria toxin"
~ ANlfmA1IOH

· 26 " In a healthy individual, fasting leads to increased lipolysis, increased accumulation of fatty IAA[
·27 "
· 28 "
acids, and increased production of acetyl-coenzyme A (acetyl-CoA). Acetyl-CoA is an allosteric
activator of an important mitochondrial enzyme that cata lyzes an irreversible step in
gluconeogenesis.
· 29 "
· 30 " What is the product of the reaction catalyzed by this enzyme?

··3233 ""
• 31 " 134415: zheng
We value your feedback!
A. Fructose-1,6-bi sphosphate

B. Glucose
· 34 "

· 35 " C. Glucose-6-phosphate
· 36 "

·37 "
· 38 "
D. Oxaloacetate

E. Phosphoenolpyruvate
· 39 "
F. Pyruvate
· 40 "
• 41 "
· 42 "
· 43 " The correct answer is D. 38% chose this.
·44 " In a healthy individual. fasting eventually leads to gluconeogenesis. Synthesis of glucose from
pyruvate or lactate requires seven reactions of glycolysis that are reverSib le. However,
three glycolytic reactions that are irreversible and must be circumvented by four alterrlate
,Pad 9 2307 58« _ '

+ G R< UI usmle rx c om/ap l#qmax1 + III :


Item ' 38 of 46
QID 42 15 i
L • Mark --<l
Pre Ylous
t:>
Next
_!
la b~ ues
~.
Notes
C';'\I
Cakul~ tor

The correct answer 15 O. 38% chose this.


· 26 " In a hea lthy individual, fasting eventually leads to gluconeogenesis. Synthesis of glucose from

··2728 ""
pyruvate or lactate requires seven reactions of glycolysis that are reversible. However, there are
three glycolytic reactions that are irreversible and must be circumvented by four alternate

·29"
· 30 "
reactions that favor the synthesis of glucose. The first roadblock in the synthesi s of glucose
from pyruvate is the irreversible conversion of phosphoenolpyruvate to pyruvate by pyruvate
kinase. To convert pyruvate to phosphoenolpyruvate, pyruvate is first carboxylated to
oxaloacetate by pyruvate carboxylase, the first enzyme in the pathway. Pyruvate carboxylase is

··3233 ""
• 31 "
activated by acetyl-eoA and requires biotin and adenosine tripho sphate as cofactors.
Biotm Glucose Adeoosme Triphosphate

· 34 " A Is not correct. 11% chose this.


· 35 " Fructose-1 ,6-bisphosphate is converted to fructose-6-phosphate by fructose-1 ,6-
bisphosphatase.
· 36 "

·37 "
· 38 "
Fructose

B is not correct. 4% chose this.


The fina l reaction in gluconeogenesis is the conversion of glucose-6-phosphate to glucose by
· 39 "
glucose-6-phosphatase in the liver.
· 40 " Glucose

• 41 " C is not correct. 9% chose this.


· 42 "
Glucose-6-phosphate is converted to glucose by glucose-6-phosphatase. Von Gierke's disease
· 43 " results from a deficiency of this enzyme.
·44 " Glucose

E is not correct. 24% chose this.

· 26 " Glucose-6-phosphate is converted to glucose by glucose-6-phosphatase. Von Gierke's disease

·27 "
· 28 "
results from a deficiency of this enzyme.
Glucose

· 29 " E is not correct. 24% chose this.

· 30 "
Phosphoenolpyruvate carboxykinase cata lyzes the conversion of oxaloacetate to
phosphoenolpyruvate. This reaction, which requires guanosine triphosphate, is activated by
glucagon and inhibited by insulin.

··3233 ""
• 31 "
glucagon (rONA) Insulin

F is not correct. 14% chose this.


· 34 "
Pyruvate is the substrate in the reaction that is activated byacetyl-CoA.
· 35 "
· 36 "

·37 "
· 38 "
Bottom Li ne:
Pyruvate carboxylase is the first irreversible enzyme in the process of gluconeogenesis.
Pyruvate carboxylase is activated by acetyl-CoA, requires biotin and adenosine triphosphate as
· 39 " cofactors, and results in the conversion of pyruvate to oxaloacetate.
· 40 " Biotin Adenosine

• 41 "
· 42 "
· 43 " Iii lif, , ., I,' for year: 2015
·44 " FIRST .0.10 FAC TS
,Pad 9 2307 58« _ '

+ G R< UI usmle rx c om/ap l#qmax1 + III :


Item ' 39 of 46
QID 2690 i
L • Mark --<l
Pre Ylous
t:>
Next
_!
la b~ ues
~.
Notes
C';'\I
Cakul~ tor

· 26 " A pregnant woman in Nairobi, Kenya, develops nausea, vomiting, and watery diarrhea

··2728 ""
associated with abdom inal pain. Stool testing is negative for ova and parasites and stool
culture is negative. She has a low-grade fever and stable vital signs but dies within 2 weeks
of developing symptoms.
·29"
· 30 " Tissue biopsy of the liver is most likely to show which of the following results?

··3233 ""
• 31 " 134415 : zheng
We value your feedback!
A. Hepatocyte congestion
B. I rregu lar mass lesion
· 34 "

· 35 " C. Micronodular cirrhosis


· 36 "

·37 "
· 38 "
D. Patchy necrosis

E. Well-circumscribed granuloma
· 39 "
· 40 "
• 41 "
· 42 " The correct answer is D. 37% chose this.
· 43 "
Th is patient has acute hepatitis E, which is classically associated with fatalities and fulminant

·44 " hepatitis in pregnant women. Acute hepatitis E infection consists of two phases: a prodromal
phase and an icteric phase. In the prodromal phase, typical symptoms include fever, myalgias,
nausea, and vomiting. The icteric phase manifests with symptoms of cholestasis, such as
jaundice, dark urine, pale stools, and pruritus. Although the patient has not yet reached

· 26 " This patient has acute hepatitis E, which is classically associated with fatalities and fulminant

·27 "
· 28 "
hepatitis in pregnant women. Acute hepatitis E infection consists of two phases: a prodromal
phase and an icteric phase. In the prodromal phase, typical symptoms inc lude fever, myalgias,
nausea, and vomiting. The icteric phase manifests with symptoms of cholestasis, such as
· 29 " jaundice, dark urine, pale stoo ls, and pruritu s. Although the patient has not yet reached the
· 30 " icteric phase of disease, her symptoms are consistent with the prodromal phase. Hepatiti s E is
transmitted via the fecal-oral route, and infection rates are high in sub-Saharan Africa. Acute
hepatitis infections are associated with focal hepatocyte damage, which results in patchy

··3233 ""
• 31 "
necrosis of liver ce ll s.
Nausea VomIting Pruri tus Hepatocyte Fulminant hepatitis Acute hepatItiS Glycogen Storage Disease Type VI Hepatitis

· 34 " A is not correct. 22% chose this.


· 35 " Hepatocyte congestion suggests right-sided heart fa ilure and nutmeg liver.
· 36 " Hepatocyte Heart Fa ilure. Rlght·S ided nutmeg <ll iergen ic eKt r<lct

·37 "
· 38 "
B is not correct. 11% chose this.
An irregu lar mass lesion would be typical for hepatocellular carcinoma.
· 39 " C is not correct. 17% chose this.
· 40 " Micronodular cirrhosis is associated with chronic liver disease and is a result of disorders such
• 41 " as alcoholism, Wilson disease, and hemochromatosis. Note that postinfectious or drug-induced
· 42 " hepatitis usually results in macronodular (nodules> 3 mm) cirrhos is.
Hemochromatosis Hepatolenticular Degeneration Hepatitis. Drug-Induced
· 43 "

·44 " E is not correct. 13% chose this.


A well-circumscribed granuloma suggests Echinococcus infection.
Gr<lnuloma
,Pad 9 2307 58« _ '

+ G R< UI usmle rx c om/ap l#qmax1 + III :


Item ' 39 of 46
QID 2690 i
L • Mark --<l
Pre Ylous
t:>
Next la b~
_!
ues
~.
Notes
C';'\I
Cakul~ tor

Mieronodular cirrhosis is associated with chronic liver disease and is a result of disorders such
· 26 "
as alcoholism, Wilson disease, and hemochromatosis. Note that postinfectious or drug-induced

··2728 ""
hepatitis usually results in macronodular (nodules > 3 mm) cirrhosis.
Hemochromatosis Hepatolemicular Degeneration Hepatitis, Drug·lnduced

·29"
· 30 "
E is not correct. 13% chose this.
A well-circumscribed granuloma suggests Echinococcus infection.
G'<lnuIOm(!

··3233 ""
• 31 "

Bottom Line:
· 34 "
Hepatitis E can be fatal, particularly to pregnant women, due to cirrhosis, seen as patchy

t
· 35 "
necrosis on biopsy.
· 36 " Necrosis Hepatitis

·37 "
· 38 "

· 39 " ul;f;"'''' for year: 2015


fIRST "ID f"CTS
· 40 "
• 41 " FAI5, p . 166.1
· 42 " He patitis viruses
I'lRlI'i IRAMSM";51~ (AR~I[R IN(UBAliOM H(UIiSl
· 43 "

·44 " HAVO RNt\


pieorn,j\'iTlls
Fecal-oral No Short (weeks) No Asymptomatic
(usually). Acute.
Alone (no carrie rs)

· 26 " An 8-day-old neonate is brought to the pediatrician by his mother, who notes that he cried

·27 "
· 28 "
all through the night. She also says that he appea rs "flo ppy" lately and moves very little . On
physical examination, the newborn's skin appears yellow. His sclerae are also yellow. Testing
of his reflexes reveals decreased responses. The baby is subsequently admitted to the hospital.
· 29 "
· 30 " Low levels of which of the following proteins can lead to this condition?

··3233 ""
• 31 " 13441 5 : zheng
We va lue your feedback!
A. Albumin

B. Ceruloplasmin
· 34 "

· 35 " C. Ferritin
· 36 "

·37 "
· 38 "
D. Hemoglobin

E. Intrinsic factor
· 39 "
· 40 "
• 41 "
· 42 " The correct answer is A. 39% chose this.
· 43 "
Jaundice is a condition in which the patient's sclerae and skin appear yellow as a result of

·44 " hyperbilirubinemia. The patient in this vignette also has developed kernicterus, or excess
unconjugated bilirubin damaging the central nervous system (CNS), which is characterized by
neurologic deficits including hypotonia, lethargy, and poor reflexes. Albumin binds
unconjugated bilirubin in the blood, and the albumin-bilirubin complex is unable to pass
,Pad 9 2308 58« _ '

+ G R< UI usmle rx c om/ap l#q max1 + III :


Item ' 40 of 46
QID 3020 i
L • Mark --<l
Pre Ylous
t:>
Next
_!
la b~ ues
~.
Notes
C';'\I
Cakul~ tor

Jaundice is a condition in wh ich the patient's sclerae and skin appear yellow as a result of
· 26 "

··2728 ""
hyperbilirubinemia. The patient in this vignette also has developed kern icterus, or excess
unconjugated bil irubin damaging the central nervous system (eNS), wh ich is characterized by
neurologic deficits including hypotonia, lethargy, and poor reflexes . Albumin binds
·29"
· 30 "
unconjugated bil irubin in the blood, and the albumin-bilirubin complex is unable to pass
through the blood-brain barrier, preventing bilirubin from affecting the eNS. Lower levels of
albumin would result in less complexing of the unconjugated bili rubin, resulting in kernicterus.
There are many causes of hyperbilirubinemia. Decreased hepatic enzyme activity in the

··3233 ""
• 31 "
newborn, or even increased reabsorption of bilirubin from the gut (the "enterohepatic
circulation"), also can produce jaundice in the newborn. Jaundice and kernicterus also may occur
in syndromes cha racterized by a deficiency of the hepatic enzyme uridine diphosphate-
· 34 "
glucuronyl transferase, which conjugates bilirubin . The resulting inability to conj ugate bilirubin
· 35 " leads to less excretion through such vehicles as urine, and an increase in serum bili ru bin levels.
· 36 " The first-line treatment for hyperbilirubinemia in the neonate is phototherapy; treatment for

·37 "
· 38 "
refractory cases includes exchange transfusion. Phototherapy is the use of light to break down
unconjugated bilirubin to smaller, water-soluble product s so they may be excreted.
Kernicterus Muscle hypotonia Newborn (finding) Uridine Phototherapy Water Hyperbilirubinemia Blood brain barrier function
· 39 " Neonatal )aundice
· 40 "
B is not correct. 24% chose this_
• 41 "
Ceruloplasmin is a plasma protein that serves as the major storage protein for copper. A
· 42 "
deficiency of ceruloplasmin may result in Wilson's disease, an autosomal recessive disease.
· 43 " Patients w ith Wilson's disease exhibit neuropsychiatric symptoms caused by deposition of
·44 " copper in the basal ganglia. Additionally, copper may deposit around the iris, forming Kayser-
Fleischer rings. Also, copper depositi on in the liver leads to liver fa ilure. Long-term management
of Wi lson's disease includes copper chelation therapy with D-penicillamine and eventua l Ii
I

I
deficiency of ceruloplasmin may result in Wilson's disease, an recessive disease.
Patients with Wilson's disease exhibit neuropsychiatric symptoms caused by deposition of
copper in the basal ganglia. Additionally, copper may deposit around the iris, forming Kayser-
Fleischer rings. Also, copper depositi on in the liver leads to liver fa ilure. Long-term management
· 29 " of Wi lson's disease includes copper chelation therapy with D-penicillamine and eventua l liver
· 30 " transplantation.
Basal Ganglia Kayser.Fle ischer ring Copper Iris (Eye) Lrver Failure Penic illamine

··3233 ""
• 31 "
C is not correct. 12% chose this_
Ferritin is the main intracellular iron -binding protein. Low levels offerritin are indicative of an
· 34 "
iron deficiency, whic h can be treated with iron supplementation. During anemia of chronic
disease, ferritin levels may actua lly be elevated because of its role as an acute-phase reactant
· 35 " and because of increased intracellular hoarding of iron. Ferritin levels may also be elevated
· 36 " during iron overload disorders such as hemochromatosis. These disorders are treated with iron
·37 "
· 38 "
chelators.
Intracellular ferritin Hemochromatosis Anem ia of chronic disease

· 39 " o is not correct. 16% chose this.


· 40 " Hemoglobin is an iron-binding protein in RBCs that is respo nsible for the transport of oxygen in
the body. Deficiencies of hemoglobin result in anemia, wh ich can cause fatigue, deteriorating
• 41 " neurologic status, and high-output cardiac failure. Low hemoglobin values can be the result of
· 42 "
iron deficiency, blood loss, or even chronic disease (eg, malignancy). The yellowing seen in this
· 43 " patient more strongly suggests that the process involves bilirubin, not hemoglobin.
·44 " Oxygen Anemia
,Pad 9 2308 58« _ '

+ G R< UI usmle rx com/ap l#qmaxl + III :


Item ' 40 of 46
QID 3020 i
L • Mark --<l
Pre Ylous
t:>
Next
_!
la b~ ues
~.
Notes
C';'\I
Cakul~ tor

, 26 " Intrinsic factor is a glycoprotein produced by parietal cells in the stomach that binds vitamin
, 27 "
81 2- Pernicious anemia is an autoimmune disease that leads to the destruction of parietal cells.
The resulting B12-deficient state is characterized by a macrocytic anemia and peripheral
, 28 "
neuropathy.
, 29 " lntrinsk f~(tor Stomach Anem i ~. Pernicious Per iphe,,,1 NeurOpilthy

' 30 "

' 31 "
Bottom Line :
, 32 "
Low levels of albumin results in decreased comprexing of unconjugated bilirubin, and may
, 33 "
cause kernicterus in the neonate.
, 34 " Kernicten.15
, 35 "
, 36 "

, 37 " Iii itt'


fIRST ""D FA CTS
' ·iI .1 for year: 2015
, 38 "

' 39 " FA15, p . 37 1.3


, 40 " UnconJugated Ilemolytie, ph ys iologic (nl'll horns), Crigler-Naijar, Gilbert syndrome.
, 41 " (Indirect)
hyperbilirubinemia
, 42 "
Conjugated (direct) Biliary tract ohstruetion : gallstones, cholangiocarcinoma, pancreatic or lil'er cancer. liver Auke
, 43 " hyperbilirubinemia Biliary tract disease:
• 10 sclerosing cholangitis
, 44 "
• 10 bili;lry cirrhosis
Excrction defect: Dubin-Johnson synd rome, Rotor syndrome.

, 26 " A neonate with puru lent umbilical discharge for 1 day presents with fever, irritability, and
diffuse flushing , One day later she is covered in large, fluid-filled blisters that rupture easily,
, 27 " leaving raw red areas beneath, Blood cultures are taken, which within 24 hours grow an
, 28 " organism that is subsequently Gram stained with the results shown below.
r.. -..." -r -•.
... .
, 29 "

.•.4~ ' ;" ~. ~J ;"'1'~~


' 30 "
I .... . \ _ .. .
' 31 "
, 32 "
.j' ~
~. { " " ~ .. .. 4.
·· r V ·
~.,.4(6 "~
, 33 "
, 34 "
.. .... • " ~.;1. -r. .-~
, 35 "
~~.~\.t!.
, 36 "

, 37 "
.'
)-,,"-4--
, 38 "
." " /.
' 39 " • f
, 40 "
Image courtesy of CDC/Dr. Richard Fack/am
, 41 "
, 42 "
The skin symptoms observed in this case are due to the involvement of which of the following
, 43 "
intercellular structures?
, 44 "
We va lue your feedback!
A. Desmosomes
,Pad 9 2308 58« _ '

+ G R< UI usmle rx c om/ap l#q max1 + III :


Item ' 41of46 L • Mark --<l t:> _! ~. C';'\I
la b~ Cakul~ tor
QID 4890 i Pre Ylous Next ues Notes

We va lue your feedback.


.. .
· 26 " A. Desmosomes

··2728 ""
B. Gap junctions

·29"
· 30 "
C. Hemidesmosomes

D. Intermediate junctions

··3233 ""
• 31 " E. Tight junctions

· 34 "

· 35 "
The correct answer is A. 58% chose this.
· 36 "
The image shows gram-positive cocci in clusters. Staphylococcal scalded skin syndrome (SSSS)
·37 "
· 38 "
is caused by the release of two exotoxins (epidermolytic toxins A and B) from Staphylococcus
aureus. Desmosomes (also ca lled "macu la adherens") are responsible for binding epithelia l cells
to one another to form a coherent who le. The exotoxins that are released bind to a molecu le
· 39 "
within the desmosome called desmog lein 1, thereby disrupting cell adhesion. In SSSS, the
· 40 " epidermis separates at the stratum granulosum due to the binding of exotoxins to
• 41 " desmosomes in this layer. Clinica lly, this results in bullous lesions and a positive Nikolsky's
· 42 " sign.
Oesmosomes Epidermis Staphylococcal Sca lded Skin Syndrome Epithe lial Cells Stratum granulosum
· 43 "

·44 " B Is not correct. 7% chose this.


Gap junctions are circu lar intercellular contact areas that permit the passage of small
between adjacent cells, allowing communication to facilitate electrotonic and metabolic

The image shows gram-positive cocci in clusters. Staphylococcal scalded skin syndrome (SSSS)
· 26 "
is caused by the relea se of two exotoxins (epidermolytic toxins A and B) from Staphylococcus
·27 "
· 28 "
aureus. Desmosomes (also ca lled "macula adherens") are responsible for binding epithelia l cells
to one another to form a coherent whole. The exotoxins that are released bind to a molecu le
· 29 " within the desmosome ca lled desmog lein 1, thereby disrupting cell adhesion. In SSSS, the
epidermis separates at the stratum granulosum due to the binding of exotoxins to
· 30 "
desmosomes in this layer. Clinica lly, this results in bullous lesions and a positive Nikolsky's
sign.

··3233 ""
• 31 "
Oesmosomes EpIdermis Staphylococcal Sca lded Skin Syndrome Epithe lIal Cells Stratum granulosum

B is not correct. 7% chose this.


· 34 " Gap junctions are circu lar intercellular contact areas that permit the passage of small molecules
· 35 " between adjacent cells, allowing communication to facilitate electrotonic and metabolic
· 36 "
function.

·37 "
· 38 "
Gap Junctions

C is not correct. 21% chose this.


· 39 "
Hemidesmosomes are present on the basa l surface of epithelial cell s adjacent to the basement
membrane, and serve to connect epithelia l cells to the underlying extracellular matrix.
· 40 " Hemidesmosomes Basement membrane Extrace llular EpIthe lIal Cells

• 41 "
D is not correct. 5% chose this.
· 42 "
Intermediate junctions lie deep to tight j unctions, comprised of actin filaments forming a
· 43 "
continuous band around the cell, provid i ng structural support just below tight junctions.
·44 " TightJunctions
,Pad 9 2309 57« _ '

+ G R< UI usmle rx c om/ap l#q max1 + III :


Item ' 41of46 L • Mark --<l t:> _! ~. C';'\I
QID 4890 i Pre Ylous Next la b~ ues Notes Cakul~ tor
•J • P 9 J P • 9
· 26 " continuous band around the cell, providi ng structural support just below tight j unctions.

··2728 "" Tight junctions

E Is not correct. 9% chose this.


·29 "
· 30 "
Tight junctions are located beneath the l uminal surface of simple columna r epithelium (eg,
intestinal lining) and seal the intercellular space to prevent diffusion between cells.
TightJunction~ fxtr~(e l lular Sp.3ce

• 31 "
··3233 "" Bottom Line :
· 34 " In staphylococcal scalded skin syndrome, the epidermis separates at the stratum granu losum
· 35 " as a result of binding of epidermolytic toxins A and B to desmoglein 1 in this layer.
Ep;dermls Staphyiococ(ill Sca lded Skin Syndrome Striltum grilnulosum
· 36 "

·37 "
· 38 "
L~

iii lif, , ., .. , for year: 2015


· 39 " fl ll.S T ... ID f ... CTS

· 40 "
FA15, p. 440 .1
• 41 "
Skin infec:tiDns
· 42 "
Bacterial infections
· 43 " Impetigo Very snperfie ial skin infection. Usually from S. Iwreus or S. p)"og<mes. Highly cOllla gions. 1·loney-

·44 " colored crusting fl.


Bullous impetigo (] has lIullae and is uSllally caused lIy S. mmus.
'i Ii

· 26 " A 32-year-old woman complains of a ' ·month history of weight loss, fever, cough, and IAAI
·27 "
· 28 "
difficulty breathing. Physical examination reveals cervical lymphadenopathy and bilateral
decreased breath sounds, as well as white patches on her tongue and buccal mucosa. The
patient's chest x-ray is shown here. The physician prescribes trimethoprim-su lfamethoxazole .
· 29 "
· 30 "

• 31 "

··3233 ""
· 34 "

· 35 "
· 36 "

·37 "
· 38 "

· 39 "
· 40 "
• 41 "
· 42 "
· 43 "

·44 "
." .
· 29 •
· 30 •

· 32 •
· 33 •
· 34 •
. 35 •
. 36 .
· 37 •

· 38 .
·39 .
Which of the following laboratory techniques should be utilized to confirm the underlying cause of
.40 . all these patient's current symptoms?
.41 .
134415 zheng
· 42 .
A. Immunoassay
. 43 .
We va lue your feedback!
·44 • 8. Northern blot

C. Polymerase cha in reaction

Which of the following laboratory techniq ues should be utilized to confi rm the underlying cause of
· 26 . all these patient's current symptoms?
· 27 •
134415: zheng
· 28 .
A. Immunoassay
· 29 .
We va lue your feedback!
· 30 . 8. Northern blot
.31 . C. Polymerase cha in reaction
· 32 •
· 33 • D. Restriction fragment length polymorphism
· 34 . E. Southern blot
. 35 •
. 36 .
· 37 •

· 38 . Th e correct a ns wer Is A. 43% chose thi s .


. 39 . Th is woman has signs and symptoms of HIV infection, given the descriptions of oral thrush,
.40 . weight loss, lymphadenopathy, and Pneumocystis jirovecii infection (evidenced by the interstitial
infi ltrates on the x-ray). Thus, we need to determine which technique confirms a positive HIV
.41 . status. The current CDC algorithm recommends the validation of HIV infection with fourth-
· 42 . generation immunoassays. These immunoassays are more specific and produce fewer
. 43 . indeterminate results than previous diagnostic methods (Western blot and indirect fluorescence
.44 . assays). Current HIV testing includes a series of immunoassays. The initial immunoassay tests
for the the presence of HIV-1 and HIV-2 antibodies and HIV-1 p24 antigen in a blood
from the patient. Positive specimens are subjected to more specific immunoassays,
I
,Pad 9 2309 57« _ '

+ G R< UI usmle rx c om/ap l#q max1 + III :


Item' 42 of 46 Mark --<l t:> _! ~. C';'\I
. ..
L •

QID 1239 i Pre Ylous Next la b~ ues Notes Cakul~ tor


. . ,, - " ,

· 26 " This woman has signs and symptoms of HIV infection, given the descriptions of oral thrush,

··2728 ""
weight loss, lymphadenopathy, and Pneumocystis jirov€cii infection (evi denced by the interstitial
infiltrates on the x-ray). Thus, we need to determi ne which technique confirms a positive HIV
status. The current CDC algorithm recommends the vali dation of HIV infection with fourth-
·29"
· 30 "
generation immunoassays. These immunoassays are more specific and produce fewer
indeterminate res ults than previous diagnostic methods (Western blot and indirect fluorescence
assays). Current HIV testing includes a series of immunoassays. The initial immunoassay tests

··3233 ""
• 31 " for the the presence of HIV-' and HIV-2 antibodies and HIV-' p24 antigen in a blood specimen
from the patient. Positive specimens are subjected to more specific immunoassays, which
distinguish among HIV-l, HIV-2, and acute HIV- l infections. This procedure will be used to
· 34 " reveal the patient's HIV status and the strain of virus.
L)II1lphadenopat~ We~tern Blot Immunoa~~ay method Oral candidlMi~ Human Immur.odefic i encyVlru~ (HIV) ~tatlK
· 35 "
· 36 " B is not correct. 11% chose this.
·37 "
· 38 "
Northern blotting is a technique that measures gene expression by indentifyin g RNA in a given
sa mple. There are complementary DNA probes that hybridize to unid entified RNA molecules
after the RNA sample of interest is cut by restriction enzymes and then run on gel
· 39 " electrop horesis.
· 40 " Electrophoresis Gene Expression

• 41 " C is not correct. 30% chose this.


· 42 "
Polymerase chain reaction (peR) is a techn ique by which a short DNA or RNA sequence is
· 43 " amplified enormo usly by the use of primers and pOlymerase enzymes, with fu rther DNA
·44 " synthesis occurring in repeated cycles. Although PCR may be used as part of this procedure, it
can neith er specify the HIV virus nor confirm its presence.
PoJymer~se Chilin Reilction

i i (peR) DNA or RNA sequence


amplified enormously by the use of primers and polymerase enzymes, with further DNA
synthesis occurring in repeated cycles. Although peR may be used as part of this procedure, it
can neith er specify the HIV virus nor confirm its presence.
Po~merase Chilin Reaction
· 29 "
· 30 " D is not correct. 7% chose this.
A restriction fragment length polymorphism techn ique detects a polymorphic difference in DNA

··3233 ""
• 31 " seq uences between individual s that can be recogn ized by restriction endonuclease. It is not a
technique used in the detection of a protein .
GenetIC P~ymorphism

· 34 "
E Is not correct. 9% chose this.
· 35 "
Southern blotting is-a standard method-for analyzing the structure of-BNA-cleaved by restriction
· 36 " enzymes, but is not used for detecting HIV.
·37 "
· 38 "
Bottom Line:
· 39 "
The immunoassay is the confirmatory test for HIV infection. Fourth-generation immunoassay
· 40 " protocols are more sensitive than previous diagnostic strategies (Western blot and indirect
• 41 " ....fluoreseenee asssays) and ean distinguish among HIY- ,HIY-Z:, and acute HIV-1 infection~
· 42 " Immur.oassay IT1{!thod Western Blot

· 43 "

·44 " hl;fi'·'!·' foryear: 2015


~IRST "'10 ~"'CTS
,Pad 9 2310 57« . '

+ G R< UI usmle rx c om/ap l#qmaxl + III :


Item' 42 of 46
QID 1239 i
L • Mark --<l
Pre Ylous
t:>
Next
_!
la b~ ues
~.
Notes
C';'\I
Cakul~ tor

A restriction fragment length polymorphism technique detects a polymorphic difference in DNA


· 26 "

··2728 ""
sequences between individuals that can be recogn ized by restriction endonuclease. It is not a
technique used in the detection of a protein.
GenetIc P~ymorphism

·29"
· 30 "
E Is not correct. 9% chose this.
Southern blotting is a standard method for analyzin g the stru cture of DNA cleaved by restriction
enzymes, but is not used for detecting HIV.

··3233 ""
• 31 "

Bottom Line:
· 34 "
The immunoassay is the confirmatory test for HIV infection. Fourth-generation immunoassay
· 35 " protocols are more sensitive than previous diagnostic strateg ies (Western blot and indi rect
· 36 " fluorescence asssays) and can distinguish among HIV- ' , HIV-2, and acute HIV-1 infection.

·37 "
· 38 "
Immurooassay IT1{!thod Western Blot

· 39 "
· 40 "
hl;fi.·H.' for year: 2015
~llST " 10 ~" CTS

• 41 "
FA 15, p. 78.1
· 42 "
Blotting procedures
· 43 "

·44 " Southern blot t\ D NA S<lmpl c is cn'-Ymatic~ II}' clc;"·cd into


smallcr pieccs, eledrophoTc5cd on a gel. ,md
thc n tmn sfc rrt..:J to a filter. T hc fi lter is thc n
S~oW DRo P:
Southern '" DNA
Northcrn'" RNA

· 26 " A 5-year-old boy is seen at the pediatrician's office for reqular follow-u p for his qenetic IAA[
·27 "
· 28 "
disorder. Phys ica l examination reveals coarse facial features, clo uded corneas, and restricted
range of motion of his joints . Laboratory tests show an elevated serum hydrolase level .

· 29 "
A defect in which of the following organe lles is most likely the cause of his disease?
· 30 "
134415; zheng
We value your feedback!
A. Golg i apparatus

··3233 ""
• 31 "

8. Mitochondria
· 34 " C. Nissl body
· 35 "
D. Rough endoplasmic reticulum
· 36 "

·37 "
· 38 "
E. Smooth endoplasmic reticulum

· 39 "
· 40 "
• 41 "
The correct answer is A. 60% chose this.
· 42 " The cl inical description of this patient is that of I-cell disease , which is due to the failure of the
· 43 "
addition of mannose-6-phosphate to lysosomal proteins. Mannose-6-phosphate is added to

·44 " lysosomal proteins in the Golg i apparatus to target them to the lysosome. The enzymes are
secreted outside of the cell in I-cell disease, which elevates the serum hydro lase .
Mannose Goigi Apparatus
,Pad 9 2310 57« . '

+ G R< UI usmle rx c om/ap l#qmaxl + III :


Item ' 43 of 46
QID 255 1 i
L • Mark --<l
Pre Ylous
t:>
Next la b~
_! ues
~.
Notes
C';'\I
Cakul~ tor

Mitochondria are the site of aerobic reSpiration and therefore provide energy for the cell. There
· 26 " is no relationsh ip between mitochondrial defects and the mis-targeting of lysosoma l proteins in

··2728 ""
I-cell disease.
Mitochondria

·29"
· 30 "
C is not correct. 4% chose this.
Nissl bodies are stacks of rough endoplasmic reticulum found in neurons. They synthesize
enzymes and other proteins. A defect in Nissl bodies does not result in I cell disease.

··3233 ""
• 31 " Neurons Rough endoplasmic reticulum Type II Mucolipid~ i s

D Is not correct. 16% chose this.


· 34 " Roug h endoplasm ic reticulum is a site of protein synthesis. It does not playa role in the mis-
targeting of lysosomal proteins in I-cell disease.
· 35 " Rough endopl~smi( reticulum
· 36 "

·37 "
· 38 "
E is not correct. 8% chose this.
Smooth endoplasm ic reticulum is the site of steroid synthesis and detoxification of poisons and
drugs. The smooth endoplasm ic reticulum plays no role in the pathogenesis of I-cell disease.
· 39 " Oetoxific~tion procedure P~thogenesi5 Smooth Endopl~sm i c Reticulum

· 40 "
• 41 "
Bottom line:
· 42 "
· 43 "
I-cell disease is due to the failure of the addition of mannose-6-phosphate to lysosomal

·44 " proteins, ca using them to be sec reted outside of the cell.
Mannose

· 26 " Smooth endoplasm ic reticulum is the site of steroid synthesis and detoxification of poisons and
·27 "
· 28 "
drugs. The smooth endoplasmic reticulum plays no role in the pathogenesis of I-cell disease.
Oetoxification procedure Pathogenesis Smooth Endoplasmic Reticulum

· 29 "
· 30 " Bottom li ne:
I-cell disease is due to the failure of the addition of mannose-6-phosphate to lysosomal

··3233 ""
• 31 "
proteins, causing them to be secreted outside of the cell.
Milnnose

· 34 "

· 35 "
· 36 " ul;fi"'''' for year: 2015
·37 "
· 38 "
FIII.5T ... I D F"'CTS

FA15. p , 73. 1

· 39 " Cell trafficking Goigi is the distrihution eenler for proleins and lipids from the ER to th e I'esicles and plasma
memiJrane. i\"lodifies N-oligosaecimridC"s on asparagi ne , Adds O -oligosaC"C"haridC"s on scrinC" and
· 40 " tlirt'(lnin c. l\dd s marirlosC"·6-phosphalC" 10 pr01l'ins for Iraffi C"kill g 10 Iysosomes
Elldosollles arc wrtillgee1l1l'rs for mat er ial frOIll ollt side the cell or frolll th e C o lg i. sClldill g it 10
• 41 " lrsosollles for dcstruction or ]>;,ck to the III cIIII"'ln clColgi for fnrther usc
· 42 "
I-c:e ll disease (inclusion ed l d isease)- inhe rited I)'sowmal storage disorder; defect in
· 43 " N-;Ketrlghu:osa lllill )·l-l-phosphotransfcrasc - failure of the Golgi to phosphorrlate Inallll QSC

·44 " resid" es (i.e .. I mannose-6-phosphate) on glreoproki ns - proteins arc secreted ex trae ellularly
rather th an delivcrcil to I)'sosomes. Resu lt s in eoarS<." facial features, clouded corne as. restricted
joint mOl'emcnt. and high plasma levels of I)'sosomal cnzymes. O ften fatal in chi ld hood.
· .
Item ' 44 of46 L • Mark -<l t:> _! ~. 1:;\'1
QID 1S47 i Pre Ylous Next la b~ ues Notes Cakul~ tor

.24 . Kabuki make-up syndrome (KMS, or Niikawa· Kuroki syndrome) is a very rare genetic disorder
of unknown cause that presents in neonates as mental retardation, unusua l skin ridging in
IAAI
· 25 •
the hands, fingers, and toes, and an everted lower eyelid. Estimates of worldwide prevalence
·26 . range from 1:100,000 to 1:10,000,000.
· 27 •
· 28 • If a new genetic test for KMS were developed that had a sensitivity of 98% and a specificity of 97%,
.29 . what positive predictive value wou ld be expected of this test?
· 30 . 134415 : zheng
· 31 • A. It will be high because of a low prevalence
· 32 • We value your feedback!
B. It will be high because of high accuracy
· 33 •
. 34 . C. It will be low because of a bias
· 35 • D. It will be low because of a low prevalence
· 36 .
E. It will be low because of low accuracy
· 37 •
· 38 •
· 39 •

..4241 ..
• 40 .
The correct answer is D. 62% chose this .
As the disease prevalence decreases, the likelihood of a positive test being a true-positive
decreases. In diseases with very low prevalence, a positive test is more likely a fa lse-positive. In
· 43 . this case, if the prevalence was 1:1,000,000, and the sens itivity and specificity are both 99%,
· 44 • then performing the test on 100,000,000 people will yield 99 true-positives and 999,999 false-
positives, for a positive predictive value of approximately 0.00009 .
. 45 .
.- ~ A Is not correct. 15% chose this .

8 ~
....... ....,..... V ' }I. ...... "''' ,.,.- " . ... v,, ~"')J .. '1 ""''' ' "T .' •

Item ' 44 of46 L • Mark -<l t:> _! ~. 1:;\'1


QID 1S47 i Pre Ylous Next la b~ ues Notes Cakul~ tor

As the disease prevalence decreases, the likelihood of a positive test being a true-positive
.24 . decreases. Tn diseases with very low prevalence, a positive test is more likely a fa lse-positive. In
· 25 • this case, if the prevalence was 1:1,000,000, and the sensitivity and specificity are both 99%,
· 26 . then performing the test on 100,000,000 people will yield 99 true-positives and 999,999 false-
positives, for a pos itive predictive value of approximately 0.00009.
· 27 •
A is not correct. 15% chose this.
· 28 •
As disease prevalence decreases, the positive predictive value will decrease, not increase .
. 29 .
B Is not correct. 19% chose this.
· 30 .
Accuracy measures validity and not reliability, but it changes between different tests instead of
· 31 •
between prevalences. Positive predictive value is a measure that changes as prevalence
· 32 • changes.
· 33 • C is not correct. 3% chose this.
· 34 • There is no reason to suspect bias in this test. Bias would imply that the information given is
· 35 • inaccurate.
· 36 . E is not correct. 2% chose this.
· 37 • Accuracy measures validity and not reliability, but changes between different tests instead of
· 38 •
between prevalences. Positive predictive value is a measure that changes as prevalence changes

· 39 .

..4241 ..
. 40 • Bottom 1I ne:
For diseases with low prevalence, the positive predictive value of any test is low, even if the
test is highly sensitive and specific.
· 43 .
· 44 •
· .
Item ' 44of46 L • Mark -<l t:> _! ~. 1:;\'1
QID 1S47 i Pre Ylous Next la b~ ues Notes Cakul~ tor

A IS not correct. 15% chose thiS .


. 24 . As disease prevalence decreases, the positive predictive value will decrease, not increase.
· 25 •
B Is not correct. 19% chose this.
' 26 . Accuracy measu res validity and not reliability, but it changes between different tests instead of
· 27 • between prevalences. Positive predictive value is a measure that changes as prevalence
· 28 • changes .
. 29 . C is not correct. 3% chose this .
' 30 . There is no reason to suspect bias in this test. Bias would imply that the information given is
inaccurate.
' 31 •
E is not correct. 2% chose this.
' 32 •
Accuracy measures validity and not reliability, but changes between different tests instead of
· 33 •
between prevalences. Positive predictive value is a measure that changes as prevalence changes
.34 .
· 35 •
Bottom II ne:
' 36 .
· 37 •
For diseases with low prevalence, the positive predictive value of any test is low, even if the
test is highly sensitive and specific.
· 38 •
' 39 .
' 40 .
ul;fi.·,.·1
FIIU T -' 10 F-' CTS
for year: 2015
' 41 .
. 42 .
FAI5,p, 49.1
' 43. evaluation of Uscs 2 x 2 t~ blc eOlliparing test res ults with th e 00...
' 44 • diagnostic tests act ual prescnce of disl'asc. TP = truc positive; <1) e
F P = fal sc pru itil'c; T N = truc ncgativc; FN =
;I

A 65-year-ol d man with a history of hypertension and hyperlipidemia presents to his


.25. physician complaining of constant, dull groin pain, as well as edema in the right leg for
' 26 . several weeks. The patient denies fever, hematuria, or fl an k pain. A fulln ess is palpable in the
· 27 • right lower quadrant, and a bruit is appreciated. CT of the pelvis confirms t he diagnos is.
' 28 .
. 29 • Which of the following is the most likely cause of this patient's findi ngs?
. 30 . 13441 5 : zhe ng
We value your feedback!
· 31 • A. Abdominal aortic aneurysm
· 32 •
B. Bladder calculus
· 33 •
.34 . C. Right common iliac artery aneurysm

' 35 . D. Right ureteral calculus


' 36 .
E. Urinary tract infection
' 37 •
• 38 •
. 39 .

' 40 .
The correct answer is C. 68% chose this .
. 41 .
The cause of this patient's signs and symptoms is an ane urysm of the right common iliac artery.
' 42 . The common il iac arteries lie posterior and medial to the ureters. Common iliac artery
.43 . aneurysms are uncommon but can present with groin pain and un ilateral edema if the
· 44 • lymphatics are com pressed by the growing aneurysm. The patient's age and sex , his long
history of hypertension and hyperlipidemi a, and his constant dull pain suggest an
the etiology.
+ G ~ UI usmle rx com/apl#qmax1 + iii :
Ite m. 45 of 46
QID 2979 i
k • M k
ar
<l
Previo us
C>
N~"I
ii!
ta b~U"$
C:: '
N'OIH
I:.~I
ca lcu lator

· 24 " The cause of this patient's signs and symptoms is an aneurysm of the right common iliac artery.
· 25 " The common iliac arteries lie posterior and medial to the ureters. Common iliac artery
aneurysms are uncommon but can present with groin pain and unilateral edema if the
· 26 "
lymphatics are compressed by the growing aneurysm. The patient's age and sex, his long
·27 "
· 28 "
history of hypertension and hyperlipidemia, and his constant dull pain suggest an aneurysm as
the etiology.
Ar.eury.;m Edem<l Hyperlipidemi<l
· 29 "
· 30 " A is not correct. 17% chose this.
· 31 "
The abdominal aorta is more medial and will cause symptoms of a pulsating sensation in the

·32 " abdomen and radiating pain, often to the back. It would not account for a palpable mass in the

·33 "
· 34 "
right lower quadrant.
B is not correct. 4% chose this.
The clinical presentation does not suggest a ureteral or bladder calculus. Typical symptoms
· 35 "
would include sharp, intermittent, excruciating pain in the lower back, abdomen, or testicu lar
· 36 " region. Fever, nausea, vomiting, and hematuria are usually present as well.
· 37 " Niluseil VomIting Urirlil'Y Blildder Cillculi (disorder)

·38 "
· 39 "
o is not correct. 10% chose this.
The clinical presentation does not suggest a ureteral or bladder calculus. Typical symptoms
· 40 " would include sharp, intermittent, excruciating pain in the lower back, abdomen, or testicu lar
region. Fever, nausea, vomiting, and hematuria also are usually present.
· 41 " Niluseil Vomlling Urirlil'Y Blildder Cillculi (disorder)
· 42 "
· 43 " E is not correct. 1% chose this.

·44 " Urinary t ract infections can lead to the development of calculi, but, because of the lack of fever
and hematuria, it is highly unlikely that this is the etiology responsible for th is D"'sent.3ti,,,,

· 25 "
o Is not correct. 10% chose this.
· 26 "

·27 "
· 28 "
The clinical presentation does not suggest a ureteral or bladder calculus. Typical symptoms
would include sharp, intermittent, excruciating pain in the lower back, abdomen, or testicular
region. Fever, nausea, vomiting, and hematuria also are usually present.
NallSeil Vommng Unrlil'Y Bladder Calculi (dIsorder)
· 29 "
· 30 " E is not correct. 1% chose this.
· 31 " Urinary tract infections can lead to the development of calculi, but, because of the lack of fever
·32 " and hematuria, it is highly unlikely that this is the etiology responsible for this presentation.

·33 "
· 34 "
Urirlil'Y IrilCI Infection

· 35 " Bottom line:


· 36 " A common iliac artery aneurysm is a rare but dangerous aneurysm that must be repaired, as
· 37 " the incidence of rupture is 50%. Presenting symptoms and signs vary but include dull pelvic
· 38 "
pain, uni lateral edema, decreased urine (if the ureter is compressed), constipation (if the colon
is compressed), and a palpable mass.
· 39 " Aneurysm Edema PelvIC Pain Ureter
· 40 "

· 41 "

· 42 "
· 43 "
uhf,
~U.ST
' ·".1
.... 0 ~"'CfS
for year: 2015

·44 "
+ G ~ UI usmle rx com/apl#qmax1 + iii :
Ite m. 46 of 46
QID 2850 i
k • M k
ar
<l
Previous
C>
N~"I
ii!
tab~ U"$
C:: '
N'OIH
I:. ~I
calculator

·25 "
· 26 "
A 32-year-ol d woman with diabetes gives birth at term to a 4000-g (S.8-lb) girl. The Apgar
score at 1 minute is 7, but worsens to 4 at 5 minutes; the newborn's fingers and lips turn

··2728 ""
blue and she becomes quiet. Pulses are equal in all four limbs.

·29"
· 30 "
Which of the following is most likely responsible for the chi ld 's condition?
134415 : zheng
A. Communication between the lower esophagus and the trachea

··3233 ""
• 31 "
8. Defect in the septum primum

C. Defect in uridine diphosphate-glucuronyl transferase


· 34 "

· 35 " D. Failure of the aorticopulmonary septum to spira l


· 36 " E. Patent ductus arteriosus
·37 "
· 38 "

· 39 "
We value your feedback!

· 40 "
• 41 "
· 42 "
· 43 "

·44 "
.. .. •
.:'

• . .
Ii
• • • .. A

0
76 Sis
4 KB/s

lock End Bloc k

+ G
.
~ u i .usmle~rx . com/api#qmax1
+ iii
Item: 46 of 46
QIO: 2850

·25 "
.1
• Mark <l
Previous
C>
N~"I ",iL
~.
.'
NOles

Transposition of the great vessels is a congenital cardiac malformation in which the


[::;.
calculator

aorticopulmonary septum fails to spiral during development and the aorta arises from the right
· 26 " ventricle wh ile the pulmonary artery arises from the left ventricle, completely separating the
·27 "
· 28 "
systemic and pulmonary circu lations. These parallel ci rcuits prevent proper oxygenation of
systemic blood in the newborn, leading to almost immediate cyanosis, which is characterized by
a blue coloration in the extremities and lips. Survival with transposition requires the presence of
· 29 " a shunt to allow mixing of blood from the right (pulmonary) and left (systemic) sides (eg,
· 30 " ventricular septal defect. patent ductus arteriosus, patent foramen ovale) or surgical correction.
Infants of women who were diabetic before pregnancy are at increased risk of atrioventricular

··3233 ""
• 31 " ma lformations. Additionally, this is the only congenital anomaly listed in the answer choices
that causes cyanosis. Other cyanotic congenital heart diseases that should be considered are
tricuspid atresia, total anomalous pulmonary venous return, tetralogy of Fallot, and truncus
· 34 " arteriosus.
Newborn (finding) Pregnancy Tetralogy of Fallot Transposition of Great Vessels Aorta Entire right ventricle Entire pulmonary artery
· 35 "
Entire left ventricle Ventricular Septal Defects Patent ductus arteriosus Truncus Arteriosus, Persistent
· 36 "

·37 "
· 38 "
A is not correct. 5% chose this.
Tracheoesophagea l fistulas occu r in three major variations. The most common variation is
· 39 " characterized by atresia of the upper esophagus with communication between the trachea and
the lower esophagus. Infants with this variation present with copious sa livation, choking,
· 40 " coughing, and cyanosis during feeding, and they are at risk for aspiration pneumonia.
• 41 " Tracheoesophagea l fistulas can be a part of the VACTERL association, which is characterized by
· 42 " Vertebral anomalies, Anal atresia, Cardiovascular anomalies, Tracheo Esophagea l fistula, Renal
· 43 "
deformities, and limb deformiti es.

·44 " Atresia Coughing Tracheoesophageal Fistula Trachea Choking

B is not correct. 10% chose this.


Atrial septal defects (ASDs) usually arise from defects in the septum secundum; however,
+ G ~ UI usmle rx com/apl#qmax1 + iii :
Ite m. 46 of 46
QID 2850 i
k • M k
ar
<l
Previo us
C>
N~"I
ii!
ta b~ U"$
C:: '
N'OIH
I:. ~I
ca lcu lator

Atnal septal defects (ASDs) usually arise from defects In the septum secundum; however, those
·25 "
· 26 "
associated with Down's syndrome arise from defects in the septum primum. ASDs allow left-to-
right shunting of blood during diastole, i ncreasing pulmonary pressures, and present with fixed,

··2728 ""
wide splitting of 52. AS Os are also associated with Eisenmenger's syndrome, which is
characterized by a reversal in shunt direction as a result of prog ressive pulmonary hypertension
and resistance secondary to chronic overloading of the right heart. An ASD would not result in a
·29"
· 30 "
cyanotic appearance. Of note, the following congenital heart defects are noncyanotiC: ASD,
ventricular septal defect, and PDA.
Atrial Septal De fects Right side of heart Conge nital Heart Defects Ventricu lar Septal Defects

··3233 ""
• 31 "
C is not correct. 4% chose this.
Uri dine diphosphate-glucuronyl transferase conjugates bilirubin in the liver. Defects in this
· 34 "
enzyme would increase serum levels of unconjugated bilirubin, and result in jaundice and
kernicterus, not cyanosis. Although the baby would have declining Apgar scores, the sclera and
· 35 " skin would appear yellow, not blue.
· 36 " Uridine Kernicterus Sde ra

·37 "
· 38 "
E is not correct. 13% chose this.
Patent duct us arteriosus (PDA) is a congenital heart defect characterized by persistence of the
· 39 " communication between the pulmonary artery and the aorta. In the fetus, there is no need for
· 40 " blood to travel to the lungs for oxygenation due to the surrounding amniotic fluid, so blood in
the pulmonary artery is diverted through the ductus arteriosus into the aorta for systemic
• 41 " travel. Normally after birth, the ductu s arteriosus closes. If, however, the ductus arteriosus
· 42 "
remains open, blood travels the opposite way, from the aorta to the pu lm onary artery, due to
· 43 " the decreased pulmonary vasculature resistance after birth. This can be appreciated on phys ica l
·44 " exam with the presence of a continuous machine-like mu rmu r. These patients can also present
later with progressive pulmonary hypertension due to increased flow through the pulmonary
artery. Patients with PDA who require its closure are administered indomethacin or
I I i

·25 "
I I
later with prog ress ive pulmonary hypertension due to increased flow through the pulmonary
· 26 " artery. Patients with PDA who require its closure are administered indomethacin or may

·27 "
· 28 "
undergo surgical ligation. PDA would not result in a cyanotic appearance as described in this
Vignette. (Another point to remember is that newborns with cyanotic congenital cardiac defects
may be given prostaglandin E to maintain the patency of the ductus arteriosus, to allow mixing
· 29 " of blood from either side of the heart.)
Paterlt ductus arter iosus [rodomethilcirl Congerlital Heart Def ects entire pulmonary artery Aor\<!
· 30 "

··3233 ""
• 31 "
Bottom Line:
Cyanotic congenital heart diseases include transposition of the great vessels, tricuspid atresia,
· 34 " total anomalous pulmonary venous return, tetralogy of Fallot, and truncus arteriosus. In
· 35 " transposition of the great vessels, the aorticopulmonary septum fails to spiral during
development, and the systemic and pulmonary circulations are completely separate such that
· 36 "

·37 "
· 38 "
the body does not receive oxygenated blood.
Tetralogy of Fallot Trarlspositlorl of Great Vessels TrullCus ArteriOSUS, Persistent

· 39 "
· 40 " I ill jfj , .j 1.1 for year: 2015
FIRS T AI D FA CTS
• 41 "
· 42 "
FA1S, p. 288 .1
· 43 " Congenital heart diseases

·44 " RI6HHO' UfT SHUNTS E~rl r cya nosis-"bl uc babies," Oflen diagnosed
prenatall), or bcrome c\'idenl immediately
Th e ;: Ts,
1. Truncus 3rteriosm (I "essel)
after birth. Usuall)' require urgent ~lITgieal
eorrection andlor nl3intcna nec of 3 rDA
,Pad 9 2017 330 . -

+ G R< UI usmle rx com/ap l#Qmax1 + • :


Ite m 1 of 44
QID 208S
L

J.
• Ma rk -<l
PreYlDU'
t>-
NeKt
all
lab"" ues
~.
Notes
~'t
c .. kul~tor

· 1
· 2
•• A 3·year-old boy is brought to his pediatrician because of worsening cough and rhinorrhea.
His parents state that he has had multiple similar episodes over the past year with two brief
IAAI
·3
· 4
•• hospitalizations for pneumonia. Physical examination of the skin reveals lesions like those
seen in the image. Laboratory tests show very low serum levels of IgA.
·5
· 6 ••
· 7
·8
••
.,0 •
· 9
.
· 11 •
· 12 •
· 13 •
· 14 •
· 15 •

·16.
· 17 •
· 18 •
· 19 •

•••
· 2
134415 : zheng
We value your feedback!
·3 A. A noninflamed skin abscess
· 4
·5
· 6
•• B. A repaired cleft palate

C. Frequent fall s
· 7
·8
•• D. Skin granulomas

.,0 •
· 9 E. Visua l hallucinations
.
· 11 •
· 12 •
The correct answer is C. 42% chose this.
· 13 •
The image shows spider angiomas, which common ly occur on the ears and cheeks of patients
· 14 •
with ataxia-telangiectasia. Ataxia-telangiectasia is an autosomal recessive disorder caused a by
· 15 • mutation on chromosome 11q22-23 that results in a defect in DNA repair enzymes . It manifests

·16.
· 17 •
as a variable combination of progressive neurologic impa irment, cerebellar atax ia, variable
immunodeficiency (usua lly IgA deficiency) with susceptibil ity to sinopulmonary infections,
impaired organ maturation, x-ray hypersensitivity, ocular and cutaneous telang iectasia, and a
· 18 •
predisposition to malignancy.
· 19 . A is not correct. 25% chose this.
A noninflamed or "cold" abscess is characteristic of Job syndrome which results from a failure of
interferon·y production by helper T cells. The lack of interferon-y leads to the failure
,Pad 9 2017 330 . -

+ G R< UI usmle rx com/ap l#Qmax1 + • :

'..5.4'3..
Ite m 1 of 44
QID 208S
L

J.
• Ma rk -<l
PreYlDU'
t>-
NeKt
all
lab"" ues
~.
Notes
~'t
c .. kul~tor

p p • •y• p
coarse facies, retained primary teeth, and high levels of rgE.
' 2 •
B is not correct. 8% chose this.
Cleft palate is a (ommon feature of thymic aplasia (DiGeorge syndrome). Thymic aplasia results

..9...
from failure of the third and fourth pharyngeal pouches (and thus the thymus and parathyroid

'6
'7
.,
glands) to develop. The disease often presents with congenital defects such as cardiac
abnormalities, cleft palate, and abnormal facies. Thymic aplasia can also present with tetany
due to hypocalcemia.
o is not correct. 25% chose this .
Granulomas are collections of cells seen in (among other things) chron ic granulomatous
.,0 . disease. This disease is caused by an inabi lity of neutrophils to kill bacteria once they have
phagocytosed them.
· 11 •
E is not correct. 0% chose this.
· 12 • Visual hallucinations are not a symptom of any of the known immune defi ciencies.
· 13 •
· 14 •
Bottom line:
· 15 •
Atax ia-telangiectasia is a genetic disorder in chi ldren characterized by cerebellar ataxia,
'16.
· 17 •
vari able immunodeficiency (usually IgA), and spider angiomas.

· 18 •
· 19 •
Uhf1,.,..1 for year: 2015
fIRST AID FAC TS

•••
A 41-year-ol d HIV-positive man has a (04+ cell count of 185/mm 3 on a recent monitoring
· 2 test. His physician strongly recommends starting sulfamethoxazole and trimethoprim for
·3 Pneumocystis jiroveci prophylaxis, as well as ganciclovir for prophylaxis against
· 4 cytomegalovirus infection.
·5
· 6
•• Which of the following is a possible adverse effect of treatment with ganciclovir?

., •
•7 134415 : zheng

•.
We value your feedback!
A. Cinchonism

.,0 •
· 9 B. Cutaneous flushing

C. Hemolytic anemia
· 11 •
· 12 • D. Photosensitivity
· 13 •
" E. Thrombocytopenia
· 14 •
· 15 •

'16.
· 17 •
The correct answer is E. 55% chose this.
· 18 •
Ganciclovir is commonly used in immunocompromised patients to prevent cytomegalovirus
' 19 . infection. Its toxicities include leukopen ia, neutropenia. thrombocytopenia, and nephrotoxicity.
Combining ganciclovir with other antiretroviral drugs (eg, zidovudine) increases the incidence of
neutropenia.
,Pad "" 2018 33« _ •

+ G R< UI usmle rx com/ap l#qmax1 + III :


Item ' 2 of 44
QID 1957 i
L • Mark --<l
Pre Ylous
t:>
Next
_!
la b~ ues
~.
Notes
C';'\I
Cakul~ tor

· 1
· 2
•• Ganciclovir is commonly used in immunocompromised patients to prevent cytomegalovirus
infection. Its toxicities include leukopenia, neutropenia, thrombocytopenia, and nephrotoxicity.

·3
· 4
•• Combining ganciclovir with other antiretroviral drugs (eg, zidovudine) increases the incidence of
neutropenia.
Ga[)(iCIOvir Leukopenia Neutropenia Thrombocytopen ia Zidovvdir.e

·5
· 6
•• A is not correct. 7% chose this.
Cinchonism (a combination of blurred vision, tinnitus, headache, and/or disorientation) is a
· 7
·8
•• common toxicity of the ctass IA antiarrhythmic quinidine.
Cinchonism (disorder) Tinn itus Heildilcr.e Quin idine

. ,0 •
".
· 9 B is not correct. 7% chose this .
. Cutaneous f1ush inq is a common adverse effect of vancomycin , adenosi ne , niacin, and
calcium channel blockers .
.· 12 • Adenosine N'i1cln Calcium

· 13 • C is not correct. 18% chose this.


· 14 • Hemolytic anemia is an adverse effect of ribavirin , which is commonly used to treat respiratory
syncytial virus or chronic hepatitis C infections.
·,5 .
·16.
· 17 •
IUbilvirin Hepatitis C, Chronic

D is not correct. 13% chose th is.

· 18 •
Photosensitivity reactions are a common adverse effect of sulfonamides, amiodarone, and
tetracyclines .
· 19 . Am iodilrOne

· 2
·3
••
·5 •
Bottom line:
· 4

· 6 •• Ganciclovir inhibits cytomegalovirus, herpes simplex virus, and varicella virus thymidine
kinase, but in cross-over reactions with normal cells can cause host leukopenia, neutropenia,

••
thrombocytopenia, and renal toxicity.
· 7 Gilnciclovlr Herpes SImplex Infections Leukopenia Neutropenlil Thrombocytopenlil

·8

.,0 •
· 9
.
.".
· 12 •
· 13 •
Iii !;fi I ·" .) for year: 2015
FI II.S T A I D FAC T S

Ganciclollir
MWWIISM 5'-monophosplmk formetl by a C~'I V I'iml kinase. Guanosine an~log. Triphosphate formcd by
FA1S, p . 193.3

· 14 •
cellular kin ascs. Prefcrcntially inhibits "iral DNA polrmcmse. Prefcrcntially inhibill'iml DNA
·,5 . pol)'mcmse hy el13in tcrmin:llion.

·16.
· 17 •
WN I(Al Us<'

tOl l{l!r
Cl\·IV. especi all y in immunQC{)mpromised paticnts. Valgan ciciol'ir. ,I pr{)(lrug of ganeielOl'ir. ha s
bct lcroral bio:lI'aibhilit)'.
u>ukopcnia. neutropenia, thrombocytopenia, rC1l<IIIOXicil),. MOTe toxic to host enz)'mes than
· 18 • 3erelo,"i r.
· 19 • MWWIISM Of ! l '>lSTANH ~ I lltatcd ,"iral kinase.
,Pad 9 2018 330 . -

+ G R< UI usmle rx com/ap l#Qmax1 + • :


Item ' 3 of 44
QID 2635
L

J.
• Mark -<l
PreYlDu,
t>-
NeKt
all
lab"" ues
~.
Notes
~'t
c .. kul~tor

· 1
· 2
•• A 68-year-old man with a 6·month history of back pain and fatigue presents to the
emergency department because of severe low back pain. MRI shows multiple lytic bone
·3
· 4
•• lesions scattered throughout the man's body, and laboratory results show a serum M spike
and light chain proteinuria. Bone marrow biopsy demonstrates an excessive number of the cells
·5
· 6 •• shown in the

· 7
·8
••
.,0 •
· 9
.
· 11 •
· 12 •
· 13 •

"..
· 14 •
· 15 •

.· 17 •
· 18 •

."

•••
· 2
Which of the following is most likely to be present in these cells?

·3 134415 : zheng
· 4 A. Abundant mitochondria
·5
· 6
•• B. Abundant rough endoplasmic reticulum
We value your feedback!

· 7
·8
•• C. Abundant secondary Iysosomes

D. Abundant smooth endoplasmic reticulum

.,0 •
· 9
. E. Abundant Nissl bod ies

· 11 •
· 12 •
· 13 •
The correct answer Is B. 66% chose this.
i
.".
· 14 •
Th is patient has multiple myeloma, a neoplastic proliferation of plasma cells in the bone
· 15 • marrow that often leads to lytic bone lesions and pathological fractures. The plasma cell is a
differentiated B lymphocyte that can produce and sec rete large amounts of antibody specific to
a particular antigen. Rough endoplasmic reticulum is the site of synthesis of secretory proteins;

.".
· 17 •
thus antibody-secreting plasma cells are rich in RER. Normally there is a polyclonal distribution
· 18 • of immunoglobulins of different isotypes and antigen specificities in the serum. In multiple
myeloma, however, the majority of plasma cells are producing immunoglobulin of one isotype
and antigen specifically, which can be detected as an M spike by serum protein electrophoresis.
The free light chains (either kappa or lambda) can often be detected in the urine; this is
to as Bence-Jones proteinuria. Patients with multiple myeloma might also manifest
,Pad "" 2018 33« _ •

+ G R< UI usmle rx com/ap l#qmax1 + III :


Item ' 3 of 44
QID 2635 i
L • Mark --<l
Pre Ylous
t:>
Next
_!
la b~ ues
~.
Notes
C';'\I
Cakul~ tor

· 1
· 2
•• to as Bence-Jones proteinuria. Patients with multiple myeloma mig ht also manifest anemia,
increased susceptibil ity to infection, or clotting abnormal iti es secondary to the red uced

·3
· 4
•• production of normal blood components.
Anemia Plasma Cells Bone Marrow lymphocyte Rough endoplasmic reticu lum Immunoglobulins Serum protem electrophoresis

••
Bence.Jones proteinuria
·5 A is not correct. 8% chose this.
· 6
· 7
·8
•• The primary function of mitochondria is the synthesis of ATP. They are abundant in cells that
requi re a large amount of energy, such as myocytes.
Milochondri<l

. ,0 •
· 9
C is not correct. 9% chose this .

.".
· 12 •
· 13 •
· 14 •
. Secondary Iysosomes are formed when a primary lysosome, with its hydrolytic enzymes, fuses
with materials for degradation. Secondary Iysosomes have substrates at different stages of
digestion. Cells such as macrophages, which are responsible for phagocytosis of cell debris,
may contain mu ltiple secondary Iysosomes.
Digestion Phagocytosis

·,5. D is not correct. 9% chose this.

·16.
· 17 •
Smooth endoplasm ic reticulum (S ER) is the site of steroid synthesis and detoxification of drugs
and poisons; thus, cells like hepatocytes and steroid-producing cells of the adrenal cortex are
rich in SER.
· 18 •

·19.
Detoxification procedl1re Smooth Endoplasm ic Reticl1ll1m Adrenal Cortex

E Is not correct. 8% chose this.


Areas of rough endoplasm ic reticulum in neurons are called Nissl bodies. This is the site of
enzyme and peptide neurotransmitter synthesis.

•••
· 2
·3 Bottom Li ne:
· 4 Multiple myeloma is a malignancy in which dysplastic plasma cells (with abundant rough
·5
· 6
•• endoplasmic reticulum) secrete excess ive quantities of immunoglobulin, resulting in renal
fa il ure, hypercalcemia, lytic bone lesions, and blood component deficiencies.

••
Hypercalcemia Plasma Cells Rough endoplasmic retlCl1ll1m
· 7
·8

.,0.• hl;fi,·,.·1 for year: 2015


· 9

.".
FIRST ,0.10 FACTS

FA 15 ,p. 401 .1
· 12 • Multiple myeloma Monoc lona l plasma cell (" fri ed cgg" Thin k CRA B:
· 13 • appe amncc) canccr tha i arisc s in thc marrow HyperC alcemia
and prod uccs brgc amo unts of IgG ( )5%) or Renal inl'oll'emcnt
· 14 •
Igi\ (25%). ",los t cOlll mon IQ tn lllor arisin g Anem ia
·,5 . wi th in bone in people > 40- 50 years old. Bone lytic les ionS/ B;] c ~ 1);]i11

·16.
· 17 •
Assoc ialed wi( h,
1 Sllsccptibilit y to i" fcclioll
• Pri",,,ry ;I!llyloidosis (1\ 1..)
• Pu nched·out I)"tic bolle lesions on x·ray rJ
" I"ltiple " l yelom:1: " Ionoc lolla l " l prote i11
spi~e
D istinguish from W(1ldensl rtim
m<JCToglobulinemi(1 - M sp ike == Ig~'"
· 18 •

·19. • " ,I spike on SCHUll protein ek'drophoresis


• 19 li ght chains in mine (Bence Jones
prote in)
• Roulea ux formation rn (R BCs st'1cked like
- hypc f" is.cos it ), synd rome (e.g" bln rred
vision. R:J)"naud plwnomenon); no "C RA B"
fi nd ings.
,Pad "" 2018 33« _ •

+ G R< UI usmle rx com/ap l#qmax1 + III :


Item' 4 of 44
QID 1526 i
L • Mark --<l
PreYlous
t:>
Next
_!
la b~ ues
~.
Notes
C';'\I
Cakul~tor

· 1
· 2
•• A 58-year-old man with a history of alcohol abuse presents to the hospita l with a protuberant
abdomen. slurrinq of speech, and confus ion. On physical examination there are multiple
IAA[
·3
· 4
•• spider ang iom ata on hi s chest , and a fluid wave is elicited on abdomina l examination. When
his hands are extended outward a flappinQ tremor appears. He is promptly treated with
·5
· 6
•• medications to reduce the levels of the offensive toxic metabolite .

· 7
·8
•• Normally, where do the in iti al steps of conversion of this tox ic metabolite take place?

We value your feedback!


134415 : zh eng

.,0 •
A. Hepatocyte cytoplasm
· 9

.".
· 12 •
· 13 •
· 14 •
. B. Hepatocyte endoplasmic reticulum

C. Hepatocyte mitochondria

D. Kupffer's cell cytoplasm

E. Portal circulation
·,5 .
·16.
· 17 •
· 18 • The correct answer is C. 43% chose this.
·19. This patient is presenting with decompensated liver failure and signs of hepatic
encephalopathy. His protuberant abdomen with a fluid wave, spider angiomata, and alcohol
history all point toward portal hypertension caused by alcoholic cirrhosis. His confusion,
slurring of speech, and flapping of hands when outstretched (asterixis) are all signs of

This patient is presenting with decompensated liver failure and signs of hepatic

•••
· 2 encephalopathy. His protuberant abdomen with a fluid wave, spider angiomata, and alcohol
·3 history all point toward portal hypertension caused by alcoholic cirrhosis. His confusion,
slurring of speech, and flapping of hands when outstretched (asterixis) are all signs of
· 4

·5
· 6
•• encephalopathy due to buildup of ammonia. Patients presenting with hepatic encephalopathy
are often treated with agents to reduce ammonia levels, including lactulose and antibiotics,
both of which work in the gut to bind ammonia and elim inate urease-producing bacteria,
· 7
·8
•• respectively.
In a healthy person, ammonia (the potentially toxic by-product of protein degradation) is
converted to urea in the liver. Urea is a more neutral compound that is eliminated by the kidney

.,0 •
· 9 in the urine. The conversion from ammonia to urea constitutes the urea cycle, wh ich takes place

.".
· 12 •
· 13 •
· 14 •
. in hepatocytes. The initia l steps of the urea cycle (the convergence of ammon ia and carbon
dioxide to form carbamoyl phosphate; the convergence of carbamoyl phosphate and ornithine
to form citrulline) all occur in the mitochondrial matrix of hepatocytes. Citrulline is then
exported from the mitochondria, and the ensuing steps occur in the cytosol, where urea is
ultimately generated.
ConfU5ion Ammon ia measurement La'tu lose Ornithine Citrulline Mitochondri<1 Liver Failure Hepatl( Enc:ephalopathy Spider neVU5

·,5. Hypertension. Portal Liver Cirrhosis. Akoholi' C<1rbon DiOXide Mllochondri<11 m.ltrix

·16.
· 17 •
A is not correct. 30% chose this.
Many steps of the urea cycle occur in the hepatocyte cytoplasm. However, the initial steps
· 18 • involving ammonia occu r in the hepatocyte mitochondria l matrix.

·19. Hepatocyte Cytoplasm Ammonia measurement MItochondrial matrix

B is not correct. 19% chose this.


hepato(:yt'es is the site of detoxification of many
I I
,Pad "" 2019 32« _ •

..3·2,..
+
Item' 4 of 44
QID 1526
G
i
R<
L •
UI usmle rx c om/ap l#qmax1

Mark

.
--<l
PreYlous

D is not correct. 5% chose this.


.
t:>
Next
_!
la b~ ues
~.
Notes
C';'\I
Cakul~tor
+ III :

.4
·5 .
.
Kupffer's cells are specialized macrophages lining the sinusoids of the liver. They have
immunologic function and secrete cytokines, as well as ingest antigens. They do not have a role

.
in clearance of ammonia.

..,7.
·6 Immune System Processes Ammorlia measuremem

E is not correct. 3% chose this •


Ammonia produced by gut bacteria is absorbed into the portal circulation where it then travels
to the liver to be converted into urea via the urea cycle. No molecular conversion takes place
., while ammonia travels through the portal circulation .

.".
. ,0 .
· 12 •
· 13 •
· 14 •
Ammorlia measuremem

Bottom line:
Ammonia levels increase in decompensated liver failure and can lead to hepatic
encephalopathy. Normally, ammonia is converted to the less-toxic urea in healthy liver cells.
·,5 . The steps of the urea cycle take place both in hepatocyte mitochondria and in hepatocyte

·16.
· 17 •
cytosol.
Ammon;~ me~5urement Hepiltocyte M;tochondr;~ Liver F~;lure Hepil(;c Enceph<llopilthy

· 18 •

·19. ul;fi"'''' for year: 2015


FlJI.ST A'D FAC TS

•••
A 72-year-old woman is admitted to the intensive care unit because of septic shock. Blood
· 2 cultures are positive for Candida albicans. The patient is started on amphotericin Band
·3 flucytosine.
· 4

·5
· 6
•• Flucytosine works via which of the following mechanisms?
13441S ; zheng

., •
· 7 A. Disrupts microtubule function

., •
We value your feedback!

• 8. Forms pores in the cell membrane

".
. ,0 •

.· 12 •
· 13 •
· 14 •
C. I nhibits DNA synthesis

D. I nhibits the convers ion of lanosterol to ergosterol

E. I nhibits the convers ion of squalene to lanosterol

·,5 .
·16.
· 17 •
The correct answer is C. 52% chose this.
Flucytosine is a pyrimidine antimetabolite similar to the chemotherapeutic agent 5-fluorouraci l.
· 18 •

·19. It inhibits DNA synthesis by conversion to fluorouracil, which competes with uracil. It is
selective for fungal cells because the two enzymes that are required for its
(membrane permease and cytosine deaminase, respectively) are found in
and activation
i I low levels in
mammalian cells. It is used, along with amphotericin B, to treat systemic fungal ilr1fe,(ti "n5
,Pad "" 2019 32« _ •

+ G + III :

,..4'2'3...
R< UI usmle rx c om/ap l#qmax1

Item ' 5 of 44
i
L • Mark --<l t:> _!
la b~
~. C';'\I
Cakul~ tor
QID 1917
. Pre Ylous
..
Next ues Notes

Griseofulvin disrupts microtubule function by interfering with the spindle apparatus, thus
interfering with mitosis .
Mitos is

''65..
'7..
B is not correct. 11% chose this.
POlyenes such as amphotericin B bind to ergosterol in the cell membrane. They are amphipathic
and cause the formation of pores in fungal cell membranes.
Amphotericin B
.,
., D is not correct. 18% chose this .
The conversion of lanosterol to ergosterol is inhibited by the azole antifungal agents, such as
.,0 . ketoconazole and fluconazole. They inhibit the 14.a.demethylation of lanosterol.

"'.
· 12 •
· 13 •
· 14 •
Ketoconazole Fluconazole AntIfungal Agents

E is not correct. 12% chose this.


The conversion of squalene to lanosterol is faci litated by the enzyme squalene epoxidase.
Terbinafine is the antifungal agent that inhibits this enzyme.

"5
'16..
· 17 •
Squalene terbinafine

Bottom li ne:
· 18 •
Flucytosine is a pyrimidine antimetabo lite that inhibits DNA synthesis by conversion to

'19. fluorouracil, wh ich competes with uracil.


Flucytosine Fluorour<lcil

•••
The conversion of lanosterol to ergosterol is inhibited by the azole antifungal agents, such as
· 2 ketoconazole and fluconazole. They inhibit the 14-a-demethylation of lanosterol.
·3 Ketoconazole Fluconazole AntlfUng<Il Agents

· 4
E is not correct. 12% chose this.
·5
· 6
•• The conversion of squalene to lanosterol is facilitated by the enzyme squalene epoxidase.
Terbinafine is the antifungal agent that inhibits this enzyme.

·, •
SqUillene terbinafine
•7

·, •
.,0 •
Bottom line:
. Flucytosine is a pyrimidine antimetabo lite that inhibits DNA synthesis by conversion to

"'.
· 12 •
· 13 •
· 14 •
fluorouracil, which competes with uracil.
Flucytosine F1uorour<lcil

I ii I ;fi , .j 1.1 for year: 2015


" 5 .

'16.
FIRST AID FAC TS

· 17 • FA1S. p . 190.3

· 18 • Antifungal therapy

'19.
,Pad "" 2019 32« _ •

+ G R< UI usmle rx c om/ap l#qmax1 + III :


Item ' 6 of 44
QID 3647 i
L • Mark --<l
Pre Ylous
t:>
Next
_!
la b~ ues
~.
Notes
C';'\I
Cakul~ tor

· 1
· 2
•• A baby girl is born to a 38-year-ol d mother with little prenatal care. Physical eva luation of the
infant is notable for low birth weight, small head size, small right orbit, cleft lip and pa late, A
I'" AI
·3
· 4
•• and an extra right-sided digit com ing off the thumb. A holosystolic murm ur is appreciated
over the precordium, and severe hypotonia is noted. The attend in g physician remarks that the
·5
· 6
•• diagnosis could have been made before birth by amniocentes is and chorionic villus sampling.

· 7
·8
•• What is the mechanism for the disease producing this constellation of physical findings?
13441S : zheng

.,0 •
A. Defect in degradation of branched-chain amino acids
· 9

.".
· 12 •
· 13 •
· 14 •
.

"
B. Deletion of the short arm of ch romosome 5

C. I ntracellu lar glycogen accumu lation

D. Maternal nondisjunction

E. Microdeletion on chromosome 22
·,5 .
·16.
· 17 •
F. Nonenzymatic glycosylation

· 18 •

·19. We value your feedback!

•••
· 2 inclludinq ,,:i(~::C:~~:::~;I~~i~:'tC~~'htl,alimiia polydactyly, hypotonia suggestive of I nervous
system !fl and I heart disease suggested by a heart murmur. Advanced maternal
·3 age is a ris k factor for nondisjunction, the primary mechanism for chromosomal aneuploidies.
· 4 Microcephilly Polydilctyly Muscle hypotoniil Cor.genit31 heilrt diseilse Heilrt murmur

·5
· 6
•• A is not correct. 4% chose this.
This describes maple syrup urine disease, in which decreased activity of Q-ketoacid
· 7
·8
•• dehydrogenase causes manifestations of central nervous system defects, mental retardation,
and death.

. ,0 •
Ment31 Ret3rd<ltion
· 9

.". . B is not correct. 14% chose this .


This describes cri-du-chat synd rome , which shares features of growth retardation,
microcephaly, and congenital heart disease with Patau's synd rome. Other features of cri-du-chat
· 12 •
include the unusual cat -like cry for which the syndrome is named, distinctive facial features
· 13 • including epicanthal folds, and common ly visceral anomalies.
· 14 • Growth retard<ltion Microcephaly Cor.gemta l heart disease

·,5 . C is not correct. 5% chose this.


·16.
· 17 •
Glycogen storage diseases are the result of abnormal glycogen metabolism and subsequent
intracellular accumulation of glycogen. Presentation of the 12 types vary, but include Von
· 18 •
Gierke's disease, Pompe's disease, and McArdle's disease.

·19. Metabolism Glymgen Storage Disease Glycogen storage disease type II

E Is not correct. 13% chose this.


Th is describes 22q11 or DiGearge's syndrome. Common features of DiG.,orcl.·s
'Vlnd l, mnpinclude cl I and I heart disease. Other
,Pad"" 20 19 32 Y0 . •

+ G R< UI usmle rx com/ap l#Qmax1 + • :


Item " 6 of 44
QID 36<17
L

J.
• Mark -<l
PreYlDu,
t>-
NeKt
all
lab"" ues
~.
Notes
~'t
c .. kul~tor

· 1
· 2
•• This describes 22ql1 syndrome or DiGeorge's syndrome. Common features of DiGeorge's
syndrome include clefting, abnormal facies, and congen ital heart disease. Other features of
·3
· 4
•• 22ql1 syndrome are thymic aplasia leadi ng to immunodeficiency and parathyroid aplasia
leading to hypocalcemia.
Hypocalcemi<l (orqeniwl heart dise3se CO<19"'nit.1i ;;ob5ence of thymus

·5
· 6 •• F is not correct. 4% chose this.

••
Infants of diabetic mothers have an increased risk of congenital anomalies, including well
· 7 known associations such as caudal dysgenesis and transposition of the great vessels. The
·8 congenita l abnormalities in this case do not exclude the possible etiology of maternal diabetes.

.,0 •
· 9 However, amniocentesis and chorionic villus sampling are not used for diagnosis of diabetes
. during pregnancy.
Amniocentesis Congenital musculoskeletal anomalies Transposition of Great Vessels ChorionIC Villi Sampling Gestauonal Diabetes
· 11 •
· 12 •
· 13 •
Bottom line:

"..
· 14 • Trisomy 13 presents with microcephaly, microphthalmia, polydactyly, hypotonia, and heart
defects. It is caused by maternal nondi sj unction, for which advanced maternal age is a risk
· 15 •
factor.
MICrocephaly Polydactyly Muscle hypotonia Congenital Heart Defects
.· 17 •
· 18 •

." iii I ;fi , .j I.) for year: 2015


flll.ST AIO fACTS

•••
A paternity dispute leads to the testing of a mother, a child, and the potential father.
· 2

·3 Which of the following tests was most likely performed on DNA from all three parties in order to
•4 ascertain paternity?
·5
· 6
•• A. Enzyme-linked immunosorbent assay
- - - - - - - - - - - 134415 : zheng
We va lue your feedback!

•7

• •
L Il~ J
B. Ligase chain reaction
·8

.,0 •
•9 C. Northern blot
. D. Southern blot
· 11 •
· 12 • E. Western blot
· 13 •

".
· 14 •
· 15 •
The correct answer is O. 73% chose this .
.· 17 • A Southern blot is used to detect DNA by DNA-DNA hybridization. The bands can then be

.".
· 18 •
analyzed to determine if the child and the father share certain genes that the child could not
have inherited from the mother as well.
A is not correct. 5% chose this.
Enzyme-linked immunosorbent assay (EUSA) is a test of antigen-antibody hybridization u5ed to
detect an antigenic match in a patient's blood sample. EUSA is used to determine if a
,..3·2.
,Pad "" 2019 32« _ •

+ G R< UI usmle rx c om/ap l#qmax1 + III :


Item ' 7 of 44 Mark --<l t:> _! ~. C';'\I
.. . .
L •

i la b~ Cakul~ tor
.
QID 1362
. Pre Ylous
.
Next ues Notes

Enzyme· linked immunosorbent assay (ELISA) is a test of antigen-antibody hybri d ization used t o

.·5
4.. detect an antigen ic match in a patient's blood sample. ELISA is used to determine if a person's
immune system has encountered a particular antigen .
Immune system Enzyme-linked [mmunosorbent Assay

·6...
·7
.,
B is not correct. 7% chose this.
Ligase chain reaction (LCR) is a techn ique performed on mutant alleles for the pu rpose of
detecting single-point mutations. LCR is used as a diagnostic tool in patients suspected of
carrying disease genes .
., e is not correct. 10% chose this .

".
. ,0 .
.· 12 •
· 13 •
· 14 •
A Northern blot is used to detect specific RNA sequences through RNA-DNA hybridization .
E is not correct. 5% chose this .
A Western blot is used to detect proteins through protein-antibody hybridization.
Western Blot

·,5 . Bottom Line:


·16.
· 17 •
Southern blot hybridization assays use sample DNA in conjunction with labeled DNA probes,
and are often used in paternity testing.
DNA Damage
· 18 •

·19.
hli1',"'''' for year: 2015
fiRST ... ID f ... C TS

I I I

•••
detect an antigenic match in a patient's blood sample. ELISA is used to determine if a person 's
· 2
immune system has encountered a particular antigen.
·3 Immune system Enzyme-linked Immunosorbent Assa~

· 4

·5
· 6
•• B is not correct. 7% chose this.
Ligase chain reaction (LCR) is a techn iq ue performed on mutant alleles for the pu rpose of
detecting single-point mutations. LCR is used as a diagnostic tool in patients suspected of

., •
· 7 carrying disease genes.

., •
e Is not correct. 10% chose this .

.,0 •
A Northern blot is used to detect specific RNA sequences through RNA-DNA hybridization .

".
.· 12 •
· 13 •
· 14 •
. E is not correct. 5% chose this .
A Western blot is used to detect proteins through protein-antibody hybridization.
Western Blot

Bottom Li ne:
·,5.
·16.
· 17 •
Southern blot hybridization assays use sample DNA in conjunction with labeled DNA probes,
and are often used in paternity testing.
DNA Damage

· 18 •

·19. hli1',"""
fUUT ""0 F"'CTS
for year: 2015
,Pad"" 2019 32« _ •

+ G R< UI usmle rx com/ap l#qmax1 + III :


Item ' 8 of 44
QID 41S4 i
L • Mark --<l
Pre Ylous
t:>
Next
_!
la b~ ues
~.
Notes
C';'\I
Cakul~ tor

· 1
· 2
•• A 55-year old man is brought to the emergency department by police after sustaining a
laceration in a street fight. He was detained by the police for several hours before com ing to
IAA[
·3
· 4
•• the hospital, and on examination, he is diaphoretic and t remoring . His liver is firm 12 em
below the right costal margin.
·5
· 6
•• What is the biochemical process that explains the chronic changes to this man's liver?
· 7
·8
•• A. Atherosclerosis in the liver
We value your feedback!
134415 : zheng

.,0 •
· 9 B. Decreased NADH:NAO+ ratio

.".
· 12 •
· 13 •
· 14 •
.
C. I ncrease in the citric acid cycle

D. I ncreased NADH:NAD+ ratio

E. I ncreased protein synthesis

·,5 .
·16.
· 17 •
The correct answer is D. 65% chose this.
· 18 •

·19. Ethanol is broken down to acetaldehyde by alcohol dehydrogenase. This biochem ical step
requires reduction of NAD to NADH. In chronic alcoholics, the overall NADH:NAD+ ratio in
hepatocytes leads to changes in metabolic pathways such as inhibition of fatty acid oxidation.
Th is helps promote fatty changes in the liver. Prolonged fatty changes and repeated
the liver leads to scarring and cirrhosis of the liver.

•••
· 2 A is not correct. 3% chose this.
·3 Injuries to the liver that lead to cirrhosis or massive fatty change may lead to portal
· 4 hypertension, but the converse is not tru e. Occlusive changes in the portal vein do not cause
·5
· 6
•• fatty changes in the liver.
lead Fatty degeneration Hypertension, Portal Entire trunk of portal vein

· 7
·8
•• B is not correct. 24% chose this.
The NADH:NAO+ ratio increases, not decreases.

. ,0 •
NADH
· 9

.".
· 12 •
· 13 •
· 14 •
. C is not correct. 3% chose this •
In alcoholics, there is a reduction in the citric acid cycle, leading to shunting of acetyl-coenzyme
A into fatty acid synthesis.
Citric Acid Cyde

E Is not correct. 5% chose this.


In alcoholics, protein synthesis is decreased and VLDL is not able to escape the liver, leading to
·,5.
·16.
· 17 •
triacylglycerol accumulation in the liver.
Serum V\.Ol cholester~ measurement

· 18 •

·19. Bottom Line:


Fatty liver in alcoholics is due to an increased NAOH:NAD ratio.
NAOH Fatty Liver
A 27 Sis
v 43 KS/s

8
l ock
0
End Block
,..3·2.
,Pad "" 2019 32« _ •

+ G R< UI usmle rx c om/ap l#qmax1 + III :


Item ' 8 of 44 L • Mark --<l t:> _! ~. C';'\I
.
QID 41S4 i
. .. Pre Ylous
. .. .
Next la b~ ues
. .'
Notes
..
Cakul~ tor
.. . ... . . .
A into fatty acid synthesis.

.·5
4..
Citric ACid Cycle

E is not correct. 5% chose this .

·6..
In alcoholics, protein synthesis is decreased and VLDL is not able to escape the liver, leading to
triacylglycerol accumulation in the liver.
Serum VLOL cholesterol measurement

. 7 •
., Bottom li ne:
.,

.".
. ,0 . Fatty liver in alcoholics is due to an increased NADH:NAD ratio .
NADH Fatty Liver

· 12 •
· 13 •
ul;f;"'''' for year: 2015
fIRST ""D F... CTS
· 14 •

·,5 .
·16.
· 17 •
Eth"nol metabolism
FA15,p.95.1

· 18 •

· 19 .

•••
A 52-year-old man presents with a history of blisters. On phys ical examination, t he physician
· 2 notes that most of the bull ae are flat and also sees that slight rubbin g of the skin results in
·3 exfo liati on of the outer layer of skin " Immunofluorescent microscopy of a skin biopsy
· 4 specimen is performed after staini ng with anti-imm unog lobulin antibodies (see image).
·5
· 6 ••
., •
· 7

., •
• ... ... - ~/ ." .

".
. ,0 • ~ .. -...." . . ..... "-.' ,"'

'- •.\'.~·~X-c.~ · ·:i~ ~


.· 12 •
· 13 •
· 14 •
·
·

..
o
i,;'I..
t
~

'
~ '" >
.... " ~.
.
.
~.,. 10 .1. 1 •.
' . .......
<

..~

·,5 . -.' .,
·16.
· 17 •
,.

· 18 •
· 19 • Which of the following diseases, paired with the correspond in g autoantibody, is the patient most
likely suffering from?
,Pad "" 2020 32« _ •

+ G R< UI usmle rx c om/ap l#qmax1 + III :


Item ' 9 of 44
QID 2396 i
L • Mark --<l
Pre Ylous
t:>
Next
_!
la b~ ues
~.
Notes
C';'\I
Cakul~ tor

· 1
· 2
•• A. Bu llous pemphigoid + anti-type XVII collagen antibodies
134415 : zheng

·3
· 4
•• 8. Goodpasture's syndrome + anti-type IV collagen antibodies

·5
· 6
•• X C. Pemphigus vulgaris + anti-desmoglein 3 antibodies

· 7
·8
•• D. Pemphigus vulgaris + anti-type IV collagen antibodies

E. Scleroderma (CREST syndrome) + anticentromere antibodies

.,0 •
· 9

.".
· 12 •
· 13 •
· 14 •
. We value your feedback!

·,5 .
·16.
· 17 • The correct answer is C. 92% chose this.
· 18 • The autoantibody produced by patients suffering from pemphigus vulgaris is directed against

·19. desmoglein 3, a cell adhesion molecule present within desmosomes. Desmosomes are
intercellular junctions that connect epidermal cells to each other; disruption of this connection
leads to sloughing and blistering of the skin. The biopsy shown above demonstrates that there
immlmC)globuli ln5 present within the intercellular of the . Nikolsky's
I I

· 2
Oe5mo50me5 Epidermi5 PemphigU5 lmmunoglobullrl5
·3
· 4
A is not correct. 3% chose this.
Bullous pemphigoid is characterized by autoantibodies targeting type XVII collagen of
·5
hemidesmososomes. Type XVII collagen is a component of the basement membrane; therefore
· 6 a biopsy specimen from an individua l with bullous pemphigoid wou ld demonstrate a linear
· 7 pattern of immunofluorescence at the epidermal-derma l junction. A patient would typica lly
·8 present with tense bullae and not shallow, flaccid blisters.
Collagen Ba5ement membrane BullolJ5 pemphIgoid
· 9

.".
. ,0 . B is not correct. 2% chose this .
While Goodpasture's syndrome is characterized by the presence of anti-type IV collagen
antibodies, the pattern of immunofluorescence is different than the one in this image. Type IV
· 12 •
collagen is a component of the basement membrane; thus, a biopsy from an individual with
· 13 • Goodpasture's syndrome wou ld demonstrate a linear pattern of immunofluorescence at the
· 14 • epidermal-dermal junction.
·,5 . Collagen Ba5ement membrane

·16.
· 17 •
o is not correct. 3% chose this.
Anti-type IV collagen antibodies identify a patient with Goodpasture's syndrome, a disease
· 18 • primari ly of the lung and kidney in which a linear pattern of immunofluorescence at the

·19. epidermal-dermal junction would be seen.


Collagen

E is not correct. 1% chose this.


While CREST (Calcinosis. Raznaud
,Pad "" 2020 32« _ •

+ G R< UI usmle rx c om/ap l#qmax1 + III :


Item ' 9 of 44
QID 2396 i
L • Mark --<l
Pre Ylous
t:>
Next
_!
la b~ ues
~.
Notes
C';'\I
Cakul~ tor

· 1
· 2
•• epidermal-dermal junction would be seen.
Collagen

·3
· 4
•• E Is not correct. 1% chose this.
While CREST (Calcinosis, Raynaud phenomenon, Esophageal motility disorders, Sclerodactyly,
·5
· 6
•• and Telangiectasia) syndrome is associated with anticentromere antibodies, staining of this
nuclear protein would yield a different pattern of immunofluorescence than is seen here.

· 7
·8
•• MotI lIty

.,0 •
Bottom II ne:

".
· 9
. Pemphigus vu lgaris is an autoimmune disease against the intracellular junctions between
epidermal cells (desmosomes). Nikolsky's sign is seen in pemphigus vulgaris .
Oesmosomes Pempll igus
.· 12 •
· 13 •
· 14 •
Ul;fi'·'I.' for year: 2015
·,5 . ~III.ST "10 ~"CTS

·16.
· 17 •
Autoantibodies AUT04H TI BOOY ASSOCIATED ~SD ~ O(~
FA15,p . 2131

· 18 •

·19. A"ti·ACh rece ptor M )'asthe1\i ~, grJ"is


A"ti-b~,seme1\t melllhr~lI e Goodpasturc sy"drome
A"tic~rdiolipi".llIp"s all t ico~ gllbllt SLE. alltipllOspllOlipid SY'UlrolllC

•••
A 35 year-old man who is a long-term intravenous drug abuser presents to the emergency
· 2 department with a cough and fever. X-ray of the chest reveals faint bilateral interstitial
·3 infiltrates. Histologic analysis of induced sputum reveals trophozoite forms of P. jiroveci.
· 4

·5
· 6
•• Which of the following correctly states the function of the surface proteins of the virus that resulted
in this patient's current presentation?
· 7
·8
•• A. Bind to CD4 receptors
We value your f eedback!
134415 : zheng

.,0 •
· 9 B. Bind to COB receptors

.".
· 12 •
· 13 •
· 14 •
.
C. Kill B cells

D. Kill cytotoxic T cells

E. Kill T helper cells

·,5 .
·16.
· 17 •
The correct answer is A. 63% chose this.
· 18 •

·19. The etiologic agent of AIDS is the HIV virus. HIV surface proteins, gp41 and gp120, bind to (04
receptors on T helper lymphocytes. After binding occurs, the HIV is engu lfed into the cell and
the viral envelope fuses. Pneumocystis jirovecl (formerly carlnff pneumonia is an AIDS-defi ning
illness, typically occurring when a patient's (D4+ T lymphocyte count decreases to
,Pad "" 2020 32« _ •

+ G R< UI usmle rx c om/ap l#qmax1 + III :


Item ' 10of44 L • Mark --<l t:> _! ~. C';'\I
QID 1827 i Pre Ylous Next la b~ ues Notes Cakul~ tor

· 1
· 2
•• The surface proteins of HIV bind to CD4 receptors, not CDS receptors.
C is not correct. 4% chose this .
·3
· 4
•• The surface proteins of HIV bind to CD4 receptors. They do not ki ll cells.
D is not correct. 8% chose this.
·5
· 6
•• The surface proteins of HIV bind to CD4 receptors. They do not ki ll cells.
E is not correct. 16% chose this.
· 7
·8
•• The function of the surface proteins of HIV is to bind to the CD4 receptors on T lymphocytes,
allowing entry into the cells. Although the T helper cell will ultimately die from infection, it is

.,0 •
not due to the surface proteins but rather to the uncontrolled replication of the virus.
· 9

.".
· 12 •
· 13 •
· 14 •
.
Bottom line:
Pneumocystis jiroveci is an AIDS-defining illness that occurs when (D4+ counts drop below
200/fJl. The two HIVenvelope proteins gp41 and gp120 bind to CD4 receptors on T
lymphocytes and macro phages.

·,5 .
·16.
· 17 •
I il1ifi'·'I.) for year: 2015
FI II.S T AI D FA CTS

· 18 •

·19. HIV
FA15,p.I68 .1

.".
.,0 . A 24-year-old medical student presents to the clinic with a sore throat accompan ied by
lymphadenopathy. A rapid strep test is pos itive for streptococcus-induced pharyngitis. The
physician decides to give the patient penicillin, but the patient indicates that the last time
· 12 • that he was given penicillin, he experienced moderate airway edema and difficulty breathing that
· 13 • required hospitalization.
' 14 .
. ,5 •
.16.
· 17 •
Which of the following is the best choice for treating this patient?

A. Amoxicillin
We value your feedback!
13441S : zheng

.19.
· 18 •

.20 .
B. Cephalexin

C. Erythromycin
· 21 •
D. Metronidazole
· 22 •
E. Vancomyc in

.24.
· 23 •

· 25 •

·26. The correct answer is C. 53% chose this.


In a penicillin-allergic patient (such as the patient described), erythromycin is considered the
next best choice. Erythromycin is a macrolide antibiotic that has activity against gram-positive
organ isms (especially Pneumococcus, Streptococcus, Staphylococcus, and Corynebacterium),
EnteroCOCCi, and gram-negative organisms.
,Pad "" 20 20 32 Y0 . •

+ G R< UI usmle rx com/ap l#Qmax1 + • :


all

.
Item 11 of 44 L • Mark -<l t>- ~. ~'t
QID 362J J. PreYlDU' NeKt lab"" ues Notes c .. ku l~tor

"0
." .
In a penicillin-allergic patient (such as the patient described), erythromycin is considered the
next best choice . Erythromycin is a macrolide antibiotic that has activity against gram-positive
organisms (especially Pneumococcus, Streptococcus, Staphylococcus, and Corynebacterium),
Enterococci, and gram-negative organisms.

..
· 12 •
· 13 • Erythromycifl

"4.
"5
A is not correct. 10% chose thi s.
AmoxicilHn is a penicillin-based drug. Given that the patient describes an anaphylactoid reaction
., to penicillin, treating his pharyngitis with amoxieiliin is not an option .
Amo~i(1 lin Anaphylactoid reaction Pharyng itiS
· 17 •

.
' 18 • B is not correct. 19% chose thi s.
· 19 • Cephalexin is a cephalosporin, a class of antibiotics related to the penicillins. Although not
every patient who is allergic to penicillins is allergic to cephalosporins, there is a significant
.20 . cross-over (5%-10%). Given this cross-allergy, cephalosporins would not be a good choice for
." treatment of the patient's pharyngitis. This reflects the current standard of treatment for all
· 22 • patients with penicillin or cephalosporin allergies.
Cephillexifl Ph<lryng'tis
· 23 •
.24 . o is not correct . 4% chose thi s.
· 25 • Metronidazole is an antibiotic that forms toxic metabolites in the bacterial cell. It is typically
' 2• • used to treat protozoal infections and anaerobic infections from Clostridium or Bacteroides
species. It is not used to treat streptococcal infections.
Metronidilzole Protozoan Infections Strept<xocca l [nfecuoll5

E i s not correct. 14% chose th is.


Vane·orrlve;n inhibits cell wall formation by binding to D-a la D-a la portion of cell wall

"0 .
." .
Vancomycin inhibits cell wall formation by binding to D-ala D-ala portion of cell wall precursors.
It is used to treat serious infections with gram-positive, multidrug-resistant organisms such as
Staphylococcus aureus and Clostridium difficile. Because the organism causing strep throat
(Streptococcus pyogenes) is susceptible to penicillin, using a drug reserved for more viru lent

...
· 12 •
pathogens wou ld be unreasonable. Furthermore, vancomycin is administered intravenously and
· 13 •
would require intravascu lar access. Thi s option would lead to unnecessary procedures and
' 14 . inconvenience in a stable patient who will benefit from outpatient follow-up and ora l antibiotics.

"5
.,
· 17 •
Streptococc<ll sore thrOilt

Bottom line:
.,
"9.
." .
. 20 .
' 22 •
· 23 •
.24 .
Streptococcus infections are best treated with penici llin; however, if the patient is allergic to
penicillin, then erythromycin is the next-best choice .
Erythromy<in

iii I jfj ,
FIRST AIO FACTS
.j 1.1 for year: 2015
· 25 • FAI5.p.I86 .1
' 2• • M OIcrolides t\ ;o;ithromyc in. clarithro111)'cin. erythro111)'ci n.
MWIAlIISM Inhibit prote in synthes is by blocking tran slocati on ("'llacroslides"); bind to the 23S rR NA of th e
50S ril)Qson);,1 subunit. Bacter iostatic.
(lIN I(AlUS< t\t ),pic;1i pneUlnonia ~ (fo,"rcop/asllw. Clrlamrdia. Legioneil<l). STIs (Chlamrdi<l). gram·positive cocc i
(streptococcal infec tions in patients ;,lIergic to penicillin ). ;md B, pertussis
,Pad "" 2020 32« _ •

+ G R< UI usmle rx c om/ap l#qmax1 + III :


Item ' 12 of 44
QID 4431 i
L • Mark --<l
Pre Ylous
t:>
Next
_!
la b~ ues
~.
Notes
C';'\I
Cakul~ tor

IAA[
.".
"0. The mean blood glucose concentration for a cohort of 100 infants is 95 mg/dL. The standard
deviation for these measurements is 15 mg/dL.

..
· 12 •
What percentage of glucose measurements falls between 65 and 125 mg/d l ?
· 13 •

' 14 . 134415 : zheng


We value your feedback!
A. 100%
" 5 •
., B. 5%
· 17 •
., C. 68.7%

.".
"9.
.20 .
' 22 •

· 23 •
. 24 .
0. 95%

E. 99.7%

The correct answer is O. 75% chose this .


. 25 . This question is asking the reader to select the percent distribution that encompasses 2
' 2• • standard deviations (50s) above and below the mean. For this example, the SO is 15, so twice
the SO is 30, yielding the range 65 to 125 mg/dL [(95 - 30), (95 + 30)]. The 95% confidence inteIVal
is ±1.96 times the 50. The blood glucose measurements of 65 and 125 mg/dL fall roughly within
this range.

.".
"0. A is not correct. 3% chose this.
A 2-50 range around the mean does not hold all measured va lues .
B is not correct. 3% chose this.

..
· 12 •
· 13 • Th is is the inverse of the correct answer because only 5% of values fall outside this range.
' 14 . C is not correct. 9% chose this.

"5
., .
· 17 •
This percentage (68.7%) of measurements fall within 1 50 around the mean. In ot her words,
68.7% of measurements fall between 80 and 110 mg/dL.
E is not correct. 10% chose this.

., This percentage (99.7%) of measured va lues fall within 3 50s around the mean. In this case,

." .
" 9 •
. 20 .
' 22 •

· 23 •
99.7% of values fall between 40 and 140 mg/dL.

Bottom Li ne:
For a norma l distribution, 68% of the va lues fall within 1 50 from the mean, 95% of the val ues
fa ll with in 1.96 50s, and 99% of the values fall within 2.58 50s.

~
. 24 .
. 25 . uj;fj,·jj' h oryear: 2015
' 2• • fIR S T ... 10 f ... CTS •

FA15, p. 53. 1

Statlstlul distribution
,Pad"" 2021 32« _ •

+ G R< UI usmle rx com/ap l#qmax1 + III :


Item ' 13 of 44
QID 4129 i
L • Mark --<l
Pre Ylous
t:>
Next
_!
la b~ ues
~.
Notes
C';'\I
Cakul~ tor

IAA[
.".
"0. A 21-year-old man presents to the clinic with a painless ulcer on his genitals. Upon
questioning, the doctor discovers that he was hospitalized as a child after developing a fever,
a rash. and trouble breathing after being treated for an ear infection. The doctor explains

..
· 12 • that he cannot have a certain antibiotic to treat the current infection because of his previous
· 13 • adverse reaction.
' 14 .

" 5 • What is the mechanism of action of the antibiotic that is contraindicated for this patient?
., 134415: zheng
· 17 •
.,

.".
"9.
. 20 •

' 22 •

· 23 •
. 24 .
A. Binds to the 305 subunit and prevents attachment of aminoacyl-tRNA

B. Blocks cell wall synthesis by inhibition of peptidoglycan cross-linking

C. Blocks peptidoglycan synthesis

D. I nhibits bacterial dihydrofolate reductase

E. I nhibits protein synthesis by blocking translocation


. 25 .
' 2• •
We value your feedback!

i, i i

.".
"0. penicillin with respect to allergies . The patient presented with a typi cal symptom of syphilis (a
painless ulcer), for which the treatment is penicillin. The mechanism for pen icillin is that it
blocks cell wa ll synthesis by inhibiting peptidog lycan cross-linking.

..
· 12 •
Amo~icllhn
· 13 •

' 14 .
A is not correct. 5% chose this.

"5
., .
· 17 •
Th is is the mechanism of action for tetracyclines, which is not contraindicated by a penicillin
allergy.
C Is not correct. 17% chose this.

., This is the mechanism of action for vancomycin, which is not contraindicated by a pen icillin

.
allergy.

"9.
. 20 .
o is not correct. 4% chose this .
This is the mechanism of action for trimethoprim, which is not contraindicated by a peni cillin
." allergy.
' 22 • E is not correct. 3% chose this.
· 23 • This is the mechanism of action for mac rolides, which are not contraind icated by a penicillin
. 24 • allergy.
. 25 .
' 2• • Bottom II ne:
Antibiotics wit h the same mechan ism of action as penicillin (ie, amoxicillin and
cepha losporins) are contraindicated in patients who are allergic t o penicillin.
Amoxici llin
,Pad "" 2021 32« _ •

+ G R< UI usmle rx c om/ap l#q max1 + III :


Item ' 14of44 L • Mark --<l t:> _! ~. C';'\I
QID 3416 i Pre Ylous Next la b~ ues Notes Cakul~ tor

.".
"0. A 73-year-o ld man with a history of renal artery stenosis arrives at his primary care
physician's office complaining of a headache and nausea. While in the waiting room, he
becomes confused and restless. He is seen immediately. and the physician notes

..
· 12 • papilledema and retinal hemorrhages on the ophthalmologic examination. The patient is
· 13 • immediately transported to the emergency department by ambulance. On arrival, his heart rate is
' 14 .
95/min, temperature is 36.7°( (98°F), respiratory rate is lS/min, and blood pressure is 230/160 mm

"5
., .
· 17 •
Hg. The intern on call begins an intravenous infusion of sodium nitroprusside and sodium
thiosulfate .

What physiologic complication is the intern trying to prevent by admini stering the sod ium
.,

.
thiosulfate?

"9.
.20 . A. Arsenic poisoning
13441S : zheng

." B. Cyanide poisoning


We value your feedback!
' 22 •

· 23 • C. Dissipation of the mitochondrial proton gradient


.24 . O. Presynaptic neuromuscular blockade
. 25 .
' 2• • E. Producti on of carbon monoxyhemoglobin

i i i i i

.".
"0. Malignant hypertension is a complication of hypertension characterized by very elevated blood
pressure and organ damage in the eyes, brain, lung, and/or kidneys. It differs from other
complications of hypertension in that it is accompanied by papilledema, as in this patient.

..
· 12 •
Systolic and diastolic blood pressures are usually >200 and >140 mm Hg. respectively. as is the
· 13 • case with this patient. Cyanide is produced by the metaboli sm of nitroprusside and can lead to
' 14 . toxicity by inhibiting cytochrome oxidase. This leads to a block in the electron transport chain.

"5
., .
· 17 •
resulting in decreased oxygen utilization, lactic acidosis, and possibly death. By administering
sodium thiosulfate. the intern is hoping to convert the cyanide ion into thiocyanate. Rhodanese
is the enzyme that catalyzes the reaction in which cyanide and thiosulfate combine to produce
thiocyanate .
.,

.
Metabolism Nitroprl.l5side Oxygen Sodium NitroprU5slde MaligrKInt Hypertension In<:rease in ~ood pressure PapH ledema Acidosis Sod ium

"9.
. 20 .
A is not correct. 4% chose this .
Arsenic is not a byproduct of nitroprusside metabolism .
." Arsenic N,troprl.l5side Metabolism

' 22 • C Is not correct. 9% chose this.


· 23 • 2,4·Dinitrophenol (DNP) destroys the mitochondrial membrane's proton gradient, thereby
.24 . uncoupling electron transport from ATP synthesis. This promotes the mobilization of
. 25 . carbohydrates and fats. and is the reason why DNP was used as a diet pill in the 1930s.
However. many patients died as a result of taking the drug and its popu larity faded rapidly. It
' 2• •
does not have the same mechanism of action as cyanide.
Mllocr.ondrial Membranes

o is not correct. 5% chose this.


The metaboli sm of nitroprusside has no relation to presynaptic neuromuscu lar blockade .
,Pad "" 2021 32« _ •

+ G R< UI usmle rx c om/ap l#q max1 + III :


Item ' 14of44 L • Mark --<l t:> _! ~. C';'\I
QID 3416 i Pre Ylous Next la b~ ues Notes Cakul~ tor

".
"0. The metabolism of nitroprusside has no relation to presynaptic neuromuscular blockade. This IS
the mechanism of acti on for bot ulinum toxin .
Metabol,sm N,troprlJS5ide Injection of botulinum (OXII'l ObselVation of Neuromuscular Block

..
· 12 •
E Is not correct. 7% chose this.
· 13 •
Sod ium nitroprusside does not lead to the production of carbon monoxyhemoglobin. This
' 14 . substance is actually produced in ca rbon monox ide poisoning, and patients present with cherry

"5
., .
· 17 •
red lips and a pinkish hue to their skin.
Sodium Carbon Monoxide

.,

.
Bottom li ne:

"9.
. 20 .
Sodium nitroprusside can result in the creation of cyanide, which inhibits cytoc hrome oxidase,
leading to toxicity. Th is can be count ered by using sodium thiosulfate concurrently.
." Sod ium

' 22 •
· 23 •
. 24 . hl;fi,·,.·1 for year: 2015
FIRST ,0.10 FACTS
. 25 .
' 2• • FA15, p . 304 4

Hypertensive Drugs includc clcl'idipinc. fCl1oldopam.1abctalol. ni cardipin c. nitroprusside.


emergency
Nitroprusside Short ;Icling; I cC~dP via direct releasc of NO . COl" ca lise cyanide toxicit ), (releases cpnidej.
Fenoldopam Dopaminc DI rcccptor agonist - coronary. periphcral. rcnaL and splanch nic \·asodila tion. I Br.

"0.
.".
A 6·year-old boy is brought to the pediatrician by his mother, who is very concerned about
his health. Physical exam ination shows an extensive erythematous reticular skin rash on the
face, trunk. and extremities, as seen in the image. He also exhibits swelling around the

..
· 12 • wrists that causes him pai n on moveme nt at the joint. He is in no acute distress, but his mother is
· 13 • very anxious.

"4.
" 5 •
.,
· 17 •
.,

.
" 9 •
. 20 •

."
· 22 •
· 23 •
. 24 •
Which of the following is the best next step in treatment for this illness?

. 25 . 134415: zheng

' 2• •
X A. No treatment necessary; it is a self.limiting disease

B. Presc ribe a corticosteroi d cream

C. Prescri be an oral corticosteroid


,Pad "" 2021 32« _ •

+ G R< UI usmle rx c om/ap l#q max1 + III :


Item ' 1Sof44 L • Mark --<l t:> _! ~. C';'\I
QID 1845 i Pre Ylous Next la b~ ues Notes Cakul~ tor

.".
"0. The correct answer is A. 72% chose this .
Th is description is class ic for fifth disease, wh ich is caused by parvovirus 819. It is a pediatric
ill ness common in child ren 3-12 years old. The rash is called erythema infectiosum and develops

..
· 12 •
after fever has reso lved as a bright, bla nchable erythema on the cheeks ("slapped cheeks'1 with
· 13 •
perioral pa ll or. A more diffuse rash appea rs on the t ru nk and extremities and may wax and
' 14 . wane wit h tem perature changes over 3 weeks. However, the disease itself is self-limiting,

"5
., .
· 17 •
requiring no treatment . I n ad ults, t he disease more freq uently causes joint pain. It ca n be
dangerous for patients wit h sickle cell anemia beca use the vi rus can infect erythroid precursors
and inhibit bone marrow production of R8Cs, resulting in profound anem ia.

., Erythema Anemiil Erythema lnfectiosum Anemiil. Sid le Cell Bone Marrow

.
"9.
. 20 .

."
' 22 •

· 23 •
.24 .
B is not correct. 9% chose this.
The ras h seen in fifth disease will dissipate on its own after a few weeks and wi ll not respond to
corticosteroid cream .
C is not correct. 14% chose this.
Although the rash of erythema infectiosum is a centrally distributed mac ulopapular eruption
(li ke the rashes that accompa ny rickettsial illnesses, drug.ind uced eruptions, and Sti ll's
diseases), it is self. limiting, requiring no treatment. There is a role for the use of oral
corticosteroids in th e treatment of Still's disease, wh ich is a manifestation of juvenile
. 25.
rheumatoid arthritis that presents with high spi king fever, rash witho ut itching, and arthritis.
' 2• • Th is patient has no fever, making Still's disease unlikely.
Arthritis ErytherTlil lnfectiosum Maculop<lpular eruption Ar1t1ritis.Jwenile Rhellmatoid

D is not correct. 5% chose this.


Erythema infectiosum is caused by infection wit h parvovirus 819. Viral illnesses do not

i i ii
Th is patient has no fever, making Still's disease un,, ,,,,y.
Arthritis ErytherTlil Infectiosum Milculop<lpulilr eruption Anhritis.Jwenile Rhellmiltoid

..
D is not correct. 5% chose this.
· 13 • Erythema infectiosum is caused by infecti on wit h parvovirus 819. Viral illnesses do not respo nd
' 14 . to anti biotics.
erythema lnfectiosum
" 5 •
.,
· 17 • Bottom li ne:
.,

.
"9.
. 20 •

."
' 22 •

· 23 •
. 24 .
~\RST
Fifth disease (erythema infectiosum) is a self-limited condition caused by parvovirus 819.
ErytherTlil Infectiosum

Iii lif, ,
"'\0
., I.) for year: 2015
~"'CTS

FA1 S . p, t 58, 1

. 25. DNA viruses


VIULf~I4 I LV UIV(LIl4'( ON ~ 51ROCTUR( I4£DK~L l l4f'O Rl~N ((
' 2• •
Herpesviruses ¥\:s DS and lin e", IISV· I _ur..J (a nd sumc gC lli tal) lesiuns. spu ntancuus
tcmpom llo~ cncephalitis, kcratoconjunctivitis
HSV-2 -gcnit al (<lnd some ora l) lesions
VZV (I II-I V.,)-d .ickc1l1)()X. ws tcr (shingl\:s)
EBV (1II IV.i )-mononucleosis, Burkit t l)'I llphoma.
I ",.'nph,,,,,". ,," ~.ph")'''g''' '
,Pad "" 2022 32« _ •

+ G R< UI usmle rx c om/ap l#q max1 + III :


Item' 16of44 L • Mark --<l t:> _! ~. C';'\I
QID 1724 i PreYlous Next la b~ ues Notes Cakul~tor

IAA[
.".
"0. A 26-year-ol d medical student undergoes a PPD test after com pleting a rotation in Bot swana.
The test is posit ive, so she undergoes x-ray of t he chest, which shows no si gns of active or
prior lung disease. She is HIV negative with no other medical issues. The clinician prescribes

..
· 12 • a 6-mo nth course of monotherapy prophylaxis aga inst tuberculoS iS.
· 13 •

' 14 . Which of the following is the mechanism of action of the drug t hat was most likely prescribed?

"5
., .
· 17 •
134415 : zheng

., A. Bi nds directly t o bact erial cell membra nes, leading t o membrane disruption

.
"9.
.20 .
."
' 22 •

· 23 •
. 24 •
"
B. I nhibits DNA-dependent RNA polymerase

C. I nhibits ergosterol synthesis, allowing leakage of ions through membrane pores

O. I nhibits mycolic acid synth esis, interfering with cell wall maintenance

E. I nterferes with microtubule function, ca using disruption of mitos is

. 25 .
' 2• • We value your feedback!

I I
tuberculosis. drug effectively penetrates human I and interacts wit h intracellul ar
pathogens. The precise mechanism of action of IN H is unknown, but the end res ult is the
disruption of mycolic acid synth esis . I NH is also used as tu berculosis prophylaxis in patie nts

..
with a positive PPD test . The most frequent serious adverse effect is hepatitis, especia lly in
· 13 • patients with prior liver disease.
' 14 . Tuberculosis Hepatitis

"5
., .
· 17 •
A is not correct. 5% chose this .
Polymyxins act like detergents, binding t o and disrupting bacterial cell membranes. They are
used to treat resistant infecti ons with gram-negative rods .
.,

.
B is not correct. 16% chose this.

"9.
.20 .
Rifampin in hibits bacterial RNA synthesis by bind ing to the ~ subunit of DNA-dependent RNA
polymerase, block ing RNA transcription . Rifampin is used as part of a multidrug therapy to treat
tuberculos is.
." lUfampin Tuberculosis
' 22 •
C is not correct. 5% chose this.
· 23 •
Amphotericin Band azole antifungal agents (as we ll as many others) inhibit ergosterol
. 24 .
synthesis, an important step in fungal cell mem brane prod uction . INH has no effect on
. 25 . ergosterol synthesis.
' 2• • Amphotericin B Antifungal Agents Plasma membrane

E Is not correct. 3% chose this.


Griseofulvin is an antifungal drug t hat disrupts mitosis by interfering with mi crot ubu le function.
It is primarily used to treat hair and nail infections.
,Pad "" 2023 32« _ •

+ G R< UI usmle rx c om/ap l#q max1 + III :


Item' 16of44 L • Mark --<l t:> _! ~. C';'\I
QID 1724 i PreYlous Next la b~ ues Notes Cakul~tor

".
E IS not correct. 3% chose thi s.

"0
.
. Griseofulvin is an antifungal drug that dis rupts mitosis by interfering with microtubu le function .
It is primarily used to treat hair and nail infections.

..
· 12 • Mitos is

· 13 •

' 14 .
Bottom Line :

"5
., .
· 17 •
JN H is an anti mycobacterial drug used to treat t uberculosis that acts by inhibit ing mycolic acid
synthesis, interfering with cell wa ll maintenance.
Tuberculos is

.,

.
"9.
. 20 .

."
' 22 •

· 23 •
. 24 .
141 ;fi ,
fI RS T AI D FA CTS

Isoniazid
/II((H.\lIISIoI
., I.) for year: 2015

I srnl hcsis of mycolic acids. lJaclcria l catalase-


FAtS,p.I89.1

peroxidase (encoded h)' K~tC) needed 10


. 25 . com-e.! INII to <lctive metabolite.
' 2• • (lIN I(AlUSl: MycoJwderium tuberculosis. Th e only age nt Different 11'1 11 h~lf.li\"e5 in f;.st n _slow
nsed as wlo proph)'laxis aga i nst TB. acetylator!;.
1011(111 Ncuroto~icit )', h cp<.toto~ ic;ty. Pyridox ine 11\'11 Iniu res Neu rons and Il ep,I\oc)'tes
(,·ita",i" B6 ) can pren:nt nellroto~icity.

MWWIISMOf IlSiSIAII([ l'o'I utations leading to undere~pressio" of KaIC .

IAA[
.".
"0. A 16-year-ol d boy is brought to the clin ic by his mother after wrestl ing with his older brother.
On physical exam ination he appears to have left wrist drop and numbness on the dorsum of
his left hand.

..
· 12 •
· 13 •
Whic h nerve and vessel are most likely damaged in th is pati ent?
' 14 .

"5
., .
· 17 •
A. Axillary artery, median nerve

8. Ax illary artery, ulnar nerve


We value your feedback! ) 134415; zheng

.,

.".
"9.
. 20 •

' 22 •

· 23 •
. 24 •
X
C. Radial collateral artery, med ian nerve

D. Radial collateral artery, radial nerve

E. Ulnar artery, radial nerve

F. Ulnar artery, ul nar nerve

. 25 .
' 2• •
The correct answer Is D. 67% chose this.
The radia l collateral artery is a bra nch of the deep brac hial artery and runs in the radial groove
of the humerus along with the radial nerve. The radial nerve originates from C5, C6, (7, C8, and
T1 of the brachial plexus. It innervates the posterior compartments of the arm and
all owing for extension of the wrist; the radia l nerve also innervates the posterior cut:aneci
,Pad "" 2023 32« _ •

+ G R< UI usmle rx c om/ap l#q max1 + III :


Item ' 17 of 44 L • Mark --<l t:> _! ~. C';'\I
QID 2767
-
i
. .' Pre Ylous
.. . ,.
Next
. la b~ ues Notes Cakul~ tor

.".
"0. The radial collateral artery is a branch of the deep brachial artery and runs in the radial groove
of the humerus along with the radial nerve. The radial nerve originates from (5, (6, (7, C8, and
T1 of the brachial plexus. It innervates the posterior compartments of the arm and forearm,

..
· 12 •
allowing for extension of the wrist; the radial nerve also innervates the posterior cutaneous
· 13 • hand except for the distal digits and the last one and a half fingers. In this case the boy has
' 14 . wrist drop (inability to extend the wrist), suggesting a radial nerve injury. Injury to the radial

"5
., .
· 17 •
nerve should be suspected in a patient with a midshaft humerus break.
Structure of radial nerve Brachial plexus structure

A is not correct. 3% chose this.


.,

.
The axillary artery is proximal to the midshaft of the humerus. Damage to the median nerve

"9.
. 20 .
would produce deficits in flexion of the hand and fingers and sensory deficits in the thumb,
lateral palm, and fingers .
B is not correct. 3% chose this .
." The axillary artery becomes the brachial artery and is proximal to the midhumerus. Damage to
' 22 • the ulnar nerve would cause loss of many intrinsic movements of the hand.
· 23 • C Is not correct. 7% chose this .
. 24 . The median nerve innervates muscles that flex the fingers and pronate the forearm. It runs
. 25 . along the medial aspect of the arm with the brachia l artery and veins and is not closely
associated with the humeru s. It can become damaged when the humerus is broken more
' 2• •
distally, such as in a supracondylar fracture of the elbow. The median nerve is the only nerve to
pass through the carpal tunnel, and an inability to flex the fingers may result from compression
as in carpal tunnel syndrome. Pronation is not affected in carpal tunnel syndrome, as the
muscles for pronation are proximal to the location of compression. The median nerve I
provides cutaneous innervation to the palmar aspect of the first three and half digits and

.".
"0. as in carpal tunnel syndrome. Pronation is not affected in carpal tunnel syndrome, as the
muscles for pronation are proximal to the location of compression. The median nerve also
provides cutaneous innervation to the palmar aspect of the first three and half digits and the
dorsal aspect of the distal tips of these digits.

..
· 12 •
Carpal Tunnel Syndrome
· 13 •

' 14 . E is not correct. 17% chose this.

"5
., .
· 17 •
Damage to the radial nerve would resu lt in the nervous deficit that is described in the vignette.
However, the ulnar artery splits off the brachial artery distal to the elbow and thus would not be
typically injured with a humerus fracture.
Structure of radial nerve
.,

.
"9.
.20 .
."
' 22 •

· 23 •
. 24 •
F is not correct. 3% chose this.
The ulnar nerve runs along near the median nerve down a more superficial course than the
radial nerve. It innervates multiple intrin sic hand muscles, and a deficit would cause a loss of
finger movements such as grip, not a wrist drop. The ulnar nerve also provides cutaneous
innervation to the last one and a half digits, on both the palmar and dorsal aspects. And as
noted above, the ulnar artery typically is not injured with humerus fractures.
Structure of radial nerve Humeral Fractures

. 25 .
' 2• • Bottom line:
Injury to the radial nerve should be suspected in a patient with a midshaft humerus break and
wrist drop.
Structure of radial nerve
,Pad "" 2023 32« _ •

+ G R< UI usmle rx c om/ap l#q max1 + III :


Item ' 17 of 44
QID 2767 i
L • Mark --<l
Pre Ylous
t:>
Next
_!
la b~ ues
~.
Notes
C';'\I
Cakul~ tor

.".
"0. FA15,~.418.1

Upper eKtremity nerves


NERVE <AlISlIOf INIURY PRfl,(NTW(lH

Axillary «(5-(6) Fractured ~urgica lllcck of humerus; 311 tcrior Fbttelled dell oi d

..
· 12 •
disloca tion ofll1l1llems Loss of arm ~hduelion ,Ii shoulde r (> 15 degrees)
· 13 • Loss of sens,liion o"er delloid musc le and l;,jcr;11
arm
' 14 .

"5
., .
· 17 •
Musculocutaneous
(C5 -Cll
Radial «(S -Tl )
Upper (milk compression

l\'l idsh~ft
fractu re of hUlllerus; COtll]lrcssion of
axilb. e.g., du e to crul ch~ or sk'('pi ng wi th
Loss of forearm fl ex ioll and sl ' pination
Loss of sensation OIW lateral forearm
\Vri st drop: loss of elbow, \\'rist, and fi nger
extens ion

., ann oye r cllai r ("Sahmlay n ight palsy") I gr ip strength (wrist ext ension necessary for

.
maximal action of fl exors)

"9.
.20 . Median «(S-Tl ) Snpracondylar fractu re of hUlllerus (proxima l
Loss of sensation o ....'r posterior arm/forearm and
dorsa llmnd
"Ape ha nd~ and "Popes blessing"

." lesiQn); carpal hlllrlci syndrome and wrist


1;lCcrat ion (disl;.llcsio ll)
Loss of wrist fl l'Xion , flex ion of bter,,1 fin gers,
thnmh oPIXIS ition, luml)f icals of 2nd ;md 3rd
' 22 • digits
Loss of sensation ol'er thenar emine nce and
· 23 •
dorsa l and palmar aspects of btera l 3\1: fin gers
. 24 . wi t h proximallesioll
Tinel sig n (tingling on pcrcmsion) in carpal
. 25 . tunnel syn drome
' 2• • Ulnar «(8-11 ) Fracture of mcd iall'picond)'lc of hllm rru~ "Ulna r claw" on digit ex tension
"(UIll!), bono: - (I'rOxilllall<:sioll); fradlln:d 1'001: Radial d(" 'ial ;OI' of" rist "IXJn flexiOl ' ("roxi",,,1
of hamate (Ilistallcsion) lesion)
Loss of wrist flexion. flex ion of medial fin gers.
abduction and adduction of fingers (in terossei),
actions of mcdial 2 1u11lhricallll uscies

.".
"0. A 46-year-old woman is found to have a prolapsed fallopian tube and underqoes
hysterectomy, Five weeks later, a speculum examination reveals friab le red tissue in the
vaginal vau lt , which is identi fi ed as granulation tissue ,

..
· 12 •
· 13 •
Histologic analysis of the tissue over time will show an increase of a molecule with what structure?
' 14 .
134415; zheng
" 5 •
.,
· 17 • A. A glycoprotein that can be denatured and electrophoresed into two distinct
., bands

." .
" 9 •
. 20 .
' 22 •

· 23 •
. 24 •
B. A polymer of cross ~ sheets
C. A polymer with a backbone consisting of phosphate and suga r groups

D. A tetramer of four protein subunits constituting a heterocyclic ring

E. Repeated blocks of glycine-X-Y, with X and Y being proline, hydroxyproline, or


hydroxylysine
. 25 .
' 2• • We value your feedback!
,Pad "" 2023 32« _ •

+ G R< UI usmle rx c om/ap l#q max1 + III :


Item ' 18 of44 L • Mark --<l t:> _! ~. C';'\I
QID 4OS8 i Pre Ylous Next la b~ ues Notes Cakul~ tor

.".
"0. Granulation tissue is the perfused fibrous connective tissue that replaces a fibrin clot in a
healing wound. Collagen forms its main stru ctural support. As wound healing progresses, type
III collagen is replaced with type I collagen to form an avascular scar. Collagen is composed of
glycine-X-Y polypeptid es, with X and Y being proline, hydroxyproline, or hydroxylysine.

..
· 12 •
· 13 • Procollagen is a precursor of collagen, and is not a major organizational component of
granu lation tissue.
' 14 .

"5
., .
Collagen GfyClfle Proline Hydro)(j'proline measuremem Fibrous connective t issue

A is not correct. 10% chose this •


Immunoglobulins are composed of light and heavy chains. Each chain has a constant and a
· 17 •
., variable region. The number of antibodies would be expected to decrease in the final stages of

.
wound healing.

"9.
.20 .
lmmunoglobll lH1S Wound healed (findill9)

B is not correct. 11% chose this .


." ~-Pleated sheets are very common seco ndary molecular structures, but extracellularly are most
' 22 • notable for being the structure of amylOid. Amyloid is pathologic and not involved in wound
· 23 • healing. Cross ~ sheets are the quaternary structure of amyloid.
Wound healed (finding)
.24 .
. 25 . C is not correct. 9% chose this .
' 2• • DNA, or deoxyribonucleic acid, is a polymer consisting of nucleotides attached to an alternating
sugar and phosphate backbone. It would be present in decreasing quantity as wound healing
progresses.
D is not correct. 9% chose this.
Hemoglobin is the oxvo',n-ca,rrvina

.".
"0. Hemoglobin is the oxygen-carrying molecule in blood. As wound healing progresses, perfusion
decreases as vascular granulation tissue is replaced by avascular scar.
Orygen Perfusioo (procedure)

..
· 12 •
· 13 •

' 14 . Bottom Line:

"5
., .
· 17 •
Collagen is composed of glycine-X-Y (with X and Y being proline, hydroxyproline, or
hydroxylysine), and strengthens extracellu lar matrix in bone, skin, tendon, carti lage, basement
membrane, and granulation tissue.
., CoHagen Glycine Proline Hydroxyproline measurement Extracellular Cilrtililge Basement membrane

." .
" 9 •
. 20 .
' 22 •

· 23 •
.24 .
ul;f." ·,,,,
cts
for year: 2015
fiRST ... I D f ...

WDund healing
..,
FAtS, p . 229 .2

Tissue mediators MUlWOR

. 25 . I'DCF SeerctNi by activated platelets and nJaerophages


Induces vascu la r remodeling and smoolh muscle
' 2• • cell mi gra tion
Slimulaies fibroblast growlh for collagen sy nlh esis
FCF Stimulates angiogenesis
ECF
,Pad "" 2023 32« _ •

+ G R< UI usmle rx c om/ap l#q max1 + III :


Item ' 190f44 L • Mark --<l t:> _! ~. C';'\I
QID 3903 i Pre Ylous Next la b~ ues Notes Cakul~ tor

.".
"0. A type of bunyavirus seen in the United States is characterized clinica lly by a prodromic
phase of nonspecific symptoms including fever, myalgia , headache, nausea, and vomiting .
This initial phase is followed by a very rapid transition to a cardiopulmonary phase

..
· 12 • characterized by pulmonary edema (due to leaky pulmonary capillaries), hypotens ion, and
· 13 • coagulopathy.
' 14 .

" 5 • What is a natural reservoir for this pathogen?


., We value your feedback!
134415 : zheng
· 17 • A. Bats
.,

.".
"9.
. 20 •

' 22 •

· 23 •
. 24 •
B. Rodents

C. The reservoir is unknown

D. Ticks

. 25 . The correct answer is B. 65% chose this.


' 2• • The clinical scenario describes hantavirus cardiopulmonary syndrome. Rodents are the natural
reservoir of the hantavirus. Contrasting with most other bunyavirus, which are transmitted by
arthropod vectors (mosquitoes, ticks, or sand flies), transmission of hantavirus occurs through
contact with deer mice feces. In the United States rare cases of human disease have been
described in rural areas of the Southwest. The only other bunyavirus that might present
Crim"ar,-Conao hem"rrha"iiicfever. This i however is not

.".
"0. The clinical scenario describes hantavirus cardiopulmonary syndrome. Rodents are the natura l
reservoir of the hantavirus. Contrasting with most other bunyavirus, which are transmitted by
arthropod vectors (mosquitoes, ticks, or sa nd flies), transmission of hantavirus occurs through

..
· 12 •
contact with deer mice feces. In the United States rare cases of human disease have been
· 13 • described in rural areas of the Southwest. The only other bunyavirus that might present with
' 14 . similar symptoms is Crimean-Congo hemorrhagic fever. This virus, however, is not known to be

"5
., .
· 17 •
present in North America (rather, Africa, the Middle East, and Eastern Europe).
A is not correct. 14% chose this .
It has been hypothesized that fruit bats are a reservoir for the filovirus ebola in sub-Saharan
., Africa. They are not a reservoir for hantavirus .

.".
"9.
. 20 .

' 22 •

· 23 •
. 24 .
C is not correct. 8% chose this.
Rodents are the known reservoir for hantavirus .
D is not correct. 13% chose this .
Ticks are the vector for a number of viral, bacterial. and protozoal diseases. They are not,
however, the reservoi r or the vector for hantavirus.

Bottom Li ne:
. 25 .
Hantavirus is carried by rodents, and presents with nonspecific symptoms followed by
' 2• •
pulmonary edema, hypotension, and coagulopathy.
Hypotension Pulmonary Edeffiil
,Pad "" 2024 32« _ •

+ G R< UI usmle rx com/ap l#qmax1 + III :


Item ' 20 of 44
QID 1940 i
L • Mark --<l
Pre Ylous
t:>
Next
_!
la b~ ues
~.
Notes
C';'\I
Cakul~ tor

.".
"0. A 7+year-old boy comes to the pediatric clinic for the evaluation of a sore throat and fever
that have persisted over the last few days. Th is morning his mother also noticed the
appearance of a sandpaper-like rash on his neck and chest. A rapid strep test comes back

..
· 12 • positive. The boy is started on amox icillin, which follows first-order elim ination.
· 13 •

' 14 . Which of the following statements describes characteristics of first-order elimination?

"5
., .
· 17 •
A. The amount of drug eliminated per unit time is constant
134415 : zheng

., B. The fraction of drug eliminated per unit time is variable

.".
"9.
.20 .
' 22 •

· 23 •
. 24 .
"
C. The plasma concentration of the drug decreases linearly with time

D. The rate of elim ination is independent of drug concentration

E. The rate of elimination is proportional to the drug concentration

We value your feedback!


. 25 .
' 2• •

eiinei:ab;Qljism i enzymes, i i
most clinica l situations, the concentration of the drug is much less than the Michaelis constant
(Km), 50 the Michaelis-Menten equation reduces to: V = rate of drug metabolism = Vmax )(
concentration I Km, where Vmax is the maximum rate of drug metabolism. Therefore, in first-

..
order elimination, the rate of el imination is directly proportional to the drug concentration.
· 13 •
Metabol,sm
' 14 .

"5
., .
· 17 •
A is not correct. 12% chose this.
A constant amount of drug is eliminat ed per unit time in zero-order elimination, not in first-
order elimination.
B is not correct. 12% chose this .
.,

.
"9.
. 20 .
."
' 22 •

· 23 •
.24 .
In first-order elimination, a constant fraction of drug is metabolized per unit time because the
rate of drug metabolism is directly proportional to th e concentration of free drug. Conversely, in
zero-order elimination a constant amount of drug is eliminated per unit time .
Metabol,sm

C is not correct. 9% chose this.


The plasma concentration of the drug decreases linearly with time in zero-order elimination, not
in first-order elimination. In zero-order eliminat ion, th e enzyme is saturated by a high free-drug
concentration, and th e rate of metabolism remains constant over time .
. 25 . Metabol,sm
' 2• •
o Is not correct. 12% chose this.
' 27 •
In first-order elimination, the drug concentration decreases exponentially with time. The rate of
· 2• • el imination changes in proportion to the drug concentration .
. 29 .
A 69 Sis
v 32 KS/s

8
l ock
0
End Block
,Pad "" 2024 32« _ •

+ G R< UI usmle rx com/ap l#qmax1 + III :


Item ' 20 of 44
QID 1940 i
L • Mark --<l
Pre Ylous
t:>
Next
_!
la b~ ues
~.
Notes
C';'\I
Cakul~ tor
- ..

.".
"0. D is not correct. 12% chose this .
In first-order elimination, the drug concentration decreases exponentially with time. The rate of
elimination changes in proportion to the drug concentration.

..
· 12 •
· 13 •

' 14 . Bottom Line:

"5
., .
· 17 •
In first-order elim ination of drugs, the rate of elimination is proportional to the drug
concentration.

.,

.
"9.
. 20 •

."
' 22 •
· 23 •
. 24 .
14 i if.,
FIRS T AI D FACTS
'0'i.i for year:
Elimination of drugs
Zero-o rder
eliminatio n
2015

Ratc of elimination is con slant regard less of C p


(i.e., consta nt am ol1nt of (lrug elim inal l'd per
Capacit y-limited elim ination.
I' EA. (A pea is rou nd. sha ped like the
FA15,p . 244 .1

uO~ in
IlI,it tiIIlC). C p I linearl ), wit l, lime. E~'nnpl cs "c r<H) rder.)
. 25 . of drll gs - l'hcnrIOill. Et hall ol. ~",l Aspirill (;11
high or lo~ic cOll celllr"ti on $).
' 2• •
Fir5t-o rde r eliminatio n R;lle of el imin;ltion is d irect lr propo rtioll,,1 Fl o,,'-{Iepc •• dell t el i.ni ••;lti01 '
to th e drug eoncell tr;l1i oll (i. c .. COll st;mt
fr;Iction of drug elimi"" ted pe r I1l1il hmc). C p
I e~pol1e l1hally wi th lim e.

."9.
., A 12-year-old boy with sickle cell disease presents to the emergency department with

." .
' 20 .

· 22 •

· 23 •
' 24 .
compla ints of severe pa in in the area of his right humerus. His temperature is 37.1 O( (98.8°F),
blood pressure is 100/60 mm Hg, pu lse is 89/min, and respiratory rate is 22/min. A
rad iograph is obta in ed that shows lytic changes and periostea l elevation in the middl e and distal
humeral shaft .

What is the most likely pathogen responsible for this patient's condition?
134415 : zheng
We value your feedback!
A. Staphylococcus aureus
. 25 •
• 2• • B. Candida albicans
· 27 •
C. Mycobacterium tuberculosis
· 2• •
. 29 . D. Neisseria gonorrhoeae
.30 . E. Pasteurella multocida
'3' . F. Pseudomonas aeruglnosa
· 32 •

' 33 • G. Salmonella
.34 .
H. Staphylococcus epidermidis
.35.
' 3• •
,Pad "" 2024 32« _ •

."8"9..
+ G R< UI usmle rx com/ap l#qmax1 + III :
Item' 21 of44 L • Mark --<l t:> _! ~. C';'\I
QID 4020 i PreYlous Next lab~ ues Notes Cakul~tor

." The correct answer is G. 48% chose this.


Sickle cell disease is a disease in which patients are heterozygous for HbS. Sickle cell anemia is a
disease in which patients are homozygous for HbS. The biochemical abnormality responsible for
' 20 . this condition is the substitution of va line for glutamic ac id on the f3-globin cha in. Therefore
· 21 • patients create HbS instead of HbB. At low oxygen levels, HbS aggregates and distorts red blood
cells into a sickle shape. In the United States, 1 in SOD African-Americans carry the sickle cell
· 22 •
trait, making this ethnic group at highest risk for sickle cell disease. Historically, heterozygotes
· 23 • for HbS are believed to carry a survival advantage with protection from malaria. Patients with
' 24 . sickle cell anemia often suffer from aplastic crisis, splenic sequestration crisis, acute chest
· 25 • syndrome, and osteomyelitis. Aplastic anemia often arises from infection with parvovirus 819
(fifth disease). Child ren with this disease are also very susceptible to infection from
.26 . encapsulated bacteria, as thei r spleen is often non-functional after the age of 4 years.
· 27 • Osteomyelitis in sickle cell patients is often caused by Staphylococcus aureus and Salmonel/a.
' 28 . O.:ygen Malaria Ar.emia. Sid:lf! Cf!11 SiCklf! Cf!1I Tra it Acutf! ChE'st Syndromf! A~astic Ar.em ia Splf!f!n

.29 . A is not correct. 20% chose this_


.30 . Staphylococcus aureus is the most common cause of osteomyelitis in patients in general and the
' 31 . second most common cause of osteomyelitis in sickle cell anemia patients. This gram-positive
· 32 • coccus is often responsible for superficial skin infections and also causes joint infections after
orthopedic procedures. S. aureus is often carried in the nares or perineum of indiviudals,
' 33 •
sometimes leading to auto-inoculation. The organism can be sensitive to methicillin or resistant
.34 . to meth icill in, making laboratory sensitivities often essentia l to its successful treatment .
. 35 . Anemia. Sickle Cell Skin Diseases. Infectious Orthopedic Procedures Entire perineum

' 36 . B is not correct. 4% chose this.

"8
"9..
' 20 .
B is not correct. 4% chose this.
Candida albicans should be suspected in intravenous drug users who present with osteomyelitis.
Candida is the most common funga l pathogen affecting humans. It is a fungi that can form
· 21 • pseudo hyphae and true hyphae. One of the most common manifestations of Candida infection
is as thru sh, a creamy white plaque, in immunocompromised patients. It is also respons ible for
· 22 •
vulvovagin iti s in females, often described as having a "cottage-Cheese" consistency.
· 23 • Osteomyelitis due to Candida is ra re in the genera l population; therefore, drug use should be
' 24 . suspected if a patient presents with osteomyelitis due to Candida.
car.dida albicans f!>ctrac\ Vulvovaginitis
. 25 •
. 26 . C is not correct. 5% chose this_
· 27 • Mycobacterium tuberculosis usually causes osteomyelitis from hematogenous spread in
immunocompromised patients or in areas of endemic infection. Additionally, tuberculosis
.29.
' 28 .

.30 .
usually involves the vertebra l bodies (Pon disease), not the humerus. Infection occurs by
inhaling aerosolized particles of M. tuberculosis from an infected person. As this organism is
slow growing, it can rema in inert for many years before causing symptoms. Pott disease, also
' 31 . know as tuberculous spondyl iti s, is rare in developed countries, where antituberculous drugs
are readily available. Patients present with a combination of osteomyelitis and arthritis
· 32 •
involving more than one vertebra. They often have a history of untreated tuberculosis infection
' 33 • and complain of back pain for several weeks .
. 34 . TuberculOSiS Arthr itis Entire vertebra

.35 . D Is not correct. 5% chose this.


' 36 . Neisseria gonorrhoeae is a rare cause of osteomyelitis in sexually active people, but is more likely
to result in sept ic arthriti s (remember the mnemonic for reactive arthritis: can't see,
i
,Pad "" 2024 32« _ •

."8"9..
+ G R< UI usmle rx com/ap l#qmax1 + III :
Item' 21 of44 L • Mark --<l t:> _! ~. C';'\I
QID 4020 i PreYlous Next lab~ ues Notes Cakul~tor

." Neisseria gonorrhoeae is a rare cause of osteomyelitis in sexually active people, but is more likely
to result in septic arthriti s (remember the mnemonic for reactive arthritis: can't see, can't pee,
can't bend my knee ). This gram-negative, aerobic diplococcus can lead to pelvic inflammatory
disease in females jf left untreated, leading to infertility and tubal scarrin g. Other serious
' 20 . consequences include Fitz-Hugh and Curtis syndrome, in which the infection spreads from the
· 21 • reproductive organs to the liver capsule, causing acute perihepatitis.
Infertility Acute suppura~ve arti1r itis due to bacteria PeMc Inflammatory Disease Fitz.Hugh-Curtis syndrome
· 22 •
· 23 • E is not correct. 5% chose this.
' 24 . Pasteurelfa multocida is a cause of osteomyel iti s in persons with a history of cat and dog bites or
· 25 • scratches. This gram-negative, non-spore-forming bacterial species commonly reside in
domesti c pet species, transferring to humans with an animal bite or scratch. Locally, the
.26 . infection causes cellulitis or abscesses to develop in the area of injury. If untreated, it can
· 27 • become blood-borne, and infected persons can develop osteomyelitis or septic arthritis. These
' 28 . complications are more likely in cat bites, as cat bites tend to be smaller and more frequently
. 29 . go unnoticed .
Cellul itis Dog Bite Animal Me Acute suppurative artt1ritis due to bacteria
.30 .
' 31 . F Is not correct. 9% chose this.
· 32 • Infection by Pseudomonas aeruginosa, like Candida spp., should be suspected in intravenous drug
users who present with osteomyelitis. This gram-negative rod classically produces a blue-green
' 33 •
pigment with a sweet odor. Pseudomonas is an opportunistic pathogen seen in intravenous drug
.34 . users, immunocompromised persons, and in burn wound sepsis. Osteomyelitis due to
.35 . Pseudomonas can be blood-borne, from a systemic infection, or it may be contiguous, spread
' 36 .
from penetrating wound trauma. Persons with a history of puncture wounds of the feet are at
ri sk for Pseudomonas infection.
Pseudomonas Infections

Ime",,;'n spp.,

"8
"9..
' 20 .
users who present with osteomyelitis. This gram-negative rod classically produces a blue-green
pigment with a sweet odor. Pseudomonas is an opportunistic pathogen seen in intravenous drug
users, immunocompromised persons, and in burn wound sepsis. Osteomyelitis due to
Pseudomonas can be blood-borne, from a systemic infection, or it may be contiguous, spread
· 21 • from penetrating wound trauma. Persons with a history of puncture wounds of the feet are at
· 22 • risk for Pseudomonas infection.
Pseudomonas Infections
· 23 •

' 24 . H is not correct. 4% chose this.


.25 . Staphylococcus epidermidis, along with S. aureus, can cause osteomyelitis in patients who have
undergone prosthetic replacements. S. epidermidis is gram-positive and coagUlase-negative,
.26 . whereas S. aureus is gram-positive and coagulase-positive. S. epidermidis is found on laboratory
· 27 • cultures due to skin contamination, where it may reside. However, it is a major cause of

.29.
' 28 .

. 30 .
infection in the immunocompromi sed and in patients with indwelling catheters. Its ability to
form a biofilm on prosthetic devices makes it a possible cause of infection in those with joint
replacements .

' 31 .
· 32 •
Bottom line:

' 33 • In patients with sickle cell anemia, consider both Salmonelfa and Staphylococcus aureus as
causes for osteomyelitis. The lytic bone changes and periosteal elevation signifies an infection
.34 . of bone and therefore osteomyelitis .
. 35 . Anemia. Sickle Cell lnfecuon of bone

' 36 .
,Pad "" 2024 32« _ •

+ G R< UI usmle rx com/ap l#qmax1 + III :


Item ' 21 of44 L • Mark --<l t:> _! ~. C';'\I
QI D 4020 i PreYlous Next la b~ ues Notes Cakul~tor

·,8 •
"", ,',' I. II ,· • • • • I I . · •• "f- """ I'" ••••• '

cultures due to skin contamination, where it may reside. However, it is a major cause of
. infection in the immunocompromised and in patients with indwelling catheters. Its ability to
·,9 . form a biotilm on prosthetic devices makes it a possible ca use of infection in those with joint
· 20 . replacements.
· 21 •
· 22 • Bottom line:
· 23 • In patients wit h sickle cell anemia, consider both Salmonelfa and Staphylococcus aureus as
·24 . causes for osteomyelitis. The lytic bone changes and periosteal elevation signifies an infection
· 25 • of bone and therefore osteomyelitis.
Anemia. Sickle Cell Infection of bone
.26 .
· 27 •

· 28 .
141 ;fi , .j 1.1 for year: 2015
.29 . FIII.5T AI D FACTS

.30 .
FAI5, p, 173,4
· 31 .
Osteomyelitis ASSOC I AI(DINF!CII~
· 32 • s. ,mrellS (most common ol·erall )
Asslime if no olher in formation is avai lable
· 33 • Sexuallyactil'e Neiswria gonorrilOeae (rarc), seplic arth ritis morc
. 34 . common

.35 . Sickle cel l disease


I'ro,thetic ioint replaccment
Sillmollella and S. aI/reus
S. <l!JreIlS and S. rpidertuidis
· 36 . Vcrlcbml invoil-clllcnl

An 8S-year old man presents to the emergency department complaining of severe left lower
·,8 . quadrant pa in over the past 2 days. He describes the pain as crampy and says that he has
·,9 . been experiencing vom it ing, diarrh ea , and flatulence over the past 48 hours. The attending
· 20 . physician suspect s ac ute diverticulitis and the diag nosis is confi rmed on CT. Following surgery, the
patient devel ops gram-negative seps is and is t reated with intravenous antibiotics. Clinically the
· 21 •
patient begins to improve, but routine blood work demonstrates a creatinine level of 1.8 mg/d L.
· 22 •
· 23 •
With which of the following antibiotics was the patient most likely treated?
·24 .
13441S: zheng
.25 . A. Ampicillin
We valu e your feedback!
.26 .
· 27 • B. Aztreonam
· 28 . C. Clindamycin
·29.
.30 . D. Gentamicin

· 31 . E. Tetracycline
· 32 •
· 33 •
· 34 •
. 35 . The correct answer is O. 58% chose this.
· 36 . Gentamicin is from the antimicrobial family of am in oqlycosides, which are bactericidal and act
by inhibiting the formation of the initi atio n complex and ca using m isreading of mRNA .
requi re oxyqen for uptake and are therefore ineffective against anaerobes. They are
,Pad "" 2024 31 « _ •

."8"9..
+ G R< UI usmle rx com/ap l#qmax1 + III :
Item ' 22 of 44
QID 4004 i
L • Mark --<l
Pre Ylous
t:>
Next
_!
la b~ ues
~.
Notes
C';'\I
Cakul~ tor

." Gentamicin is from the antimicrobial family of aminoQlycosides, which are bactericidal and act
by inhibiting the formation of the initiation complex and causing misreading of mRNA . They
require oxyqen for uptake and are therefore ineffective against anaerobes. They are used to
treat severe gram-negative rod infections . Toxicities include nephrotoxicity, ototoxicity, and
' 20 . teratogenicity.
· 21 • Oxygen OtotoKic ity

· 22 •
A is not correct. 8% chose this.
· 23 • Ampicillin is an aminopeniciliin antimicrobial with a wid er spectrum than penicillin. It is
' 24 . penicillinase-sensitive but is often combined with clavu lanic acid to enhance its spectrum. It is
· 25 • used to treat certain gram-positive and gram-negative infections, including Haemophilus
influenzoe, Escherichia calf, Listeria, Proteus, Salmonella, enterococci, and Shigella. Toxicities
.26 . include hypersensitivity reactions and pseudomembranous colitis. Ampicil lin could lead to renal
· 27 • failure due to acute interstitial nephritis; however, renal failure from ampicillin is much less
' 28 • frequent than with aminoglycosides.
Ampicillin Clavulanic Acid Sh ige lla Infections Hypersensitivity pseudomembraoous colitis Acute interstitial nephritis
. 29 .
.30 . B is not correct. 10% chose this.
' 31 . Aztreonam is a monobactam antimicrobial that is resistant to ~-Iactamases. It acts by inhibiting
· 32 • cell wall synthesis and is used clinically to treat gram-negative rod infections. It is generally
nontoxic, although in some patients it causes drug reactions (such as a rash or ph lebitis) and/or
' 33 •
gastrointestinal upset .
. 34 . Aztreonam Adverse reaction to drug

.35 . C is not correct. 16% chose this.


' 36 .
Clindamycin is a bacteriostatic antimicrobial that acts by blocking peptide bond formation at
the 50S ribosomal subunit. It is not useful against gram-negative bacteria, but is eff·e ctive

"8
"9..
' 20 .
Clindamycin is a bacteriostatic antimicrobial that acts by blocking peptide bond formation at
the 50S ribosomal subunit. It is not useful against gram-negative bacteria, but is effective
against anaerobes such as Bacteroides. Its major toxicities include pseudomembranous colitis,
· 21 •
fever, and diarrhea.
Clir.o:lamycin Oiarrhea pseudomembranous colitis
· 22 •
· 23 • E is not correct. 8% chose this_

' 24 . Tetracycline is a bacteriostatic antimicrobial that acts by bind in g to the 305 ribosomal subunit
and preventing the attachment of aminoacyl-tRNA. Its main toxicities include gastrointestinal
.25 . distress, discoloration of teeth and inhibition of bone growth in children, and photosensitivity.
. 26 . It is also a teratogen.
· 27 • Tetracyc lu'\e

..3029..
' 28 •

Bottom li ne:
Gentamicin is an aminoglycoside used for treatment of gram-negative rod infections, often
' 31 . found post-gastrointestinal surgery.
· 32 •

' 33 •
. 34 . hl;fj'"", for year: 2015
.35 . fiRST ""0 f ... CTS

' 36 .
FA15, p . 184.2

Aminoglycosides
,Pad "" 2024 31 « _ •

."8"9..
+ G R< UI usmle rx com/ap l#qmax1 + III :
Item ' 23 of 44
QID 4062 i
L • Mark --<l
Pre Ylous
t:>
Next
_!
la b~ ues
~.
Notes
C';'\I
Cakul~ tor

." A 3·year-old boy presents with invo luntary, sustained muscle contractions in his arms and IAA[
legs and a systemic infection. Seru m analysis reveals a calci um level of 3.8 mq/dL and
decreased immune function. White plaques are noted in the patient's oropharynx .
' 20 .
· 21 • What reg ion of this patient 's lymph node will be atrophied?
· 22 •
134415 ; zheng
We value your feedback!
· 23 • A. Capsul e
' 24 .
8. Follicle of t he cortex
· 25 •
. 26 . C. Medullary cords
· 27 •
D. Medullary sinus
' 28 .
. 29 • E. Paracortex
. 30 .

' 31 .
· 32 •
The correct answer is E. 64% chose this.
' 33 •
DiGeorge's syndrome results from a failure of the thi rd and fourth pharyngeal pouches to
.34 . descend and is caused by deletion on chromosome 22 (22q11.2). Fai lure of the parathyroid
.35 . glands to develop leads to hypocalcemia and resultant tet any. The thymus also fails to develop,
' 36 . resulting in an immune deficiency of T lymphocytes that commonly manifests as vira l or
infections. The paracortex is the reg ion of the lymph node where T lymphocytes are I
atrophied in patients with DiGeorge's syndrome, because they have aT-lymphocyte

"8
"9..
' 20 .
The capsule encloses the lymph node and is unaffected in DiGeorge's syndrome.
B is not correct. 16% chose this.
The follicle of t he cortex cont ains B lymphocytes and is unaffected in DiGeorge's syndrome.
· 21 • C is not correct. 8% chose this.
· 22 • The medu llary cords conta in plasma cells and are unaffected in DiGeorge's syndrome.
Plasma Cells
· 23 •

' 24 . D is not correct. 8% chose this .


. 25 . The med ullary sinus contains macrophages and is unaffected in DiGeorge's syndrome .
. 26 .
· 27 • Bottom Line:

.29.
' 28 .

. 30 .
DiGeorge's syndrome, which results from the failure of the third and fourth pharyngeal
pouches to descend, should be suspected in any pediatric patient with tet any secondary to
hypocalcemia who concurrent ly has a viral or fungal infection .
TeLlny Hypoca lcemia
' 31 .
· 32 •

' 33 •
.34 . ul;fi"'''' for year: 2015
FIII.5T ... ID F"'CTS

.35 .
FA15, p, 214,1
' 36 .
Immunodefi cien cies
,Pad 9 2025 310 . '

+ + • :

."9.
G R< UI usmle rx com/ap l#Qmax1

Item " 24 of 44
QID 396<1
L

J.
• Mark -<l
PreYlDu,
t>-
NeKt
all
lab"" ues
~.
Notes
~'t
c .. ku l~tor

." A 37-year-ol d man with a history of HIV presents to the emergency department with altered
· 18 •
mental status, hemipa resis, dysphasia, aphasia, ataxia, and focal seizures. Coronal Tz-
weighted MRI shows high signal in the temporal lobes and hippocampus. His CD4 count is
' 20 . 150 cells/~ l (normal : 500-1500 cells/jJL). Polymerase chain reaction testing of his cerebrospinal
flu id was completed, and a transmissio n electron micrograph of this pathogen is made and shown
· 21 •
in t he image. After an initia l infection, the causative pathogen can remain dormant in dorsal root
· 22 • ganglia for years while successfu lly the host's immune
· 23 •

' 24 .
· 25 •
• 2• •
· 27 •

· 2• •
' 29 •
. 30 .
· 31 •
· 32 •

' 33 •
' 34 •
. 35 .
' 3• •

."9.
,,
' 20 .
During this latency period, which vi ral com ponent can be detected in infected neurons?

A. Double-stranded DNA
We value your feedback!
134415; zheng

· 21 •
B. Latency-associated proteins
· 22 •
· 23 • C. Single-stranded DNA
' 24 . D. Viral nucleocapsids
.25 .
E. Viral reverse transc riptase
. 2• •

· 27 •

· 2• •
. 29 .
The correct answer is A. 51% chose this .
. 30 .
Herpes simplex virus type 1 (HSV- l ) is a double-stranded DNA virus that can establish latency by
· 31 •
integrating its viral genome into the host cell's own DNA. At th e initial sites of infection, which
· 32 • include skin and mucous membranes in the oropharynx or genitals, the virus repl icates and
' 33 • produces vesicular skin lesions that contain numerous infectious virus particles. Following the
initial infection, HSV-l and HSV-2, as wel l as varicella-zoster vi rus, all establish latency in the
. 34 .
dorsal root ganglia associat ed with that skin dermatome or mucous membrane area. The latent
.35 . virus can subsequently be reactivated by various stimuli, inc luding stress, menstruation, and
' 3• • exposu re to ultraviolet light. During this latency phase, the virus continues to prod uce single-
stranded latency-associated messenger RNAs (mRNAs). These mRNA transcripts do not
trans lation into proteins, thus allowing the virus to evade the host's immune I
,Pad "" 2025 31 « _ •

."8"9..
+ G R< UI usmle rx c om/ap l#qmax1 + III :
Item ' 24 of 44
QID 3964 i
L • Mark --<l
Pre Ylous
t:>
Next
_!
la b~ ues
~.
Notes
C';'\I
Cakul~ tor

." stranded latency-associated messenger RNAs (mRNAs). These mRNA transcripts do not undergo
trans lation into proteins, thus allowing the virus to evade the host's immune surveillance
system. On reactivation, vira l proteins are synthesized. These particles get detected by an
immunocompetent host, leading to symptomatic resolution in a few weeks' time. Thus, during
' 20 . latency, only the viral DNA (double-stranded) and latency-associated mRNAs (single-stranded)
· 21 • can be detected in infected gangl ia.
Herpes Slm~ex Infections Mel'lStruation Immunologic SLJrveiliance Skin lesion
· 22 •
· 23 • B is not correct. 19% chose this.
' 24 . Latency-associated mRNAs (single-stranded) are not translated into protein products.
· 25 • C is not correct. 10% chose this •
. 26 . HSV-1 is a double-stranded DNA virus. It produces single-stranded mRNA during latency, which
· 27 •
can be tested for with RT-PCR. It does not produce single-stranded DNA, which would be found
in the parvovirus.
' 28 .
. 29 .
o is not correct. 13% chose this .
Viral nucleocapsids are not produced during the latency period .
. 30 .
E Is not correct. 7% chose this.
' 31 .
Viral reverse transcriptase is not associated with HSV-1 or HSV-2, but rather HIV and human T-
· 32 •
cell leukemia viru s type 1.
' 33 • leukemi..

. 34 •
. 35 .
Bottom Line:
' 36 .
HSV-1 infects skin and mucous membranes, from which it establishes dormancy within
dorsal ganglia of nerve roots associated with those areas. The virus incorporates its

"8
"9..
' 20 .
HSV-l infects skin and mucous membranes, from which it establ ishes dormancy within the
dorsal ganglia of nerve roots associated with those areas. The virus incorporates its DNA
genome into the host's DNA. During the latency period, HSV-1 only synthesizes latency-
associated mRNAs, which do not get translated into proteins.
· 21 •
· 22 •
· 23 •
ul;f.,I,'cts
.. , for year: 2015
' 24 . fiRST ... I D f ...

.25 .
.26 . He rpesv iru ses
FAtS, p. 159.1

· 27 •
HSV-l Ci ngi,·oSlom~lili s. ler~toco njuncli"iti s IJ. herpes l~bi~ lis D . IcmporJI lobe ence ph ~ lili s (mosl

.29.
' 28 .

. 30 .
COllllllon canse of sporad ic e ncephalitis. ca n present with a llerI'd me nta l stalus. seizures. andlor
aph;,sia).
Tr,lIlsmitied by re~pi mtory st-'C rct ions, 5.1li' ·a .
HSV-2 H(.·rpes gen ita lis BI, neonata l herpes. Lltent in 5.1c ml gangli a.
' 31 . Tmn smitled by sex ual conta ct. perin:tt:lll )'

· 32 • VZV Va ricella·zoste r (chicke npox I!J, sh ill gles 0). ence phalit is. pne umon ia. Latent in dorsal root o r
tr ige minal ganglia. Most com mon complica tion of shinglcs is post.herpetic nc ura lgia.
' 33 • Tmnsmitted by respiratory sec retions
. 34 . EBV l\.'lononud eos is. C haracleri l'.c.:I II)' fcvc r. hcpatos ple nomegaly. ph aryngi ti s. and lympha dc nopal hy
(C!Jpcciall )' pos terior cervical 1lQ(1('!) OJ .
. 35 .
Tr;II IS",itted by rcsl'iralory st-"Cret ions amI sali"a; a lso c"l lcd "kissin g disease'" since co"",,o nly seell
' 36 . in teells. yonn g ;lIlults
I"fects B cel ls through CD2l . Atrpicallymphocytes seen on j:.eripheral bloo';] SmC;tr fiI ;t re not
infec tcd R cells bnt r;tth e. rc act i"e ertoto~ie T ce ll s.
,Pad "" 2025 31 « _ •

."8"9..
+ G R< UI usmle rx c om/ap l#qmax1 + III :
Item' 2S of 44
QID 4209 i
L • Mark --<l
PreYlous
t:>
Next
_!
la b~ ues
~.
Notes
C';'\I
Cakul~tor

." A 40-year-old boy cuts his finger on a rock while playi ng outside. After clean ing the wound, his IAA[
mother applies an ointment containing a bactericida l agent that is rarely used systemically
because of its adverse effect of nephrotoxicity.
' 20 .
· 21 • What is the mechanism of action of this antibiotic?
· 22 •
134415 ; zheng
We value your feedback!
· 23 • A. Damages DNA
' 24 .
8. Inhibits DNA topoi50merases
· 25 •
. 26 . C. I nhibits mRNA synthesis
· 27 •
D. I nhibits nucleotide synthesis
' 28 .
. 29 • E. I nhibits peptidoglycan synthesis
. 30 .
' 31 .
· 32 •
The correct answer is E. 33% chose this.
' 33 •
Bacitracin prevents the transfer of mucopeptides into the growing bacterial cell wall. It is active
.34 . against many gram-pos it ive and some gram-negative bacteria. Because there is a serious risk of
.35 . nephrotoxicity, it is most often used in topical form.
' 36 . Bacitracin

"8
"9..
B Is not correct. 11% chose this.
Fluoroquinolones interfere with DNA gyrase and other DNA topoisomerases during DNA
replication. This ultimately results in DNA strand breakage and cell death.
' 20 . DNA Replication

· 21 • C is not correct. 24% chose this.


· 22 • Rifampin works by blocking mRNA synthesis.
· 23 • lUfampin

' 24 . D is not correct. 16% chose th i s.


. 25 . Trimethoprim and sulfamethoxazole, as well as all sulfonamides, work by preventing bacterial
. 26 . nucleotide synthesis .
· 27 •

.29.
' 28 .

.30 .
Bottom Line :
Bacitracin is a high ly effective topical antibiotic against gram-positive bacteria that works by
inh ibiting bacteria l wall synthesis.
B3c itf3cin
' 31 .
· 32 •

' 33 •
.34 . ul;f;"'''' for year: 2015
fIRST ""D f ... CTS

.35 .
FA15. p. 180.1
' 36 .
AntimicrDbial therapy
,Pad "" 2025 31 « _ •

."8"9..
+ G R< UI usmle rx c om/ap l#qmax1 + III :
Item' 26 of 44
QID 4001 i
L • Mark --<l
Pre Ylous
t:>
Next
_!
la b~ ues
~.
Notes
C';'\I
Cakul~ tor

." A 25-year-old woman presents to her primary care physician with complaints of intermittent IAA[
episodes of chest pain, palpitations, diaphoresis, flushing, and headaches. She has no
history of substance abuse and has never had any health problems in the past. but she has a
' 20 . family history significant for thyroid cancer and parathyroid tumors. Her temperature is 36.9°(
(98.4°F), pulse is 95/min, respiratory rate is 22/min, and blood pressure is 150/85 mm Hg.
· 21 •
· 22 •
Which of the following is the most appropriate initial treatment for this patient's condition?
· 23 •

' 24 . 134415 : zheng


We va lue your feedback!
A. Dobutamine
· 25 •
. 26 . B. Isoproterenol
· 27 •
C. Phenoxybenzamine
' 28 •
. 29 . D. Phenylephrine
.30 . E. Propranolol
' 31 .
· 32 •

' 33 •
. 34 . The correct answer is C. 61% chose this .
. 35 . This patient is presenting with classic symptoms of a pheochromocytoma, a tumor derived from
' 36 . chromaffin cells of the adrena l medulla that secretes catecholamines including epinephrine,
norepin ephrine, and dopamine. Diagnosis is based on elevated urinary vanillylmandelic
levels and plasma catecholamines. Pheochromocytomas may occur spontaneously or

medulla secretes catecholamines including epinephrine,

"9.
' 20 .
· 21 •
norepinephrine, and dopamine. Diagnosis is based on elevated urinary vanillylmandelic acid
levels and plasma catecholamines. Pheochromocytomas may occur spontaneously or in the
setting of familial multiple endocrine neoplasia syndromes associated with medullary
carcinoma of the thyroid or tumors of the parathyroid gland. Patients present with episodic
· 22 • hyperadrenergic symptoms including hypertension, headache, tachycardia, diaphoresis, and
pa llor. Initial medical treatment involves control of symptoms via a-blockade with
· 23 •
phenoxybenzamine, a nonselective irreversible a-receptor antagonist.
' 24 . Headache Tachycardia Phenoxybenzamine AdreMI Medulla Epinephrine Multiple Endocrine Neoplasia Parathyroid Neoplasms

.25 . A is not correct. 4% chose this .


. 26 .
Dobutamine is a ~,-adrenergi( receptor agonist used to treat shock and heart failure. It is not
· 27 • indicated for the treatment of pheochromocytoma.

.29.
' 28 .

. 30 .
Oobutamine Heart fai lure

B is not correct. 6% chose this .


Isoproterenol is a nonselective r3-ad renergic receptor agonist used to treat atrioventricular
' 31 . block. It is not indicated for the treatment of pheochromocytoma.
Isoproterenol
· 32 •

' 33 • o is not correct. 5% chose this .


. 34 . Phenylephrine is an Q,-adrenergic receptor agonist used to treat nasa l decongestion, dilate
.35 . pupils, or induce vasoconstriction in the setting of hypotension and shock. It is not indicated for
the treatment of pheochromocytoma.
' 36 .
Phenylephrine Hypotension
,Pad "" 2026 31 « _ •

."8"9..
+ G R< UI usmle rx c om/ap l#qmax1 + III :
Item ' 26 of 44
QI D 4001 i
L • Mark --<l
Pre Ylous
t:>
Next
_!
la b~ ues
~.
Notes
C';'\I
Cakul~ tor
.. . .
." Propranolol is a nonselective f3-adrenergic receptor antagonist used to treat hypertension,
angina, myocardial infarction, supraventricular tachycardia, and congestive heart failure . 13-
Blockers can be used to treat pheochromocytoma only after sufficient Q-blockade has been
' 20 . achieved. I3-Blockade in the absence of an a-blocker can cause peripheral vasoconstriction by
· 21 •
antagonizing the vasodilatory effects of peripheral!3-receptors and would worsen hypertension.
Whereas propanolol would be appropriate to treat the symptoms of hyperthyroidism. this
· 22 •
patient's symptoms and fa mi ly history suggest pheochromocytoma. Pheochromocytoma often
· 23 • presents as a classic triad of headache, tachycardia, and sweating. In contrast, thyrotoxicosis
' 24 . would be suspected in an individual with a history of hyperthyroidism. However, this patient had
no previous symptoms suggestive of Grave's disease (thinning hair, exopthalmos, history of
· 25 •
symptoms related to hyperthyroidism) .
. 26 . Propranolol HYiJ@rthyroidlsm H@aoacl\@ Tachycard ia Thyrotoxicos is Myocardial Infarction Suprav@ntriculartachycardia
· 27 • Conges!ivf! hear! failure
' 28 •
. 29 .
Bottom li ne:
. 30 .
' 31 . Phenoxybenzamine is used to treat the symptoms of pheochromocytomas, adrenal tumors
that cause hypertension, headache, tachycardia, and diaphoresis.
· 32 •
Phenorybemamine Headache Tachy<ardia Adrena l Gland Neoplasms
' 33 •
. 34 •
. 35 .
hl;fi,·,.·1 for year: 2015
FIRST ,0.10 FAC TS
' 36 .

· 22 • A child is brought in to clinic by his parents, because they are concerned that he has not
been moving his bowels regularly. They cannot remember the last time the child had a bowel
IAAI
' 23 •
movement, and he has been getting increasingly irritable and listless. A biopsy of the distal
' 24 . segment of the child's colon is performed, and the results are shown in the image .
. 25 .

' 26 .
' 27 •
' 28 •
• 29 •
• 30 •
. 31 .

· 32 •
. 33 •
. 34 . Image
' 35 .
' 36 . Which of the following is another type of receptor that functions in the same way as the receptors
· 37 • of the intestinal gang lion cells?
. 38 . 134415: zheng
We va lue your feedback!
' 39 .
,Pad "" 2026 31 « _ •

+ G R< UI usmle rx c om/ap l#qmaxl + III :


Item' 27 of 44
QID 1S43 i
L • Mark --<l
Pre Ylous
t:>
Next
_!
la b~ ues
~.
Notes
C';'\I
Cakul~ tor

134415 : zheng
· 22 • We va lue your f eedback!
·23 .
. 24 • B. p,
. 25 . C. Dopamine,
· 2• •
D. Histamine,
·27 .
· 28 . E. Vasopress in2
. 29 •
. 30 .
· 31 •

·32 . The correct answer is D. 37% chose this.


· 33 • The bioposy results indicate an aganglionic colon. The enteric ganglion ce ll s contain
.34 . musca r in ic, (M,) receptors, wh ich trans m it their signa l via t he phosphol ipase
Clphosphatidyli nositol bisphosphate/inositol triphosphate (phospholipase C- PIP2-IP3) pathway.
· 35 . Other types of receptors t hat function in the same way are the histamine, (H,) receptors of the
· 3• • respiratory tract, a, sympathetic receptors, vasopressin, (V,) receptors, and M3 parasympathetic
· 37 • receptors .
.38 . Inositol Histamine Vasopressin (USP)

· 39 . A is not correct. 19% chose this.


Q2 Receptors act vi a an inhibitory G protein to decrease intracellular cAMP levels and thus
reduce the activity of prot ein kinase A.

· 22 • 13, Sympathetic receptors act via the adenylate cyclase-cAMP pathway to increase the activity of
· 23 •
protein kinase A. They act via a stimulatory G protein.

· 24 . C is not correct. 19% chose this.


.25 . Dopam ine, (0,) receptors act via a stimulatory G protein to increase intracellular cyclic AMP and
thus increase protein kinase A activity.
· 2• •
E is not correct. 11% chose this.
·27 . Vasopressin2 (V2) receptors act via a stim ulatory G protein to increase cAMP and protein kinase
· 28 . A activity .
. 29 . VoJsopressln (USP)

.30 .
· 31 •
Bottom Line:
· 32 •
The M3 receptor is a Gq receptor, which activates phospholipase C to cleave PIPz int o IP3 and
· 33 •
diacylglycerol (OAG). 1P3 increases calcium ions for use in calmodulin activity; OAG activates
.34 . protein ki nase C. The muscarinic acetylcholine receptors function in th e pa rasympathetic
· 35 . nervous system. Other Gq receptors include H" V" and M,.
(~ I CH.lm Acetylcholirn:' PM~symp<llheli( Nervous System
· 3• •
· 37 •
. 38 .
· 39 . 141;£0,,·,..1
FHUT "10 F"CTS
for year: 2015
· 40 .
,Pad "" 2026 31 « _ •

+ G R< UI usmle rx c om/ap l#qmax1 + III :


Item' 28 of 44
QID 3630 i
L • Mark --<l
Pre Ylous
t:>
Next
_!
la b~ ues
~.
Notes
C';'\I
Cakul~ tor

· 22 • Strength iva, a new drug t hat is used to treat chronic inflammatory polyne uropathy disorder, IAA[
has a narrow therapeutic range. The medication is given intravenously at a constant drip.
·23 . The drug has the following characteristics:
. 24 •
. 25 . Clearance: 0.5 Umin
· 2• • I nfusion rate: 12.0 mg/min
Volume of distribution: 5.0 L
· 27 •
. 28 •
. 29 .
Based on this information, what is the steady st ate concentration of Strength iva?
. 30 .
134415: zheng
· 31 • We value you r feedback!
A. 12.0 mg/L
·32 .
· 33 • B. 120.0 mg/L
. 34 . C. 24.0 mg/L
· 35 .
0 .48.0 mg/L
· 3• •
· 37 • E. 6.0 mg/L
.38 . F. 60.0 mg/L
· 39 .

state concentration =Infusion rate/Clearance = (12 i


·23 . Alternatively, steady state concentration can be figured out the maintenance dose is given.
. 24 .
Maintenance dose = Plasma concentration )( (Clearance/B ioavailability). Rearrang ing this
equation for the plasma concentration (equiva lent to the steady state) gives: Steady stat e
.25 . concentration = Maintenance dose)( (Bioavailabi lity/Clearance).
· 2• •
A is not correct. 7% chose this.
· 27 . Th is answer is half the correct answer. The equation for determination of steady state
. 28 . concentration for an int ravenous medication (which, by definition, is 100% bioava ilable) is
. 29 . Infusion rate/Clearance. No mu lti plication factor is needed .
. 30 . B is not correct. 19% chose this .
· 31 • Th is answe r is incorrect. The equati on for determination of co ncentratio n is I nfusion
rate/Clearance.
·32 .
D is not correct. 10% chose th is.
· 33 •
Th is answer is twice the correct answer. The equation for det ermination of concentration is
.34 . Infusion rate/Clearance . No mu ltiplication factor is needed.
· 35 . E is not correct. 10% chose this.
· 3• •
Th is answer is incorrect. The equation for determination of concentration is Infusion
· 37 • rate/Clearance .
. 38 . F is not correct. 7% chose this.
· 39 . Th is answer is incorrect. The equation for determination of concentration is Infusion
rate/Clearance.
,Pad "" 2027 31 « _ •

+ G R< UI usmle rx com/ap l#qmax1 + III :


Item ' 28 of 44
QI D 3630 i
L • Mark --<l
Pre Ylous
t:>
Next
_!
la b~ ues
~.
Notes
C';'\I
Cakul~ tor
• • - I . •

· 22 • Th is answer is incorrect. The equation for determination of concentration is Infusion


· 23 . rate/Clearance.
. 24 . F is not correct. 7% chose this •
.25 . This answer is incorrect. The equation for determination of concentration is Infusion
rate/Clearance.
· 2• •
· 27 .
.28 . Bottom line:
.29 . Steady state concentration is calcu lated by dividing infusion rate by clearance for an
intravenous medication if the infusion r ate remains constant.
.30 .
· 31 •

· 32 .
· 33 •
Ul;fi'-'I.' for year: 2015
~IRST "10 FACTS

.34 .
FA15, p. 2432
· 35 .
Dosage calculations C xVd In renal or liver disease, maintenance dose I and
· 3• • Loading dose = ~ loading dose is lIslIally lIneh:mgt..:l.
· 37 • C XCLXT T ime to steady state depends primarily on

.38 . "'iainlclwilce dose =- ' -F- - tll2 and is independent of dose and dosing
freq uency.
C == large! plasma conccn!ratioll at slcad)' state
· 39 . v
t = dosage interv:Li {time between doses}, if nol
ad ministered COlllillllollsl),

· 22 • A 55-year-old man with a history of kidney transplantation 1 year ago presents with
decreased urine output. Laboratory analysis shows:
· 23 •
. 24 . Na+: 142 mEq/L
.25 . K+: 4.5 mEq/L
· 2• •
(1-: 101 mEqlL
HC03-: 23 mEqlL
· 27 . Blood urea nitrogen: 15 mg/dL
·28 . Creatinine: 1.3 mg/d l
.29 .
.30 . Repeated creatinine 2 days later is 1.5 mg/dL. He is currently taking a f3-blocker. low- dose
methylprednisolone , and long-term tacrolimus.
· 31 •

· 32 .
After work-up and acute management, which other medication should be avoided?
· 33 •
.34 . A. Azathioprine
We va lue your feedback!
134415 : zheng

· 35 .
· 3• • 8. Cyclosporine
· 37 •
C. Paclitaxel
.38 .
· 39 . D. Prednisone

E. Vincristine
,Pad "" 2027 31 « _ •

+ G R< UI usmle rx com/ap l#qmax1 + III :


Item ' 29 of 44
QID 2408 i
L • Mark --<l
Pre Ylous
t:>
Next
_!
la b~ ues
~.
Notes
C';'\I
Cakul~ tor

· 22 • Both cyc losporine and tacrolimus are immunosuppressant agents that are frequently prescribed
to transplant recipients. These medications act to prevent activation ofT lymphocytes that
·23 . might mediate transplant rejection. More specifically, they form protein -drug complexes that
.24 . interfere with the intracellular signaling events that occur after T-Iymphocyte receptor ligation.
.25. Unfortunately, both of these agents can be potent inducers of nephrotoxicity, which seems to
be a problem in this patient with tacrolimu5 alone .
· 2• •
Cyclosporine Tacrolimus GraftRejection T-lymphocyte
· 27 .
· 28 .
A is not correct. 14% chose this.
. 29 .
Azathioprine is an antimetabolite derivative of 6-mercaptopurine that interferes with the
metabolism and synthesis of nucleic acids. It is used clinically as an immunosuppressive agent
. 30 . in patients with autoimmune disorders (in cluding glomerulonephriti s and hemolytic anem ia) or
· 31 • those who receive kidney transplants. Azathioprine is toxic to proli ferating lymphocytes.
·32 . However, it does not commonly have the adverse effect of nephrotoxicity, wh ich is one of the
significant adverse effects oftacrolimus.
· 33 •
Azathioprine Metabolism mercaptoP'Jrine Kidr.eyTranspiantation Tacrolimus
.34 .
C is not correct. 7% chose this.
· 35.
Paclitaxel is an antineoplastic agent that hyperstabilizes microtubules, inhibiting anaphase
· 3• •
breakdown of the mitotic spindle. It may induce myelosuppression, but it does not common ly
· 37 • cause nephrotoxicity.
. 38 . Paclitaxel Microtubules Ar>aphase Mitotic Spindle Apparatus

· 39 . o Is not correct. 9% chose this.


There are several adverse effects of long -term steroid use, including hypertension, diabetes,
weight gain, cataracts, and adrenal suppression. Nephrotoxicity is not a common
however.

There are several adverse effects of long-term steroid use, including hypertension, diabetes,
· 22 •
weight gain, cataracts, and adrenal suppression. Nephrotoxicity is not a common concern,
·23 . however.
. 24. Bilateral cataracts (d'sorder)

.25 . E is not correct. 7% chose this.


· 2• • Vincri stine is an antineoplastic agent that inhibits polymerization of microtubules, thereby
· 27 . leading to neuropathy in many patients. However, it does not commonly cause nephrotoxicity.
Vincnstir.e Microtubules
· 28 •
. 29 •
. 30 . Bottom li ne:
· 31 • Both cyclosporine and tacrol im us are immunosuppressant agents that can cause
·32. nephrotoxicity.
Cyclosporine Tacrol,mus
· 33 •
.34.
· 35 .
· 3• •
hl;fi,·,.·1 for year: 2015
FIRST ,0.10 FACTS

· 37 •
. 38 . FA15, p . 218 .1

Immunosuppressants Agcuis Ihal bloc\; lymphoc) te ad il'alion and proliferation . Rcd l1cC aCl1 lc Irdllsplan l reicd ion by
· 39 . mf'p"";"" c.d· 1 " 1,, il . Frr<jl1Cnl l)' co",bined 10 achicvc grcatc , efficacy will, I lo.~icit }'.
. , I Ii
,Pad 9 2027 31 0 . '

+ G R< UI usmle rx com/ap l#Qmax1 + • :


Item " 30 of 44
QID 3706
L

J.
• Mark -<l
PreYlDu,
t>-
NeKt
all
lab"" ues
~.
Notes
~'t
c .. ku l~tor

· 22 • A 52-year-old woman present s with new-onset jaundice. After extensive work-up, a liver
biopsy is ca ll ed for. The histologic section is shown in the image.
· 23 •

· 24 . ••• • • • •
•·
. 25 .
· 26 . . ; •, ".
•, " •"
"' ;
:~~ .4 -",
"' : J
· 27 •

· 28 •
. .." ~
\

,.
• • )I' •
., ....
~

. 29 .
·'0 .
· 31 •
'. -..
~'.
ooC"'t:
f
• • • i",'

"
.. •-
· 32 •
.. ,;-
.'. .
· 33 • ,
.'4 .
·'5 . ., ~ •
•• • , "
·'6 . • • .' ;.:", !'h •
Image courtesy a/U.S. National Library oj Medicine

· 37 •
· 38 •
The ce ll s indicated by the arrows are most likely undergoing whic h of the following processes?
·'9 .
· 40 . We value your feedback!
134415: zheng
A. Apoptosis

· 22 • The ce ll s indicated by t he arrows are most likely undergoing whic h of the following processes?
· 23 • 134415: zheng
We value your feedback!
. 24 • A. Apoptosis
. 25 .
B. Caseous necrosis
· 26 .
· 27 •
C. Coag ulative necrosis

· 28 . D. Fat necrosis
.29 .
E. Li quefactive necrosis
.'0 .
· 31 •

· 32 •
· 33 •
The correct answer is A. 61% chose this .
.'4 . The programmed death of cells, apoptosis, commonly occurs in virally infected cells and is
·'5 . med iated extrinsically by killer T lymphocytes. Intrinsic apoptosis occ urs after inju rious stimuli
·'6 . and is also a norma l part of embryolog ic development, homeostasis, and aging. Apoptos is is
unique in that cell death is active, whic h requires gene ex pression and energy consumption.
· 37 •
Morphologic changes t hat distinguish apoptosis from other forms of cell death involve cell
.'8 . shrinkage, chromatin condensatio n in w hi ch chromatin aggregates peripherally, formation of
·'9 . cytoplasmic blebs and apoptotic bodies, and phagocytosis of apoptotic cells by mac rophages.
These changes are evident in the cells indicated in the image.
Apoptosis Aging Phagocytosis Gene Expression
,Pad"" 2036 30« • •

+ G R< UI usmle rx com/ap l#qmax1 + III :


Item ' 30 of 44
QID 3706 i
L • Mark --<l
Pre Ylous
t:>
Next
_!
la b~ ues
~.
Notes
C';'\I
Cakul~ tor
.. ., .
·18 • Tuberculosis

"9.
' 20 .
· 21 •
· 22 •
· 23 •

'24 •
• 25 •
. 26 .
· 27 •
' 28 .
' 29 •
. 30 .
· 31 •
·32 •
· 33 • [mage courtesy of Dr. Robert Novak
' 34 . Caseous necrosis, which occurs in the setting of tuberculosis infection, grossly displays a "white
· 35 • and cheesy" appearance. Microscopic findings reveal amorphous granu lar debris su rrounded by
. 36 .
a granulomatous reaction, as shown in this image .
C is not correct. 20% chose this.

' 20 .
· 21 •
· 22 • • Glomerulus
· 23 •

' 24 . Tubules
' 25 •
. 26 .
· 27 •
' 28 .
' 29 .
' 30 .
· 31 •
. 32 . Image courtesy of Dr. Robert Novak
· 33 •
Coagulative necrosis, which occurs in hypoxic death of all tissues except the brain, is
' 34 . characterized by the preservation of original tissue architecture, as shown in this image.
· 35 • Ultimately necrotic cells are removed due to proteolysis by wacs and engulfment by
. 36 . macrophages .
o Is not correct. 8% chose this.
,Pad "" 20 36 30 Y0 . •

+ G R< UI usmle rx com/ap l#Qmax1 + • :


Item ' 30 of 44 L • Mark -<l t>- all ~. ~'t
QID 3706 J.
. .,
PreYlDu,
.,'
NeKt lab"" ues Notes c .. ku l~tor

·18 • Fa! r.ecrosis Acute pancreatltis Calcium

"9.
' 20 .
· 21 •
· 22 •
· 23 •

'24 .
· 25 •
. 26 .
· 27 •
· 2• •
' 29 .
' 30 .
· 31 •
;"L.
· 32 •
[mage courtesy of Dr. Robert Novak
· 33 •

' 34 .
Fat necrosis, which common ly occurs in the setting of acute pancreatitis, involves focal areas of
fat destruction. On histology, shadowy outlines of necrotic fat cells with basophilic ca lcium
.35 . deposits are seen, surrounded by an inflammatory reaction, as shown in this image .
. 36 .
E is not correct. 4% chose this.
Qig<!SlIOn l iquefaCl1Vf! neUOSI$

."9.
.,
' 20 .
Digestion Liquefacti~ necrosIs

· 21 •
· 22 •
· 23 •

' 24 •
• 25 •
. 26 .
· 27 •
· 2• •
' 29 •
. 30 .
· 31 •
· 32 • [mage courtesy of Dr. Robert Novak
· 33 •
Liquefactive necrosis, which occurs characteristically in focal bacterial or fungal infections,
' 34 .
involves the complete digestion of dead celis, as shown in this image. Subsequently, the
.35 . residual tissue becomes a liquid viscous mass .
. 36 .
,Pad "" 2036 30« • •

+ G R< UI usmle rx c om/ap l#q max1 + III :


Item ' 30 of 44
QID 3706 i
L • Mark --<l
Pre Ylous
t:>
Next
_!
la b~ ues
~.
Notes
C';'\I
Cakul~ tor
• ., J

·18 •
liquefactive necrosis, which occurs characteristically in focal bacterial or fungal infecti ons.

"9.
' 20 .
involves the complete digestion of dead celis, as shown in th is image. Subsequently, the
residual t issue becomes a li quid viscous mass.
· 21 •
· 22 • Bottom line:
· 23 •
Apoptosis is prog rammed cell death that involves ch romati n agg regation, phagocytos is,
'24 . cytoplasmic blebs, and apoptotic bod ies .
. 25 . Apoptosis Phagocytosis

.26 .
· 27 •
' 28 .
I ill jfj , .j I.' for year: 2015
FIII. S T AI D FA CTS
' 29 •
. 30 . FA15,p.222.1

Apoptosis Progr,ullllled cell death; I\TP required. Intrinsic or ex trinsi c pathway; both p.l l hwa)'li - act iva tion
· 31 •
of cylosolic c aspascs th~1 media tc cel lular breakdown.
·32 • Nosignificimt inll<111l1n11li0l1 (unlike nL'CTOsis).
· 33 • Ctmr:lcterized b)' (leepl)' eosinophilic cytoplasm. ce ll sh rinkage. nnelelr sh rinbge (pyknosis) 'liid
basophilia. membrane blcbbing. nuclear fragmentation (br),orrhexis). and format ion of apoptotie
' 34 . bodies. which arc thcn phagocytosed.
· 35 • DNA bdder ing is a sensitivc ind icator of apoptosis; dming br)"orr hcxis. cndonlld cascs cleave at
illternudeoso,n al reg ions. )'iel din g frJgmenis in mlllt iples of ISO bp. Ibdi~tion ther;I]l)' eau scs
. 36 . "IXlPtosis of tu"'ors ,,,,<1 snrrollndin g tissue via free rJ<l ic,,1 fonnation and dsDNA bre" b ge.
It apidly divid ing cd ls (c.g., skin. cr lllUCOS.l) arc ,"cry suscc ptibl e to radiation th erapy- induced

.,8 . A pharmaceutical company is actively researching a novel compoun d that shows promising

"9.
' 20 .
results as a chemotherapeutic agent. After administering the drug to a culture of cancer
cells, microscopic evaluation reveals arrest in the prophase of mitosis in all celts.

· 21 • Whic h of the following intracellular targets was most likely in hibited by the drug?
· 22 •
134415; zheng
' 23 . We value your feedback!
A. Actin
'24 .
B. Centromere
' 25 •
. 26 . C. Microfilament
· 27 •
D. p53
' 28 .
' 29 . E. Topoisomerase
' 30 .
· 31 •
. 32 .
· 33 • The correct answer is B. 48% chose this.
Centromere M,crotubutes Mitos,s Meiosis Chromosomes Centrosome Kinetochores
' 34 .
· 35 •
. 36 .
,Pad"" 2041 29« _ •

+ G R< UI usmle rx com/ap l#qmax1 + III :


~
. Mark <l I>- -I ~
.' ' C';'\I
QID 3943 i PreYlous Next l a b ' ues Notes Cakul~tor
The correct answer IS B. 48% chose this.
·18 •

"9.
' 20 .
Centromere Micro!ubules MitOSIS Meiosis Chromosomes Centrosome Kinetochores

· 21 •
· 22 •
· 23 •

'24 •
. 25 •
. 26 .
· 27 • Kinetochore Kinetochore
microlubules
' 28 .
' 29 .
.30 . Centromere
Chromatid
· 31 •
. 32 .
· 33 •

' 34 . The centromere is the part of a chromosome that lin ks sister chromatids at the center such that
· 35 • it assumes the classic "X" configuration shown in the diagram. It is at the centromere of the
.36 . chromosome that the kinetochore attaches to the microtubules, allowing the chromatids to
migrate to opposite sides of the cell toward the centrosomes (a lso known as microtubule
centers, MTOCs). The spindle is a cytoskeletal structure consisting

.,8 . The centromere is the part of a chromosome that links sister chromatids at the center such that

"9.
' 20 .
it assumes the classic "X" configuration shown in the diagram. It is at the centromere of the
chromosome that the kinetochore attac hes to the microtubules, allowing the chromatids to
migrate to opposite sides of the cell toward the centrosomes (a lso known as microt ubule
· 21 • organ izing cent ers, MTOCs). The spindle apparatus is a cytoskeletal structure consisting of
· 22 • microt ubules that function during cell division (mitosis or meiosis) to sepa rate sister
' 23 . chromatids or homologous chromosomes. The assembly of the spindle apparatus begins during
pro-meta phase, in which the centrosomes (MTOCs) migrate to oppos ite poles of the cell.
'24 .
Microtubule polymerization proceeds in a counterparallel fas hion from each centrosome .
. 25 . Kinetochores that had preassembled on each chromosome's centromere capture some of the
.26 . growing microtubule ends, leading to chromosomal alignment at the metaphase plate.
· 27 • Kinesins, guanosine triphosphate-powered molecula r motors, "walk" along the microtubules in
a unidirectional manner to both separate the sister chromatids and further elongate the spindle
'28 .
apparatus as the microtubu les themselves actively depolymerize and repolymerize. Therefore, a
' 29 . drug t hat inhibits t he function of the microtubul es, ki nesi ns, ki netochores, or centromere woul d
.30 . severely impair the assembly of the spind le apparatus and force the cells to arrest in prophase.
· 31 • A is not correct. 11% chose this .
. 32 . Actins are globular prot ein subunits t hat assemble in a helica l fas hion to give rise to
' 33 •
microfilament s of the cytoskeleton and thin filaments of the muscl e contractile appa ratus. They
are important to muscle contraction, basic cell motil ity, and organellelvesicular transport, as
' 34 .
well as cytokinesis. They are not involved in the assembly of the spindle apparatus.
· 35 • Microfilaments Microtubules assO(iated WIth cytoplasmic filaments Globu lar proteinuria Motility

.36 . C is not correct. 23% chose this.


The microfilament is one of the th ree
,Pad "" 2041 29« _ •

+ G R< UI usmle rx com/ap l#qmax1 + III :


Item ' 3 1 of 44 L • Mark --<l t:> _! ~. C';'\I
QID 3943 i Pre Ylous Next la b~ ues Notes Cakul~ tor

The mlcrofilament IS one of the three types of filaments making up the eukaryotlc cells
· 18 • cytoskeleton. It is made of actin subun it s. Microfilaments are not involved in the formation of
·,9 . the spindle apparatus.
Microlubules ~ssociated with cytopl ~smic fil~ments Microfil<lments
·20 .
· 21 • o is not correct. 8% chose this.
· 22 • p53 is an intracellu lar protein that can arrest the progression from G, phase to S phase when
· 23 •
DNA damage is detected. Many cancer cells have mutations in p53, allowing for uncontrolled
growth. Inhibiting p53 would increase, not arrest, the growth of cancer cells.
·24 . DNA O"m.lge
. 25 .
E is not correct. 10% chose this .
. 26 .
Topoisomerase inhibitors such as etoposide would iinh ibit S-phase and G2-phase, but would not
· 27 •
affect mitosis directly.
• 28 . Etoposide Mitosis

· 29 •
. 30 .
Bottom line:
· 31 •
The spindle apparatus is a complex structure that is essentia l for mitosis and meiosis. It
·32 •
involves microtubule polymerization at opposing centrosomes, and microtubule capture by
· 33 • kinetochores located on each chromosome's centromere. I nhibition of any of these elements,
·34 . with the exception of centrosomes, may lead to arrest or failure of the apparatus to assemble.
Mitosis Meiosis Kir>etochores Centromere
· 35 •
. 36 .

Topoisomerase inhibitors such as etoposide would iinh ibit S-phase and G2-phase, but would not
.,8 . affect mitosis directly.
·,9. Etoposide Mitosis

·20 .
· 21 •
Bottom line:
· 22 •
The spindle apparatus is a complex structu re that is essentia l for mitosis and meiosis. It
· 23 . involves microtubule polymerization at oppos ing centrosomes, and microtubule capture by
·24 . kinetochores located on each chromosome's centromere. Inhibition of any of these elements,
. 25 . with the exception of ce ntrosomes, may lead to arrest or failure of the apparatus to assemble .
.26 . Mitosis Meiosis Klr>etochores Centromere

· 27 •
• 28 .
I il1ifi'·'I.) for year: 2015
· 29 . FI RS T AI D FA CTS

. 30 .
FAt5.p , 72.1
· 31 •
Ce ll cy cle ph illSes Chedpoinh con lrol 1 r,"'sili o ll ~ between plhlses of cell cycle. Thi~ process is regulated b)' crc1 ins.
·32 • C)din.dependel1 t kina ses (CDKs). and tUlllor suppressors. M phase (shorkst phase of cel l crc1e)
· 33 • includes mitosis (prophase. promctaphase. metaphasc. anaphase. telophase) and crtokin csis
(cytoplasm spli ts in two). G 1 and Go arc of variable dumt ion.
RtGUlATION Of em (vm

CDKs COll5titutive and in adi"e.


Cyclins
,Pad 9 20 4 3 29« • •

+ G R< UI usmle rx com/ap l#qmax1 + III :


Item ' 32 of 44
QID 24n i
L • Mark --<l
Pre Ylous
t:>
Next
_!
la b~ ues
~.
Notes
C';'\I
Cakul~ tor

· 24 " A 58-year-ol d man comes to the doctor complaini ng of arthralgia in his hands and knees
lasting several weeks and mild fever. He reports a history of high blood pressure and states
· 25 "
that he is t aking medications to help control it. His medicati on regimen was changed
· 26 " recently when he complained that his previous medication caused a "nasty cough." Physical
·27 "
· 28 "
examination reveals hepatomegaly, but there are no abnormal skin findings or focal neurologic
deficits. Laboratory studies rule out hematologic abnormal ities and renal disease; however, the
patient tests positive for anti-nuclea r anti body and negative for anti-DNA antibodies .
· 29 "
· 30 "
Whic h of the following medications is most likely responsible forth is patient's symptoms?
· 31 "
·32 "
· 33 "
A. Bleomycin
We value you r feedback! 13441 5; zh eng

· 34 " 8. Enalapril
· 35 "
C. Haloperidol
· 36 "

· 37 " D. Hydralazine

·38 "
· 39 "
E. Rifampin

· 40 "

The correct answer is D. 65% chose this.


Th is is the clinical picture of drug-induced lupus, which affects male and fe"m~;a~:I~e,~;~~~J~att~d
eq ually (as opposed to systemic lupus erythematosus, which has a higher ir

· 24 " This is the clinical picture of drug-induced lupus, which affects male and female populations
equally (as opposed to systemic lupus erythematosus, which has a higher incidence in the
· 25 "
female population). Anti-nuc lear antibodies are seen commonly in drug-induced lupus, unlike
· 26 "
anti-DNA antibodies, wh ich normally are present in systemic lupus erythematosus. Among the
·27 "
· 28 "
drugs listed, only hydralazine has been shown to be associated with drug-induced lUpus. Other
drugs known to cause lupus are proca inamide, isoniazid, chlorpromazine, penicillamine,
sulfasalazine, methyld opa, and quinidine.
· 29 "
Hydralazine Procainamide Chlorpromazine Penkillam ine Sulfasalazine quinIdine Drug·induced lupus erythematosus
· 30 "
lupus Erythematosus. Systemic
· 31 "
·32 "
· 33 "
A is not correct. 6% chose this.
Bleomycin is an antin eoplastic agent that most commonly is associated with pu lm onary
fibrosis. It would not be used to treat hypertension, nor has it been associated with drug-
· 34 " induced lUpus.
· 35 " Bleomycin P1.Ilmonary Fibrosis Drug·induced lupus erythematosus

· 36 "
B is not correct. 22% chose this.
· 37 " Enalapril, an angiotensin-converting enzyme inhibitor, is associated most often with a dry
·38 "
· 39 "
cough. Enalapril also can cause a severe anaphylactoid reaction with a rash and angioedema in
a very sma ll number of patients.
Enalapril Anaphylactoid reactioo Angioedema Dry tough
· 40 "
C is not correct. 4% chose this.
Haloperidol can lead to hepatomegaly, but would not cause the other symptoms associated
with drug-induced lUpus. One common adverse effect of haloperidol is tard ive dyskinesia
(random, uncoordinated, and involuntary movements and other extrapyram idal effects).
,Pad 9 20 4 3 29« • •

+ G R< UI usmle rx com/ap l#qmax1 + III :


Item ' 32 of 44 Mark --<l t:> _! ~. C';'\I
..
L •

QID 24n i
- p .. Pre Ylous Next

p 9 y,
la b~ ues Notes Cakul~ tor

with drug-induced lUpus. One common adverse effect of haloperidol is tard ive dyskinesia
Y P .
, 24 "
(random, uncoordinated, and involuntary movements and other extrapyram idal effects).
, 25 "
Haloperidol is not used to treat hypertension.
, 26 " Haloperidol Drug·ioolKed lupus erythem.lt05us Tardive Dys ki nesia

, 27 "
E is not correct. 3% chose this.
' 28 " Isoniazid, not rifampin (both of which are first-line treatments for tuberculosis), is associated
, 29 " with drug-induced lUpus. The most serious adverse effect of rifampin is hepatotoxicity, for
, 30 " which patients need frequent liver function tests. Other adverse effects include gastrointestinal
, 31 "
disturbances, fever, and discoloration of body fluids such as urine and breast milk to an orange-
red color, a benign but frightening change. Rifampin is not used to treat hypertension.
, 32 "
Mampin Tuberculosis Drug· induced lupus erythematosus Hepatotoxicity Liver function Breast
, 33 "
, 34 "
Bottom Line:
, 35 "
, 36 " Although drug-induced lupus presents with many of the same symptoms as systemic lupus
erythematosus, patients with drug-induced lupus often will test positive for anti-nuclear
' 37 "
antibodies also. Hydralazine, a common antihypertensive medication, has been associated
, 38 " with drug-induced lUpus.
, 39 " Hydralazine Drug·induced lupus erythematosus Lupus Erythematosus, Systemic

, 40 "

I ill if., , .j 1.1 for year: 2015


t
FIRS T A I D FACTS

A 12-year-old boy is brought by his mother to the clin ic because of persistent rash and
, 24 "
peculiar behavior. She reports that over the past few years the boy has developed a rough,
, 25 "
red rash on his face, neck, and hands that gets worse when he is outdoors . She reports that
, 26 " he has been "tottering like an old man" si nce the beginning of the year and that his hands are
, 27 "
consistently shaky. He is not doing well in school. More often than not his stools are loose and
watery. The mother reports that he eats a normal, balanced diet. On physical examination, the boy
' 28 "
appears well nourished, with an erythematous, scaling rash on his face, neck, and exposed
, 29 " extremities. He has bi lateral nystagmus and an intention tremor. He walks with a teetering, wide-
, 30 " based gait. Urinalys is revea ls significant am inoaciduria.
, 31 "

, 32 " Deficiency of which amino acid is likely responsible for this patient's presentation?

' 33 " 13441S : zheng


We value your feedback!
, 34 " A. Cystine
, 35 "
B. Histidine
, 36 "

' 37 " C. Isoleucine

, 38 " O. Tryptophan
, 39 "
, 40 "
E. Tyrosine
,Pad 9 20 4 3 29« • •

+ G R< UI usmle rx com/ap l#qmax1 + III :


Item ' 33 of 44
QID 1539 i
L • Mark --<l
Pre Ylous
t:>
Next
_!
la b~ ues
~.
Notes
C';'\I
Cakul~ tor

Th is patient IS presenting With photosens it ive dermatitis, diarrhea, and central nervous system
· 24 " disturbances pointing to the triad of pel lagra. Pellagra is the result of niacin deficiency and can
· 25 " occur because of insufficient dietary intake or an inherited error of metabolism. Considering the
· 26 " boy appears well nourished on exam with no history of insufficient dietary intake, the possibil ity

·27 "
· 28 "
of an inherited error of metabolism is higher on the differential. This possibility is further
suggested by t he boy's sign ificant aminoacidu ria. There are several disorders caused by
deficient renal tubu lar reabsorption of am ino acids that can result in high levels of amino aci ds
· 29 " lost in the urine. One of t hese disorders is known as Hartnup's disease, which is caused by
· 30 "
mutation in the transport of neutral amino acids, including that of tryptophan. Tryptophan is a
precursor to niacin. Hartnup's disease therefore present s as pellagra due to tryptophan, and
· 31 " consequently niacin, deficiency.
·32 " Diarrhea Pellagra MeL3bolism AminoAcids Muwtion Tryptophiln Niacin Derffiiltitis

·33 "
· 34 "
A is not correct. 16% chose this.
Deficiency of cystine results in cystinuria, the most common genetic error of amino acid
· 35 " trans port. It is a disorder caused by impaired transport of cystine and dibasic amino acids
· 36 " (lysine, ornithine, and arginine) in the proximal rena l tubule and gastrointestinal tract. Patients
present with rena l stones resulting in obstructive nephropathy, pyelonephritis, and, ra rely, renal
· 37 "

·38 "
· 39 "
fai lure.
Cystinuria lYSII"\e Ornithine Arginine Kidney Tubules. PrOXimal Gastrointestinal tract structure

B is not correct. 6% chose this.


· 40 "
A relatively common inborn error of metabolism affecting histidine is histidinemia, caused by a
deficiency in the enzyme histidase, necessary for the catabolism of histidine. These individuals
have elevated levels of histidine in the blood and urine. It is considered to be a benign disorder.
Histidine Histidinemla Cawbolism Inborn Errors of Metabolism

A metabolic disorder involving isoleucine (as well as other branched-chain amino acids) is maple
· 24 " syrup urine disease. Th is condition is caused by a deficiency in branched-chain a-keto acid
· 25 " dehydrogenase, leading to elevated levels of valine, leucine, and isoleucine in plasma and urine.
· 26 " Centra l nervous system problems are common, as are feeding problems, vomiting, dehydration,

·27 "
· 28 "
severe metabolic acidosis, and a characteristic maple syrup odor to the urine. Treatment
involves dietary restriction of branched-chain amino acids.
Am ino Acids leucine Vomiting Dehydration Metabolic Diseases Ma~e Syrup Urine Disease feeding difficulties

· 29 "
E is not correct. 19% chose this.
· 30 "
Deficiency of tyrosine is found in phenylketonuria (PKU), caused by deficiency of the enzyme
· 31 " phenylalanine hydroxylase, which converts phenyla lanine to tyrOSine. PKU is the most common
·32 " inborn error of amino acid metabolism. Affected individua ls have high phenylalanine levels,

·33 "
· 34 "
mental retardation, and hypopigmentation (fair hair, light skin color, blue eyes). There is a
neonatal screen for PKU . If detected early, these adverse sequelae can be prevented by adhering
to a diet low in phenylalanine.
· 35 " Tyros ine Phenylalanine Mental Retardation Neonatal Screen ing

· 36 "

· 37 " Bottom line:


· 38 "
Pellagra, caused by niacin deficiency, can be a result of insufficient dietary intake or an inborn
· 39 "
error of metabolism leading to reduced renal tubular reabsorption of tryptophan (Hartnup
· 40 " disease).
· 41 " Pellagra Tryptoph.ln Niacin Inborn Errors of Metabolism HanrnJp Disease
,Pad 9 20 4 3 29« • •

+ G R< UI usmle rx com/ap l#qmax1 + III :


Item ' 34 of 44
QID 4128 i
L • Mark --<l
Pre Ylous
t:>
Next
_!
la b~ ues
~.
Notes
C';'\I
Cakul~ tor

· 24 " A 56-year-old male patient in the intensive care unit develops a fever of unknown origin
several days post-operatively. Blood cu ltures are drawn and found to have Enterococcus.
· 25 "
Gentamicin therapy is initiated, but within 2 days, it is apparent that the treatment is not
· 26 " clearing the infection.
·27 "
· 28 " Which of the following is the most common mechanism of resistance to gentamicin?
· 29 "
134415 : zheng
· 30 " A. Decreased permeabi lity
· 31 " We va lue your feedback!
·32 "
· 33 "
B. I nactivation by acetylation

C. Modification of the ribosomal binding site


· 34 "
D. Production of a hydrolase
· 35 "
· 36 " E. Production of an efflux pump
· 37 "

·38 "
· 39 "
· 40 " The correct answer is B. 38% chose this.
Gentamicin is an aminoglycoside. Bacterial resistance is usually achieved by plasmid-mediated
production of group transferases that cova lently modify (via acetylation, adenylation, or
phosphorylation) the antibiotic such that it is no longer active. Adverse effects of
am inoglycosides include ototox icity, nephrotoxicity, and teratogenicity.

The correct answer Is B. 38% chose this.


· 24 "
Gentamicin is an aminoglycoside. Bacterial resistance is usually achieved by plasmid-mediated
· 25 " production of group transferases that cova lently modify (via acetylation, adenylation, or
· 26 " phosphorylation) the antibiotic such that it is no longer active. Adverse effects of

·27 "
· 28 "
am inoglycosides include ototoxicity, nephrotoxicity, and teratogenicity.
Bacterial resisliInce Phosphoryiatlon Ototoxicity

· 29 " A is not correct. 7% chose this.


· 30 " Reduced permeability of the bacterial cell to antibiotics is a form of resistance to some drugs
such as macrolides (for example, erythromycin, azithromycin, and clarithromycin). It is not the
· 31 " most frequently occurring form of resistance to aminoglycosides. Adverse effects of macrolides
·32 " include gastrointestinal discomfort, acute cholestatic hepatitis, eosinophilia, and skin rash.
·33 "
· 34 "
ErythromyCin Amhromycon Clarithromycon Disorder characterized by eosinophilia

C is not correct. 32% chose this.


· 35 " Modification of the ribosomal bind ing site for antibiotics is a common form of resistance to
· 36 " tetracyclines, ch loramphenicol, and macrolides, but not aminoglycos ides.
Chloramphenicol
· 37 "
· 38 " D is not correct. 5% chose this.
· 39 " I3-Lactam drugs inhibit bacterial growth by inhibiting crosslinking of peptidoglycan in the cell
· 40 " wall. Bacteriall3lactamase hydrolyzes 13-lactam antibiotics and is the most common means of
resistance.
· 41 " Determination of bacteflalgrowth

E is not correct. 18% chose this.


Active efflux of antibiotics is an important cause of resistance to tetracyclines but less
,Pad 9 2043 29« • •

+ G R< UI usmle rx com/ap l#qmax1 + III :


Item ' 34 of 44
QID 4128 i
L • Mark --<l
Pre Ylous
t:>
Next
_!
la b~ ues
~.
Notes
C';'\I
Cakul~ tor

· 24 " E Is not correct. 18% chose this.


· 25 " Active efflux of antibiotics is an important cause of res istance to tetracyclines but less
· 26 " important for aminog lycosides. Adverse effects of tetracyclines include tooth discoloration and

·27 "
· 28 "
photose nsitivity.

· 29 "
Bottom Li ne:
· 30 " The mechan ism for aminoglycosi de resist ance is covalent modification via acetylation,
adenylation, or phosphorylation.
· 31 "
·32 "
· 33 "
Phosphoryl<llion

· 34 "
hl;*,i.·,.·1 for year: 2015
· 35 " FaST "" D FACTS

· 36 "
FA15, p . 154.2
· 37 " C Cllt~llIicin . Nl"Omrcill. Amib(:ill, "" lea n" (amillogl)cosidc) GNATS caNNOT

·38 "
Aminogl),CDsides
Tobramyc ill. Streptomycin l.:ill anaerobes
· 39 " MWWIISM B;lcte ric i(bl; irrc\crsiblc inhibition of initi;ltioll
complex through bindiug of the 30S subun it.
· 40 " Can ca use misreading of mR NA. Also hlock
translocation. Require 0 , for uptake; tlll"rcfore
incffcrtil·e against amlrr~bes
WN I(AlUS< Severe graill-negative T()(l infections. Synergi.ltie
\\ilh Il-lactaillantibiotics

· 24 " A 14-year-ol d gi rl with cystic fi brosis is admitted to the hospit al with fulminant pneumonia. IAA[
Bro nchoalveolar lavage revea ls Pseudomonas along with several other organisms. After fa iled
· 25 "
therapy with several other anti biotics, im ipenem, a broad-spectrum antibioti c from the
· 26 " carbapenem class, is administered. Cilastatin, another agent, is almost invariably administ ered
·27 "
· 28 "
with imipenem in the treatment of severe bacterial infections.

· 29 " What is the functio n of cilastati n?


· 30 " 13441S: zheng

· 31 "
·32 "
· 33 "
A. Disruption of t he bact erial cell wa ll to allow ent ry of im ipenem

· 34 " B. I nhibition of bacterial f3-lactamase


· 35 "
C. I nhibition of bacterial folic aci d synthesis
· 36 "
D. I nhibitio n of DNA gyrase
· 37 "
·38 "
· 39 "
E. I nhibition of imipenem meta bolism at the proximal t ubule of the kidneys

· 40 "
We value your feedback!
,Pad 9 2043 29« • •

+ G R< UI usmle rx com/ap l#qmax1 + III :


Item ' 3S of 44
QID 4109 i
L • Mark --<l
Pre Ylous
t:>
Next
_!
la b~ ues
~.
Notes
C';'\I
Cakul~ tor

Cilastatin is an inhibitor of dehydropeptidase-l, an enzyme in the brush border of the proximal


· 24 "
convoluted tubule that metabolizes imipenem. By inhibiting this enzyme. cilastatin maintains
· 25 " effective serum levels of imipenem and prevents the nephrotoxicity that can be caused by
· 26 " imipenem metabolites. Of note, meropenem, another carbapenem, is resistant to
·27 "
· 28 "
dehydropeptidases and does not require coadministration of cilastatin.
Cil<lswtin I(jdneyTubule5, Pro~im,, 1 lmipenem meropenem

· 29 " A is not correct. 13% chose this.


· 30 " Cilastatin itself has no effect on bacteria. An extended-spectrum J3-lactam drug, imipenem itself
functions by disrupting cell wa ll synthesis and would not require another drug to do so.
· 31 "
·32 "
· 33 "
Cil"stiltin Imlpenem

B is not correct. 14% chose this.


Although based on the J3-lactam chemical structure, imipenem is inherently resistant to 13-
· 34 "
lactamases and thus does not require coadmin istration of J3-lactamase inhibitors. I3-Lactamase
· 35 " inhibitors are used in combination with penicillins, such as in the combination drug amoxicillin-
· 36 " clavulanate.
Imipenem Amoxicillin
· 37 "
·38 "
· 39 "
C is not correct. 4% chose this.
Cilastatin does not act as an antimetabolite. Sulfa drugs such as sulfamethoxazole
· 40 " competitively inhibit the production of folic acid by bacteria.
Cil"stiltin fol,c Ac id

D is not correct. 4% chose this.


Cilastatin has no intrinsic antimicrobial properties . Inhibition of DNA gyrase is the
of action of the fluoroquinolones.

· 24 " Bottom Line:


· 25 "
Imipenem is used with cilastatin during severe bacteria l infections involving aerobic and
· 26 " anaerobic gram-positive and gram-negative bacteria (including Pseudomonas aeruginosa and
·27 "
· 28 "
Enterococcus species). Cilastatin inhibits renal degradation, thereby preventing rapid
deactivation of imipenem. Notably, cilastatin is not a first-line antibiotic because of the fear of
increasing bacteria l resistance. It is used only when severe infections have been found to be
· 29 "
resistant to most other agents.
· 30 " Im ipenem Cil<lstiltin a"cter i<ll resistilnce a"cteri<llinfections Fe"r (find ing)

· 31 "
·32 "
· 33 " hl;fi'",,' for year: 2015
FIII.ST AI D FA CT S
· 34 "
· 35 " FAI5.p, 183,1
· 36 " Carbapenems Im ipcnem. mcropenem . crtapcncm. dori pc ncm.
· 37 " M(HAHISM Im ipenem is a broad-speclfllm, p-lael:lmaSC- W ith imipcnem, 'Ihe kill is laslin ' lI' ilh

·38 "
· 39 "
res isla nl earb"pe nem, Always adm in iSlered
wilh ei!asla!in (inhi bil or of ren al
dchp lrol>c plidase I) 10 I i n ~cti \''' lion of drug
ci laslalin."
Newe r ear bape nems incl nde erlapenem (limi!ed
I'seudomonas Co\'er;lge) alHl doripe l\el\'
in ren"llnhnles.
· 40 "
(lIN I(AlUSf Cram-posili\'e cocci, gram-negali\,e rods. ;lIld
anae robes, Wide spect rnm. bnl signifi ea nl side
dfl"C ls li milnse 10 lifl'-Ihrea k n ing ink"C lions
or afler olhe r dTllgs ha"c fail ed . i'To'I eropenem
has a I risk of seizn Tes and is stable 10
dchydrope pli dase 1
,Pad 9 20 4 3 29« • •

+ G R< UI usmle rx com/ap l#qmax1 + III :


Item ' 36 of 44
QID 1258 i
L • Mark --<l
Pre Ylous
t:>
Next
_!
la b~ ues
~.
Notes
C';'\I
Cakul~ tor

· 24 " Rotenone is a naturally occurring chemical that is used primarily as a fish poison. It exerts its IAA[
effects by competitively inhibiting reduced nicotinamide adenine dinucleotide
· 25 "
dehydrogenase, the first complex in the electron transport chain . Application of rotenone
· 26 " will halt cellular respiration at this stage.
·27 "
· 28 " Which of the following combinations describes the effects of rotenone on the enzyme kinetics of
· 29 " NADH?
· 30 " 134415 : zheng

· 31 "
·32 "
· 33 "
A. I n competitive inhibition, the Michaelis-Menten constant increases while the
maximum reaction rate remains the same
· 34 " B. The Michaelis-Menten constant decreases while the maximum reaction rate
· 35 " remains unchanged
· 36 " C. The Michaelis-Menten constant increases while the maximum reaction rate
· 37 " decreases

·38 "
· 39 "
D. The Michaelis-Menten constant remains the same while the maximum reaction
rate decreases
· 40 "
We value your feedback!

· 24 " Rotenone competitively inhibits reduced nicotinamide adenine dinucleotide dehydrogenase


· 25 " (NADH). As a result, the Michaelis-Menten constant increases because rotenone competes with
NADH at the enzymatic active site. However, the maximum reaction rate for a given
· 26 "

·27 "
· 28 "
concentration of enzyme does not change, because it can eventually be achieved by increasing
the amount of substrate (ie, NADH) available to the enzymes .
NAOH

· 29 " B is not correct. 18% chose this.


· 30 " The Michaelis-Menten constant decreasing with the maximum reaction rate remaining the same
· 31 " is not a known pattern of inhibition.
·32 "
· 33 "
C is not correct. 14% chose this.
This combination does not describe competitive or noncompetitive inhibition.
· 34 " o is not correct. 14% chose this.
· 35 " This combination of the Michaelis-Menten constant remaining constant and the maximum
· 36 " reaction rate decreasing describes noncompetitive inhibition. However, this is not the observed
mechanism of action of rotenone.
· 37 "
·38 "
· 39 "
Bottom Li ne:
· 40 " Competitive inhibitors increase the Michaelis -Menten constant (Km), which is the substrate
concentration at which half of the enzyme in the sample is saturated. However, competitive
· 41 " inhibition has no effect on the maximum reaction rate . Competitive inhibition can be
· 42 " overcome by increasing the concentration of the natural substrate. Noncompetitive inhibitors
· 43 " decrease the maximum reaction rate but do not change the Km.
A 86/s
v 06/s

8
l ock
0
End Block
,Pad 9 20 4 3 29« • •

+ G R< UI usmle rx com/ap l#qmax1 + III :


Item ' 36 of 44
QID 1258 i
L • Mark --<l
Pre Ylous
t:>
Next
_!
la b~ ues
~.
Notes
C';'\I
Cakul~ tor

· 24 "
· 25 "
· 26 "
ul;fi,·ij·, for year: 2015
FIRST "10 FACTS

·27 "
· 28 "
FA15, p. 242 .1

Enzyme kineti cs
· 29 "
Michae l i5~Menten [S) = CQllce1l1r~t ioli of substrate; V = ..docit)'. Kill is invcrsel), related 10 the ~ffinitr of the
· 30 " kinetics ~rlz)'1tI" for ih Sl1Lst r~!I~.
Ym.. is directly proportional to the Cllz)"ne
· 31 "

·32 "
· 33 "
CQllCC1l1rJliOll
~·I osl cn~ym,lIic rc~ctiOIl S follow ~ hn>crholic
cun'e (i.e .. ~To'Iich<lclis-~'l entcn kiuctics);
hO\\C<"Cf. enzyma tic reactions thai exhibit a
· 34 " sigmoid cnf"C l1>[mll), indicate coopt'rnlive
· 35 " kinetics (i,e., hemoglobill)
Lineweaver-Burk pial t r-;IItcrcepl. I V" .. ,
· 36 "
The further to the right th e ~_illtcrcept (i,c_. closer
· 37 " 10 zero), the grC;l ter the K,n ~1\d the luwer the

·38 "
· 39 "
affinil)'.

Enzyme inhibition RevCfsible eompclilivc illhibi lors cross each


· 40 "
othcr compcli li"e! r, ",hercds noncompeliti"c
inhibitors do nol.

· 24 " A 46-year-old man presents with a temperature of 38.6°( (1 01.5°F). He was fine 2 weeks ago,
but started feeling poorly a few days ago after a dental examination. Physical examination
IAAI
· 25 "
reveals tender raised lesions on the beds of his fingers and toes, and painless, erythematous
· 26 " lesions on hi s palms and soles, like those in the images. On further questioning, the physician
·27 "
· 28 "
discovers that the man has a history of rheumatic fever as a child. Blood cultures are drawn.

· 29 "
· 30 "

· 31 "

·32 "
·33 "
· 34 "
· 35 "
· 36 " Left: Image courtesy oJ Tsagarcas, et 0/. West} Emerg Med. Feb 2012; 13(1): 92-93. do;: 1O.5811/wescjem.2011.6.6806. Right: Image

·37 "
· 38 "
courtesy oj Roberto}. Galindo

· 39 " The most likely causative organism will have which characteristics?
· 40 " 134415: zheng

· 41 "

A. Gram-positive cocc i, catalase-negative, ~-hemo l ytic, bacitracin-resistant

Gram-positive cocci, catalase-negative, ~-hemo l ytic, bacitracin-sensitive


,Pad 9 2044 29« • •

+ G R< UI usmle rx com/ap l#qmax1 + III :


Item ' 37 of 44
QID 4019 i
L • Mark --<l
Pre Ylous
t:>
Next
_!
la b~ ues
~.
Notes
C';'\I
Cakul~ tor

A. Gram-positive cocci, catalase-negative, !3.hemolytic, bacitracin-resistant


· 24 "
· 25 " B. Gram-positive cocci, catalase-negative, l3-hemolytic, bacitracin-sensitive
· 26 "

·27 "
· 28 "
C. Gram-positive cocc i, catalase-negative, a-hemolytic, optochin-resistant

D. Gram-positive cocci, catalase-positive


· 29 " E. Gram-positive, weakly acid-fast rod
· 30 "

· 31 "
·32 "
· 33 "
We value your feedback!

· 34 "
· 35 "
· 36 "

· 37 "

·38 "
· 39 "
The correct answer is C. 49% chose this.
The patient is likely suffering from subacute bacterial endocarditis, likely precipitated by his
· 40 " recent dental examination. He has classic symptoms of Osiers nodes (tender raised lesions on
finger or toe pads--Ieft-hand image) and Janeway's lesions (non-tender erythematous lesions on
the palms or soles, right-hand image). Viridans streptococci are a common cause of this form of
endocarditis, which generally occurs in the setting of dental procedures on patients with
diseased heart valves. Vi ridans streptococci are gram-positive, catalase-negative, I

· 24 "
Streptococcus agalactiae, or Group B streptococci, are gram-positive, l3-hemolytic cocci. They are
bacitracin resistant. They cause pneumonia, meningitis, and sepsis, mainly in babies, and thus
· 25 "
are unlikely to be responsible for this patient's condition.
· 26 " a<l(it'<lcin Pneumoni<l Meningitis

·27 "
· 28 "
B is not correct. 25% chose this.
Streptococcus pyogenes is a gram-positive, l3-hemolytic group A streptococcus. These organisms
· 29 " are bacitracin sensitive. This organ ism is most commonly associated with illnesses such as
· 30 " pharyngitis, cellulitis, impetigo, scarlet fever, toxic shock synd rome, rheumatic fever, and acute
glomerulonephritis-but not with endocard iti s.
· 31 "
·32 "
· 33 "
a<lcit'<lcin PharyngItIS Cellu lItis Impetigo Scarlet Ff!ver RheumatIC Fever Acute glomeru lonephritis

D is not correct. 8% chose this.


· 34 "
Staphylococcus aureus is a common cause of acute endoca rditis. These organisms often are the
culprit in the setting of previously healthy intravenous drug users. They are gram-positive,
· 35 "
catalase-positive cocci.
· 36 " CATALASE Acute endocard itis

· 37 "
E Is not correct. 4% chose this.
·38 "
· 39 "
Nocardia asteroides is a gram·positive, weakly acid·fast rod. It is known to cause pulmonary
infection in immunocompromised individual s. It is not known to cause bacterial endocarditis.
· 40 " a<lcte, i<ll ErxIoc<l,ditis

Bottom Line:
,Pad 9 2044 29« • •

+ G R< UI usmle rx com/ap l#qmax1 + III :


Item ' 37 of 44 Mark --<l t:> _! ~. C';'\I
..
L •

QID 4019 i
-
Pre Ylous
" .- Next la b~ ues Notes Cakul~ tor

, 24 " Nocardia asteroides is a gram-positive, weakly acid·fast rod. It is known to cause pulmonary
infection in immunocompromised individuals. It is not known to cause bacterial endocarditis.
, 25 "
Bacterial Er.docarditis
, 26 "

, 27 "
' 28 "
Bottom Li ne:
, 29 " Viridans streptococci are a group of gram-positive, catalase-negative cocci that are also alpha
, 30 " hemolytic. They can cause subacute endocarditis, generally in the setting of previously
diseased heart valves.
, 31 " CATALASE Subacute er.docarditis

, 32 "
, 33 "
, 34 " Iii lif, , .j I,' for year: 2015
FIRST AID FACTS
, 35 "
, 36 "
FAtS, p . 1292
' 37 " Viridans group a -he molylic. They arc no rmal flora of Sa nguinis = blood . Think. ··th ere is lots of
, 38 " streptococci the oropharynx that cause de nta l cari N bl ood in the hea rt '· (endoca rd iti s). S. lKluguiuis
(Strcptococcl!S HUJtlms) and subacute makes dextrallS, whi eh bind to fihrin-platelet
, 39 "
haeteri al e ndocardit is at damal(ed heart al(l!:reeate$ on dam31(ciI heart l"lI k cs.
, 40 "

, 24 "
A 19-year-old student presents with complaints of a low-grade fever and general fatigue for
approximately 9 days. His temperature is 38°( (100AOF), blood pressure is 108/65 mm Hg,
, 25 " pu lse is 87/min, and respiratory rate is 20/min. Physical examination reveals cervical
, 26 " lymphadenopathy and m ild hepatos plenomega ly . Results of a urine sediment analysis are shown
, 27 " in the image ,_""'__- -
r
' 28 "
, 29 "
, 30 "
, 31 "

, 32 "

' 33 "
, 34 "
, 35 "
, 36 "

' 37 "
, 38 "
, 39 "
, 40 "
, 41 "
,Pad 9 20 4 4 29« • •

+ G R< UI usmle rx com/ap l#qmax1 + III :


Item ' 38 of 44
QID 4024 i
L • Mark --<l
Pre Ylous
t:>
Next
_!
la b~ ues
~.
Notes
C';'\I
Cakul~ tor
We value your feedback!
A. Cytomegalovirus
· 24 "
· 25 " 8. Epstei n-Ba rr virus
· 26 "

·27 "
· 28 "
C. Human herpesvirus 1

D. Human herpesvirus 8
· 29 "
E. Varicella-zoster virus
· 30 "

· 31 "

·32 "
· 33 " The correct answer is A. 49% chose this.
· 34 " The patient present s with sym ptoms of an infectious mononucleosis-like syndrome . These
· 35 " symptoms inc lude malaise, low-grade fever, cervica l lymphadenopat hy, and splenomega ly.
· 36 " Althoug h mono nucleosis is due to Epstein-Ba rr virus, frequent ca uses of mononucleosis-like
syndromes incl ude cytomegalovirus «(MV), acut e toxoplasmosis, and acute HIV infection. In this
· 37 "
case, the image shows the cha racteristic "owl's eye" inclusions of (MV.
· 38 " Splenomegilly To~oplasm05is Infectious Mononucleosis CefVIcilllymphildenopilthy Acute HIV infection Eye

· 39 "
B is not correct. 41% chose this.
· 40 "
Epstein-Ba rr virus causes mono nucleosis in young adults, but does not cause numerous giant
inclus ion cells.
e is not correct. 5% chose this.
h;i:~~~i~~;:~~~t;;yp;.e~ti;1.'is transm itted through respiratory sec retions and saliva and
" ke r atocor'1u r1Ctivi t i 5~ t"m " orallobe Ii. and

Human herpesvirus type 1 is transmitted through respiratory secretions and saliva and can
· 24 " result in gingivostomatitis, keratoconj unctivit is, temporal lobe encephaliti s, and herpes labia lis.
· 25 " It is not a known cause of mononucleosis.
· 26 " Gingivostomatitis Encephilhtls Herpes ldblahs Emlre temporililobe

·27 "
· 28 "
D is not correct. 3% chose this.
Human herpesvirus type 8 is transmitted through sex ual contact and can result in Kaposi 's
· 29 " sarcoma in patients co-infected with HJV. It is not a known cause of mo nonucleosiS .
Sarcoma
· 30 "

· 31 " E is not correct. 2% chose this.


·32 " Va ricella-zoster is a type of herpesvirus that causes ch ickenpox, shing les, encephalitis, and

·33 "
· 34 "
pneumon ia. It does not result in monon ucleosis .
Chi"':enpo~ Encepha litis Pnellmorua

· 35 "
· 36 " Bottom line:
· 37 " (MV causes a monon ucleosi s-type syndrome t hat is Monospot negative. Microscopically, (MV
· 38 " shows characteristic "owl 's eye" incl usion bod ies.
Eyo
· 39 "
· 40 "

· 41 "
hl;fi.·,.·1 for year: 2015
FIRST ,lID F,l CT S
,Pad 9 20 4 5 29« • •

+ G R< UI usmle rx c om/ap l#qmax1 + III :


Item ' 39 of 44
QID 1816 i
L • Mark --<l
Pre Ylous
t:>
Next la b~
_! ues
~.
Notes
C';'\I
Cakul~ tor

· 24 " A family who recently em igrated to the United States from Romania brings their 7-year-old IAA[
child to the pediatrician with complaints of red, itchy eyes and some swelling around them.
· 25 "
The chil d has had coughing with a runny nose and high fever for 3 days. Small lesions with
· 26 " white centers are seen in his oral cavity.
·27 "
· 28 " Which of the following is the most likely cause of this child's symptoms?
· 29 "
13441 5 : zheng
We value your feedback!
· 30 " A. Diphtheria
· 31 "

·32 "
· 33 "
B. Pertussis

C. Roseola
· 34 "
D. Rubella
· 35 "
· 36 " " E. Rubeola
· 37 "

·38 "
· 39 "
· 40 " The correct answer is E. 53% chose this.
Rubeola, also ca lled measles, is a relatively rare ill ness in the United States because of the
ubiquity of the measles/mumps/rubella (MMR) vaccine. It presents with the prodrome described
in this patient. The rash that spreads from head to toe over a 3-day period develops 1 or 2 days
after the appearance of Koplik's spots, which are red oral lesions with white centers.

Dip htheria is an illness virtually unknown in the United States because of the prevalence of the
· 24 "
diphtheria/tetanus/pertussis (DTa P) vaccine. It is caused by CorynebacterIum dlphtheriae and is
· 25 " characterized by a pseudomembranous pharyngitis. This is a life-threatening cause of sore
· 26 " throat.

·27 "
· 28 "
Oipf1theria Tewnus Pholryngl1is Pertussis Sore ThrO<lt

B is not correct. 5% chose this.


br~nd of Phenol

· 29 " Pertussis, or whooping cough, is also rare due to widespread vaccinations but has been on the
· 30 " upswing recently because of a decrease in the vaccination rate. It is a respiratory infection of
children that cha racteristically produces coughing spasms followed by a loud inspiratory
· 31 " whoop .
·32 " Vaccination Pertuss is

·33 "
· 34 "
C is not correct. 13% chose this.
Roseola is a febrile disease of very young children that begins with a high fever and progresses
· 35 " to a rash similarto measles. Infants and young children are most at risk. It is believed to be
· 36 " caused by human herpesvirus 6.
Measles
· 37 "
· 38 " D is not correct. 20% chose this.
· 39 " Rubella, also known as German measles, is a less severe viral exanthem . Many infections are
· 40 " subclinica l, but rubella can cause severe birth defects when infection occ urs during the prenatal
period.
· 41 " Rubella Measles Viral el«lnthE'm
,Pad 9 20 4 5 29« • •

+ G R< UI usmle rx c om/ap l#qmax1 + III :


Item ' 39 of 44
QID 1816 i
L • Mark --<l
Pre Ylous
t:>
Next
_!
la b~ ues
~.
Notes
C';'\I
Cakul~ tor

Meas les
· 24 "
· 25 " D is not correct. 20% chose this.
· 26 " Rubella, also known as German measles, is a less severe viral exanthem. Many infections are

·27 "
· 28 "
subclinica l, but rubella can ca use severe birth defects when infection occurs during the prenatal
period.
Rubel la Measles Viral exanthem
· 29 "
· 30 "
Bottom line:
· 31 "
·32 "
· 33 "
Remember the 3 C's of measles (rubeola)- Cough, Coryza (runny/stuffy nose), and
Conjunctivitis - and Koplik's spots. The rash of rubeola typically progresses in a crania l-to-
cauda l fashion beginning 1 to 2 days after the appearance of Koplik's spot s.
· 34 "
· 35 "
· 36 " I i i I if' , .j ,., for year: 2015
FIII. S T AI D FA CTS
· 37 "
·38 "
· 39 " Measles (rubeola) A parnm)'xoviHls that CHU~ measles. Usual 3 C's of mcasles:
FA1S, p. 1642

· 40 " virus presentation inl'oll'es prodroma l fel'er Co ugh


with cough, cor)'1.a. and conjunctil'itis, CorYZ:I
thcn evcntua lly Koplik spots 13, followed Conjunctil'itis
by a maculopaplllar rash [J that sta rh al Vitamill A sllpplemclllalioll ca ll r~,<llIce measles
Il,c hc,uUllcck and spre,uls dowllward mortality i1\ 1Il;,llIourishell or I'it;nuiu-dcficicnt
Lymph"dcllitis with W"rthi ,,-Finkcldcy gi;lIlt childrcn
I:, I

· 24 " A scientist is performing restriction mapping of various mutant and wild -type genes
involved in Drosophila fly wi ng shape. A specimen of interest is treated with restriction
· 25 "
enzymes, pi petted into a well of aqarose gel. and subjected to an electrica l field. Next. the
· 26 " gel is stained with eth idi um brom ide and visual ized under ultraviolet light .
·27 "
· 28 " What laboratory technique does this describe?
· 29 "
--w
- e-v-a-'u-e-y-
o-u-r~fe-e-d-b-a-C-k-'- 134415 : zheng
· 30 " A. Enzyme-linked immunosorbent assay
· 31 "
·32 "
L ll~ J
" B. Gel electrophoresis

· 33 " C. Northern blot


· 34 "
D. Polymerase cha in reaction
· 35 "
· 36 " E. Sequencing
· 37 " F. Southern blot
·38 "
· 39 "
G. Western blot
· 40 "

The correct answer is B. 59% chose this.


The stem describes gel electrophores is. Gel electrophoresis uses an electric fie ld to
,Pad 9 20 4 5 29« • •

+ G R< UI usmle rx c om/ap l#qmax1 + III :


Item ' 40 of 44
QID 1232 i
L • Mark --<l
Pre Ylous
t:>
Next
_!
la b~ ues
~.
Notes
C';'\I
Cakul~ tor

· 24 " The stem describes gel electrophores is. Gel electrophoresis uses an electric field to separate
molecules based on their sizes. The negatively charged DNA migrates in the electric field toward
· 25 "
the positive end. Smaller fragments move more rapidly through the gel. Bands of DNA can be
· 26 " visua lized by staining the gel with fluorescent dyes such as ethidium bromide.
·27 "
· 28 "
Electrophoresis

A Is not correct. 6% chose this.


· 29 " Enzyme-linked immunosorbent assay (ELISA) is an immunologic technique to determine
· 30 " whether a particular antibody is present in a patient's blood. Labeled antibodies are used to
detect whether the serum contains antibodies against a specific antigen precoated on an ELISA
· 31 "
·32 "
· 33 "
plate.
Enzyme·Linked Immunosorbent Assay

C is not correct. 4% chose this.


· 34 "
Northern blots are similar to Southern blots except that in Northern blotting, RNA is separated
· 35 "
by electrophoresis instead of DNA.
· 36 " Electrophoresis

· 37 "
Is not correct. 7% chose this.
·38 "
· 39 "
D
Polymerase chain reaction is a laboratory technique used to produce many copies of a segment
of DNA. In this procedure, DNA is mixed with two specific primers, deoxynucleotides and a
· 40 " heat-stable polymerase. The solution is heated to denature the DNA and is then cooled to allow
reannealing and synthesis. Twenty cycles of heating and cooling amplify the DNA more than a
million times.
Polymerase Chilin Reaction

Sequencing is a laboratory technique that utilizes dideoxynuc leotides to randomly terminate


· 24 "
growing strands of DNA Gel electrophoresis is used to separate the varying lengths of DNA.
· 25 " The DNA sequence can then be read based on the position of the bands on the gel.
· 26 " Electrophoresis

·27 "
· 28 "
F Is not correct. 16% chose this.
In a Southern blot, DNA is separated with electrophores is, denatured, transferred to a filter, and
· 29 " hybridized with a labeled DNA probe. Regions on the filter that base pair with the labeled DNA
· 30 " probes can be identified when the filter is exposed to film that is sensitive to the radi olabe led
probe.
· 31 "
·32 "
· 33 "
Electrophoresis

G is not correct. 4% chose this.


In a Western blot. a protein sample is separated by electrophoresis and labeled antibodies are
· 34 "
used as a probe. Thi s technique can be used to detect the existence of an antibody to a
· 35 " particular protein.
· 36 " Western Blot ElectrophoresIs

· 37 "
·38 "
· 39 "
Bottom line:
Gel electrophoresis uses an electric field to separate charged molecules based on their size.
· 40 "
DNA is a negatively cha rged molecule that moves from the anode to the cathode .
Electrophoresis
,Pad 9 20 4 5 29« • •

+ G R< UI usmle rx c om/ap l#qmax1 + III :


Item ' 40 of 44
QID 1232 i
L • Mark --<l
Pre Ylous
t:>
Next
_!
la b~ ues
~.
Notes
C';'\I
Cakul~ tor
p p
· 24 " Western Blot ElectrophoresIs

· 25 "
· 26 "
Bottom II ne:
·27 "
· 28 "
Gel electrophoresis uses an electric field to separate charged molecules based on their size.
DNA is a negatively charged molecule that moves from the anode to th e cathode.
· 29 " Electrophoresis

· 30 "

· 31 "
·32 "
· 33 "
Iii lif, , .j I,' for year: 2015
FIII.$ T .0. 10 FACTS

· 34 " FA15, p . 78.3

· 35 " Enzyme-linked Used 10 det ect the prCSl'IlCC of eit her ~ spcrilic Used in many labora tories to det ermine ",hether
immunosorbent assay 31ltigen or a specific an!ibod), in a patient's a particular antibod)' (e.g .. ant i- H [V ) is prcsent
· 36 "
blood samplf". in a patient's bl ood sample. Both the sensi tivity
· 37 " Palient's blood sample is probed with either: and speeificit)' of ELISA approach 100%, but

·38 "
· 39 "
• Direct ELISA: uses 3 tesl antiho(l), 10 sce if
3 sl)Ccific :lllligcn is presen t. The anlibod),
is directly coupled to a color-genera tin g
IHlt h fa[sc-posit ive and falsc-ncgati,'c resu lt s
occur.

enzyme to <leted th e ~"'tigen


· 40 "
• Indirect ELI SA: uses either a lest ;lIltigen
or ,mti bod)' to see if;L speci fic ,mtihody or
311 tigen. respectively. is present. A secon(131)
311 tibodr eoup[ed to a color-generating
enzyme is "ddl'{) to detect the anti body-

· 24 " A 53-year-ol d man is t reated for Helicobacter pylori-related peptic ulcer disease wit h a 2-week
course of triple t herapy includ ing amoxicillin, clarithromycin, and lansoprazole. One week
· 25 "
lat er t he patient experiences fatigue and a temperat ure of 385°C (1 01.3°F). This is followed
· 26 " by the appea rance of diffuse, generalized erythemat ous macules and patches, which are tender to
·27 "
· 28 "
the touc h and accom panied by a burning sensation of t he skin.

· 29 " Based on the history and clinical presentation, what is the most likely diag nosis?
· 30 " 134415: zheng
We value your feedback!
· 31 " A. Cut aneous flushing

·32 "
· 33 "
B. Drug-induced acute urtica r ia

· 34 " C. Erythema nodosum


· 35 "
D. Pyoderma gang renosum
· 36 "
E. St evens-Johnso n syndrome
· 37 "
·38 "
· 39 "
· 40 "
The correct answer is E. 52% chose this.
Stevens-Johnson syndrome lies on the urti caria-erythema multiforme-toxic epiderma l necrolysis
spectrum of acute epidermal disorders. It is most commonly associated with drug exposures,
especially to penici llins, sulfonamides, eth osux imide, and lamotrigine. The question
presents a typical clin ical picture, includi ng the prodrome of fever and fatigue, foll loY,ed
,Pad 9 20 4 5 29« • •

+ G R< UI usmle rx c om/ap l#qmax1 + III :


Item ' 41 of 44 L • Mark --<l t:> _! ~. C';'\I
QID 3649 i Pre Ylous Next la b~ ues Notes Cakul~ tor

Stevens-Johnson syndrome lies on the urtICaria -erythema multlforme·toxlc epiderma l necrolysis


, 24 " spectrum of acute epidermal disorders. It is most commonly associated with drug exposures,
, 25 " especially to penicillins. 5ulfonamides, ethosuximide, and lamotrigine. The question stem
presents a typical clinical picture, includi ng the prodrome of fever and fatigue, followed by skin
, 26 "
and mucocutaneous lesions. Skin lesions begin as erythematous macules, progress to form
, 27 " bullae, and subsequently slough.
' 28 " unkilri<l 13motrigine Prodrome Stevens-John50n Syndrome Erythemil Multiforme To~ic Epjdermill Necrolysis Skin lesion

, 29 " A is not correct. 5% chose this.


, 30 " Cutaneous flushing can be induced by drug exposure, including niacin, adenosine and
, 31 " vancomycin. Fevers, fatigue, and pain on palpation are more characteristic of Stevens-Johnson
syndrome.
, 32 "
Ni,lCin AdeflO5 ine Stevens·johmon Syndrome
, 33 "
, 34 " B is not correct. 24% chose this.
, 35 " An allergic drug reaction could present as urticaria, or hives, which are raised lesions (transient
edematous papules and plaques). Angioedema is a larger edematous area that involves the
, 36 "
dermis and subcutaneous tissues. While many antibiotics, in clud ing amoxicillin, can cause
' 37 " acute urticaria, the clinica l description is more consistent with the much more severe Stevens-
, 38 " Johnson syndrome.
, 39 " Urticaria Ang ioedemil Amoxici llin Acute urticaria Stevens·johmon Syndrome

, 40 " C Is not correct. 12% chose this.


Erythema nodosum is characterized by the appearance of painful erythematous nodules on the
lower legs. Like Stevens-Johnson syndrome, fever and malaise are common symptoms. Drugs,
infections, and inflammatory diseases are some of the diverse etiologic associations with this
condition.

e,y","" ," norjosum i i


lower legs. Like Stevens -Johnson sv"d,'orne, are common symptoms. Drugs,
infections, and inflammatory diseases are some of the diverse etiolog ic associations with this
condition .
Erythema Nodosum Stevens·johnson Syndrome
, 27 "
D Is not correct. 7% chose this.
' 28 "
Pyoderma gangrenosum, seen common ly in association with ulcerative colitis, is not related to
, 29 "
medication use and is characterized by boggy, red ulcers with purulent, necrotic bases. Both
, 30 " pyoderma gangrenosum and Stevens-Johnson syndrome are painful to the touch.
, 31 " Pyodermil Ulcerat~ Col itis Pyodermil Gangrenosum Ste~ns·johnson Syndrome Touc:h sensation

, 32 "
, 33 "
Bottom li ne:
, 34 "
Stevens-Johnson syndrome, a urticaria-erythema multiforme-toxic epidermal necrolysis
, 35 " disorder, is associated with exposure to penici llins, sulfonamides, ethosuximide, and
, 36 " lamotrigine.
Urticaria lamotrigine Stevens·johnson Syndro~ Erythemil Mu ltifor~ Toxic Epidermal Necrolysis
' 37 "

, 38 "
, 39 "
, 40 " Ul;fi'·'I.' for year: 2015
~IRST "10 ~"C T S

FAt5, p . 441.1

Blistering skin disDrders


,Pad"" 2046 290 _ •

+ G R< UI usmle rx com/ap l#Qmax1 + • :


Item" 42 of 44
QID 1918
L

J.
• Mark -<l
Pre YlDu,
t>-
NeKt
all
la b"" ues
~.
No t es
~'t
c .. k ul~ tor

..
' 24 . A 25-year-old man is brought to the hospita l with symptoms of confusion and lethargy.
.25 . Laboratory work-up reveals markedly elevated liver transaminases and decreased
ceruloplasmin. Physical examination reveals the finding shown in the image below.
' 26 •

."
' 28 •

."
. 30 .
· 31 •
· 32 •
. 33 •
. 34 . The drug used to treat the cause of th is patient's underlying disease is an example of which class of
' 35 . agent?
' 36 .
134415 : zheng
' 37 •
We value your feedback!
A. Chemical antagonist
.38 .
B. Competitive antagonist
.39 .
' 40 . C. Irreversible antagonist
. 41 .
D. Partial agonist

E. Pharmacologic antagonist

.
A chemica l antagonist binds directly to the a90nist , thus preventing the agon ist from binding
' 24 .
its target . D-penacillamine is an example of a chemical antagonist ; it binds toxic metals such
.25 . as copper. This patient has Wilson's disease as evidenced by low serum ceruloplasmin and the
' 26 . presence of Kayser-Fleischer rings in his eyes. He is suffering from fulm inant
hepatoencephalopathy, a diagnosis supported by elevated transaminases .
." Kilyser-Relscher ring Eleyated tr<lnsamlJJ.lses Copper
' 28 .
. 29 . B is not correct. 22% chose this •
. 30 . A competitive antagonist is a pharmacologic agent that binds the receptor, thus preventing the
agonist itself from binding to the receptor. This antagonism can be overcome with increasing
· 31 •
concentration of agonist.
· 32 •
C is not correct. 14% chose this •
. 33 .
An irreversible antagonist is a pharmacologic agent that binds the receptor, preventing the
.34 . agonist itself from binding. However, unlike a competitive antagonist, this antagonism cannot
' 35 . be overcome by increasing the concentration of agonist.
' 36 . D is not correct. 6% chose this.
' 37 . When compared to an agonist, a partial agonist is a drug that produces a reduced effect on
binding to its receptor. Depending on its dose it can act as either an agonist or an antagon ist.
· 38 •
. 39 . E Is not correct. 12% chose this .
A pharmacologic antagon ist is a drug that binds to the receptor but does not activate it. In this
' 40 .
case the drug used to treat Wilson's disease binds the metal directly, not to a receptor.

Bottom Line :
A chemical antagonist works by bind ing directly to an agonist, thus preventing the
,Pad 9 20 4 6 29 « • •

+ G R< UI usmle rx com/ap l#qmax1 + III :


Item ' 42 of 44
QID 1918 i
L • Mark --<l
Pre Ylous
t:>
Next
_!
la b~ ues
~.
Notes
C';'\I
Cakul~ tor
. . -- - - - - - - - - - -- - -- --
· 24 "
· 25 " Bottom line:
· 26 " A chemical antagonist works by binding directly to an agonist, thus preventing the agonist
·27 "
· 28 "
from binding its target.

· 29 "
· 30 " Iii lif, , ., I.) for year: 2015
~IRST "'10 FAC TS

· 31 "
·32 "
· 33 " Receptor binding
FA15, p . 246.'

· 34 "
· 35 "
n 1)0

....
-
--------------- 100 --------------------

· 36 "
i
I
· 37 " ,i!! \0 ---------
1
,
·38 "
· 39 "
~ ~
· 40 "

AGONlSTWlTH
o -

o Competitive
0,1

--
l,e 10 100

EFFECT
1000


o -
OLIO

--
-------------------
10 100 1000


!XAMPlE
0 ,) 1.0 to 100 \OCI(l

· 24 " A 62-year-old man with a history of tobacco use presents to his physician complaining that
his skin has been "turning yellow." Physical examination reveals a nontender mass in his
· 25 "
right upper quadrant and both erythema and tenderness of his left leg. CT of the abdomen
· 26 " reveals an underlying malignancy.
·27 "
· 28 " Which of the following is the embryologic origin of the most likely site of this patient's tumor?
· 29 "
13441 5 : zheng
We value your feedback!
· 30 " A. Dorsal mesentery
· 31 "
·32 "
· 33 "
B. Lung bud

C. Pancreatic islet cells


· 34 "
D. Ureteric bud
· 35 "
· 36 " " E. Ventral pancreatic bud
· 37 "
·38 "
· 39 "
· 40 " The correct answer is E. 59% chose this.
This patient is presenting with signs and symptoms of a pancreatic adenocarcinoma. These
tumors arise most commonly in the pancreatic head, and they may not present until an
advanced stage when they have obstructed the common bile duct, resulting in jaundice.
Patients with pancreatic adenocarcinoma may also experience weight loss and ab,do lmilnal
radiating to the back. This patient manifests two other signs sometimes seen i
,Pad 9 2046 29« • •

+ G R< UI usmle rx com/ap l#qmax1 + III :


Item ' 43 of 44
QID 2607 i
L • Mark --<l
Pre Ylous
t:>
Next
_!
la b~ ues
~.
Notes
C';'\I
Cakul~ tor
. . .
This patient is presenting with signs and symptoms of a pancreatic adenocarcinoma . These
· 24 "
tumors arise most commonly in the pancreatic head, and they may not present until an
· 25 "
advanced stage when they have obstructed the common bile duct, resulting in jaundice.
· 26 " Patients with pancreatic adenocarcinoma may also experience weight loss and abdominal pain
·27 "
· 28 "
radiating to the back. This patient manifests two other signs sometimes seen in pancreatic
adenocarcinoma: migratory thrombophlebitis and a palpable gallbladder. Given both the
epid emiology of this cancer and the patient's jaundice, it is likely that his tumor arose in the
· 29 "
head of the pancreas, which is derived from the ventral pancreatic bud. Other derivatives of the
· 30 " ventral bud include the uncinate process and main pancreatic duct.
Adenocarc inoma Abdom inal Pain Thrombophlebitis migrans
· 31 "

·32 "
· 33 "
A Is not correct. 16% chose this.
The dorsal mesentery gives rise to, among other things, the adult spleen. This patient has an
· 34 " adenocarcinoma of the head of the pancreas. The pancreatic head is derived from the ventral
· 35 " pancreatic bud, not the dorsal mesentery.
Meser'ftery Adenocarcinoma Speen
· 36 "

· 37 " B is not correct. 7% chose this.

·38 "
· 39 "
Th is patient has adenocarcinoma of the pancreatic head. The lung bud plays a role in
development of the adult lungs, but it does not contribute to any part of the pancreas.

· 40 "
C is not correct. 10% chose this.
Pancreatic islet celis, like pancreatic acinar celis, are derived from endoderm.
D is not correct. 8% chose this.
The adult kidneys (except for the nephrons) are derived from the ureteric bud. This

· 24 " D is not correct. 8% chose this.


· 25 " The adult kidneys (except for the nephrons) are derived from the ureteric bud. This patient,
· 26 " however, presents with an adenocarcinoma of his pancreas.

·27 "
· 28 "
Adenocamnoma Pancreas

· 29 " Bottom line:


· 30 " Pancreatic cancer most commonly occurs in the ventral pancreatic bud. Symptoms of
· 31 " pancreatic adenocarcinoma include obstructive jaundice, weight loss, and abdominal pain
·32 " radiating to the back.

·33 "
· 34 "
Adenocarcinoma jaundice. ObstrtJ(tIV~ Abdominal Pam

· 35 "
ul;fi""" foryear: 2015
· 36 " FIR S T A I O FACTS

· 37 "
FA'5,p . 377.'
· 38 "
Pancreatic '\"emge survi"al - I ),ear after diagnosis. Very
· 39 " aggressi\'e tumor arising from pancreatic
· 40 " ducl s (disorg3llizcd glandulm slruclnrc with
cellular in lihl"Jt ion Il); alread), metastasized
· 41 " at prt"lentation: tumors mo re common in
p;mcrealic head [] (- ohstrllcti,'e jamulice).
Associatell ",ith CA lq-q hUllOr marker (<l!so
CEI\. bs sJX.'£ ilic).
Ri sl factors:
,Pad 9 20 4 6 28« • •

+ G R< UI usmle rx com/ap l#qmax1 + III :


Item ' 44 of 44
QID 4486 i
L • Mark --<l
Pre Ylous
t:>
Next
_!
la b~ ues
~.
Notes
C';'\I
Cakul~ tor

· 24 " A 35·year-ol d white man presents to the emergency department 15 days after returning from I'" AI
a boating trip . He reports a 4-day history of headache; fatigue; low back pain; mya lgi a, A
· 25 "
especia lly of the ca lves; vomiting; and a fever as high as 40.6°( (105.1OF). On physical
· 26 " examination the man has erythema of t he left flank, with scattered tiny red macules on the legs.
·27 "
· 28 "
Laboratory studies show prot einuria and a creatinine level of 4.0 mg/dL. Results of cerebrospinal
flu id examination are normal. A monos pot test is negati ve .
· 29 "
What is the most likely diagnosis?
· 30 "
134415 : zheng
· 31 " We value your feedback!
·32 "
· 33 "
A. Hepatitis A

B. I nfectious mononucleosis
· 34 "
C. Leptospirosis
· 35 "
· 36 " D. Lyme disease
· 37 "
·38 "
· 39 "
E. Naegleria meningitis

· 40 "

The correct a nswer is C. 62% chose this .


Leptospirosis is common after recreational freshwater expos ures . Salient clinica l features
include high fever with uniform rena l impairment and mild hepatitis.

B is not correct. 5% chose this.


Because the monos pot test was negative, infectious mononucleosis can be ruled out. Also, the
·27 "
· 28 "
cl inical presentation wou ld be different with infectious mononucleosis.
Monospot test Infectious Mononucleosis

· 29 " o Is not correct. 13% chose this.


· 30 " Lyme disease usually begins with a tick bite and presents with a classic circular rash.
Lyme Oise~se V~cc i ne Tick bite (disorderl
· 31 "
·32 "
· 33 "
E is not correct. 13% chose this.
Because the cerebrospina l fluid examination was normal, Naegleria meningitis can be rul ed out.
Mening itis
· 34 "
· 35 "
· 36 " Bottom line:
· 37 " Leptospirosis commonly occurs after freshwater ingestion and ca uses high fever, renal
·38 "
· 39 "
impairment, and mild hepatitis.
leptosp<rosis Renal impaIrment HepatItis

· 40 "

iii Iif., , .j 1.1 for year: 2015


fllI.ST AIO FACTS
,Pad "" 2047 28 Y0 . •

+ G R< UI usmle rx com/ap l#Qmax1 + • :


Item 1 of 44
QID 4027
L

J.
• Mark -<l
PreYlDu,
t>-
NeKt
all
lab"" ues
~.
Notes
~'t
c .. kul~tor

· 1
· 2
•• An 87-year-old man presents to his urologist with complaints of urinary retention. His past
medical history is significant for Parkinson's disease, congestive heart failure, and benign
·3
· 4
•• prostatic hypertrophy. He notes that his urinary retention has significantly worsened since
changes in his medi cat ions a few days ag o . Additionally, he has also noticed difficulty reading his
·5
· 6 •• morning paper, but he attributes his blurred vision to "old age." On physical examination, the
patient has flushed skin with warm extremities. His mucosal membranes appear dry and he is
slig htly disoriented to time and place. The physician immediately refers this patient to the
· 7
·8
•• emergency department due to these findings.

.,0 •
· 9 What medication should be administered immediately in the emergency department?
. We value yourfeedback! 134415 : zheng
· 11 • A. 8enzodiazepines
· 12 •
B. Benztropine
· 13 •
· 14 • C. Dig italis antidote
· 15 • D. Naloxone
·16.
· 17 • " E. Physostigmine
· 18 •
· 19 •
The correct answer is E. 59% chose this.
Th is patient is presenting with symptoms of anti muscarinic toxicity, likely as a result of

i i i

•••
overdosing on his Parkinson's medication. Symptoms of antimuscarinic toxicity include
· 2
increased body temperature, rapid pulse, dry mouth, dry flushed skin, cycloplegia, constipation,
·3 and disorientation. Antimuscarinic agents such as benztropine are second-line drugs for the
· 4 treatment of Parkin so n's disease. They can improve symptoms of tremor but not rigidity or
·5
· 6
•• bradykinesia. Overdoses are treated with physostigm ine, which is a cholinomimetic.
Physostigmine has the added benefit of being able to cross the blood-brain barrier.

· 7
·8
•• Benztropine Bradykinesia Physostigm ine Blood braIn barrier function

A is not correct. 4% chose this.

.,0 •
Benzodiazepines are used for management of agitation or seizures. These medications would be
· 9
.
.".
appropriate if the patient's cholinergic toxicity was so great as to cause seizures involving
severe agitation and mental status change; however, our patient does not demonstrate this
level of toxicity. Additionally, benzodiazepines would not reverse the other adverse effects of
· 12 • atropine and wou ld be inappropriate for use in this sce nario.
Seizures Atropine
· 13 •
· 14 • B is not correct. 15% chose this.
· 15 • 8enztropine is a muscarinic cholinoreceptor blocker which acts at the central nervous system

·16.
· 17 •
level and is used in the treatment of Parkinson's disease. It would exacerbate the patient's
symptoms.
Benztrop,ne
· 18 •
C is not correct. 17% chose this.
· 19 . In cases of digitalis poisoning, the proper t reatment is to stop the med ication, normal ize
potassium levels, and administer lidocaine, anti-dig Fab fragments, and magnesium. Digitalis is
a po sitive inotrope that binds to Na+/K+ -ATPase, thus inhibiting active transport of sodi
I I I
,Pad 9 2047 28« • •

+ G R< UI usmle rx com/ap l#qmax1 + III :


Item ' 1 of 44
QID 4027 i
L • Mark --<l
Pre Ylous
t:>
Next
_!
la b~ ues
~.
Notes
C';'\I
Cakul~ tor


·2
.
.3
•9 P 9 pp
potassium levels, and administer lidocaine, anti-dig Fab fragments, and magnesium. Digitalis is
P •

.4 .
a positive inotrope that binds to Na+/K+ ·ATPase, thus inhibiting active transport of sodium. This
drug is indicated for the treatment of congestive heart failure, atrial fibrillation, and atrial

·5 . flutter. With toxicity, the patient will experience hypokalemia, nausea, vomiting, blurred vision

.
with a yellow tint, and cardiac arrhythmias . Alth ough our patient has blurred vision, his other

·6
.7
.,
. symptoms (urinary retention and flushed skin) do not indicate digitalis toxicity.
Udocaine Digitalis preparallOn Hypokalemia NallSea Vomiting Urinary Retention Potassium Magnesium Sod,um

Atrial Fibr illation Atria l Flutter


Congesti~e heart fa ilure

., D is not correct. 5% chose this .

.".
.,0 . Naloxone is used to reverse the effects of opioid toxicity (respiratory depression, sedation, and
hypotension). The triad of centra l nervous system depression, miosis, and respiratory
depression should alert any physician to possible opioid tox icity from sou rces such as heroin (in
· 12 • the intravenous drug user) or fentanyl pain patches (in a chronic pain patient). Ou r patient's
· 13 • symptoms are not typical of opioid toxicity.
Naloxone Hypotension Hero;n Fentanyl Respiratory Oepression Sedation Chron ic pam
· 14 •

·,5 .
·16.
· 17 •
Bottom line:
Learn to recognize atropine toxicity usi ng its mnemonic: "hot as a hare, dry as a bone, red as a
· 18 • beet, blind as a bat, and mad as a hatter." The antid ote to atropine toxicity is physostigmine.

·19. Atropine Physostigmine

hVIDoter15iionil. i nervous system i ,

•••
· 2 depression should alert any physician to possible opioid tox icity from sources such as heroin (in
the intravenous drug user) or fentanyl pain patches (in a chronic pain patient). Our patient's
·3
symptoms are not typi ca l of opioid tox icity.
· 4

••
Naloxone Hypotension Heroin Fentanyl Respiratory Depress,on Sedation Chron ic pam

·5
· 6

., •
Bottom II ne:
· 7

., •
Learn to recognize atropine tox icity using its mnemonic: "hot as a hare, dry as a bone, red as a

• beet, blind as a bat, and mad as a hatter. " The antidote to atropine toxicity is physostigmine.

".
Atropine Physostigmine
. ,0 •

.· 12 • Ul;fi'·'I.' for year: 2015


~IRST " 10 ~" C T S
· 13 •
· 14 •
FA1S, p. 2512
·,5 .
·16.
Atropine 1\'l l15cari n ic antagollisl, Ust-"iI to Ircal bradrcardia and for ophthalmic applications.
AClION Hom
· 17 • Ey. I pnpil dilation. cycloplegia Bloch DU:-" 1BBc LSS. Skddalmnsclc and
Airway I st-'C rction s e NS excitation mcdialNl by nicotinic
· 18 •

·19. Stomach

Go<
Bladder
I acid secrelion
I motilit )'
rcceptors. See prcI'ions page.
,Pad 9 20 4 8 28« • •

+ G R< UI usmle rx com/ap l#qmax1 + III :


Item' 2 of 44
QID 1674 i
L • Mark --<l
Pre Ylous
t:>
Next
_!
la b~ ues
~.
Notes
C';'\I
Cakul~ tor

· 1
· 2
•• A 6·month·old boy is brought to the physician, because he seems smaller than other kids his
age and has occasional vomiting assoc iated with bottled milk feedings. He was born at A
I'" AI
·3
· 4
•• home and did not undergo newborn screening. On physical examination, the patient is
found to have jaundice, bi lateral clouding of eye lenses, and hepatomegaly. The patient also shows

·5
· 6
•• signs of developmental delay.

· 7
·8
•• Which of the following enzymes is likely to be defective in this cond iti on?

134415 : zheng

.,0 •
A. Q, Hydroxylase
· 9

.". . We value your feedback!


B. Aldolase B

C. Ga lactose-l-phosphate uridyltransferase
· 12 •
· 13 • D. Iduronate sulfatase
· 14 •
E. Transmembrane copper-transporting ATPase
·,5 .
·16.
· 17 •
· 18 • The correct answer is C. 58% chose this.
·19. This boy's presentation suggests classic galactosemia. Classic galactosemia has autosomal
recessive inheritance and is caused by deficiency of galactose-l-phosphate uridyltransferase.
Without this enzyme, phosphorylated galactose cannot be converted to phosphorylated
to be used in glycolysis and gluconeogenesis. As a result, galactose-l-phosphate and

•••
· 2
recess ive i i and caused by of galactose- -phosphate uridyltransferase.
Without this enzyme, phosphorylated galactose cannot be converted to phosphorylated glucose
·3
to be used in glycolysis and gluconeogenesis. As a result, galactose-l-phosphate and ga lactitol
· 4 accumulate in va rious organs in the body, including the central nervous system, liver, and eyes.
·5
· 6
•• Galactose and lactose (milk) should be avoided in patients with this condition . Ga lactokinase
deficiency resulting in galactosemia is also a possibility in this case. However, the presenting

· 7
·8
•• symptoms would be less severe, causing only infanti le cataracts and developmental delays
related to sight (lack of development of a social smile and inability to track objects).
G.II<1ctose CI<lssie<l1 g<l1<lctosemi<l Glucose Oeficiency of g<l1<letokin;lse

. ,0 •
· 9
A is not correct. 5% chose this .

.".
· 12 •
· 13 •
· 14 •
. Q1 Hydroxylase converts calcidiol to calcitriol, and operates in the proximal tubule of the
kidneys. It is essential for creating 1,25(OH)-vitamin D. Its absence may be associated with
hypocalcemia and increased parathyroid hormone formation. Hypocalcemia is classically
associated with tetany and neuromuscu lar irritability.
Calcitriol Kidney Tubules. Proximal Hypocalcem ia Tetany

·,5 . B is not correct. 16% chose this.

·16.
· 17 •
Aldolase B deficiency causes fructose intolerance, resulting in fructose- l -phosphate
accumulation. As a result, available phosphate drops, and both gluconeogenesis and
glycogenolysis are inhibited. The clinical manifestations include hypoglycemia, jaundice,
· 18 • cirrhosis, and vomiting. This is a presentation sim ilar to that of classic galactosemia, but

·19. cataracts are typically not a presentation feature in aldolase B deficiency.


Fructose Vomltlf)g Heredn<lry fructose intoler<lnce syndrome CI<lss icalg<llaetosemi<l Bil<lter<ll c<ltar<lC[5 (dIsorder)
,Pad 9 20 4 8 28« • •

+ G R< UI usmle rx com/ap l#qmax1 + III :


Item' 2 of 44
QID 1674 i
L • Mark --<l
Pre Ylous
t:>
Next la b~
_! ues
~.
Notes
C';'\I
Cakul~ tor

· 1
· 2
•• Iduronate su lfatase is deficient in Hunter's synd rome, an X-linked recessive condition. Hunter's
syndrome can present with hepatosplenomegaly and developmental delay. This patient has
·3
· 4
•• signs, but corneal clouding is not a typical presentation of Hunter's syndrome.
E Is not correct. 4% chose this.
·5
· 6
•• A transmembrane copper-transporting ATPase would be at fault in Wi lson's disease, an
autosomal recessive condition. Wilson's disease is caused by an accumulation of toxic levels of
· 7
·8
•• copper in the eye, liver, and brain. The ophthalmic clinical sign for Wi lson's disease is the
appearance of Kayser-Fleischer rings, which are greenish-yell ow rings around the iris. Wilson's
disease does not genera lly present before 6 years of age.


".
· 9 l<.<Iyser·Fleischer ring Copper Eye Iris (Eye)

. ,0 •
Bottom Line:
.· 12 •
Galactose-'-phosphate uridyltransferase deficiency causes ga lactosemia, manifesting clinically
· 13 • with jaundice, cataracts, hepatomegaly, and mental retardation within months of birth.
· 14 • Galactose Class ical galactosemia Bilateral cataracts (disorder) Mental Retardation

·,5 .
·16.
· 17 • 141 ;fi
fiRS T A I D fAC TS
, .jI.) for year: 2015
· 18 •

·19. Disorders of galactose metabolism


FA15. p . 103.2

Galactokinase I-I e red ila rr deficie ncy of glilactokinlisc. C~ 1aditol acc nmulates if ga lado:;e is pre:;e nt in diet
Relati vely mild cond ition. Autos,onml rcressil'c.

•••
A 13-year-old boy is diagnosed with pharyngitis and is prescribed an antibiotic for treatment.
· 2 Three days later, he returns to the physician compla ining of fever and a diffuse rash that has
·3 been worsening since he began his treatment. The boy is otherwise in excellent health, and
· 4 his medica l history is unremarkable. Laboratory stud ies show:
·5
· 6
•• WBCs: 11 ,OOO/mm 3 , 71% neutrophils, 15% lymphocytes, 9% eosinophils
Hematocrit: 37%
· 7
·8
•• Platelets: 220,OOO/mm 3
Urinanalysis: No albumin, 1+ protein, 2+ WBC casts


".
· 9
. ,0 • Which of the following medications was this patient most likely given?

134415 : zhe ng
We value your feedback!
.· 12 • A. Aztreonam
· 13 • B. Clindamycin
· 14 •
C. Penic illin V
·,5 .
·16.
· 17 •
O. Tetracycline

E. Vancomycin
· 18 •

·19.
,Pad 9 20 4 8 28« • •

+ G R< UI usmle rx com/ap l#qmax1 + III :


Item ' 3 of 44
QID 2499 i
L • Mark --<l
Pre Ylous
t:>
Next la b~
_! ues
~.
Notes
C';'\I
Cakul~ tor

· 1
· 2
•• Although resistance is a common problem, penicillin can still be used to treat some infections,
such as streptococca l pharyngitis. Penicillin is most often used to treat infection with
·3
· 4
•• Treponema, gram -positive cocci such as streptococci, and gram -negative aerobes such as
gonococci. Allergy to penicillin can manifest wit h va rious symptoms from rash to frank

·5
· 6
•• anaphylaxis. Interstitial nephritis, eosinophilia, and hemolytic anemia can also be seen. Notably,
amoxicillin may be used in place of penicillin in child ren, as this medication is more eas ily
tolerated orally in this popu lation.
· 7
·8
•• anaphylaxIs Amoxlcilhn Streptococcal sore throat Disorder characterized by eosinophilia

A is not correct. 6% chose this .

. ,0 •
· 9 Aztreonam has no effect on gram-positive organisms, such as the streptococcal pharyngitis t he

.".
· 12 •
· 13 •
· 14 •
. patient likely has, and usually has few tox ic effects .
Aztreonam Streptococca l sore throat

B is not correct. 10% chose th is.


Clindamycin is used to treat anaerobic infections, not the patient's likely streptococcal infection.
It is commonly associated with Clostridium difficife infection.
Clindamycin Clostridium diff.cile infection
·,5 .
·16.
· 17 •
o Is not correct. 8% chose this.
Tetracycline may lead to gastrointestinal upset, hepatic damage, photosensit ivity, and renal
tubular dysfunction. Although it is effective against a wide array of both gram-positive and
· 18 •

·19. gram-negative organ isms, it is not indicated for the treatment of streptococcal infections.
Tetracyclir.e liver damage Streptococcal Infections

E is not correct. 11% chose this.


Vancomycin can cause a diffuse rash (red man syndrome) with in minutes of intravenous

•••
Vancomycin can cause a diffuse rash (red man syndrome) with in minutes of intravenous
· 2
infusion. Vancomycin, although effective against gram-positive bacteria, would not be a fi rst-
·3 line treatment for strep throat.
· 4 Intraven0u5 infu5ioo procedures Streptococcal sore throat

·5
· 6
••
· 7
·8
•• Bottom Line:
Allergy to penicillin can present with symptoms ranging from rash to frank anaphylaxis,
among others.


".
· 9 anaphylaxis

. ,0 •

.· 12 • I ill ;fi , .j 1.1 for year: 2015


FIII.ST AID FACTS
· 13 •
· 14 • FAtS. p. 180.2

·,5 . PenicillinG, V Penicillin C (IV ,md 11\,1 form). penicillin V (oT~,I ), Prol otype ~.I ;,ctam antibiotics

·16.
· 17 •
MWIAlIISM Bind penicillin·binding prolcills (IT;II1 speptidascs),
BlocllrJ nspeptid"sc cross-l in lin g of peptidoglycJlI in cell wall
I\ctiv<lle autolytic cn~yllle~.
· 18 • (lIN I(ALUS< 1\'105tl)' uscd for g,," n-positi\"c o rg,,,,isI1l5 (5. pIHWl1louiill'. 5. pyogeues. !\ctirlOl1l)"("t'l;).I\l so used for

·19. gram-negali\'\: cocci (",ain)y N. meuiugilidis) and spirochetes (n;1lneiy T. /J<II/idum ). Bacter ic id;tI
for granl-posili"e cocci. gram-positive rods. gram-negati,'c cocci. and spirochetes. Pcnicillinase
scnsiti,·c.
,Pad 9 20 4 8 28« • •

+ G R< UI usmle rx com/ap l#qmax1 + III :


Item' 4 of 44
QID 4110 i
L • Mark --<l
PreYlous
t:>
Next
_!
la b~ ues
~.
Notes
C';'\I
Cakul~tor

· 1
· 2
•• A 38-year-old man with a history of HIV/AIDS visits an ophthalmologist complaining of blurry
vision in his right eye. He also reports the sensation of "floaters" in his visua l field and
IAA[
·3
· 4
•• occasional flashes of briqht liqhts. Fundou5copic exam ination revea ls fluffy yellow-white
lesion s of the right retina w ith small areas of hemorrhage . The patient admits being poorly
·5
· 6
•• compliant with his HIV antiretroviral therapy, and a laboratory test reveals a (D4+ count of
35/mm 3 , Following in iti ation of therapy with a new drug to treat this AIDS-related infection, the
ophthalmologist observes significant improvement on fundoscopic examination. However, the
· 7
·8
•• doctor has to halt therapy 2 weeks later after follow-up laboratory tests revea l an absolute
neutrophil count of 400/IJL and a creatinine leve l of 1.9 mg/dL .


".
· 9
. ,0 • What is the mechanism of action of the newly prescribed drug?

13441S : zheng
.· 12 • A. Binding to the gp41 subun it of the viral envelope
· 13 • B. I nhibition of cleavage of viral polyproteins
· 14 •
C. I nhibition of the cross-linki ng of peptidog lycan
·,5 .
·16.
· 17 •
" O. I nhibition of the incorporation ofdGTP by viral DNA polymerase

E. I nterference with the release of viral progeny


· 18 •

·19. We value your feedback!

•••
· 2 Th is patient has cytomegalovirus (CMV) retinitis, an infection most common ly seen in patient s
with advanced AIDS and a CD4 count <50/mml. The diagnosis is made clinically, based on the
·3
finding of retinitis on funduscopy in the clinical context of severe immunocompromise. CMV is a
· 4 DNA virus of the herpesvirus family and is effectively treated with ganciclovir. This drug is
·5
· 6
•• activated by phosphorylation by viral protein kinases and acts as an analog to dGTP,
competitively inhibiting the incorporation of dGTP into replicating viral DNA. It is important to

· 7
·8
•• monitor patients on ganciclovir for the development of myelosuppression and nephrotoxicity,
two sig nificant adverse effects of the drug.
Ganciclovir PhosphorylaUon

. ,0 •
· 9
A is not correct. 12% chose this .

.".
· 12 •
· 13 •
· 14 •
. Th is describes the mechanism of action of the peptide-based HIV fusion in hibitor enfuvirtide,
which prevents entry of the virus after bi nding to host cells . Although the patient's long-term
survival depends on effective HIV therapy, enfuvirtide wou ld not directly treat the patient's
cytomegalovirus retinitis. Common toxicities of enfuvirtide include depression, insomnia,
peripheral neuropathies, and pancreatitis.
enfuvirtide CytomegalovirU5lletinitis
·,5 .
·16.
· 17 •
B is not correct. 15% chose this.
Th is describes the mechanism of the anti-HIV protease inhibitors, such as ritonavir. Although
encouraging better compliance by the patient with his anti ·HIV drugs is undoubtedly of high
· 18 • importance, protease inhibitors would not directly treat cytomegalovirus retinitis. The most

·19. common adverse effects of protease inhibitors are gastrointestinal disturbances and
pa resthesias.
IUtonavir Cytomega lovirus ReUnlUs
,..·23.
,Pad 9 20 4 8 28« • •

+ G R< UI usmle rx com/ap l#qmax1 + III :


Item' 4 of 44
i
L • Mark --<l t:> _!
la b~
~. C';'\I
Cakul~tor
.
QID 4110
. . PreYlous
. Next ues Notes

This describes the action of 13-lactam antibiotics, such as penicillin. These are active against

.4 .
bacteria but are not antivirals and would not treat this patient's cytomegalovirus retinitis .
Cytomegalovirus RetIn itis

·5
.7..
·6 . E Is not correct. 9% chose this.
Oseltamivir is a treatment for influenza that functions by inhibiting viral neuraminidase and
preventing the release of vira l progeny from infected celis. It wou ld not be of use in treating
cytomegalovirus retinitis. Gastrointestinal disturbances are the most common adverse effects
., of this drug .
., Oseltamivir Influenza Cytomegalovirus RetlnillS

.".
. ,0 .
· 12 •
· 13 •
· 14 •
Bottom li ne:
Ganciclovir is activated by a viral kinase and inhibits viral DNA polymerase by acting as a dGTP
analogue. Adverse effects include myelosuppression and renal toxicity.
GanciclOVlr

·,5 .
·16.
· 17 •
iii I if., ,
FIII. S T A I O FA CTS
.j 1.1 for year: 2015
· 18 •

·19. Ganciclovir
/II((HAHISloi
FA1S, p. 193.3

5'.monophosphalc forme<! by a C",IV viral kinase. Gu anosine analog. Triphosphate formed by


cellular kinases. t . it 'i DNA pol)'merase. Preferentially inhibill'irJI DNA
I

•••
A 54-year-old alcoholic is brought to the emergency department by fire rescue after being
· 2 found lying face down in the street. He is incoherent and is unable to walk in a straight line.
·3 His pulse is 110/min, hi s blood pressure is 135/80 mm Hg, and his respiration rate is 20/min.
· 4 Physical examination revea ls a diaphoretic man with generalized wea kness passing in and out of

·5
· 6
•• consciousness. A glucose fingerstick test shows a glucose level of 45 g/dL.

., •
This patient's hypoglycemia most likely resulted from an elevated ratio of which of the following?
· 7

., •
134415: zheng
We value your feedback!

.,0 •
A. NAO+:NAOH

.".
· 12 •
· 13 •
· 14 •
. B. NAOH:NAO+

C. NADP:NADPH

D. NADPH:NADP

E. Pyruvate:lactate
·,5 .
·16.
· 17 •
· 18 • The correct answer is B. 60% chose this.

·19. This patient is suffering from hypoglycemia related to his alcohol use . Ethanol is metabolized to
acetaldehyde, which is then metabolized to acetate (acetaldehyde dehydrogenase).
steps of ethanol metabol ism, NAOH is generated from NAO+. With an elevated NAoUC1:N All
ratio in the liver, pyruvate is diverted to lactate, whi le oxaloacetate is diverted
,Pad 9 20 4 8 28« • •

..3·2,..
+
Item ' 5 of 44
QI D 1171
G
i
R<
L •
UI usmle rx com/ap l#qmax1

P
Mark --<l
Pre Ylous
- 9
t:>
Next

YP 9Y
_!
la b~ ues
~.

.
Notes
C';'\I
Cakul~ tor

acetaldehyde, which is then metabolized to acetate (acetaldehyde dehydrogenase), Du ring both


+ III :
. .
.4 .
steps of ethanol metabol ism, NADH is generated from NAD+, With an elevated NADH:NAO+
ratio in the liver, pyruvate is diverted to lactate, while oxaloacetate is diverted to malate, thus

·5 . inh ibiting gluconeogenesis. The metabo li sm of ethanol is also responsible for the hepatic fatty

.
changes see n in chronic alcoholi cs (increased fatty acid synthesis).

..,7.
·6 Metabol,sm NADH Fany degeneration Ethanol

A is not correct. 18% chose this •


An elevated NAO+: NADH ratio does not result from alcohol metabolism. In any event, lactate
., would generate pyruvate, and gluconeogenes is would not be inhibited .

".
NAOH Metabolism
.,0 .
C is not correct. 9% chose this .
An elevated NADP:NADPH ratio results from a fa ulty pentose shunt. NADPH is used for fatty
.· 12 • acid synthesis and to reduce glutathione. The above presentation and decreased
· 13 • gluconeogenesis are consistent with an elevated NADH:NAD+ ratio.
· 14 • Glutathione NADH

·,5 . D is not correct. 8% chose this.


·16.
· 17 •
NADPH is produced by the pentose pathway. NADPH is used for fatty acid synthesis and to
reduce glutathione. Glutathione helps prevent oxidative damage to cells by reducing hydrogen
peroxide. An elevated NADPH:NADP ratio would not inhibit gluconeogenesis.
· 18 •

·19. Glutathione Hydrogen

E Is not correct. 5% chose this.


An elevated lactate:pyruvate ratio would be seen in alcohol intoxication rather than an
Py,·u'.t,,, I , 'ct,"e ratio . alcohol metabolism, the elevated NADH:NAD+ ratio

•••
· 2 An elevated lactate:pyruvate ratio would be seen in alcohol intoxication rather than an elevated
pyruvate: lactate ratio. During alcohol metabolism, the elevated NADH: NAD+ ratio leads to
·3
diversion of pyruvate to lactate.
· 4

••
Metabol,sm NADH Alcoholic Intoxicauon
·5
· 6

., •
Bottom II ne:
· 7

., •
NAD+ is converted to NADH du ring several reactions used to metabolize ethanol. An increased

. ,0 •
NADH:NAD+ ratio in the liver leads to inhibition of gluconeogenesis and subsequent
hypoglycemia .

.".
· 12 •
· 13 •
· 14 •
. NAOH Ethanol

I il1ifi'·'I.) for year: 2015


FIRS T A I D FAC TS

·,5 .
·16.
FA15. p.95.1
Ethanol metabolism

· 17 •
· 18 •

·19.
,Pad 9 20 4 8 28« • •

+ G R< UI usmle rx com/ap l#qmax1 + III :


Item ' 6 of 44
QID 4487 i
L • Mark --<l
Pre Ylous
t:>
Next
_!
la b~ ues
~.
Notes
C';'\I
Cakul~ tor

· 1
· 2
•• The image depicts th e cell wall of a gram·negative bacterium.

·3
· 4
•• What structure is responsible for the pathogenicity of the orga nism?

·5
•• We value your feedback!
134415 ; zheng

"
A
· 6
· 7
·8
•• B

c
Il~

".
· 9
~ J
. ,0 • 0

E
.· 12 •
· 13 •
· 14 •

·,5 . The correct answer is A. 65% chose this.


·16.
· 17 •
The pathoqenicity of a gram-negative bacterium is largely due to structures in the outer
membrane . This membrane consists of a lipopolysaccharide capsule and various proteins such
as endotoxins. Endotoxins can activate th e comp lement cascade, the coagulation cascade, and
· 18 •
macrophages . The outer membrane proteins have enormous genetic and antigenic variability.
· 19 .
B is not correct. 10% chose this.
The peri plasmic space is immediately beneath the outer membrane and is adjacent to
peptidoglycan. which forms the major cell wall constituent.

••
The image depicts th e cell wall of a gram-negative bacterium.
· 2
D
·3
· 4

·5 •• A

· 6
· 7
••
·8
· 9 •• E_ _ _ _ _ }.

".
. ,0 •

.· 12 •
· 13 •
· 14 •

·,5 . What stru cture is responsible for the pathogenic ity of the orga ni sm?

·16.
· 17 • A
We value your feedback!
134415 : zheng

· 18 • B
· 19 •
"""
· 2
·3
· 4

"""
·S
The correct answer is A. 65% chose this.
· 6
The pat hoqenicity of a gram-negative bacterium is largely due to structures in the outer
· 7 membrane . This membrane cons ists of a lipopo lysaccharide capsule and various proteins suc h
·8
· 9 "" as endot oxins. Endotoxins can activate the complement cascade, the coagulation cascade, and
macrophages . The outer membrane proteins have enormous genetic and antigenic variabil ity.
Antigenic Vanatton
· 10 "
B is not correct. 10% chose this.
· 11 "

··12·14"
13""
The peri plasmic space is immediately beneath the outer membrane and is adjacent to the
peptidog lycan, which forms the major cell wa ll constituent.
C is not correct. 6% chose this.
The inner cytoplasm ic membrane contains penici llin-bind ing proteins.
· 1S "

··16"
1718 ""
D is not correct. 9% chose this.
Porins, or hydrophilic channels, trigger host cell endocytosis of the organism, an important step
in early inflammation.
··19" Inflammation

E Is not correct. 10% chose this.


Choice E represents the peptidoglycan layer. Inhibition of peptidoglycan synt hesis is the
mechan ism of action for th e pen icillins and cepha losporins.

Choice E represents the peptidoglycan layer. Inhibition of peptidoglycan synt hesis is the

"""
· 2
·3 mechan ism of action for th e pen icillins and cepha losporins.
· 4
Bottom Line:

"""
·S
· 6 Endotoxins and lipopolysaccharide present in the cell wa ll of gram-negative bacteria are
· 7 largely responsible for their pathogenicity.
·8
· 9 "" I ill jfj , .j 1.1 for year: 2015
· 10 " fIRST AID fACTS

· 11 "

··12·14"
13""
FAI5, p . 118.2

Ce ll walls
Common to both Uniquo to
g,. m-rwg. t.. o

· 1S "

··16"
1718 ""
··19"
,Pad 9 2049 28« • •

+ G R< UI usmle rx com/ap l#qmax1 + III :


Item' 7 of 44
QID 3901 i
L • Mark --<l
PreYlous
t:>
Next
_!
la b~ ues
~.
Notes
C';'\I
Cakul~tor

· 1
· 2
•• Prior to vaccine development, a particular pathogen was highly prevalent in the United
States. Culturing it from a site of infection requ ires swiping with a calcium alginate swab,
IAA[
·3
· 4
•• followed by plating onto potato aga r or Regan-lowe medium.

·5
· 6
•• What is the organism described?

•••
134415 ; zheng
We value your feedback!
· 7 A. Bacillus anthracis
·8
8. Bordetella pertussis

".
· 9
. ,0 • C. Chlamydia trachornatls

D. Clostridium tetan;
.· 12 •
· 13 •
E. Corynebacterium diphtheriae
· 14 • F. Legionella pneumophila
·,5 .
·16.
· 17 •
G. Neisseria gonorrhoeae

· 18 •

·19. The correct answer is B. 52% chose this.


The causative agent of whooping cough, the gram-negative rod Bordetelfa pertussis. may be
cultured from an infected pharynx. A ca lcium alginate swab should be used because the

The causative agent of whooping cough, the gram-negative rod Bordetelfa pertussis. may be

•••
· 2 cultured from an infected pharynx. A ca lcium alginate swab should be used because the
·3 bacteria do not transfer well on cotton. From the swab, the organisms are grown on either
pota to (Bordet-Gengo u) agar or Regan-Lowe medium (which is more commonly used today).
· 4

••
Since vaccine development, the incidence of whooping cough in the United States has dropped
·5 from several hundred-thousand cases a year to several thousand. Severa l recent reports,
· 6 however, suggest that it is still may be a frequent cause of bronchitis and chronic cough in
· 7
·8
• • adults.
Whooping cough due to unspecified organism conon allergenic extract Agar Bronchitis Entire pharynx Calcium Chronic cough

.,0 •
A is not correct. 5% chose this.

".
· 9
. Bacilfus anthracis does not require special culture conditions. A vaccine has been developed
against this gram-positive rod, but it is not given as standard practice .

.· 12 • C is not correct. 4% chose this.


Chlamvdia trachomatis is an Obligate intracellular species and requires co-culture with
· 13 •
eukaryotic cell s . Again, there is no effective vaccine.
· 14 •
D is not correct. 10% chose this.
·,5 .
·16.
· 17 •
The causative agent of tetanus, Clostridium tetoni (as well as all clostridial species), requires
anaerobic cu lture conditions for laboratory growth.
Tetanus

· 18 • E is not correct. 22% chose this.


·19. Tellurite agar or Loffler's medium is used for culturing Corynebacterium diphtheriae. Immunity
against this gram-positive rod is conferred by the combination OPT (d iphtheria-pertussis-
tetanus) vaccine.
,Pad 9 2049 28« • •

..3·2,..
+
Item' 7 of 44
QID 3901
G
i
R<
L •
UI usmle rx com/ap l#qmax1

Mark --<l
PreYlous

F is not correct. 4% chose thi s.


t:>
Next
_!
la b~ ues
~.
Notes
C';'\I
Cakul~tor
+ III :

.·5
4.. Legionel/a pneumophila is grown on BYCE agar (buffered·charcoal -yeast-extract) supplemented
with cysteine and iron. No vaccine has been developed against this gram-negative rod .

.
Ag<lr Cysteine

·6
·7
.,
.
.
G is not correct. 3% chose this.
Neisseria gonorrhoeae is grown on Thayer.Martin media, also known as VCN media. It is
chocolate agar supplemented with vancomycin, col istin, and nystatin. Although candidate
vaccines against gonococca l disease have been studied, t here is cu rrently no effective means of
., vaccination .

".
. ,0 •

.· 12 •
· 13 •
· 14 •
chocolate allergenic extract Agar Colistin Nystatin Vaccination

Bottom line:
Whooping cough is caused by Bordetella pertussis, which is collected using a calcium alginate
swa b rather than cotton and is cult ured on potato agar or Regan-Lowe medium.
·,5 . cotton <l llergenic extract Ag<lr WhOOpIng cough due 10 unspecified orgilnism

·16.
· 17 •
ul;fii,' .. , for year: 2015
· 18 •

·19.
Fll1.ST ... ID F"'Cts

FA15, p.121.1

•••
A 31 -year-ol d white woman is trying to get pregnant. She has a niece who suffers from a
· 2 genetic disease characterized by recurrent respiratory infections and pancreatic fail ure. She
·3 would like to assess her chances of having a child with this disease.
· 4

·5
· 6
•• Wh ich of the followinq laboratorv techniques cou ld be used to determine if this woman and/or her
husband is a carrier of the mutant gene?

., •
· 7 134415 : zheng

., •
A. Enzyme-li nked immunosorbent assay

.,0 •
We va lue your feedback!
B. Gel electrophoresis

.".
· 12 •
· 13 •
· 14 •
.
C. Northern blot

D. Polymerase chain reaction and sequencing

E. Western blot

·,5 .
·16.
· 17 •
The correct answer is D. 59% chose this.
· 18 •

·19. Polymerase chain reaction (peR) and seQ uencinq can be used to determine if this woman and/or
her husband is a carrier of the cystic fibrosis gene , the most common single-gene mutation in
white people. This mutation commonly presents with dysfunction of t he lunqs, pancreas, and
other orqans due to bu il dup of thick mucus. peR is used to am plify the region of interest ,
seq uencing is used to see if the cystic fibrosis mutation is present.
,Pad 9 2049 28« • •

+ G R< UI usmle rx com/ap l#qmax1 + III :


Item' 8 of 44
QID 1221 i
L • Mark --<l
PreYlous
t:>
Next
_!
la b~ ues
~.
Notes
C';'\I
Cakul~tor

· 1
· 2
•• Enzyme-linked immunosorbent assay is an immunologic tec hniq ue used in laboratories to
determine whether a particu lar antibody is present in a patient's blood. This test cannot be used

·3
· 4
•• to determine whether the woman and her husband are carriers for the cystic fibrosis gene.
Enzyme-unked lmmunosorbentAssay

·5
· 6
•• B is not correct. 13% chose this.
Gel electrophoresis uses an electric field to separate molecules based on their sizes. Gel
electrophoresis cannot be used alone to determine whether the woman and her husband are
· 7
·8
•• carriers for the cystic fibrosis gene.
Electrophores is

. ,0 •
".
· 9 C is not correct. 10% chose this .
. Northern blots test RNA levels. It cannot be used to determine whether the woman and her
husband are carriers for the cystic fibrosis gene .
.· 12 • E is not correct. 12% chose this.
· 13 • A Western blot is a test for t he presence or absence of a protein . Th is test cannot be used to
· 14 •
determine whether the woman and her husband are carriers for the cystic fibrosis gene.
Western Blot
·,5 .
·16.
· 17 • Bottom line:
· 18 • peR is a method in which a small quantity of DNA is amplified using primers that are

·19. complimentary to the segment of interest. Nucleotides and a heat-stable enzyme, called Taq
polymerase, are also added to the peR reaction. Each heating and cool in g cycle doubles the
number of copies of the DNA of interest.

A Western blot is a test for th e presence or absence of a protein . Th is test cannot be used to

•••
· 2 determine whether the woman and her husband are carriers for the cystic fibrosis gene.
Western Blot
·3
· 4

·5
· 6
•• Bottom li ne:
peR is a method in which a small quantity of DNA is amplified using primers that are
· 7
·8
•• complimentary to the segment of interest. Nucleotides and a heat-stable enzyme, called Taq
polymerase, are also added to the peR reaction. Each heating and cooling cycle doubles the

• number of copies of the DNA of interest.

".
· 9
. ,0 •

.· 12 • Ul;fi'-'I.' for year: 2015


~IRST .0. 10 ~ A CTS

· 13 •
FAt5, p . 77.4
· 14 •
Polymerase chain "'l olecul ~r hi ology lahor;Jtory procedure llsed to a mplify a desi red fragment of DNA. Usl"fu l as a
·,5. reaction diagnost ic tool (e.g .. neonatlll H IV. herpes enceplmlit is).

·16.
· 17 •
Steps;
l. Dena tu ra tion - DNA is dena tu red hy hea tin g to ge nerale 2 separal e shands
1. An ne~ling-dur i ng coo ling, exces~ prcnmdc DNA pri mers anneal to a specific sequcnce 011
· 18 • eac h strand to he 3111plifiell.

·19. Elong,llion-hc;ll-st" bl e DNA po lymer<lse replie;ltes th e DNA se(lnence followin g eac h


pru ner.
These steps <Ire repea ted Illull iple ti mes for 0 011\ sequence anlpli fieation.
t\ g:lTClSe gel ck-e1roph orcs is- uscd for size separatioll of pe R prod ucts (smaller molecu les travel
fu rth cr); compared agai nst DNA ladder.
,Pad '9 2049 28 Y0 . •

+ G ~ UI usmle rx com/ap lltqmaxl + Iil :


Item . 9 of 44
QIO 4122
~
.l.
• Mark <:]
P,e'llaus
C>-
Nexl
iiI
l~ b'" Ue$
r.pJ '
Notu
Jr.':I
Cal( u l~tor

••• [Ai
' 1
A 23-year-old man presents to the emergency department complaining of an unsteady heart
·2 beat. A prolonged QT interval is noted. The patient loses consciousness 30 minutes later,
· 3 and a repeat ECG is done, which is shown in the image.
· 4
•5 •• :111
tilt
·6
· 7
•• ifill
rtW1M tm
· 8
· 9
•• III
~.
III II
"
'1
:

"0.
· 11 •
Drugs used to treat which of the following protozoal infections are associated with this cardiac
· 12 •
problem?
· 13 •
· 14 • 134415; zheng
We value your feedback!
A. Giardia fambfia
.16.
· 15 •

· 17 •
B. Plasmodium {alciparum

C. Pneumocystis jirovecii
· 18 •
D. Trichomonas vag/nails

E. Trypanosoma cruz;

The arrhythmia shown is torsades de pointes, a polymorphic ventricular tachycardia that derives
·2
••
· 3
its name from its resemblance to a twisting ribbon. It often results from a prolonged QT
interval, which can be the result of medications or a congenital defect. Plasmodium [alciparum is
· 4
·5
•• one of the four causative agents of malaria. Although resi stance patterns vary by location, it is
the only one that is consistently susceptible to quinine (first-line therapy for chloroquine-
resistant P. lalciparum.) Quinine is an antimalarial drug that is known to cause torsades de

•••
·6 pointes by prolonging the QT interval. It is an isomer of quinidine, an antiarrhythmic drug.
· 7 Lastly, multiple antimalarial agents can prolong the QT interval. For example, chloroquine,
which is used to treat Plasmodium vivax, can also cause torsades de pointes.
· 8


Torsades de Pointes Malaria Qumine Chloroqu ine Quinidine Ventr,cular tachycardia, polymorphic Prolonged QT interval
· 9

"0.
· 11 •
· 12 •
A is not correct. 6% chose this.
Giardia lamblia is a parasite that is commonly found in areas of poor sanitation or unsafe water.
Infection of the intestinal tract causes profuse, watery, greasy diarrhea that is foul smelling.
Such infections are treated with metronidazole. This drug inhibits acetaldehyde dehydrogenase
· 13 • (as does the drug disulfiram used to treat alcohol addiction) and thus should never be taken
· 14 •
with alcohol.
Water Diarrhea Metronidazole Disulfiram Entire intestina l tract

.16.
· 15 •

· 17 •
C is not correct. 12% chose this .
Pneumocystls jlrovecll is a fungus that causes pneumonia in the immunosuppressed. It is usually
treated with trimethoprim-sulfamethoxazole, whic h is not associated with cardiac arrhythmias.

D is not correct. 6% chose this.


Like Giardia fambfia, Trichomonas vaginaNs infections are treated with metronidazole.
Im,oc,r"'"tlv. both the patient and his/her sex ual partner must also be treated to era,die:at,
,Pad 9 2049 2810 • •

+ G ~ UI usmle rx com/ap lltqmaxl + Iil :


Item 9 of 44
QID 4122 i
k • Mark <l
prevlo".
t::>
NHt
-I
~ab'" IlU
r;:"
Notu
~
(al(u la tor

••
· 1

·2
Like Giardia fambfia, Trichomonas vaginaUs infections are treated with metronidazole.
Importantly, both the patient and his/her sexua l partner must also be treated to eradicate the
· 3
· 4
•• flagellated protozoan. As mentioned above, metronidazole works by inhibitin g acetaldehyde
dehydrogenase (as does the drug disulfiram used to treat alcohol addiction) and thus should
never be taken with alcohol.
·5
·6
•• TrkhomOn<lS Infections Metronid<lzole Oisulfir<lm

E is not correct. 21% chose this.


· 7
· 8
•• Trypanosoma cruzl causes Chagas disease. Although the cardiomyopathy associated with Chagas
disease usually leads to arrhythmias, nifurtimox (the drug used to treat Chagas disease) does
· 9

"0. • not cause cardiac arrhythmias. A reservoir for this protozoan is the armadillo.
Nifurtimo~ Cholg<ls Oise<lse

"'.
· 12 •
· 13 •
· 14 •
Bottom Line:
Quinines are used to treat malaria and are associated with torsades de pointes, a life-
threatening ventricu lar arrhythmia.

.16.
M<lI<lria TOfS<ldeS de Pointes Vemri(ul<lr arrhythmi<l
· 15 •

· 17 •
Iii itt' ' oj i .1 for year: 2015
fiRST ... ID f ... C TS

FA1S.p.1S\.1
i infections

.".
,'0 .
· 12 •

' 13 •
' 14 .
A new growth factor, XGEF, has been found to be upregulated in breast carcinomas. This
growth factor appears to activate a seven-transmembrane-domain cell surface receptor on
endothelial cells. The binding of the receptor results in the increase of calcium in the cell
cytosol.

By what mechanism does XGEF exert its effect on endothelial cells?


.,5 .
'16.
134415: zheng
We value your feedback!
A. Activation of Gi
· 17 •
B. Activation of Gq
' 18 •
.,9 . C. Activation of Gs
.20 .
O. Upregulation of cGMP
· 21 •
' 22 •

· 23 •
· 24 •
The correct answer is B. 64% chose this.
· 25 •
Seven-transmembrane-domain receptors on cell surfaces are G-coupled receptors. Many drugs
bind to G-protein-coupled receptors. Activation of Gq leads to activation of phospholipase C
(PLC), which breaks down phosphatidylinositoJ bisphosphate (PIP2) into inositol 1,4,5·
triphosphate (IP3) and diacylglycerol (DAG). IP3 activates IP3 receptors in the endoplasmic
reticulum, leading to release of ca lcium from intracellular stores, whereas DAG activates protein
kinase C.
,Pad 9 2049 2810 • •

+ G ~ UI usmle rx com/ap lltqmaxl + Iil :


Item 100f 44
QID 1320 i
k • Mark <l
pre vlo".
t::>
NHt
-I
~ab'" IlU
r;:"
Notu
~
( al( ula to r

A IS not correct. 6% chose this.

.".
,'0 .
· 12 •

' 13 •
Activation of Gi leads to downregulation of adenylyl cyclase and subsequent decrease in cAMP
and downregulation of protein kinase A. Activation of Gj is not linked to an increase in
intracellular ca lcium.
C.. kium

"4
.,5..
C is not correct. 23% chose this .

"6
,,..
· 17 •

.,9.
. 20 .
Activation of Gs leads to upregulation of adenylyl cyclase and an increase in cAMP, which then
leads to activation of protein kinase A. Activation of Gs is not related to an increase in
intracellular ca lcium in endothelia l cells. However, activation of Gs does increase intracellular
calcium in cardiac muscle cells.
C"tCium Endothet i<lt Cell5 Myocytes. C",di<lC

o Is not correct. 7% chose this .


Upregulation of cGMP does not lead to an increase in intracellular calcium.
· 21 • C"lcium

' 22 •

· 23 •
Bottom II ne:
.24 .
Activation of Gq leads to activation of phospholipase C, which breaks down
· 25 •
phosphatidylinositol bisphosphate into IP3 (which promotes the release of calcium from
intracellular stores) and DAG (which activates protein kinase C).
C<l lclum

".
,'0.
.· 12 •
' 13 •
o is not correct. 7% chose this .
Upregulation of cGMP does not lead to an increase in intracellular calcium.

"4
.,5..

.
Bottom line:

"6 .
· 17 •
,,
Activation of Gq leads to activation of phosphOlipa se C, which breaks down
phosphatidylinositol bisphosphate into 1P3 (which promotes the release of calcium from
intracellular stores) and DAG (which activates protein kinase C).
C<l lcium

.,9•
. 20 .
· 21 • ul;fi.·H.'
FIlST " to F" CTS
for year: 2015
' 22 •

· 23 • FA1 S . p, 248 ,1

.24 . G-protei n- linked 2nd m esse nge rs


RfCEPTOII G-PROTlINCLAI\ MAJOR f UN(TlON'i
· 25 •
Sympathetic

I ,-ascn !ar smooth muscle contract ion. , pupi lla ry dib tor 11Iuscle
" eon tr<lction (11I),d,i ;l$is). , intestin;tI and bbdder sphin cte , muscle
eon tr;lction
! sym pat heti c olltflow, I insu li n release. I li polysis, , platelet aggregati oll.
I aqu('(l US hllmor prod uction
I heMt ra te. ' contract il ity. I relli n release. 'hpol)'si!
,Pad 9 2049 2810 • •

+ G ~ UI usmle rx com/ap lltqmaxl + Iil :


Item 11 of 44
QID 1248 i
k • Mark <l
prevlo".
t::>
NHt
-I
~a b'" IlU
r;:"
Notu
~
(al(u lator

[AI
"0.
.".
A 6-year- old girl exh ibi ts vision problems during a screening at school, and the school
nu rse tells the parents the ch ild should be fitted for glasses. Her mother is upset, because
her daughter is already much taller than her classmates, has an awkward gait, and was

."8.
· 12 • recently diagnosed with scoliosis. She is afraid that the qlasses will only add to her dauqhter's
' 13 • problems at school, where her classmates frequently tease her. When the ophthalmologist

"4 .
" 5 •
.,
observes that the patient's right lens is dis located , he suspects that her symptoms are in fact
related to an enzyme deficiency.

This patient most likely has difficulty metabolizing which substrate?


· 17 •
134415: zheng
We value your feedback!
A. Homocysteine
" 9 •
. 20 . B. Lysine
· 21 •
C. Phenylalanine
' 22 •

· 23 • O. Tryptophan
. 24 . E. Tyrosine
· 25 •

The correct answer is A. 60% chose this.


Homocystinuria is an inborn error of metabolism caused by a defect in cystathionine
the enzYme that converts homocysteine to cystathionine. In addition to Marfan-like

"0.
.". Homocystinuria is an inborn error of metabolism caused by a defect in cystathionine synthase,
the enzYme that converts homocysteine to cystathionine. In addition to Marfan-like features
an d subluxation of the lens , these patients are at increased risk of a va riety of cardiovascula r

."8.
· 12 •
derangements, including premature vascular disease and death. Atherosc lerosis is rare in
' 13 • children. Atherosclerosis in a child is hig hiy suggestive of homocystinuria.

"4
"5
.
.
,.
Homocystinuria Homocysteine measurement Subluxation of jomt AtherosclerosIs Inborn Errors of Metabolism Vascu lar Diseases

B is not correct. 14% chose this .


Lysine is another of the essential amino acids. (Recall the mnemonic PVT TIM HALL used to
· 17 • remember the 10 essential amino acids: Phenyla lanine, Valine, Tryptophan, Threonine,
I soleucine, Methionine, Hi stidine, Arginine, Lysine, and Leucine). It is not related to

"9
.20 .
.
· 21 •
homocystinuria,
lysine AminoAcids Homocystlnuria

C is not correct, 9% chose this.


Phenylalanine accumulates in patients with phenylketonuria (PKU). The stereotypi ca l patient
' 22 •
with PKU presents with severe mental retardati on, se izu res, and a "mo usy" odor .
· 23 • Phenylalanine Seizures Severe menta l retardation (I.Q. 20-34)
.24 . o is not correct. 9% chose this.
· 25 •
Tryptophan is an amino acid often confused with tyrosine. It is an essentia l amino acid and does
not have any relationship to homocystinuria. Tryptophan is converted into niacin (vitamin 83),
and its deficiency can lead to pellagra (the "3 Os" of 0 iarrhea, 0 ermatitis, and 0 ementia) .
Tryptophan Tyrosine Homocystinuna Niacin Pe llagra
,Pad 9 2050 2810 • •

+ G ~ UI usmle rx com/ap lltqmaxl + Iil :


Item 11 of 44
QID 1248 i
k • Mark <l
prevlo".
t::>
NHt
-I
~ab'" IlU
r;:"
Notu
~
(al(ulator

E IS not correct. 8% chose thi s.

"0.
.". In phenylketonuria (PKU), a deficiency of phenylalanine hydroxylase results in an inability to
convert phenylalanine to tyrosine, making the latter an essential amino acid in patients with
PKU. It has no role in homocystinuria.

."8.
· 12 •
Phenylalanine Tyrosine Homocystinuria
' 13 •

"4
"5 .
., .
Bottom line:
Homocystinuria is an autosomal recessive disorder caused by a defect in cystathionine
· 17 • synthase, which converts homocysteine to cystathionine. Individuals with this disorder have
multisystemic complications and often suffer subluxation of the lens, premature vascular

"9
. 20 .
.
· 21 •
disease (thrombotic complications), and early death. For the U5MLE, know the various
metabolic disorders and their classic presentations .
HomocysUnuria Homocysteine measurement Subluxation of joint Vas(ular Diseases Metabolic Diseases

' 22 •

· 23 • hl;fi,·,.·1 for year: 2015


.24 . FIRST .0.10 FA CTS

· 25 • FAt5, p. 1062

HomocySlinuria Tr pes (a ll aUlosomal recessil'e): All forms result in excess homoc),stei ne.
• Cystalhi onin e s)'nlha se defic iency Fimlings: tl homoc)'steine in urine. inlell ect ual
(Ireatment: i melh ioniu e. I e),S lei ne. I BI2 disahilit y. osteoporosh. marfano id habilus.
~ml folalc ill diet) kyphosis. lens snhlllx;ltion (down,,";l rd and
• i a ffillit y of cyslathionine synthase for illw~rd). thromhos is. ~ ml ~Ih erosc le ros i s (stroke
prrido~,tl phosph ate (treatment: II B6 and ,,,,<1M!).
I

[AI
.".
"0. In response to increased body temperature in exercise, a marathon runner thermo regulates
through the increased production of sweat.

."8.
· 12 •
What are the locations and primary stimulus for secretion of the sw eat glands most active in this
' 13 •
marathon runner?

"4
"5 .
., . A. Axillae and groin; circulating catecholamines
134415 : zheng

We value your feedback!


· 17 • B. Axillae and groin; sympathetic innervation

C. Palms and soles; circulating catecholamines

"9
.20 .
.
· 21 •
D. Throughout the skin; circulating catecholamines

" E. Throughout the skin; sympathetic innervation


' 22 •

· 23 •
. 24 .
· 25 • Th e correct answer i s E. 55% chose th i s.
There are two types of sweat glands: apocrine and eccrine. Eccrine glands serve a
thermoregulatory function and would be the sweat glands likely to be most active in a
marathon runner. There are two to four million eccrine sweat glands throughout the skin, but
they are most numerous on the pa lms and soles. Eccrine glands secrete a watery solution and
are stimulated by direct sympathetic autonomic innervation. Apocrine sweat glands are
over the axillae, areolae, genitals, and anus, and do not become active until pUDen)l.
,Pad 9 2050 2810 • •

+ G ~ UI usmle rx com/ap lltqmaxl + Iil :


Item 120f44
QID 3634 i
k • Mark <l
p r evlo".
t::>
NHt
-I
~a b'" IlU
r;:"
Notu
~
(a l(u lator

.".
"0. There are two types of sweat glands: apocrine and eccrine. Eccrine glands serve a
thermoregulatory function and would be the sweat glands likely to be most active in a
ma rathon runner. There are two to four mill ion eccrine sweat glands throughout the skin, but
· 12 • they are most numerous on the pa lms and soles. Eccrine glands secrete a watery solution and
' 13 • are stimulated by direct sym pathetic autonomic innervation. Apocrine sweat glands are found

"4 .. over the axillae, areolae, genitals, and an us, and do not become active until puberty. Apocrine

"5
.
gland excretions are oily and viscid, and stim ulated by circulating catecholamines .

"6 .
Encounter due 10 puberty Sweal Glands Eccrine Glands Ar'lUS

A Is not correct. 7% chose this.


· 17 •
., Eccrine sweat glands are distributed throug hout the skin, not limited to the groin. They are
stimulated by direct sympathetic autonomic innervation, not catecholamines.

"9.
. 20 .
· 21 •
Eccrine Glilnds

B is not correct. 21% chose this.


While eccrine sweat glands are stimulated by direct sympathetic autonomic innervation, they
' 22 • are distributed throughout the skin and not limited to the axi llae and groin.
· 23 • Eccrine Glilnds

. 24 .
C is not correct. 3% chose this.
· 25 • Eccrine sweat glands are located throughout the skin, but they are most numerous on the
pa lms and sales. Apocrine gland excretions are oily and viscid, and stim ulated by ci rculating
catecholamines.
Eccrine Glilnds

D is not correct. 14% chose this.

"0
.
. ".
· 12 •
· 13 •
Whi le eccrine sweat glands are stimulated by direct sympathetic autonomic innervation, they
are distributed throughout the skin and not limited to the axi llae and groin .
Eccrine Glilnds

C is not correct. 3% chose this.

"4.. Eccrine sweat glands are located throughout the skin, but they are most numerous on the

"5
.
pa lms and sales. Apocrine gland excretions are oily and visc id, and st imulated by Ci rcu lating
catecholamines.

"6
"7
.
.
,.
Eccrine Glilnds

D Is not correct. 14% chose this .


Eccrine sweat glands are distributed throughout the skin. However, they are stimulated by
.'9 . direct sympathetic autonomic innervation; apocrine glands are stim ulated by Circulat ing
' 20 . catecholamines.
Eccrine Glilnds
· 21 •
· 22 •
· 23 • Bottom line:
' 24 . Eccrine glands secrete a watery sol ution and are stimulated by direct sympathetic innervation .
. 25 . Apocrine glands, which secrete oily, viscous fluid, are stim ulated by catecholamines .
. 26 . Eccr ....e Glilnds

· 27 •

iii I if') , .j I.) for year: 2015


fllI.ST AIO FACTS
,Pad "" 20 50 28 Y0 . •

+ G R< UI usmle rx com/ap l#Qmax1 + • :


Item 13 of 44
QID 210<1
L

J.
• Mark -<l
Pre YlDu,
t>-
NeKt
all
la b"" ues
~.
No t es
~'t
c .. k ul~ tor

"0. A 43-year-old man who recently returned from a one-week hiking t rip around the Great Lakes
presents to the clinic with flu-like symptoms and a productive cough. Review of symptoms is A
I·AI
' 11 .
remarkable for new-onset diffuse bone and j oint pain. Physical examination is remarkable for
· 12 •

.
pleuritic chest pain. Cultures are taken from the patient's blood, a slide from which is shown in the
· 13 • image.

"4. e- •
· 15 • 0
,'6 .
· 17 •
'. •

~

.
· 18 •
. ,9 .
' 20 • t
."
· 22 •
· 23 •
• •
' 24 •
. 25 .
Image courtesy of CDC
.26 .
· 27 •
Which of the following agents is the most appropriate t reatment for th is patient?

We value your feedback!


A. Amphotericin B

"0.
' 11 .
The correct answer is A. 59% chose this.
Th is pati ent has likely been infected by Blastomyces dermatltidis, the fungus that causes
blastomycos is. It is endemic in many parts of North America, including states east of the
· 12 • Mississippi River and in the areas surround ing the Great Lakes. The broad-based budding yeast
· 13 • shown in the image, indicated with the red arrow, is classic for Blastomyces . Blastomycosis

"4
"5 .. usually presents with flu-like symptoms: fevers, chills, productive cough, myalgia, arthra lgia,
and pleuritic chest pain. Patients commonly progress to develop chronic pulmonary infection or
widespread disseminated infection. Mild pu lm onary infections are treated with fluconazole or
,'6 . ket oconazole. Amphotericin B is used to treat systemic infections (ie, the organism is already
· 17 • circulating in the blood).

.
Biaslomycosis Arthralgia Fluconazole Ketoconazole Infl~nza · l i ke symptoms Productive cough PleuritiC pa .... Amphoteric ....
· 18 •
. ,9 . B Is not correct. 26% chose this.
' 20 . Fluconazole or ketoconazole are effective treatments for local mild pulmonary Blastomyces
infections but are ineffective if the infection has already become systemic .
." Fluconazole KelO(onazole
· 22 •
· 23 • C is not correct. 6% chose this.
Uraconazole Sporotr ichos is pustu le Microscopy Potassium Iodide
' 24 .
.25 . •
.26 .
· 27 •
,Pad "" 20 51 28 Y0 . •

+ G R< UI usmle rx com/ap l#Qmaxl + • :


Item 13 of 44
QID 210<1
L

J.
• Mark -<l
Pre YlDu,
t>-
NeKt
all
la b"" ues
~.
No t es
~'t
c .. k ul~ tor

Potassium Iodide and Itraconazole are used to treat Sporothnx schenckll infectIOns
"0.
' 11 .
(sporotrichosis). When 5. schenckii is introduced into the skin, usually by a thorn prick, it causes
the formation of a local pustule or ulcer with nodules along the draining lymphatics (ascending
lymphangiti s). Under microscopy, S. schenckii appears as a dimorphic fungus with cigar-shaped,
· 12 •
budding yeast, as shown in this image.
· 13 •

"4.
· 15 •
D is not correct. 4% chose this.
Selsun Microscopy Selenium Tinea

,'6 .
· 17 •

.
· 18 •
. ,9 .
' 20 •

."
· 22 • Image courtesy of CDC/Dr. Lucille K.
· 23 • Georg
' 24 . Selenium sulfide (Selsun) is used to treat Malassezia furfur, the cause of tinea versicolor.
. 2S . Symptoms of this infection include hypo pigmented skin lesions that occur in hot and humid
.26 . conditions. M. furfur has a "spaghetti and meatballs" appearance under microscopy. as shown in
this image.
· 27 •
E is not correct. 5% chose this.
Splenomegaly Anem ia Sod'LJm StlbogllJCOnale

"0
." . .
· 12 •
· 13 •
Selenium sulfide (Selsun) is used to treat Malassezia fUrfur. the cause of tinea versicolor.
Symptoms of this infection include hypo pigmented skin lesions that occur in hot and humid
conditions. M. furfur has a "spaghetti and meatballs" appearance under microscopy. as shown in
this image.
E is not correct. 5% chose this.

"4 .
· 15 •
. ,6 .
Splenomegaly Anemia Sod,um Subogluconale

· 17 •

.
· 18 •

" 9 •
• 20 •

." Image courtesy of CDC/Dr. Francis W


· 22 •
Chandler
· 23 •
Sodium stibogluconate is used to treat Leishmania donovani infection. L. donovani infection
' 24 .
presents with hepatomegaly and sp lenomegaly. malaise. anemia. and weight loss. It is a
.2S . protozoal parasite tran smitted via the sandfly. Microscopically. macrophages containing
.26 . amastigotes are observed. as shown in this image.
· 27 •

Bottom line:
Blastomycosis. endemic to many parts of eastern North America. presents with flu-like
svrnoltonns. Mild pulmonary infections are treated with fluconazo le or ketoconazole;
,Pad "" 2051 28« _ •

+ G R< UI usmle rx com/ap l#qmaxl + III :


Item ' 13 of 44
QID 2104 i
L • Mark --<l
Pre Ylous
t:>
Next
_!
la b~ ues
~.
Notes
C';'\I
Cakul~ tor

· 10 "
Sodium stibogluconate is used to treat Leishmania donovan; infection. L donovan; infection
presents with hepatomegaly and splenomegaly, malaise, anemia, and weight loss. It is a
· 11 "

··12"
13 "
protozoal parasite transmitted via the sandfly. Microscopically, macrophages containing
amastigotes are observed, as shown in this image.

· 14 "
Bottom Line :
·15"
· 16 "
Blastomycosis, endemic to many parts of eastern North America, presents with flu-like
symptoms. Mild pulmonary infections are treated with fluconazole or ketoconazole; systemic
··17"
18 "
infection requires amphotericin B.
BI<lstomy<:osis FlucoO<lzole Ketocofl3lole InflueJlZ~·like symptoms Amphoteric in

· 19 "
· 20 "
ul;f.,.·,,,, for year: 2015

·22 "
• 21 " Ftll.5T ""0 F"'C T 5

·23 "
· 24 "
Systemic mycoses All of thc following can canse pneumonia and can di s~cminate .
FA1S, p. 145,1

/\1 1are caused b)' dimorphic


fllngi: cold (20°C) = UlOid: heat (37°C) = yeast. Th e o nly excepti on is cOC(: idioidomycosis, "hid,
· 25 " is;1 sphcrul e (not yeast) ill ti ss11c. Treatment: flueonawle or itraeonawlc for luc al infection;
amphotericin B for sp tcm ic infection_ S)'S ielllic mycoses call mimic TB (grJnnlum;l fOrlll;,tioll).
· 26 "

·27 " DISEASE


Histoplilsmosis
except. uillike 'I'll. h;" 'e 110 person·person tr;lIl smi ss ion
EMDEMIC LOCAT IOH AHD'ATHOLDGK fEATU RES
~'l i$s i sSippi ~l1d Ohio Ri,'er ";lllc)'~. CallSes
~ons

Il isto h id es (wilhin macrophagcs). Bird or bat


p"cllmolli;" droppings.
Mac ropha ge filled wi th Hisioplasllla (smaller
thall RBC) rJ.

· 10 " A 35-year-old man develops swell ing and pain in his right leg while it is immobilized in a cast. IAA[
He is diagnosed with a deep venous thrombosis and prescribed a 6-month course of
· 11 "

··12"
13 "
warfarin. Du ring this time he develops a severe ringworm infection on his ankle and receives
antifungal treatment. Whi le being treated he develops dyspnea and hypoxia and is found to have a
pu lmonary embolism.
· 14 "
· 15 " Administration of which of the followinq aqents could have resu lted in lower serum levels of
warfarin and contributed to this patient's pulmonary embolism?
· 16 "

··17"
18 " A. Caspofungin
We value your feedback!
13441S : zheng

· 19 " B, Flucytosin e
· 20 "
C. Griseofulvin

·22 "
• 21 "
D. Ketoconazole
·23 "
· 24 "
E. Terbinafine

· 25 "
· 26 "

·27 " The correct answer is C. 54% chose this,


Griseofulvin is an antifungal agent used to treat superficial infections such as tinea or
ringworm. Adverse effects of griseofulvin include teratogenesis, carcinogenesis, conful5ic,~
headaches, and induction of cytoch rome P450. Its stimulation of the P450
,Pad "" 2052 27« _ •

+ G R< UI usmle rx com/ap l#qmax1 + III :


Item ' 14of44 L • Mark --<l t:> _! ~. C';'\I
QID 1736 i Pre Ylous Next la b~ ues Notes Cakul~ tor

9 9 P
· 10 " ringworm. Adverse effects of griseofulvin include teratogenesis, carcinogenesis, confusion,
headaches, and induction of cytochrome P450. Its stimulation of the P450 system decreases the
· 11 "

··12"
ha lf-life of molecules metabolized by this system, including warfarin. In other words, while

13"
· 14 "
taking griseofulvin, a patient metabolizes warfarin more quickly. He or she therefore has a lower
effective dose, which may be subtherapeutic. This patient is already at risk for a pulmonary
embolism secondary to his deep vein thrombosis. The reduced activity of warfarin increases his

·15"
· 16 "
chances of developing a pulmonary embolism .
A is not correct. 5% chose this.

··17"
18 "
Caspofungin is an antifungal that inhibits cell wall synthesis and is used to treat invasive
aspergillosis. Adverse effects include gastrointestinal upset and flushing. It is not known to
interact with warfarin. Caspofungin should not be taken in conjunction with cyclosporine as this
· 19 " combination may lead to elevated liver enzyme levels.
· 20 " B Is not correct. 6% chose this.
Flucytosine would not be an appropriate antifungal treatment for ringworm. Clinical use is

··2223 ""
• 21 "
confined to combination therapy, either with amphotericin B for cryptococcal mening iti s or with
itraconazole for chromoblastomycosis. Adverse effects include bone marrow toxicity with
anemia, leukopenia, and thrombocytopenia.
· 24 "
D is not correct. 31% chose this.
· 25 "
Ketoconazole is an antifungal agent that inhibits ergosterol synthesis. It inhibits the
· 26 " cytoch rome P450 system, which results i n an increase in warfarin activity. This would place the
·27 " patient at ri sk of a bleed as opposed to thrombosis.
E is not correct. 4% chose this.
Terbinafine is an antifungal agent used to treat dermatophytoses, S~'~~i;:~t~~%;
(fungal infections of the nail). Like the azole it interferes with bic>svntl,e,

D is not correct. 31% chose this.


· 10 "
Ketoconazole is an antifungal agent that inhibits ergosterol synthesis. It inhibits the
· 11 "

··12"
cytochrome P450 system, which results i n an increase in warfarin activity. This would place the

13"
· 14 "
patient at risk of a bleed as opposed to thrombosis.
Ketoconazole Thrombosis

E Is not correct. 4% chose this.


· 15 " Terbinafine is an antifun gal agent used to treat dermatophytoses, spec ifically onychomycosis
· 16 " (fungal infections of the nail). Like the azole drugs, it interferes with ergosterol biosynthesis, but
··17"
18 "
does not interact with the P450 system. Instead, terbinafine inhibits the fungal enzyme
squalene epoxi dase; this causes the accumulation of a toxic metabolite.
terb'nafine Ony<:homyi:osis SqUdlene
· 19 "
· 20 "
Bottom line:

··2223 ""
• 21 "
Griseofulvin induces the cytoch rome P450 system, thereby decreasing the half-life of
molecules metabolized by this system such as warfarin .

· 24 "
· 25 "
· 26 "
Ul;fi'·'I.' for year: 2015
FIRST .0.10 FACTS

·27 "
,Pad "" 2052 27« _ •

+ G R< UI usmle rx com/ap l#qmax1 + III :


Item ' 1Sof44 L • Mark --<l t:> _! ~. C';'\I
QID 1199 i Pre Ylous Next la b~ ues Notes Cakul~ tor

A 12-year· old boy with intellectua l disabil ity comes to the physician because of painfu l
' 10 "
swollen joints. During the examination, the physician notices that the boy makes several
, 11 "
uncontrolled spastic muscle movements. Past medical history includes a diagnosis of
, 12 " muscu lar hypotonia at 5 months of age. Laboratory values are shown in the list.
, 13 " Na+: 137 mEq/L
, 14 " K+: 4.1 mEq/L
CI' : 101 mEq/L
, 15 "
HC03': 25 mEq/L
, 16 " BUN: 15 mg/d L
, 17 " Creatinine: 0.8 mg/dl
, 18 " WBCs: 8000/mm 3
Lactic acid: 0.2 mg/dL
' 19 "
Uric acid: 8.5 mg/dl
, 20 "
, 21 "
Which of the following is the most likely cause of these findings?
, 22 "
, 23 " 134415: zheng
, 24 "
, 25 " A. A deficiency of adenosine deaminase
, 26 "
8. A deficiency of glucocerebrosidase
, 27 "
C. A mutation of an enzyme in the de novo purine biosynthetic pathway

' 10 " Muscle Spasticity HyperlJricemia Lesd~-Nyhan Syndrome Mental Retardation

~r
, 11 "

, 12 "

, 13 "
, 14 " Xanthille

" 5"
, 16 "
, 17 " Guanille Hypoxanthine
, 18 "

' 19 " PAPP


, 20 "
HPAT
, 21 "

, 22 "
PPI
, 23 "
GMP IMP
, 24 "
, 25 "
, 26 " Th is patient is suffering from Lesch-Nyhan syndrome, an X-linked recessive disorder caused by a
, 27 " deficiency in hypoxanthine guanine phosphoribosyltransferase t hat leads to the overproduction
of pu rines and the accumulation of uric ac id, as shown in the image. Clues to this diagnosis
include intellectual disabilty, spasticity, and an elevated uri c acid level. This rare biochemical
disorder is characterized I by hyperuricemia, excessive on)aLJC{lon
I
,Pad "" 2052 27« _ •

+ G R< UI usmle rx com/ap l#qmax1 + III :


Item ' 1Sof44 L • Mark --<l t:> _! ~. C';'\I
QID 1199 i Pre Ylous Next la b~ ues Notes Cakul~ tor

Th is patient IS suffenng from Lesch-Nyhan syndrome, an X-linked recessive di sorder caused by a


· 10 " deficiency in hypoxanthine guanine phosphoribosyltransferase that leads to the overproduction
of purines and the accumulation of uric ac id, as shown in the image. Clues to this diagnosis
· 11 "

··12"
include intellectual disabilty, spasticity, and an elevated uric acid level. This rare biochemical

13"
· 14 "
disorder is characterized clinically by hyperuricemia, excess ive production of uric acid, and
certain characteristic neurolog ic features, including self-mutilation, choreoathetosis, spastic ity,
and intellectual disability.

·15"
· 16 "
A Is not correct. 5% chose this.
A deficiency of adenosin e deaminase would result in severe combined immunodeficiency

··17"
18 "
disease (SeID), which prevents developm ent of both the humoral and cell-mediated immune
systems . Therefore individuals with SCID are faced with recurrent devastating bacterial, viral,
and fungal infections.
· 19 " Mutation Infarction Anemia Thrombocytopenia Bone Marrow Collapse of~ertebra

· 20 "
B is not correct. 18% chose this.
A deficiency of glucocerebrosidase would result in Gaucher disease. There are several types of

··2223 ""
• 21 "
Gaucher disease based on the type of mutation, but most forms are marked by lipid-laden
macrophages (termed Gaucher cells) that invade the bone marrow and cortex, leading to bone
infarction, vertebral collapse, anemia, and thrombocytopen ia.
· 24 "
Immune system Adenosine deamilOlse deficiency
· 25 "
· 26 " C is not correct. 23% chose this.

·27 " This boy's findings are not due to a mutation of an enzyme in the de novo purine biosynthet ic
pathway, which woul d result in deficiencies in nucleotides needed for DNA synthesis. Symptoms
may resemble cond iti ons in which dietary deficiencies impede de novo nucleotide synthesis,
such as megaloblastic anemia due to fol ic acid and/or vitamin 8 1 2 deficiency.

· 10 " An excision repair enzyme deficiency wou ld result in xeroderma pigmentosum, which is marked
· 11 " by dry and hyperpigmented skin that is extremely sensitive to exposu re to ultraviolet radiation.

··12"
Therefore, individuals with th is disease are at increased risk for severe sunburns and skin

13"
· 14 "
cancer.
Xeroderma Pigmentosum Malignant neoplasm of skin

· 15 "
· 16 " Bottom li ne:

··17"
18 "
Lesch-Nyhan syndrome is an X-linked recessive disorder caused by a deficiency in
hypoxanthine-guanine phosphoribosyltransferase, an enzyme in the pu rine salvage pathway. It
results in an accumulation of uric acid and is associated with self-mutilation behaviors,
· 19 " intellectual disability, gout, and movement disorders.
· 20 " Gout Lesch-Nyhan Syndrome Disability

··2223 ""
• 21 "

I ill ;fi , .j 1.1 for year: 2015


FllI.5T A I D FAC TS
· 24 "
· 25 " FA15,p.64.1

· 26 " Purine salvage deficiencies

·27 " Nuclfic acids NLlcI~ acids


,Pad "" 2053 27« _ •

+ G R< UI usmle rx com/ap l#qmax1 + III :


Item' 16of44 L • Mark --<l t:> _! ~. C';'\I
i la b~ Cakul~tor

.
QID 1302 PreYlous Next ues Notes

"5
"6 ..
· 17 •
A 54-year-ol d woman has had longstanding rheumatoi d art hritis. Her rhe umat ologist
rece nt ly started her on meth otrexate, a com petitive inhibitor of dihydrofol ate reductase.

.,

.".
"9.
. 20 •

· 22 •

' 23 •
.24 .
What effect does methotrexate have on d ihydrofolate red uctase (OHFR)?

A. Methotrexate acts on DH FR by decreasi ng its Miehaelis-Menten const ant

8. Methotrexate acts on DH FR by increasing its Michaelis-Menten constant


134415 : zheng

C. Methotrexate does not affect DHFR's Michaelis-Menten constant


· 25 •
' 26 . D. The maxim um reaction rate is decreased
· 27 •
E. The maximum reaction rate is increased
· 2• •
. 29 .
' 30 . We value your feedback!

"6..
· 17 •
.,

.
" 9 •
• 20 •

."
· 22 •

' 23 •
.24 .
.. .
Subltrate concentnltion lSI

· 25 • The Michaelis-Menten constant (Km) of an enzyme such as DHFR reflects the enzyme's affinity for
a particular substrate [5). suc h as methotrexate, in an inverse fash ion (shown in the graph).
' 26 .
Therefore, if the Km is high, the enzyme w ill have low affinity for the substrate. The Km of an
· 27 • enzyme is the concentrati on of substrate required to achieve a reaction velocity equal to hal f of
· 2• • the maximum reaction rate (Vmaxl. A co m petitive inhibitor binds reversibly to the same site that
. 29 . the substrate wou ld normally occupy and thus competes with the substrate for that site .
Therefore, in th e presence of a competitive inhibitor, the concentration of methotrexate
' 30 .
requi red to achieve half of Vmax wi ll be increased. It should also be noted that competitive
inhibitors can be overcome by adding more substrate.
A Is not correct. 18% chose this.
The Michaelis·Menten constant of an enzyme is increased by a competitive inhibitor.
C is not correct. 13% chose this.
The Michaelis-Menten constant of an enzyme is increased by a competitive inhibitor.
,Pad "" 2054 27« _ •

+ G R< UI usmle rx com/ap l#qmax1 + III :


Item' 16of44 L • Mark --<l t:> _! ~. C';'\I
i la b~ Cakul~tor

.
QID 1302 PreYlous Next ues Notes

"5
"6 ..
· 17 •
The Michaelis-Menten consta nt of an enzyme is increased by a competitive inhibitor.
C is not correct. 13% chose this.

., The Michaelis-Menten constant of an enzyme is increased by a competitive inhibitor.

.
"9.
. 20 .
."
· 22 •

' 23 •
.24 .
o is not correct. 12% chose this.
The maximum reactio n rate of an enzyme is unchanged by a competitive in hibitor.
E Is not correct. 2% chose this .
The Vmax is the maxim um rate or velocity in which substrate molecules are converted to product
per unit time . At high su bstrate concentrati ons. the reaction rate levels off, reflecting the
saturation of all available binding sites with substrate. At a sufficiently high substrate
concentration, the reaction velocity reaches the Vmax observed in the absence of inhibitor. Thus,
the Vmax of an enzyme is unchanged by a competit ive inhibitor, as an increase in substrate
· 25 •
outcompetes this type of inhibitor.
' 26 .
· 27 •
Bottom Line:
· 2• •
In enzyme kinetics, Km reflects the affinity of the enzyme for its substrate .
. 29 .
' 30 .

iii I if' , .j 1.1 for year: 2015


FIII.ST AID FACTS

FA15,p.4091

Antimetabolites

."9.
"6 .
· 17 •
.,
A retrospective study is designed to look at the differences in smoking in male and female
populations. In a sample of 100 men and 100 women, 2S men and 40 women admit to
smoking a cigarette in the past month .

." .
.20 .
· 22 •
What is the odds ratio of a woman smoking compared to a man smoking?

A. 1:2
( We value your feedback!
134415; zheng

l ltl~
8. 2:1
' 23 •
. 24 . C. 5:8
· 25 •
0 .40:25
' 26 .
· 27 • E. Cannot be determined
· 2• •
. 29 .
' 30 .
The correct answer is B. 36% chose this.
,Pad "" 2054 27« _ •

+ G R< UI usmle rx com/ap l#qmax1 + III :


Item ' 17 of 44
i
L • Mark --<l t:> _!
la b~
~. C';'\I
Cakul~ tor

.
QID 3742 Pre Ylous Next ues Notes

"5
"6 ..
· 17 •
The correct answer is B. 36% chose this.

Smoking
.,

.
y~ No

"9. f ~~
j~~
. 20 •

."
· 22 •

' 23 •
.24 . The odds ratio is defined as the ratio of the odds of an event occurring in one group compared
with the odds of it occurring in another group. The formu la for calc ulation is: [p I (1 - p) I [q I (1 -
· 25 •
q)]. In this case, (0.41 0.6) 1(0.25/0.75) = 2. An alternative way to approach this question is to
' 26 . construct a 2x2 table with smoking as the disease and female gender as the exposure (see the
· 27 • image for clarification). To calcu late the odds ratio from this table, the formula wou ld be
(a/c)/(b/d), OR ad/be. Thi s would then give (40)( 75) I (60 x 25) = 2. Constructing a 2x2 table
· 2• •
quickly and accurately is essentia l for success on USMLE Step 1 .
. 29 . A is not correct. 8% chose this.
' 30 .
The odds ratio of a man smoking to a woman smoking is 1 to 2, but the question asks for the
ratio of a woma n smoking to a man smoking.
C is not correct. 16% chose this.
If you divide the frequency of male smokers by the frequency of female smokers, you get .251
A, or 5:8. This is a risk ratio, called relative risk, and not an odds ratio.

."9.
"6 .
· 17 •
.,
If you divide the frequency of fema le smokers by the frequency of male smokers, you get 40 1
25, or 1.6. Th is is a risk ratio reflecting relative risk, and not an odds ratio.

.
E is not correct. 14% chose this .
Enough information is provided. The formula for ca lculating the odds ratio is (p 11 - p) 1 (q 11 -
q). Alternately, a 2x2 table can be quickly constructed as described above .
• 20 •

." Bottom li ne:


· 22 •
Calculating the odds ratio, relative risk, and attributable risk is possible given the proper data
' 23 •
from a study, and it is important to understand that odds ratios are calculated for case-control
.24 . trials, whereas relative risk is calculated for cohort stud ies. A 2x2 table can be quickly
· 25 • constructed to visualize the data in a familiar format and simplify the necessary calculations.
' 26 .
· 27 •

· 2• • iii I jfj , ., 1.1 for year: 2015


FIRST AIO FACTS
.29 .
' 30 .
Quantifying risk Definitions and formulas ;lre bmed on the classic
2 )( 2 or contingency table. <!> -- e
FA15, p. 50.1
,Pad "" 2054 27« _ •

+ G R< UI usmle rx com/ap l#qmax1 + III :


Item' 18of44 L • Mark --<l t:> _! ~. C';'\I
i la b~ Cakul~tor

.
QID 1388 PreYlous Next ues Notes

"5
"6 ..
· 17 •
A 13-year-old boy is diagnosed with neurofibromatosis after an MRI reveals bilateral acoustic
neuromas as the cause of his progressive decline in hearing.
IAA[
.,

.
By which of the following modes is this disease man ifested?

"9.
.20 . A. Autosomal dominant
We value your feedback!
134415 : zheng

." B. Autosomal recessive


· 22 •

' 23 • C. Mitochondrial inheritance


.24 .
D. Somatic mutation
· 25 •
' 26 . E. X-linked recessive
· 27 •

· 2• •
. 29 .
' 30 . The correct answer is A. 72% chose this.
Neurofibromatosis type 2 is caused by a mutation in the NF2 gene on chromosome 22; this
mutation is inherited in an autosomal-dominant manner. Autosomal-dominant inheritance
shows disease in many generations, with both males and fema les affected. It is possible for a
ma le or a female to transmit the defective gene to their offspring .
Mutation Neurofibromatosis 2 Chromosomes. Human. Pair 22

."9.
"6 .
· 17 •
.,
In autosomal recessive inheritance, a defective gene from each carrier parent is transmitted to
the offspring. Disease is often seen in on ly one generation. Males and females are equally likely
to be affected. Neurofibromatosis is not inherited in this way .

." .
.20 .
· 22 •

' 23 •
.24 .
C is not correct. 2% chose this.
In mitochondrial inheritance, all child ren (ma le and female) of an affected mother exhibit the
disease. The disease is not transmitted from fathers to any of th eir children (only maternal
transmission). Neurofibromatosis is not inherited this way .
D Is not correct. 3% chose this.
Somatic mutations occur in the somatic (non-germ) tissues; they are unable to be transmitted
to offspring and often occur in isolated individuals within a family. Males and females are
equally susceptible to somatic mutations. Neurofibromatosis is not inherited thi s way.
· 25 • Somatic mutation

' 26 .
E is not correct. 4% chose this.
· 27 •
X-linked recessive inheritance shows affected male individuals inheriting a defective copy of the
· 2• • X chromosome from heterozygous (asymptomatic) mothers. There is no male-to-ma le
. 29 . transmission .
X Chromosoll"H!
' 30 .

Bottom II ne:
Neurofibromatosis is inherited in an autosomal-dominant manner, but type 2 is associated
with bilateral acoustic neuromas.
Acoustic Neuroma
,Pad "" 2054 27« _ •

+ G R< UI usmle rx com/ap l#qmax1 + III :


Item ' 18 of44 Mark --<l t:> _! ~. C';'\I
... . . ........... .. -_.-
L •

i la b~ Cakul~ tor

.
QID 1388 Pre Ylous Next ues Notes

"5
"6 ..
· 17 •
transmission.
X Chromosome
- - - --

.,

.".
"9.
. 20 .

· 22 •

' 23 •
. 24 .
Bottom line:
Neurofibromatosis is inherited in an autosomal-dominant manner, but type 2 is associated
with bilateral acoustic neuromas .
Acoustic Neuroma

I iil;pi'·'!,) for year: 2015


F1II. S T AID FA CTS
· 25 •
' 26 .
FA15,p.83.1
· 27 • Autosomal dominant diseases
· 2• • Aulosomal dominant Bilateral. massive cn llirgcmc ilt of I: i<lnc)'s due 10 multiple large C)'sls. 85% of cases arc dllt' 10
.29 . polycystic kidney
disease (ADPKD)
mutation inl'KDI (chromosome 16; 16 lellcrs in "pol)'C)"tic kidn c(); remai nder du(' !o mutation
in I'K02 (chromosome 4).
' 30 .
Familial adenomatous Colon heeollH.'S emered will, ;ldenorn ;dOIi S polyps afler I)nherly. Progresses 10 colon ca ncer "nless
polyposis colon is resedcd . ~..Inl;ll iolls 0 11 chrolllosolllc 51.] (Arc ge ne); 5 1elkrs ill ··polyp.H
Familial Ele';'I ~1 LDL due 10 (lefecli"e or ahsenl LDL receplor Le;l(]s 10 seve re ;Ilherosclerolic di sc;Ise e~rl)'
hypercholesterolemia in life. cornca l arcus. tClldon x,lIlthonlas (classically in thc Achillcs tendon).
Hereditary Inhcrikd disordcr of blood ,·csscl5. Findi ll gs; br;lIlching s"in Icsions (1clangit:c1asias). rcrurrent
hemorrhagic cpista xis. skin discoloratiolls. artcriO"CIIOllS m;,]formations (AV/o.oIs). G I hlcedillg. hcmatnria. Also
telangiectasia " nO\, n as Osk r-\Veber- Iknd u syndromc.

"6..
· 17 •
.,
A 34-year-old woman presents to her physician with urinary urgency and dysuria. Urinalysis
shows gram-negative rods and a large number ofWBCs. The patient is prescribed an
antibiotic; however, she is cautioned to return if she experiences joint stiffness or pain, a

.
known adverse effect of this medication .

"9.
. 20 • Which of the following enzymes is inhibited by the antibiotic she was most likely prescribed?

." We value your feedback!


134415: zheng
· 22 • A. DNA gyrase
' 23 • B. DNA he Ii case
.24 .
C. DNA ligase
· 25 •
' 26 . D. DNA polymerase
· 27 •
E. RNA primase
· 2• •
. 29 .
' 30 .

The correct answer is A_ 64% chose this.


Th is patient has a urinary tract infection, and was likely prescribed a fluoroquinolone.
Fluoroquinolones such as ciprofloxacin, levofloxacin, and ofloxacin inhibit DNA gyrase, or
topoisomerase II, a bacterial enzyme that relaxes DNA supercoil s. Rare but serious adverse
reactions that follow treatment with fluoroquinolones include tendonitis or spontaneous
tendon rupture.
,Pad "" 2054 27« _ •

+ G R< UI usmle rx com/ap l#qmax1 + III :


Item ' 190f44 L • Mark --<l t:> _! ~. C';'\I
i la b~ Cakul~ tor

.
QID 1197 Pre Ylous Next ues Notes

"5
"6 ..
· 17 •
The correct answer is A. 64% chose this.
Th is pati ent has a urinary tract infect ion. and was li kely prescribed a fluoroq uinolone.
Fluoroqui nolones such as ciprofl oxacin, levofl oxacin, and ofl oxaci n inhibit DNA gyrase, or
.,

.
topoi50merase II, a bacterial enzyme that relaxes DNA supercoi ls. Rare but serious adverse

"9.
. 20 .
reactions t hat follow treatment wit h fluoroquinolones incl ude tendonitis or spo ntaneous
tendon rupture .
CiprOfloxa(;rl Levonoxac;n onoxacirl Urinary tract infection

." B is not correct. 8% chose this.


· 22 •
Helicase is a eukaryotic topoisomerase that unwinds DNA. Fl uoroquinolones ta rget only
' 23 • bacterial topoisomerase II and thus do not affect this enzyme. Evidence suggests t hat DNA
.24 . helicase inhibition in patients with herpes simplex vi rus infectio ns wil l prove therapeutic.
· 25 • Herpes Simplex InfeCtions

' 26 . C is not correct. 7% chose this.


· 27 • DNA ligase is an enzyme that unites t wo polynucleotide chains. It functions to sea l t ogether
· 2• • Okazaki fragments, which are synthesized discont in uously on th e lagging strand du ri ng DNA
. 29 . repl ication .
ONA Replication
' 30 .
o Is not correct. 18% chose this.
Polymerases synthesize new DNA strand s by using complementary strands as t emp lates.
Polymerase III is the main bacteria l DNA polymerase responsible for th e 5' exonuclease activity
of proofreading.
E is not correct. 3% chose this.
Primase is an RNA polymerase t hat creat es RNA primers for DNA replication in assa,:ialtia

"6..
· 17 •
.,
Polymerases synthesize new DNA strands by using complementary strands as t emp lates .
Polymerase III is the main bacterial DNA polymerase respo nsible for t he 5' exonuclease activity
of proofreadi ng.

.
E is not correct. 3% chose this .

"9.
. 20 .
Primase is an RNA polymerase t hat creat es RNA primers for DNA replication in association with
DNA polymerase-a .
DNA Replication

."
· 22 •
Bottom Li ne:
' 23 •
.24 . Fluoroquinolones inhibit DNA gyrase, a bacteria l enzyme t hat is needed to relax positive
supercoils during DNA replication. A ra re adverse effect of f1uoroqu inolone treatment is
· 25 •
tendon damage, which is more prevalent in chil dre n and the el derly.
' 26 . Elderly(populatioogrOlJp) DNA Replication

· 27 •

· 2• •
.29 . iii I if.., ,
FIR S T AI O FA CTS
., I.) for year: 2015
' 30 .

FA1 5 . p. 65.'
DNA replication Ellbryoti c DNA replication i$ more co mpl e~ th an th e probryoti c process but U K'!; man)'
en q111es analogo us to those listed below, In hoth proka ryotes and cubrrotes. DNA repli cati on is
scmieonscT\"d tive Hild invokes ho th contin uolls and di scontinuolls (Oka za ki fragment ) s)"nt hesis.
00 Origin of replication
,Pad "" 2054 27« _ •

+ G R< UI usmle rx com/ap l#qmax1 + III :


Item' 20 of 44
i
L • Mark --<l t:> _!
la b~
~. C';'\I
Cakul~ tor

.
QID 1378 Pre Ylous Next ues Notes

"5
"6 ..
· 17 •
A 3·vear-old qirl is brouqht to her pediatrician because of a progressive loss of motor
function and a decline in her cog nitive abilities . On physical examination, it is noted that
the patient has decreased deep tendon reflexes, truncal ataxia, and a decreased attention
.,

.
span in comparison to the child's last visit 6 months aqD. The physician knows that her pathology

"9.
. 20 •
is due to an abnormal accumulation of cerebrosi de sulfate in her brain, peripheral nerves, kidney,
and liver.

." A deficiency of wh ich of the following enzymes leads to th is condition?


· 22 •
134415 : zheng
' 23 • We va lue your feedback!
A. a-Ga lactosi dase A
.24 .
· 25 • B. j3-Ga lactocerebrosidase
' 26 . C. Arylsulfatase A
· 27 •
D. Hexosaminidase A
· 2• •
. 29 . E. Sphingomyelinase
' 30 .

The correct a nswer is C. 41 % chose this .


Arylsulfatase A converts sulfatide to galactocerebroside. Th is enzyme is deficient in patients
with metach romati c leukodystrophy , an autosomal recessive lysosomal storage disease
which patients cannot degrade sulfatides, leading to accumulation of cerebroside I

."9.
"6 .
· 17 •
.,
Arylsulfatase A converts sulfatide to galactocerebroside. Th is enzyme is deficient in patients
with met achromati c leukodystrophy , an autosomal recessive lysosomal storage disease in

.
which patients cannot degrade sulfati des, leading to accumulation of cerebros ide sulfate in
both neuronal and nonneurona l t issues. There is abnormal myelination with widespread loss of
myelination in the cent ra l nervous syst em and peripheral nerves, leading to the clinical signs .
. 20 . Metachromatic granules can be seen on histologic examination .
." Metachromatic stain reaction Leukodystrophy. Metachromatic Lysosomal Storage Diseases

· 22 • A is not correct. 14% chose this.


' 23 • a-Galactosidase A converts cerami de trihexoside to lactosyl cerebroside. This enzyme is
.24 . deficient in Fabry's disease.
· 25 • B is not correct. 26% chose this.
' 26 . j3·Galactocerbrosidase converts galactocerebroside to cerebroside. This enzyme is deficient in
· 27 •
Krabbe's disease.

· 2• • D is not correct. 11 % chose this .


.29 . Hexosaminidase A converts gang lioside M2 to ganglioside M3. This enzyme is deficient in Tay-
Sachs disease.
' 30 .
Tay·Sachs Disease

E is not correct. 8% chose thi s.


Sphingomyelinase converts sphingomyelin to cerebroside. This enzyme is deficient in Niemann-
Pick disease.
Niemann-Pick Diseases
,Pad "" 2055 27« _ •

+ G R< UI usmle rx com/ap l#qmax1 + III :


Item' 20 of 44
i
L • Mark --<l t:> _!
la b~
~. C';'\I
Cakul~ tor

.
QID 1378 Pre Ylous Next ues Notes

"5
"6 ..
· 17 •
Sphingomyelinase converts sphingomyelin to cerebroside. Th is enzyme is deficient in Niemann-
Pick disease.
Niemann-Pick Diseases

.,
"9
.".
.20 ..
· 22 •

.24.
' 23 •
Bottom II ne:
Metachromatic leukodystrophy is an autosoma l recess ive lysosomal storage disease that
results from a deficiency of arylsulfatase A, wh ich leads to tissue accumulation of cerebroside
sulfate.
Leukodystrophy, MetachromatIC Lysosomal Storage Diseases

· 25 •
I il1ifi','!,) for year: 2015
' 26 . FI RS T AID FA CTS

· 27 •
FA15, p. 111.1
· 2• •
E~c h is caused by ~ deficienc)' ill one of the many lysosomal CI1 7,)""'C$. RC~lJ lt$ ill ;111 :lCCl1l11 11I ;l\i o ll
.29 . Lysoso mal s torage
diseues of ~bllor111:,111lcl;,holic pnxillcts
' 30 . flNOI1IG5 OlFKI£Nt£NllM£ A((U/oIUtmOSUlIStRm INH(.q llAH((
Sphingolipidoses
Fabry disease Pcri phc r~ ll1Cll rOp;dh )' Of " ~l1dslfcct. Ccr;l1l1i{lc XR
;"'gi olocratom~s. nlr(' io,'~sC\ I br/rcnal trihcxosidc
discasl..
Gaucher disease Most co mmon. Gilicoccrcbrosidasc Gilicoccrcbrosidc Aft
Hcp;lIospienomcgd l),. P.ll1c)'topenia. (ll-gil leosi(bsc)
osteoporosis. Jscplic llccrosi5 of

"6..
· 17 •
.,
A 76-year-old woman presents to the emergency department complaining of blurry visio n
and a headache of 5 hours' duration. Her medical history is significant for chronic
hypertension and poorly controlled asthma . On phys ical examination, she is afebrile with a

.
heart rate of 75/min and blood pressure of 21 0/140 mm Hg. Funduscopic examination shows
swelli ng of the optic disks .
" 9 •
. 20 .
Which of the following medications sho uld be administered immediately?
." 13441S: zheng
· 22 • We value your feedback!
A. Captopril

.24.
' 23 •

· 25 •
B. Hydrochlorothiazide

C. Labetalol
' 26 .
· 27 • D. Losartan
· 2• • E. Sodium nitropru sside
.29 .
' 30 .

The correct answer is E. 55% chose this.


This patient is having a hypertensive emergency, defined as a systolic blood pressure >210 mm
Hg and/or a diastolic blood pressure >120 mm Hg.ln addition, she is exhibiting the sig ns and
symptoms of end -organ involvement: blurry vision and headache, as well as papilledema.
Sodium nitroprusside, a fi rst-line medication for hypertensive emergencies, acts by i
,Pad "" 2055 27« _ •

+ G R< UI usmle rx com/ap l#qmax1 + III :


Item' 21 of44 L • Mark --<l t:> _! ~. C';'\I
i la b~ ues Cakul~ tor

.
QID lS02 Pre Ylous Next Notes

"5
"6 ..
· 17 •
Th is patient is having a hypertensive emergency, defined as a systolic blood pressure >210 mm
Hg and/or a diastolic blood pressure >120 mm Hg.ln addition, she is exhibiting the signs and
symptoms of end-organ involvement: blurry vision and headache, as well as papi lledema.
., Sod ium nitroprusside, a fi rst-line medication for hypertensive emergencies, acts by direct

"9
.".
.20 ..
· 22 •

.24.
' 23 •
dilation of both arteries and veins. It is important to note that elevated blood pressure with
signs of end-organ damage (such as increased levels on liver fu ncti on tests, pa pilledema, retinal
hemorrhages, symptoms of stroke) should also be termed hypertensive emergency, and
nitroprusside shou ld be administered immediately. Adverse effects of nitroprusside include
reflex tachycardia, as we ll as cyanide t oxicity, especia lly in patients with liver disease.
Headache Nitroprusside Papi lledema Sodium NitroprUSSide liver function Retinal Hemorrhage Reflex tilchycard ia

A is not correct. 8% chose this .


Captopril, an angiotensi n-converting enzyme inhibit or, is an ext remely effective med ication in
· 25 • cont rolling chronic hypertension. However, it is not useful in acute hypertensive emergencies
' 26 . because its mechanism of actio n is too slow.
· 27 • Captoprl l

· 2• • B is not correct. 15% chose this .


. 29 . Hydrochlorothiazide is an effective diuretic med ication used for the treatment of chronic
' 30 . essent ial hypertension. Because of its mechan ism of action, it does not ca use a reduction in
blood pressure rapid enough to be useful in an emergent situation such as the one described
here.
Hydrochlorothiazide Essential Hypertension

C is not correct. 16% chose this.


!3-Blockers such as the nonselective agent labetalol are also first-line agents in m,onaoir.o
hypertensive emergencies, alt hough the ir mechanism is not completely understood.

."9.
"6 .
· 17 •
.,
!3-Blockers such as the nonselective agent labetalol are also first-line agents in managing
hypertensive emergenc ies, although the ir mechanism is not completely underst ood. With fi rst-
time emergent use, th e antihypertensive acti on of labetalol is thought to res ult from a decrease

.
in cardiac output. However, this patient's history of asthma makes labeta lol a suboptimal
choice, as !3-adrenergic blockade in the airways co uld precipitate bronchospasm.
labetillol Asthma
.20 .
o is not correct. 6% chose this .
." As an angiote nsin receptor blocker, losartan is a useful drug with whic h to control chro nic
· 22 •
hypertension. It has no role in the acut e management of hypertensive emergency because of its

.24.
' 23 •

· 25 •
slow onset of effect .

' 26 .
Bottom Line:
· 27 •
Patients with hyperte nsive emergency (>210 mm Hg systo lic and/or >120 mm Hg diastolic)
· 2• • should immediately be given sodium nitroprusside, which dilates both arteries and veins .
. 29 . Sod ium Nitroprusside

' 30 .

hli1',"''''
fiRS T A I D fAC TS
for year: 2015

FA15 . p . 304 .4

Drugs incl llde cle\'idipine. fel loidopaili. h,he lalol . ni c;ml ipine. nitroprusside.
,Pad "" 2055 27« _ •

+ G R< UI usmle rx com/ap l#qmax1 + III :


Item' 22 of 44
QID 1369 i
L • Mark --<l
Pre Ylous
t:>
Next
_!
la b~ ues
~.
Notes
C';'\I
Cakul~ tor

.'8 . To confirm a diagnosis of HIV, a laboratory technician runs a set of proteins from confirmed IAA[

.
.'9 . HIV-infected cells on an electrophoretic gel, transfers them to a filter membrane, and then
.20 . incubates the filter membrane with a patient's serum. He adds labeled anti-human IgG
· 21 •
antibodies.

." Which of the following tests is being performed?


. 23 .
.24 . - -w- e- v.- '-u-e-y-o-u- C"fe-e-d-b-. -Ck- ,-
r
134415 : zheng
· 25 •
A. Enzyme-linked imm unosorbent assay

·26 .

L ll~ J
B. Northern blot
· 27 •
C. Polymerase cha in reaction
.28 .
· 29 . D. Southern blot
·30 . " E. Western blot
·3' •
. 32 •
. 33 .
·34 . The correct answer is E. 58% chose this.
Th is is a Western blot. In HIV testing, the Western blot is performed to confirm HIV after an
enzyme-linked immunosorbent assay result is positive. In a Western blot, HIV proteins from
known HIV-infected cell s are separated by electrophoresis. The separated proteins are blotted
on to a filter membrane. This membrane is incubated with the patient's serum. If the
HIV-positive, the patient's serum will conta in anti-HIV antibodies. These antibodies I

.
Th is is a Western blot. In HIV testing, the Western blot is performed to confirm HIV after an
.'9. enzyme-linked immunosorbent assay result is positive. In a Western blot, HIV proteins from
.20 . known HIV-infected cell s are separated by electrophoresis. The separated proteins are blotted
· 21 • on to a filter membrane. This membrane is incubated with the patient's serum. If the patient is
." HIV-positive, the patient's serum will conta in anti-HIV antibodies. These antibodies will bind to
the HIV proteins on the membrane. Then enzyme-linked anti-human IgG antibodies are added;
. 23 .
these recognize and bind to the patient's anti-HIV antibodies. An enzymatic reaction causes
.24 . bands to form on the membrane at the position of speci fi c HIV proteins. The Western blot is
· 25 • positive in the presence of two or more bands corresponding to the HIV proteins p24, gp41, and
·26 . gp120/160, indicating that the patient's serum conta ins specific antibodies to these HIV
proteins and is therefore infected with HIV. False-positive and false-negative results are very
· 27 •
rare with the Western blot, so it is an excellent confirmatory test .
. 28 . Western Blot ElectrophoresIs Enzyme·Lmked Immunosorbent Assay

· 29 . A is not correct. 31% chose this.


·30 . Enzyme-linked immunosorbent assay (ELISA) is used as the primary screening test for HIV
·3' • infection. In ELISA, HIV proteins are tethered to a surface (usually beads or wells) to which the
. 32 . patient's serum is added. If the patient is HIV positive, the patient's anti- HIV antibodies will
.33 . become bound to the tethered HIV proteins. Then enzyme-linked anti-human IgG antibodies are
added; these recognize and bind to the patient 's anti-HIV antibodies. An enzymatic reaction
·34 . causes a color to appear where these anti-human antibodies bind, and the intensity of color is
proportionate to the concentration of the patient's anti -HIV antibody. Unlike the test described
in the question stem, ELISA does not involve electrophoresis.
Electrophofes is Erozyme·Linked ImmunosorbentAss<lY
,Pad "" 2055 27« _ •

.
Item ' 22 of 44
QI D 1369

.,
.'9 .
.20 .
G
i
R<
L

.

UI usmle rx com/ap l#qmax1

Mark

.. .
--<l
Pre Ylous
. -
t:>
Next la b~
_! ues
~.
Notes

A Northern blot is used to detect RNA by DNA-RNA hybridization. In a Northern blot, a sample
of RNA is separated by size on an electrophoretic gel. The separated RNA is transferred to a
filter membrane, where a probe that has been labeled with radioactivity. fluorescence, or dye is
added. This labeled probe is usually a sing le DNA fragment containing a sequence that
C';'\I
Cakul~ tor
+ III :

· 21 •
hybridizes with a specific sequence on the target RNA. Occas ionally a labeled RNA probe is used
· 22 •
instead of a DNA probe. The target RNA can be detected by visualizing the labeled probe to
.23 . which it is hybridized. Northern blots do not involve the use of antibodies .
. 24 . C Is not correct. 4% chose this.
· 25 • Polymerase chain reaction (peR) is used to amplify a segment of DNA, not to confirm the
· 26 . presence of antibodies. In peR, the double-stranded DNA sample is mixed with DNA
· 27 • polymerase, deoxynucleotide triphosphates (dNTPs), and primers complementary to the 5' and
3' regions of the desired DNA segment in the presence of a buffer solution. The temperature is
· 2• •
increased to denature the double-stranded DNA, yield in g single-stranded DNA, and the
· 29 . temperature is lowered, permitting the primers to anneal to the single-stranded DNA. This
·30 . forms double-stranded segments to which the DNA polymerase binds, and a new DNA strand is
synthesized. The process is repeated severa l times to amplify the desired DNA segment.
·3' • Po~merase Chain Reaction
.32 .
. 33 . D is not correct. 4% chose this .

·34 . A Southern blot is used to detect DNA by DNA-DNA hybridization. In a Southern blot, a sample
of DNA is separated by size on an electrophoretic gel. The separated DNA is transferred to a
filter membrane to which a probe labeled with radioactivity, fluorescence, or dye is added. This
labeled probe is usually a single DNA fragment that hybridizes with a specific sequence on the
target DNA. Occasionally a labeled RNA probe is used instead of a DNA probe. The target DNA
can be detected by visualizing the labeled probe to which it is hybridized. Southern blots
involve the use of antibodies.

.,.
. '5.
.'6.
· 17 •

.'9.
. 20.
A Southern blot is used to detect DNA by DNA-DNA hybridization. In a Southern blot, a sample
of DNA is separated by size on an electrophoretic gel. The separated DNA is transferred to a
filter membrane to which a probe labeled with radioactivity, fluorescence, or dye is added. This
labeled probe is usually a single DNA fragment that hybridizes with a specific sequence on the
target DNA. Occasionally a labeled RNA probe is used instead of a DNA probe. The target DNA
can be detected by visualizing the labeled probe to which it is hybridized. Southern blots do not
involve the use of antibodies .
· 21 •
· 22 • Bottom line:
.23 . Western blot, which utilizes protein-antibody hybridization, is used as the confirmatory test for
.24 . the presence of anti-HIV antibodies in a serum sample. For HIV testing, the specific proteins
. 25 .
employed are p24, gp41, and gp120/160 .
Western Blot
· 26 .
· 27 •

· 2• •
iii lif, , ., .. , for year: 2015
. 29 . flll.ST ""D f ... CTS

· 30 .
FAIS. p . 78.1
· 3' • Blotting procedures
·32 . SOUlhern blot A DNA sa mple is enzym~ t ica lly clca' ·cd into SNoW DRoP:
. 33 . small er pieecs. elcr trophorcscd on a gel. and Sout hern == DNA
then tm nsferrc<l to a fi lt er. T he filt er is then Nort hern == RNA
.34. " 'estern == Protcin
,Pad "" 2056 27« _ •

+ G R< UI usmle rx c om/ap l#qmax1 + III :

..
Item' 23 of 44
QID 3118 i
L • Mark --<l
PreYlous
t:>
Next
_!
la b~ ues
~.
Notes
C';'\I
Cakul~tor

.,5 . A 17-year· old girl (omes to the physician because she is concerned with her delayed level of
., sexual maturity. Physical examination reveals scant pubic and axillary hair. She has recently
lost 11 kg (25Ib) and is below the 50th percentile in weight for her height. She complains of
· 17 •
fatigue and light-headedness when standing too qUickly. Her skin has a bronze coloring although
,, she does not reca ll excessive sun exposure. l aboratory studies show the following:
,'9.
.20 . Blood glucose: 47 mg/dL Serum pH: 7.30
K': 6.0 mEq/L
· 21 • He03': 20 mEq/L
· 22 • Partial pressure of carbon dioxide: 36 mm Hg
· 23 •
. 24 .
.25 . Which of the following is the most appropriate treatment for correcting the patient's electrolyte
imbalance?
' 2• •
134415 : zheng
' 27 • We value your feedback!
A. Demeclocycine
· 2• •
. 29 . 8. Dexamethasone

"0.
· 31 •

' 32 •
C. Fludrocortisone

D. Ketoconazole
· 33 •
E. Spironolactone
,'4 .

.,..
.,5.
· 17 •
,,
The correct answer is C. 40% chose this .
This girl has clinical features of Addison disease, or primary adrena l insufficiency, which is
typically caused by autoimmune destruction of the adrenal cortex. Decreased corticosteroid
levels result in hyperkalemia, hypoglycemia, and hyperpigmentation. Low cortisol levels lead to
increased secretion of ACTH and its precursor, pro-opiomelanocortin (POMC). POMC is cleaved
,'9. into ACTH and melanocyte-stimulating hormone (MSH); the high MSH levels lead to the bronze
discoloration of the skin. The low mineralocorticoid level causes a metabolic acidosis because
.20 . hydrogen is not being excreted in the urine. The treatment of Addison disease requires
· 21 • replacement of the adrena l corticoids. Dexamethasone is often used initially for patients with
· 22 • Addison disease presenting in adrenal crisis. It is the preferred glucocorticoid replacement as it
· 23 • does not interfere with assays that measure cortisol levels. However, since dexamethasone
does not have mineralocorticoid activity, fludrocortisone will need to be added to correct
.24 . electrolyte abnormalities because of its mineralocorticoid activity .
. 25. melanocyte Hydrogen DeK<1methasooe F1udrocortisooe AddIson Disease Adrenal Cortex

' 2• •
A is not correct. 15% chose this.
' 27 •
Demeclocycline is a tetracycline derivative used to treat the syndrome of inappropriate ADH
· 2• • secretion (SIADH). This drug binds to the vasopressin receptors on the collecting duct of the
.29 . nephron, consequently blocking its activity. This results in a drug-induced diabetes insipidus-

"0.
· 31 •

' 32 •
like presentation. Due to these adverse effects, it has no utility for patients with Addison
disease.
Demeclocy.:line Tetracy.:line Inappropriate ADH Syndrome Vasopressin (USP) Central Diabetes Insipidus Add ison Disease

8 is not correct. 22% chose this.


· 33 •
Dexamethasone is used in a suppression test to diagnose the cause of a patient's
,'4 . syndrome. If the patient has Cushing disease (pituitary tumor secreting excess
doses of dexamethasone will but doses will cortisol levels. If a
,Pad "" 2056 27« _ •

+ G R< UI usmle rx c om/ap l#qmax1 + III :

..
Item' 23 of 44
QID 3118 i
L • Mark --<l
PreYlous
t:>
Next
_!
la b~ ues
~.
Notes
C';'\I
Cakul~tor

. ,5 . B is not correct. 22% chose this •


., Dexamethasone is used in a suppression test to diagnose the cause of a patient's Cushing
syndrome. If the patient has Cushing disease (pituitary tumor secreting excess ACTH), then low
· 17 • doses of dexamethasone will not, but high doses will suppress cortisol levels. If a patient has
,, either a non-pituitary ACTH-produeing tumor (e.g., small cell lung carcinoma) or a cortisol-
,'9. producing tumor (e.g., adrena l adenoma), then neither low nor high doses of dexamethasone
will suppress cortisol levels. Dexamethasone is also used in the treatment for patients with
.20 . Addison disease presenting in adrenal crisis. It is the preferred glucocorticoid replacement as it
· 21 • does not interfere with assays that measure cortisol levels. However, dexamethasone does not
· 22 • have mineralocorticoid activity and therefore would not be useful in correcting this patient's
electrolyte abnormalities.
· 23 •
Dexamethasone Cushll)9 Syndrome Pitultary·dependent Cushing's disease Pitu itary Neoplasms Addison Disease
.24 .
. 25 . D is not correct. 10% chose this .

' 2• •
Ketoconazole can be used to treat Cushing syndrome (hypercortisolism) because of its
antagonist effect on the synthesis of androgens and cortisol. Ketoconazole inhibits the first step
' 27 •
in cortisol biosynthesis (desmolase) as well as interferes with androgen production. In Cushing
· 2• • syndrome, increased cortisol leads to hyperglycemia, central obesity, female viril ization, and
.29 . hypertension. This drug is also used to treat fungal infections as it blocks synthesis of

"0.
· 31 •

' 32 •
ergosterols, wh ich are essential for the maintenance of the fungal cell wall.
Ketoconazole Hypergly<:emia Cush ing Syndrome Obesity

E Is not correct. 13% chose this.

· 33 •
Spironolactone is used to treat Conn syndrome, which is caused by an ald osterone-secreting
tumor or bi lateral hyperplasia leading to hyperaldosteronism. Conn syndrome is characterized
,'4 . by hypertension due to the effects of aldosterone on sodium retention, I
an increase of secretion, and metabolic alkalosis due to loss

..
,'5.
.,
· 17 •
,,
syndrome, increased cortisol leads to hyperglycemia, central obesity, female viril ization, and
hypertension. This drug is also used to treat fungal infections as it blocks synthesis of
ergosterols, wh ich are essential for the maintenance of the fungal cell wall.
Ketoconalole Hyperglycemia Cush ing Syndrome ObeSity

E Is not correct. 13% chose this.

"9.
.20 .
· 21 •
Spironolactone is used to treat Conn syndrome, which is caused by an ald osterone-secreting
tumor or bi lateral hyperplasia leading to hyperaldosteronism. Conn syndrome is characterized
by hypertension due to the effects of aldosterone on sodium retention, hypokalemia caused by
an increase of potassium secretion, and metabolic alkalos is due to loss of hydrogen ions in the
· 22 •
urine. Spironolactone is an ald osterone receptor antagonist and its administration would
· 23 • actually worsen electrolyte balance in a patient with Addison disease .
. 24 . Spirono lactone Hyperaldosteronism Hypo~alemla Hydrogen Conn Syndrome Sodium Potassium Addison Disease

.25.
' 2• •
Bottom Line :
' 27 •
Addison disease presents with hypotension, hyperka lemia, and hyperpigmentation. This
· 2• •
disease is caused by a lack of cortisol and aldosterone due to an inabi lity of the adrenal glands
.29 . to produce sufficient corticosteroids. Treatment includes exogenous glucocorticoids

"0.
· 31 •
(dexamethasone, hydrocortisone) and mineralocorticoids (fludrocortisone).
Hypotension Dexamethasone Hydrocortisone Fludrocortisone Addison Disease Adrenal Glands

' 32 •
· 33 •

,'4 . iiI! ifi , .j I,) for year: 2015


fllI.ST AI O FACTS
,Pad "" 2056 27« _ •

+ G R< UI usmle rx c om/ap l#qmax1 + III :


Item ' 24 of 44
QID 1410 i
L • Mark --<l
Pre Ylous
t:>
Next
_!
la b~ ues
~.
Notes
C';'\I
Cakul~ tor

·23 " Blood glucose is tightly regu lated after a meal by a peripheral enzyme found in muscle IAA[
tissues. This enzyme is used to phosphorylate glucose for conversion into storage glycogen.
· 24 "
In addition. this process is subject to feedback inhibition by glu(ose-6-phosphate.
· 25 "
· 26 "

·27 "
· 28 "
Which of the following peripheral enzymes is being described?

We value your feedback!


134415; zheng
A. Glucokinase
· 29 "
· 30 " 8. Hexokina se

C. Phosphofructokinase

··3233 ""
· 31 "

D. Pyruvate dehydrogenase
· 34 " E. Pyruvate kinase
· 35 "
· 36 "

· 37 "
· 38 " The correct answer is B. 58% chose this.
· 39 " Hexokinase is found in all t issues in the body and is inhibited by glucose-6-phosphate.
Glucose
· 40 "

A is not correct. 25% chose this.


Glucokinase is found in the liver and in pancreatic f3 cells. It does not und ergo feedback
inhibition, allowing the liver to take up and phosphorylate glucose at higher cD,n c"n!ra!:;D

·23 "
· 24 "
Glucose

A is not correct. 25% chose this.


· 25 "
Glucokinase is found in the liver and in pancreatic f3 cells. It does not und ergo feedback
· 26 " inhibi tion, allowing th e liver to take up and phosphorylate glucose at high er concentrations
·27 "
· 28 "
than peripheral tissues.
Glucose

· 29 " C is not correct. 9% chose this.


· 30 " Phosphofructokinase catalyzes the phosphorylation of fructose-6-phosphate to fructose-1 ,6-
bisphosphate in the glycolysis pathway.

··3233 ""
· 31 " PhosphOrylation Fructose

D is not correct. 4% chose this.

· 34 "
Pyruvate dehydrogenase catalyzes the conversion of pyruvate into acetyl-eoA.
· 35 " E is not correct. 4% chose this.

· 36 "
Pyruvate kinase catalyzes the conversion of phosphoenolpyruvate into pyruvate in the glycolysis
pathway.
· 37 "
· 38 "
Bottom li ne:
· 39 "
· 40 " Hexokinase is ubiquitous in the human body and undergoes feedback inhibition by glucose-6-
phosphate. In contrast, glucokinase is found exclusively in the liver and pancreatic f3 celis and
· 4' " is not feedback-inhibited.
· 42 " Glucose
A S Sis
v S Sis

8
l ock
0
End Bloc k
,Pad "" 2056 27« _ •

+ G R< UI usmle rx c om/ap l#qmax1 + III :


Item' 24 of 44
QID 1410 i
L • Mark --<l
PreYlous
t:>
Next
_!
la b~ ues
~.
Notes
C';'\I
Cakul~tor

·23 "
· 24 "
D is not correct. 4% chose this.
Pyruvate dehydrogenase catalyzes the conversion of pyruvate into acetyl-eoA.
· 25 " E is not correct. 4% chose this.
· 26 " Pyruvate kinase catalyzes the conversion of phosphoenolpyruvate into pyruvate in the glycolys is

·27 "
· 28 "
pathway.

· 29 " Bottom II ne:


· 30 " Hexokinase is ubiquitous in the human body and undergoes feedback inhibition by glucose-G-
phosphate. In contrast, glucokinase is found exclusively in the liver and pancreatic 13 celJs and

··3233 ""
· 31 " is not feedback-inhibited .
Glucose

· 34 "
· 35 " I il1ifi','!,) for year: 2015
FIRS T A I D FACTS
· 36 "

· 37 "
FA15, p. 98."
· 38 " Hexokinilse vs. Ph osphorylation of gl ucosc to yield glucose-6-phospha tc sen"cs as thc 1st step of glycolysis (also
· 39 " glucokinllSe scrves as the 1st step of glycogen spllhesis in the li'·cr). Reaction is ca talY7.ed br either ht"Xokinase
or glucokinasc, dcpending on the tissue. At low glucose conccn trations, hexokinase seques ters
· 40 "
glucose in th e tissue. At high glucose coneentrations, excess glucose is stored in th e lil"Cr.
He~okinase Glucokinase
Liver. ~ cells of pancreas

·23 "
· 24 "
A 21-year-old woman presents to the clin ic complaining of a painful ulcer on her bottom lip
for 3 days. On further question ing she states that she has exper ienced sim ilar lesions in the
IAA[
past. Biopsy of the lesion shows multinuc leated giant celis and intranuc lear inclusions.
· 25 "
· 26 "

·27 "
· 28 "
Whic h of the following additional findings is most likely to be present in this patient?

We value your feedback!


134415; zheng
A. Croup
· 29 "
· 30 " 8. Keratoconjunctivitis

C. Mononucleosis

··3233 ""
· 31 "

D. Neonatal herpes

· 34 " E. Pneumonia
· 35 "
· 36 "

· 37 "
· 38 " The correct answer is 8. 50% chose this.
· 39 " The recurrent nature of this patient's oral lesion and the biopsy findings suggest an infection
· 40 " with herpes simp lex virus type 1 (HSV-1). In addition to the gingivostomatitis (oral and mucosal
lesions), HSV-l causes keratoconjunctivitis, temporal lobe encephalitis, and herpes labial is.
Keratoconjunctivitis is characterized by ac ute onset of pain, blurry vision, and ocular discharge
and may lead to corneal blindness. HSV· 1 is transmitted via respi ratory secretions.
,Pad 9 2056 270 . -

+ G R< UI usmle rx com/ap l#Qmax1 + • :


Item' 2S of 44
QID 2057
L

J.
• Mark -<l
Pre YlDu,
t>-
NeKt
all
la b"" ues
~.
No t es
~'t
c .. k ul~ tor

· 23 •
The recurrent nature of this patient's oral lesion and the biopsy findings suggest an infection
with herpes simplex virus type 1 (HSV-l). In addition to the gingivostomatitis (oral and mucosal
·24 . lesions), HSV-' causes keratoconjunctivitis, temporal lobe encephalitis, and herpes labial is .
. 25 . Keratoconjunctivitis is characterized by acute onset of pain, blurry vision, and ocular discharge
• 2• • and may lead to corneal blindness. HSV-' is transmitted via respiratory secretions .
· 27 •
Herpes Simplex [r.fections GlnglvostomdtillS Encephalitis Herpes Labi~hs Entire temporal lobe
·28 .
A is not correct. 4% chose this .
· 29 •
.30 . Croup is caused by the paramyxovirus parainfluenza and is characterized by inspiratory stridor,
cough, and hoarseness. Croup typically affects children 6 to 36 months of age. It is diagnosed
· 31 . clinically on the basis of a barking cough or stridor and relieved with the use of glucocorticoids.
· 32 • Croup Parainfluenza Stridor Inspiratory stridor

· 33 • C is not correct. 21 % chose thi s.


. 34 .
Mononucleosis is a sequela of infection with Epstein-Barr virus. It is also the cause of Burkitt
· 35 . lymphoma.
· 3• • BurkJn lymphoma

· 37 . D is not correct. 20% chose th is.


· 38 . Neonatal herpes is usually caused by HSV type 2, but may also be caused by HSV type 1. The
.39. patient is infected with HSV (of either type) and, were she to be pregnant, may transmit it to her
child perinatally during vaginal delivery (although infection can also occur, less frequently, in
· 40 .
utero or postnatally).
· 41 •
E i s not cor rect. 5% chose thi s.

.".
.25 .
· 2• •
· 27 •
Neonatal herpes is usually caused by HSV type 2, but may also be caused by HSV type 1. The
patient is infected with HSV (of either type) and, were she to be pregnant, may transmit it to her
child perinatally during vaginal delivery (although infection can also occur, less frequently, in
utero or postnata lly).
E is not correct. 5% chose th is.
·28 . Pneumonia is caused by many types of bacteria, viruses, and fungi. Viruses of the herpesvirus
fami ly known to cause pneumonia include varicella zoster virus and cytomegalovirus .
. 29 .
Pneumonia
.30 .
· 31 .
· 32 • Bottom li ne:

.34.
· 33 •

. 35 .
HSV-1 infection causes gingivostomatitis, keratoconjunctivitis, and temporal lobe encephalitis,
whereas herpes genitalis and neonatal herpes are more commonly, but not exclusively, caused
by HSV-2 infection .
GingivostomatitIS Encepha lItis Gen ital Herpes Entire temporal lobe
· 3• •
· 37 •
. 38 . I iii if') , ., i.) for year: 2015
.39 . FIRST ,0.10 FACTS

· 40 . FAI 5 , p. !59!
Herpesviruses
HSV-l G ingivostomatit is. kcratoco niuncti vitis D . herpes la bialis III. tcmporJllobc cnce phalit is (most
common cause of sporadic e ncephalitis. can prese nt with altercd mc nta l stat us. seizu res. andlor
,Pad "" 2056 27« _ •

+ G R< UI usmle rx c om/ap l#qmax1 + III :


Item' 26 of 44
QID 36SO i
L • Mark --<l
Pre Ylous
t:>
Next la b~
_!
ues
~.
Notes
C';'\I
Cakul~ tor

·23 "
· 24 "
A 28-year-old African-American woman comes to the clinic because she has had fevers and
painful joints for the past month. Her temperature is 37.S0( (100°F). Physical exam ination
reveals an erythematous rash bilaterally on her cheeks. Laboratory studies show a creatinine
· 25 " level of 4.0 mg/dL, a positive Venereal Disease Research Laboratory test for syph ilis, and a positive
· 26 " antinuclear antibody test with a titer of 1:2560.
·27 "
· 28 " This patient is most likely to have which additional physical examination finding?
· 29 " 134415: zheng
· 30 " A. Painful erythema of the first metatarsophalangea l joint

X
··3233 ""
· 31 " B. Painless erythematous oropharyngeal ulcers

C. Painless purple striae on the abdomen


· 34 "
D. Painless reddish-brown wart-like lesions in the groin and perineum
· 35 "
· 36 "
E. Painless ulcer with raised margins on the labia majora

· 37 "
· 38 " We value your feedback!
· 39 "
· 40 "

·23 "
· 24 "
This patient has SLE. Oral or nasopharyngeal ulcerations occur frequently in SLE and are 1 of the
11 classification criteria for SLE. Wh ite plaques can also be present in the oral cavity. The
ulcerations and plaques seen in SLE are often painless .
· 25 " A is not correct. 21% chose this.
· 26 " Th is describes podagra, the sudden onset of severe pain in the great toe associated with gout.
·27 "
· 28 "
Although polyarthralgias or arthritis are common manifestations of SLE, the arth ritis is usually
symmetric, resembling rheumatoid arthritis. Involvement of exclusively the great toe would be
very rare.
· 29 "
Podagra Gout Arthritis Entire great toe Rheumatoid Arthritis
· 30 "
C Is not correct. 7% chose this.

··3233 ""
· 31 "
This describes a cutaneous finding in Cushing syndrome. It may be seen in patients with SLE
who have been treated with long-term high-dose steroids, but it is not characteristic of SLE
itself.
· 34 " Cushmg Syndrome

· 35 "
o is not correct. 8% chose this.
· 36 " This describes a classic secondary syphilis rash: condyloma lata. These lesions are rapid in
· 37 " onset. appearing in a matter of weeks. As mentioned above, SLE patients often have false-
· 38 " positive VORL due to the presence of anti phospholipid antibody.
VORL test
· 39 "
· 40 " E Is not correct. 17% chose this.
This describes the rash that ofte n accompanies primary syphi lis. The Venereal Disease Research
Laboratory (VORL) test for syphilis is not specific, and systemic lupus erythematosus (SLE) can
produce a false-positive VORL due to the presence of anti phospholipid antibody. There
I I iI
,Pad "" 2057 27« _ •

+ G R< UI usmle rx com/ap l#qmax1 + III :


Item' 26 of 44
QID 36SO i
L • Mark --<l
PreYlous
t:>
Next lab~
_! ues
~.
Notes
C';'\I
Cakul~tor
p p ryyp
·23 "
· 24 "
Laboratory (VORL) test for syphilis is not specific, and systemic lupus erythematosus (SLE) can
produce a false-pos it ive VDRL due to the presence of anti phospholipid antibody. There are
several causes of a false-positive VORl. Acute causes include many febrile illnesses and some
· 25 "
drugs, and chronic causes include SLE, leprosy, and rheumatoid arthritis.
· 26 " VORltesl Leprosy Prim<lry s)'pflilis Lupus Erythem<lI05U5, Systemic Rheum<l\oid Arthritis

·27 "
· 28 "
Bottom Line :
· 29 "
· 30 "
Systemic lupus erythematosus (SLE) often manifests with mucosal ulcerations that are usually
painless. Patients with Sl E often have a false-positive VORl.

··3233 ""
· 31 " VORL test Lupus Erythematosus, Systemic

· 34 " hiif." ·,,,, for year: 2015


fIRS T A I O fAC TS
· 35 "
· 36 "
FATS, p , 433, T
· 37 " Systemic lupus erythematosus
· 38 " SYMPTOMS Ci<1ssic presentation: msh, joinl pain. and f(,I '(,T. RAS II O R m i N:
· 39 "
most common ly in a female of rcprodllcti\'{~ Rash (malar IJ or discoid)
age alld African dcsccnt. Arthritis
· 40 " Libman-Sacks endocarditis- no"hacterial, "art· SofT ti ssuesfserosi ti s
like "egelat ions on hot h sides of ,aive I lcll1al ologie disordcl) (e.g .. cytopc ni'ls)
Lnpn s "ephrilis (t)'l>C III hypersensitivit y O ral hl<lsopl,ary" ge al" kers
Re n;,1di sease, R;or n,ll,d phe"omeno"
Photosens itivity. Positi,'c VDR I.JRPR

·23 "
· 24 "
A 6·year·old girl presents to her family physician complain ing of a lesion on her chin for 5
days. Physical examination reveals a lesion on the jaw line with a thick adherent crust and
several en larged cervical lymph nodes. Cultu res of pus from the lesion demonstrate colon ies
· 25 " surrounded by complete clearing on blood agar plates. Gram stains are positive for cocci in chains,
· 26 "

·27 "
· 28 "
Which of following is the most likely sequela of this infection?
134415: zheng
· 29 " We value your feedback!
A. Arthritis
· 30 "
B. Dental caries

··3233 ""
· 31 "
C. Hematuria

D. Jaundice
· 34 "
· 35 " E. Meningitis
· 36 "
F. Otitis media
· 37 "
· 38 " G. Sca lded skin syndrome
· 39 "
· 40 "

The correct a nswer is C. 38% chose this .


,Pad"" 2057 27« _ •

+ G R< UI usmle rx com/ap l#qmax1 + III :


~
. Mark <l I>- -I ~
.' ' C';'\I
QID 2047 i PreYlous Next l a b ' ues Notes Cakul~tor

·23 "
· 24 "
Th is patient likely has nonbullous impetigo caused by the !3-hemolytic, catalase-negative,
bacitracin-sensitive, gram-positive coccus Streptococcus pyogenes. Poststreptococcal
glomerulonephrit is (PSGN) is a complication of S. pyogenes infection and commonly presents
· 25 " with gross hematuria. The latent period f rom impetigo to PSGN is approximately 21 days. Renal
· 26 " function often retu rns to normal in 1-2 weeks, and hematuria resolves within 6 months. PSGN

·27 "
· 28 "
often resolves spontaneously without complication. PSGN can occur following infection with 5.
pyogenes strains that cause either impetigo or pharyngit is. However, rheumatic fever (a nd the
Jones criteria symptoms that go along with it) only occurs fo ll owing infecti on with the strains
· 29 " responsible for pharyngitis. This is an important distinction to make.
Impetigo CATALASE a<ldtr<lcin Ph<lryngltis Ren<l l fUflCtion Rheumatic Fever
· 30 "

A is not correct. 13% chose this.

··3233 ""
· 31 "
Severa l orga nisms such as Staphylococcus aureus (gram-positive, catalase-positive, coagulase-
positive), Neisseria gonorrhoeae, and Streptococcus pneumoniae can cause infectious arth ritis. N.
gonorrhoeae and S. pneumoniae do not cause impetigo. S. aureus is a common cause of impet igo;
· 34 "
however, it would appea r as a gram-pos itive cocci in clusters on a Gram stain, wh ich was not
· 35 " observed here. Infection with Streptococcus pyogenes stra ins that cause pharyngitis can read to
· 36 " polyarthritis as part of th e spectrum of symptoms seen in rheumatic fever. The Jones criteria are
· 37 "
a good way to remember t hese symptoms. Strains of S. pyogenes that cause rheumatic fever do
not ca use impeti go, ma king arthritis an incorrect answer choice for the patient described in the
· 38 " question.
· 39 " CATALASE Impeugo PharyngItiS Arthntis, Infectious Rheumatic Fever Artt1ritis

· 40 "
B is not correct. 9% chose this.
De ntal carries are a complication of infection with virida ns group streptococci (Streptococcus
mutans, for example).

·23 "
· 24 "
Dental carries are a complication of infection with viridans group streptococci (Streptococcus
mutans, for example).
D is not correct. 4% chose this.
· 25 "
Jaundice is typically a clinical mani fest ation of hepatitis.
· 26 " HepatItiS

·27 "
· 28 "
E is not correct. 8% chose this.
In this patient 's age group, the most common causes of meningitis are Streptococcus
· 29 " pneumoniae, Neisseria meningitidis, and HaemophiJus in/luenzae.
· 30 " Mening itis

F is not correct . 11% chose this.

··3233 ""
· 31 "
Common causes of otitis med ia include Streptococcus pneumoniae (catalase-negative, Q-
hemolytiC, optochin-sensitive), HaemophiJus influenzae (gram·negative coccus), and MoraxeJJa
· 34 " catarrhatis.
CATALASE Otitis Medi<l
· 35 "
· 36 " G is not correct. 17% chose this .
· 37 " Scalded skin syndrome is a toxin-mediated disease caused by StaphylOCOCCUS aureus infection .
Staphylococcal Scalded Skin Syndrome
· 38 "
· 39 "
· 40 " Bottom li ne:
Think about PSGN when acute-o nset hematuria is preceded by im petigo or pharyngitis caused
by a group A f3-hemolytic streptococcus.
,Pad "" 2057 26« _ •

+ G R< UI usmle rx com/ap l#qmax1 + III :


Item' 27 of 44
QID 2047 i
- • Mark --<l
Pre Ylous
t:>
Next
_!
la b~ ues
~.
Notes
C';'\I
Cakul~ tor

·23 "
· 24 "
CATALASE Otitis Medi ..

G is not correct. 17% chose this.


· 25 " Scalded skin syndrome is a toxin-mediated disease caused by Staphylococcus aureus infection.
Staphylococcal Scalded Skm Syndrome
· 26 "

·27 "
· 28 " Bottom II ne:
· 29 " Think about PSGN when acute-onset hematuria is preceded by impetigo or pharyngitis caused
· 30 " by a group A l3-hemolytic streptococcus.
Impetigo Pharyngitis

·32 "
· 31 "

·33 "
· 34 "
Iii !if., ,
fiRST AID FACTS
., I,' for year: 2015
· 35 "
· 36 " FA15,p . 127.1

Gram-positive lab algorithm


· 37 "
· 38 "
· 39 "
· 40 "

·21·22 "" A 27-year-old white man develops a deep vein thrombosis in his left lower leg after a 4-hour
car ride. His 59-year-old father had a th rombosis in a mesenteric vein last year, and his 52-
IAA[
·23 "
· 24 "
year-old mother has had repeated superficial vein thromboses.

Which of the following disorders does thi s patient most likely have?
· 25 "
· 26 " 13441S : zheng

·27 "
· 28 "
A. Factor V Leiden mutation
We value you r feedback!

8. Protein 5 deficiency
· 29 "
· 30 " C. Prothrombin gene mutation

D. Von Hippel-Lindau disease


·32 "
· 31 "

· 33 "
E. Von Willebrand's factor deficiency

· 34 "
· 35 "

· 36 "
The correct answer is A. 71% chose this.
· 37 "
This patient has factor V Leiden thrombophilia, an inherited condition that predisposes patients
· 38 " to thromboses, especially those in unu sual locations (eg, the mesente ric vei ns), and to repeated
· 39 " thrombotic events (superficial and/or deep). Many patients have their fi rst thrombotic event
before 50 years of age. Heterozygous patients have a slightly inc reased ri sk of thrombotic
events, whereas homozygous individuals are at a significantly in creased risk.
,Pad "" 2057 26« _ •

+ G R< UI usmle rx com/ap l#qmax1 + III :


Item' 29 of 44 L • Mark --<l t:> _!
la b~
~. C';'\I
Cakul~ tor
QID 1381

· 21 •
i
; .. ' Pre Ylous
, .. Next ues Notes

Protein S is a criti cal co-factor for the inactivation of factors Va and VIlla; deficiency of t his
· 22 • protein can increase an individ ual's risk f or thrombos is. Factor V l eiden mutations, however,
·23 . account for 40%-50% of all inherited t hrombophil ias, maki ng this explanatio n more likely.
Thrombosis FactorV Lelden mlfultion
. 24 •
. 25 . C is not correct. 6% chose this.
· 2• • A mutation in the proth rombin gene, resulting in inc reased plasma levels of factor II, can
increase an individual's risk for thrombosis; factor V Leiden mutations, however, account for
· 27 .
40%-50% of all inherited thrombophilias, making this ex planation more likely.
· 28 . Mutation Thrombosis Factor V Lelden mutation
.29 .
D is not correct. 4% chose this .
. 30 .
Von Hippel-Lindau disease is characterized by abnormal blood vessel growth leading to
· 31 •
ang iomas and hemang ioblastomas in the retina, bra in, and spinal cord as well as in other
·32 . regions of the body; it is not associated with increased risk for thrombosis.
· 33 • ThrombosiS Von Hippel·Lil1d.lu Syndrome Spil1<ll Cord

.34 . E is not correct. 6% chose this.


· 35 . Von Willebrand's deficiency causes a prolonged bleeding t ime, which manifests as increased
· 3• • bleeding after trauma or surgery, nosebleeds, and hematomas; it does not cause increased risk
for thrombosis.
· 37 •
~on Wi liebrand Disease Thrombos is Bleeding time procedure
.38 .
· 39 .
Bottom line:
Factor V Leiden thromboph ilia is associated with an increased risk of recurrent thl'ornb·ol

· 21 •
· 22 •
Bottom li ne:
·23 . Factor V Leiden thromboph ilia is associated with an increased risk of recurrent thrombotic
events, including those of the mesente ric veins .
. 24 .
Thrombophi lia
.25 .
· 2• •
· 27 . h iiI'., , .,. .1 for year: 2015
· 28 . FaST " 10 FACTS

.29 . FAtS, p. 386.2


. 30 . Coagulation cascade components
· 31 • Procoagu latio n WJrf;lrin inh ibits the cn1;r"'C "it;"ni" K
·32 . epo~ide redllCIJsc. Noon;ltCI bel e nteric
bacteri". which produce \'it;l111in K.
· 33 • Vitamin Kdeficiency: I s)'Ilt hcsis offaclors II.
.34 . VII. IX. X. protein C. protei!1 S
,,\\I F carrics/proteds VIII
· 35 . Anticoa gulation t\ntith rombin inhibits activ;ltcd forms of factors
· 3• • ThrornI:>orI-_"""",",", _ II. VII.IX,X.X I,X IL
~<fIhI S I-I cp.uin cnhanccs thc activity of antith rombin.
· 37 •
Principal targcts of antithrombin: thrombin and
.38 . fact or Xa.
· 39 . _ - - """'" - - fitlrl olysrs
~

1 de;wOljt 01 ttm meIh


Factor V Lcidcn mutation produccs a factor V
rcsistant to inhibition b)' activated protein C.
2de$lIuttion ol~ facton
tl''\ is nsed clinieallr as a thrombolytic.
,Pad "" 2058 26« _ •

+ G R< UI usmle rx c om/ap l#qmax1 + III :


Item ' 30 of 44
QID 2007 i
L • Mark --<l
Pre Ylous
t:>
Next
_!
la b~ ues
~.
Notes
C';'\I
Cakul~ tor

· 21 • A 35-year-old man comes to the physician for evaluation of a painless enlargement of a IAA[
· 22 • cervical lymph node. Biopsy of the node shows Hodqkin's lymphoma. The patient is initiated
on a chemotherapeutic reqimen that includes procarbazine . The patient is warned that he
·23 . must take certain dietary precautions whi le taking this drug .
. 24 •
. 25 . Which of the following results is this patient's doctor most likely trying to prevent?
· 2• •
134415 : zheng
· 27 . We value your f eedback!
A. Dystonia
· 28 .
. 29 • B. Hypercholesterolemia
. 30 .
C. Hypertens ive crisis
· 31 •

·32 . D. Obesity
· 33 • E. Serotonin syndrome
.34 .
· 35.
· 3• •
· 37 • The correct answer is C. 46% chose this .
. 38 . Procarbazine is an alkylating agent used in cancer chemotherapy that also inhibits monoamine
· 39 . oxidase. Monoamine oxidase in the gut normally degrades tyramine obtained from the diet.
Therefore patients taking procarbazine shou ld be cautioned against ingesting tyramine-
containing products (e.g., aged cheeses, chicken liver, beer, and wine). Excess tyramine
nerve terminals to release large amounts of catecholam ines. This massive release of

· 21 • containing products (e.g., aged cheeses, chicken liver, beer, and wine). Excess tyramine causes
· 22 •
nerve terminals to release large amounts of catecholam ines. This massive release of
catecholamines can cause a hypertensive crisis with symptoms of headache, nausea,
·23 . hypertension, tachycardia, and possibly cardiac arrhythmias and stroke .
. 24 . Tyramine chicker. aller¥r.ic extract Headache Nausea Tachycardia Hyperter.siw crisis

.25. A is not correct. 12% chose this.


· 2• •
Dystonia is a movement disorder involving sustained muscle contractions causing twisting or
· 27 . repetitive motions. Many antipsychotic agents, as well as antidepressants, can induce dystonia,
· 28 . but procarbazine genera lly does not.
Oystor'<ia
. 29 •
. 30 . B is not correct. 16% chose this.
· 31 • While hypercholesterolemia is a concern following excess ingestion of fatty food and alcohol,
the doctor in this situation is most likely trying to prevent a hypertensive crisis.
·32 . Hypercholesterolemia Hypertensive crisis
· 33 •
. 34 . o is not correct. 5% chose this .
· 35. While obesity is a concern following excessive ingestion of fatty food and alcohol, the doctor in
this situation is most likely trying to prevent a hypertensive crisis.
· 3• •
Obesity Hypertensive crisis
· 37 •
E is not correct. 21% chose this .
. 38 .
Monoamine oxidase inhibitors can cause serotonin syndrome when ingested in combination
· 39 . with other serotonergic drugs, such as selective serotonin reuptake inhibitors, tricyclic
antidepressants, or meperidine. Patients on procarbazine shou ld avoid these drugs but
restrictions alone wi ll not prevent serotonin synd rome.
,Pad "" 2058 26« _ •

+ G R< UI usmle rx c om/ap l#qmax1 + III :


Item ' 30 of 44
QID 2007 i
L • Mark --<l
Pre Ylous
t:>
Next
_!
la b~ ues
~.
Notes
C';'\I
Cakul~ tor

· 21 • E is not correct. 21 % chose this.


· 22 • Monoamine oxidase inhibitors can cause serotonin syndrome when ingested in combination
with other serotonergic drugs, such as selective serotonin reuptake inhibitors, tricyclic
· 23 . antidepressants, or meperidine. Patients on procarbazine shou ld avoid these drugs but dietary
.24 . restrictions alone wi ll not prevent serotonin syndrome .
. 25 . Meperidine Serotonin Syndrome

· 2• •
· 27 . Bottom Line:
· 28 .
Procarbazine, an antineoplastic agent, is a monoamine oxidase inhibitor. Patients taking
. 29 . procarbazine should avoid tyram ine-rich foods to prevent a hypertensive cris is,
.30 . Tyramine Hypener15ive crisis

· 31 •

· 32 .
· 33 • ul;f.,.·,.·1
cts
for year: 2015
fIRST ... I D f ...
.34 .
· 35 . FAt5, p . 523 .4

· 3• • Monoamine oxidase ' ITa n)'lc)'promin e, Phenel zi ne, isoearOOxa zid, Sdegiline (sdeeti\l~ r-.'lAO- B inhibitor).
(MAO) inhibitors (r-. IAO '1:1kcs Pride In Shanghai).
· 37 •
NOllsclec tiH! MAO inhibilion t IC\-ds of amine nenrotransmit ters (norepinephrine. 5-HT,
. 38 . dop:ulline).
· 39 . ((INICAIUSE !\t ypical depression, anxiety.
TOXJClTY H)'pcrtensiye crisis (most notably with ingestion of t)'ramine, which is fou nd in many foods .Inch
as wine 1111(1 cheese): eNS stimnlation . Con traindicated wilh SSRls. TeAs. SI. Joh n', 1I'0rl.
,

·24 . A 30-year-old missionary w ho recently returned from a trip to South America presents to the IAA[
.25 . clinic with symptoms of high fever, headache, and back pain, Physical examination reveals
yellow pigmentation at the conjunctiva of the eyes, and the patient vomits during the visit.
· 2• • The vomitus contains dark-colored blood. Results of a reverse transcription pOlymerase chain
· 27 . reaction test are positive,
· 28 •
. 29 . Which of the following viru ses is a member of the same vi ru s family as the one most likely causing
.30 . this patient's disease?
· 31 • 134415: zheng
We value your feedback!
· 32 . A. Alphavirus
· 33 • B. Arenavirus
.34 .
· 35 . C. Coronavirus
· 3• • 0_ Hepatitis C virus
· 37 •
E. I nfluenza virus
. 38 •
. 39 .
· 40 .
The correct answer is D. 50% chose this.
Yellow fever is caused by a mosquitO-borne virus belonging to the family Flavivi rid ae; this virus
is generally transmitted by Aedes mosqu itoes. Symptoms generally include high fever, I
vomitus (dark-colored blood in the vom itu s as a result of gastrointestinal bleeding),
,Pad "" 2058 26« _ •

+ G R< UI usmle rx c om/ap l#qmax1 + III :


Item ' 3 1 of 44 L • Mark --<l t:> _! ~. C';'\I
QID 1848 i Pre Ylous Next la b~ ues Notes Cakul~ tor

Yellow fever IS caused by a mosqu ito-borne virus belonging to the fam ily Flavlvlndae, this virus
· 24 " is generally transmitted by Aedes mosquitoes. Symptoms generally include high fever, black
· 25 " vomitus (dark-colored blood in the vomitus as a result of gastrointestinal bleeding), jaundice,
headache, and back pain. Diagnosis can be confirmed by testing with reverse transcriptase
· 26 "

·27 "
· 28 "
polymerase chain reaction within 6-10 days. Liver biopsy can show signs of Councilman bodies
(acidophilic inclusions in liver); however, due to the bleeding tendency of yellow fever patients,
liver biopsy is generally not recommended and only advisable postmortem to confirm the cause
· 29 " of death. Preventive treatment includes the use of live-attenuated vaccines before one travels to
an endemic area (eg, South America, Africa). The hepatitis C virus is another member of the
· 30 "
Flaviviridae family, a linear SS(+)RNA vira l family with icosahedral caps ids. This family also
contains the viruses that cause dengue fever, St. Louis encephalitis, Japanese encephalitis, and
·32 "
· 31 "
West Nile virus .
Headache Po~merase Chain Reaction Yellow Fever Vaccine Biopsy of lIVer (procedure) Hepatitis C Dengue Fever Encephal itis
· 33 "
· 34 " A is not correct. 13% chose this.
· 35 " Afphavirus is a member of the Togaviridae fami ly. It is a linear SS(+)RNA virus with an icosahedral
· 36 " capsid. The Togaviridae family includes the virus that causes rubella (German measles) and
Afphavirus (eg, Ea st and West equine encephalitis)
· 37 "
·38 "
· 39 "
Encepha litis Rubella Measles

B is not correct. 16% chose this.


Arenavirus is a member of the Arenavi ridae family. It is a circular SS(-)RNA vi ru s (in two
· 40 "
segments) and possesses a helical nucleocapsid. Other members in the family include
lymphocyt ic choriomeningitis virus (which causes lymphocyt ic choriomeningitis) and Lassa fever
encephalitis.
Encepha litis Lymphocytic Choriomeningitis Lassa Fever

Coronavirus is a member of the Coronaviridae family. It is a linear SS(+)RNA vi rus and possesses
· 24 "
a helical nucleocapsid. This family is the #2 cause of "common colds" (after rhinovirus) and is
· 25 " also associated with the 2002-2003 epidem ic of seve re acute respiratory syndrome
· 26 " Severe Acute Respiratory Syndrome

·27 "
· 28 "
E Is not correct. 7% chose this.
The influenza virus is a member of the Orthomyxoviridae family. It is a linear SS(-)RNA virus (with
· 29 " eight segments) and possesses a helica l nucleocapsid.
Influenza
· 30 "

·32 "
· 31 "
Bottom Line:
· 33 " Yellow fever is a mosquito-borne viral ill ness caused by a f1avivirus, a family of si ngle-stranded
· 34 " linear RNA viruses responsible for hepatitis C and two of the hemorrhagic fevers (yellow fever,
· 35 " dengue). Yellow fever presents with fever, nausea, and pain and can cause jaundice.
Nausea Ye llow Fever Vaccine Hepatitis C Hemorrhagic Fevers. Viral
· 36 "

· 37 "
·38 "
· 39 "
141;1','·".1 for year: 2015
~III.ST "10 F"CTS

· 40 "
FA15. p, 161,1

RNA viruses
VIAUf~Ml tT (NV(t~( RHASIAlKIUA( (Af'SIOITMM£l1t1 MEOJ(AtIMF'OAI~If((

Reoviruses No OS lillc~r
,Pad "" 2058 26« _ •

+ G R< UI usmle rx c om/ap l#qmax1 + III :


Item' 32 of 44
QID 1733 i
L • Mark --<l
Pre Ylous
t:>
Next
_!
la b~ ues
~.
Notes
C';'\I
Cakul~ tor

· 24 " A 24-year-ol d woman returns to the clin ic 2 weeks after treatment for a sore throat that was
culture positive for Neisseria gonorrheae. She was treated with a single dose of ciprofloxacin.
· 25 "
However, today she states she is sti ll symptomatic and is experiencing mild dysuria. Vital
· 26 " signs are normal. Physical exam ination revea ls her pharynx appears erythematous and her tonsils
·27 "
· 28 "
are moderately enlarged. A rapid streptococcal antigen test is negative. Another throat culture is
obtained, and shows the growth of colon ies in Thayer-Martin media.
· 29 "
What is the most appropriate treatment for th is patient?
· 30 "
134415: zheng
· 31 " We value your feedback!
·32 "
· 33 "
A. Amoxicillin

B. Cefixime
· 34 "
C. Ceftriaxone
· 35 "
· 36 " " D. Ceftriaxone plus azithromycin
· 37 "
·38 "
· 39 "
E. Ciprofloxacin

· 40 "

The correct answer is O. 57% chose this.


This patient has pharyngitis and a cultu re positive for Neisseria gonorrhoeae . Thayer-Martin
media contain va ncomycin, colistin, and nystatin. Vancomyci n is bacterici dal for all gram-
positive organisms; colistin is a polymyxin that inhibits growth of all gram-negative

answer s.
· 24 " Th is patient has pharyngitis and a cultu re positive for Neisseria gonorrhoeae . Thayer-Martin
· 25 " med ia contain vancomycin, colistin, and nystatin. Vancomycin is bactericidal for all gram-
· 26 " positive organisms; colistin is a polymyxin that inhibits growth of all gram-negative orqanisms

·27 "
· 28 "
except Neisseria species; and nystatin is microbicidal for all funqal orqan isms. In this case,
initial treatment with a sing le dose of ciprofloxacin was appropriate but did not cure her
infection. This failure could eith er be due to f1uoroquinolone resistance or because pharyngeal
· 29 " infections are more difficult to erad icate than genital-urethral infections. The most appropriate
· 30 " treatment at this time is ceftriaxone plus azithromycin. Intramuscular ceftriaxone is the most
reliable treatment for N. gonarrhoeae infection, and azithromycin (or a tetracycline) should be
· 31 " added because patients with a pharyngeal infection may have a concurrent ch lamydial infection
·32 "
· 33 "
that could be the source of dysuria.
PharyngItiS Col,stin Nystatin Ciprofloxacm Ceftriaxone Azithromycm Tetracycline

· 34 " A is not correct. 6% chose this.


· 35 " Amox icillin is an extended-spectrum penicillin. As a f3-lactam anti biotic, it is ineffective aga in st
· 36 " Chlamydia species, which do not possess a cell wa ll. Amox icillin is no longer a first-line
· 37 " treatment for gonorrhea.

·38 "
· 39 "
Amoxicillin Gonorrhea

B is not correct. 3% chose this.


· 40 " Cefixime is a third-generation cephalosporin that can be taken ora lly. It is used to treat otitis
med ia, upper and lower respi ratory tract infections, and urinary tract infections. In addition,
cefixime is a first-line treatment for gonorrhea. It wou ld not be appropriate in this patient as
she requires treatment for Chlamydia as well.
Gonorrhea Otitis Media Lower respIratory tractlnfecllon Urinary tract Infection
,Pad "" 2058 26« _ •

+ G R< UI usmle rx c om/ap l#qmax1 + III :


Item' 32 of 44 L • Mark --<l t:> _! ~. C';'\I
QID 1733 i
.. , .
Pre Ylous Next la b~ ues Notes Cakul~ tor

· 24 " Ceftriaxone is a third-generation cephalosporin that requires intravenous administration. The


· 25 " third-generation cepha losporins are active against members of the Enterobacteriaceae family,
Neisseria gonorrhoeae, and Haemophilus inf/uenzae. It is also a first-line treatment for gonorrhea.
· 26 "
However, this patient requires treatment for Chlamydia as well.
·27 "
· 28 "
Ceftriaxone Gonorrhea lntra~novs infusion procedures

E is not correct. 3% chose this.


· 29 "
Ciprofloxacin is a fluoroquinolone used primarily to treat atypical pneumonias and urinary tract
· 30 " infections. It is a first-line therapy for gonorrhea. Since this patient failed this therapy,
· 31 "
ceftriaxone is a better option for treating her gonorrhea. An anti-chlamydia l antibiotic should

·32 "
· 33 "
also be administered.
CiproflO)«lCIn Gonorrhll!a C@ftriaxom! Primary atypical prl@umonia Urinary Iract inf@ction

· 34 "
· 35 " Bottom li ne:
· 36 " Intramuscular ceftriaxone is the most reliable treatment for Neisseria gonorrhoeae infection,
· 37 " and azithromycin (or a tetracycline) should be added because patients with gonorrhea may

·38 "
· 39 "
have a concurrent chlamydial infection.
C@ftriaxone Azitl1romycin T@tracychne Gonorrhea

· 40 "

iii I if' , .j 1.1 for year: 2015


FIII.ST AID FACT S

· 24 " A 5-year-old child is brought to her pediatrician after her parents notice a rash . On review of I'" AI
systems, her parents also note recent decreased appetite and energy, and a dry cough A
· 25 "
accompanied by wheezing. Her temperature is 38.6°( (101.5°F), and she has lost 0.9 kg (2 Ib)
· 26 " since her last well-child visit 3 months ago. Abdominal examination reveals hepatosplenomegaly.
·27 "
· 28 "
On further questioning, the parents report that they purchased a puppy approximately 1 month
ago.
· 29 "
l evels of which of the following would most likely be elevated in this patient?
· 30 "
134415: zhe ng
· 31 " We value your feedback!
·32 " A. Eosinophils

·33 "
· 34 "
B. Lymphocytes

C. Mast cells
· 35 "
· 36 " D. Monocytes
· 37 "
·38 "
· 39 "
E. Neutrophils

· 40 "

The correct answer is A. 56% chose this.


@osinophi l La ..... a Migrans. Visc@ral Anor@xia StrabOsmus ParaSitIC Dis@as@s

DisOfder characterized by eosinophilia Vascular Diseases


,Pad "" 2058 26« _ •

+ G R< UI usmle rx c om/ap l#qmax1 + III :


Item ' 33 of 44
QID 3092 i
L • Mark --<l
Pre Ylous
t:>
Next
_!
la b~ ues
~.
Notes
C';'\I
Cakul~ tor

EOSinophil (see Image) levels are often elevated dunnq a parasitic infection. This patient most
· 24 " likely has a parasitic infection such as with Toxocara canis (causative agent of viscera l larva
· 25 " migrans), which can present clinically with asymptomatic eosinophilia and mild symptoms such
· 26 " as fever, malaise, anorexia/weiqht loss, couqh, wheezinq, and skin rash, or with solely ocu lar

·27 "
· 28 "
manifestations (including decreased acuity and strabismus) , The history is almost always
significant for a household dog, as this is a canine roundworm . The disease is self-limited,
subsides slowly over time, and requires no intervention unless serious complications (such as
· 29 " ocular complications) occur. Other causes of eosinophilia include Neoplasm, Asthma, Al lergy,
· 30 "
Collagen vascular disease, and Parasites (mnemonic NAACP). Eosinophils can be distinguished
by their bi-Iobate nuclei and multiple, large, eosinophilic (pinkish) granules containing
· 31 " histaminases and arylsulfatases.
·32 " B is not correct. 11% chose this.
·33 "
· 34 "
Lymphocytes, which include both 8 and T lymphocytes, have round, densely staining nuclei that
fill up nearly the entire cell. They increase in number in response to a viral infection. Normally,
· 35 " they are small and round and have densely stai ning nuclei.
· 36 " C is not correct. 17% chose this.
· 37 " These cells, notable for their cytoplasm filled with large bluish granules, are mast cells, which

·38 "
· 39 "
interact with IgE and degranulate, with subsequent release of hi stam ine, heparin, and other
chemotactic factors. Mast cells would be elevated in an allerg ic reaction, not a parasitic
infection.
· 40 " Cytoplasm Heparin mast cel l Histamlf'.e Paras itic Diseases

D is not correct. 4% chose this.


Monocytes can be easily identified by their kidney-shaped nucleu s. Important in the phagocytic
response, they increase during chronic inflammation (often as a result of viral or fungal
infections) and migrate into the tissue, where they differentiate into macrophages.

Monocytes can be easily identified by their kidney-shaped nucleus. Important in the phagocytic
· 24 " response, they increase during chronic inflammation (often as a result of viral or fungal
· 25 " infections) and migrate into the tissue, where they differentiate into macrophages.
· 26 " Monocytes ChrOllic inflammation

·27 "
· 28 "
E is not correct. 12% chose this.
Neutrophils are the first responders to bacterial infection and are vital in the acute
· 29 " inflammatory response. They have multi-lobed nuclei and pink granules that contain hydrolytic
enzymes, Iysozymes, and myeloperoxidases, which assist them in their role as phagocytes.
· 30 "
Ph<lgocytes
· 31 "

·32 "
·33 "
· 34 "
Bottom Line:
Causes of eosinophilia include neoplasm, asthma, allergy, collagen vascular disease, and
· 35 "
parasites (such as acquired with ingestion of Toxocara canis eggs from an infected pet dog).
Asthma Collagen Disorder ch<lracterized by eosinoph ili,] Vascu lar Diseases
· 36 "

· 37 "
·38 "
· 39 "
hli1',"''''
fi R ST "10 f" CTS
for year: 2015
· 40 "
FA1S. p . 383.3

Eosinophil l:Nfentis ag'lin st helminthie infecti ons (maior Eosi" = pink d),e; pliilic = lovin g.
h,.sic protein), Bilohate nucleus. Packed c,1\lSeS of eusinuphil"l = NAACP,
with large eos inophilic granulc~ of uniform Neoplasia
si7.C D. I-light)'
, . i for ;l1ltigcn. Astlnn"
,Pad "" 2059 26« _ •

+ G R< UI usmle rx com/ap l#qmax1 + III :


Item ' 34 of 44
QID 2939 i
L • Mark --<l
Pre Ylous
t:>
Next
_!
la b~ ues
~.
Notes
C';'\I
Cakul~ tor

· 24 " A 1Q-year-old boy presents with progressive neurologic and dermatologic symptoms that
were first noticed when he was about 1 year old. Of note, his mother says he has a
· 25 "
progressively worsening, staggering gait. In addition, he has thick, dry, and scaly skin. On
· 26 " physical examination he has cataracts and sensorineural hearing loss. The patient also reports that
·27 "
· 28 "
he has been having difficulty with his sense of smell. A nerve conduction study shows delayed
action potential propagation.
· 29 "
A disorder in which of the following organelles is most likely causing this patient's symptoms?
· 30 "
13441S: zheng
· 31 " We value your feedback!
·32 "
· 33 "
A. Cytoskeleton

B. Endoplasmic reticu lum


· 34 "
C. Endosomes
· 35 "
· 36 " D. Mitochondria
· 37 "
·38 "
· 39 "
E. Peroxisomes

F. Ribosomes
· 40 "

The correct answer is E. 33% chose this.


The patient in this vignette has a rare condition called Refsum disease, a disorder in myeli

The patient in this vignette has a rare condition called Refsum disease, a disorder in myelin
· 24 "
formation due to a disorder in fatty acid oxidation or synthesis. The only organelle listed that is
· 25 " involved with long-chain fatty acid metabolism is the peroxisome. Peroxisomes contain
· 26 " enzymes important in oxidation-reduction reactions in some cells, particularly in the catabolism

·27 "
· 28 "
of long-chain fatty acids and in the formation of hydrogen peroxide. Hydrogen peroxide
functions in detoxification and in microbial killing. Disorders of peroxisomes can result in
Refsum disease or adrenoleukodystrophy. Both diseases resu lt in an inability to metabolize
· 29 " very-long-chain fatty acids, and patients with both diseases present with progressive neurologic
· 30 " damage at a young age.
Metabol,sm peroxisome Catabol ism Detoxification procedure Adrenoleukodystrophy Refsum Disease Myelin Hydrogen Hydrogen Peroxide
· 31 "
·32 "
· 33 "
A is not correct. 17% chose this.
The cytoskeleton acts to provide structura l support to the cell. It is not directly involved in fatty
· 34 "
acid synthesis.
MICfotubules associated with cytoplasmic filaments
· 35 "
· 36 " B is not correct. 15% chose this.
· 37 " The endoplasmic reticulum is responsible for steroid synthesis. For example, these structures

·38 "
· 39 "
are particularly numerous in the liver, where cholesterol synthesis primarily occurs. It is not
directly involved in fatty acid synthesis.
Endoplasmic Reticulum Cholesterol

· 40 "
C is not correct. 12% chose this.
Endosomes are intracellular organelles responsible for the ingestion of foreign matter. They are
not involved in the formation of the myelin sheath.
,Pad "" 2059 26« _ •

+ G R< UI usmle rx com/ap l#qmax1 + III :


Item ' 34 of 44
QID 2939 i
L • Mark --<l
Pre Ylous
t:>
Next
_!
la b~ ues
~.
Notes
C';'\I
Cakul~ tor

Endosomes are Intracellular organelles responsible for t he Ingestion of foreign matter. They are
· 24 " not involved in th e formation of the myelin sheath.
· 25 " Organelles M~lin

· 26 " D is not correct. 19% chose this.


·27 "
· 28 "
The mitoc hondria are responsible for ATP synthesis and some fatty acid oxidation. However.
they are not involved in the metabolism of long-chain fatty acids, which are relevant to myelin
· 29 "
sheath synt hesis.
Mltochondrlil Metabolism Myelin
· 30 "
F is not correct. 4% chose this.
· 31 "
·32 "
· 33 "
Ribosomes are responsible for peptide synthesis . Disorders in ribosomes can result in a host of
dysfunctions, but not in fatty acid dysfunction.
IUbosomes
· 34 "
· 35 "
· 36 "
Bottom II ne:
· 37 " Peroxisomes are involved wit h long-chain fatty aci d metabolism, and a peroxisoma l disorder

·38 "
· 39 "
can affect myelin sheath formation.
Metabolism Peroxisomal Disorders Myelin

· 40 "

Ul;fi'·'I.' for year: 2015


~IRST .0.10 ~AC T S

· 24 " Twelve yea rs ago, a lead poisoning cli nic screened a number of local inhabitants for lead
poisoni ng and found 110 people with the disease. To determine if living in public ho using is
· 25 "
a risk factor, a group of patients without lead poison ing was compared, and resu lts are
· 26 " shown in the table below.
·27 "
· 28 "
· 29 "
· 30 "

· 31 "
·32 "
· 33 "
· 34 "
· 35 "
Given t he rarity of lead poisoning in the population studied, a patient who tests positive for lead
· 36 "
poisoning wo uld be estimated t o be how many more times likely to have lived in government
· 37 " ho using?
·38 "
· 39 " A. (100 x 4990)/(4700 x 10)
We value your feedback! 134415: zheng

· 40 "
B. (100/110)/(4700/9690)

C. (100/4800)/(10/5000)

D. (100/9800)/(10/9800)
,Pad "" 2059 26« _ •

+ G R< UI usmle rx com/ap l#qmax1 + III :


Item ' 3S of 44
QID 3559 i
L • Mark --<l
Pre Ylous
t:>
Next
_!
la b~ ues
~.
Notes
C';'\I
Cakul~ tor

· 24 " Twelve years ago, a lead poisoning clinic screened a number of local inhabitants for lead IAA[
poisoning and found 110 people with the disease. To determine jf living in public housing is
· 25 "
a risk factor, a group of patients without lead poisoning was compared, and results are
· 26 " shown in the table below.
·27 "
· 28 "
Outcome
Lead No lead
· 29 " Risk ~son;ng poisoning TOTAL

· 30 "
Exposed (public housing) ' 00 4700 4800
· 31 "
·32 "
· 33 "
Nonexposed (private housing)

TOTAL
.0

II.
4990

9690
5000

9800
· 34 "
· 35 "
Given the rarity of lead poisoning in the population studied, a patient who tests positive for lead
· 36 "
poisoning would be estimated to be how many more times likely to have lived in government
· 37 " housing?
·38 "
· 39 " A. (100 x 4990)/(4700 x 10)
We value your feedback! 134415 : zheng

· 40 "
B. (100/110)/(4700/9690)

C. (100/4800)/(10/5000)

D. (100/9800)/(10/9800)

E. 1 - ([100/4800]/[10/5000])

·27 "
· 28 " The correct answer is A. 39% chose this.
· 29 " This type of study is retrospective, which means that the proportion of people with the disease
was the investigator'S choice (cases were selected). In a case-control study, the only comparison
· 30 " that can be done to evaluate a potential risk factor is the odds ratio, which is calculated as:
· 31 " (cases exposed to risk factor x controls not exposed) I (number of controls exposed to risk
·32 "
· 33 "
factor x number of cases not exposed). It is important to note that case-control studies cannot
determine the incidence or prevalence of a disease and therefore that they cannot predict the
risk of contracting that disease. Also remember that relative risk approaches odds ratio for
· 34 " small probabilities (ie, low prevalence studies).
· 35 "
B is not correct. 15% chose this.
· 36 " This calculation is a di stracter.
· 37 " C is not correct. 32% chose this.
·38 "
· 39 "
Th is is the relative risk, wh ich can on ly be calculated in a prospective study that uses cohorts to
determine the actua l proportion of the population affected by the disease.
· 40 " D is not correct. 7% chose this.
Thi s calculation is a di stracter.
E Is not correct. 7% chose this.
This is , - relative risk, and it represents the relative risk reduction, which is typically the
of the use of a medication. This can only be determined by prospective studies.
,Pad "" 2059 26« _ •

+ G R< UI usmle rx com/ap l#qmax1 + III :


Item ' 3S of 44
QID 3559 i
L • Mark --<l
Pre Ylous
t:>
Next
_!
la b~ ues
~.
Notes
C';'\I
Cakul~ tor

· 24 " E Is not correct. 7% chose this.
· 25 " This is 1 - relative risk, and it represents the relative risk reduction, which is typically the result
· 26 " of the use of a medication. This can only be determined by prospective studies.

·27 "
· 28 " Bottom Line:
· 29 " In a case-control stUdy, the best way to calcu late the potential risk factor is by using the odds
· 30 " ratio.
· 31 "

·32 "
· 33 " I ~ I ;fi , ., 1.1 for year: 2015
FIII.5 T AI D FACTS
· 34 "
· 35 " FAI5,p.48.1

· 36 " Observational studies


SIUDrTYP! DlSKiN W[A5UAElJIIAMPLE
· 37 "

·38 "
· 39 "
Cross-sectional study Collects data from a group of people to 3S5C$S
fr{'{juency of disease (and related risl.: fact ors) at
a particular point in time.
Disc3~e prcl':!lence.
Cau show risk ['lc tor associatiou with diseasc. but
does uot establish causalit),.
Ash, "What is happening?"
· 40 " Case-control study Comp;nes a gronp of people with disease to ;1 Odds rat io (OR).
Retrospective grou p wi thout dise,lsc "Patients with COPD had higher odds of a
Looks for prior aposllfc or risk factor. history of slllokiug than those without COP D."
Asks, "What l13ppeuL>tI?"

· 24 " A 62-year-old woman with stage IV ovarian cancer presents with shortness of breath and
chest pain, Diagnostic imaging indicates a fluid collection within the right pleural space. The
· 25 "
physician prepares the patient for therapeutic thoracentesis, and she inserts the needle in
· 26 " the midaxillary line on the right side, in the lower margin of the ninth intercostal space, as shown
·27 "
· 28 "
in the image,

· 29 "
· 30 "

· 31 "

·32 "
·33 "
· 34 "
· 35 "
· 36 "

· 37 "
· 38 "
· 39 "
· 40 "

· 41 "
Image courtesy alSo/dati G, et 0/. Mu/tidiscip Respir Med (2013)

Which structure(s) does the physician aim to avoid by inserting the needle at this point?
,Pad"" 2100 26« _ •

+ G R< UI usmle rx com/ap l#qmax1 + III :


Item ' 36 of 44 Mark --<l t:> _! ~. C';'\I
.
L •

QID 2732 i Pre Ylous Next la b~ ues Notes Cakul~ tor


9
· 24 " A. Ninth intercostal nerve, artery, and vein
· 25 " We value your f eedback!
B. Parietal pleura
· 26 "

·27 "
· 28 "
C. Phrenic nerve

D. Right pericardiophrenic artery and vein


· 29 "
E. Tenth intercosta l nerve, artery, and vein
· 30 "

· 31 " F. Visceral pleura


·32 "
· 33 "
· 34 "
· 35 " The correct answer Is A. 54% chose this.
· 36 " This patient presents with a pleura l effus ion, the accum ul ation of excess fluid in the pleural
space. Pleural effusions can have a number of causes, including pneumonia, congestive heart
· 37 " fai lure, and cancer. Therapeutic thoracentesis can be performed to rel ieve symptoms and
·38 "
· 39 "
improve respiratory function. The intercost al vein, artery, and nerve run in the intercostal
groove on the inferior surface of each rib. When thoracentesis is performed, the needle is
· 40 " always inserted at the most inferior aspect of an intercostal space to avoid these struct ures
running along the superior aspect of the space.
Pneumonia Thoracentesis Pleural effusion disorder Congestive heart fa,lure

B Is not correct. 7% chose this.


The I pleura is the outer layer of the pleura and is attached to the chest wal l.

· 24 " The parietal pleura is the outer layer of the pleura and is attached to the chest wal l. When
· 25 " performing a therapeutic thoracentesis, it is necessary to pierce the parietal pleura in order to
· 26 "
access th e pleural space.

·27 "
· 28 "
Thoracentesis Pleura

C is not correct. 7% chose this.


· 29 "
The phrenic nerve is found deep in the thorax, running along the med iastinum and pericard ium;
it is too deep to be injured by thoracentesis.
· 30 " Thor<lcentesis Ent're phrenic nerve

· 31 "
·32 "
· 33 "
o is not correct. 4% chose this.
The pericardioph renic vessels travel with the ph ren ic nerve along the mediastinum and
pericardium. These vessels are too deep to be injured by this procedure.
· 34 " Entire phrenic nerve
· 35 "
E is not correct. 18% chose this.
· 36 "
The needle here is inserted above the tenth rib, in the ninth intercostal space. The tenth
· 37 " intercostal vessels and nerve run below the tenth rib, in the tenth intercostal space.
·38 "
· 39 "
Bone structure of tenth rib

F Is not correct. 10% chose this.


· 40 " The visceral pleura is the inner layer of pleura that covers the lungs and adj oining struct ures in
the thorax. It is important to avoid piercing the visceral pleura because of its association with
the lung. Avoidance of this stru cture, however, is not strong ly dependent on inserting the
needle at the inferior aspect of the intercostal space. This is instead best aided by asking
patients to hold their breath.
,Pad "" 21 00 26« _ •

+ G R< UI usmle rx com/ap l#qmax1 + III :


Item ' 36 of 44 L • Mark --<l t:> _! ~. C';'\I
QID 2732 i
, .... Pre Ylous
.'
Next
' ......
la b~ ues Notes

the thorax. It is important to avoid piercing the visceral pleura because of its association with
. . . .... . ..
Cakul~ tor

· 24 "
the lung. Avoidance of this structure, however, is not strongly dependent on inserting the
· 25 "
needle at the inferior aspect of the intercostal space. This is instead best aided by asking
· 26 " patients to hold their breath.
·27 "
· 28 "
Pleura

· 29 " Bottom li ne:


· 30 "
The intercostal nerves, arteries, and vei ns run along the inferior surfaces of their
· 31 " corresponding ribs; to avoid them, a thoracentesis needle is inserted immediately superior to
·32 "
· 33 "
a rib.
Thoracentesis

· 34 "
· 35 "
· 36 "
iii lif, , ., 1.1 for year: 2015
FIR S T AI O FA CTS

· 37 "

·38 "
· 39 "
lung relation s Ri ght lung has 3 lobes; l,eft h:lS Less Lobes (2)
and Li ngula {homolog of righ t Illiddle lobe}. spHce occupied by the heart.
FA1S.p .GO•. 1

Instead of n middle lobe. the left lung hns a

· 40 " Right hlllg is 111OreeommOil site for inhaled The relation of the pulmol1Hry artery to the
foreign body beeause the right mail) stem brOllchlls at eHch IUllg hill1l11 is described by
bronchus is wider and more ver tical than RAUi - Righ t Anterior; Left Superior.
the left

· 24 " A 5+year+old child is brought to the clinic for his chec k+up. On physical exam no apparent
abnormalities are found. Before leaving, the mother states that she has been having
· 25 "
difficulty at work, but has been attempting to make sure her son is being adequately fed. She
· 26 " spec ifi cally asks about protein nutrition, as she has heard that all amino acids are not present in
·27 "
· 28 "
foods that are labelled as being protein+ri ch.

· 29 " Which amino acid should she be sure to includ e in her son's diet?
· 30 " 134415: zheng
We value your feedback!

"
· 31 " A. Arginine

·32 "
IlDil
B. Aspartate
· 33 "
· 34 " C. Glutamate
· 35 " ~
D. Serine
· 36 "

· 37 "

·38 "
· 39 " The correct answer is A. 61% chose this.
· 40 " Arginine is a basic amino acid that may be synthesized by the body. However, during periods of
growth, such as chil dhood, biosynthesis levels are not high enough to allow adeq uate protein
synthesis and the amino acid becomes a necessary part of dietary intake.
Arginine
,Pad"" 2100 26« _ •

+ G R< UI usmle rx com/ap l#qmax1 + III :


Item ' 37 of 44
QI D 4428 i
L • Mark --<l
PreYlous
t:>
Next
_!
la b~ ues
~.
Notes
C';'\I
Cakul~tor
: "
· 24 " Aspartate is not an essential amino acid. It is an aci dic amino acid that can be used as an
· 25 " ingredient in the artificial sweetener aspartame. One of the byproducts of aspartame
· 26 "
breakdown is t he am ino acid phenylalanine and as such, the sweetener should be avoided in

·27 "
· 28 "
patients with phenylketonuria.
Aspartate Phenylalanine

C Is not correct. 14% chose this.


· 29 "
Glutamate is not an essential amino acid. It is an acidic amino acid that is a derivative of the
· 30 " inhibitory neurotransmitter GABA as well as a derivative of the antioxidant glutathione.
· 31 " Glutathione Glutamate

·32 "
· 33 "
D Is not correct. 9% chose this.
Serine is not an essentia l amino acid. It is a polar amino acid, which allows it to be one of the
· 34 " sites of D-ol igosaccharide attachment , along with threonine, when proteins are modified in the
· 35 " golgi apparatus.
Serine Goigi Apparatus
· 36 "

· 37 "

·38 "
· 39 "
Bottom li ne:
Be aware of the essentia l amino acids. Likewise, understand that argin in e and histidine are j
· 40 " required during period of growth.
Amino Acids Arg",ine Histidine
'---------------'-'

· 24 " Glutamate is not an essential amino acid. It is an acidic amino acid that is a derivative of the
inhibitory neurotransmitter GABA as well as a derivative of the antioxidant glutathione.
· 25 "
Glutilthione Gh.l\<Imate
· 26 "

·27 "
· 28 "
o is not correct. 9% chose this.
Serin e is not an essentia l amino acid. It is a polar amino acid, which allows it to be one of the
sites of a-oligosacc haride attachment, along with threonine, when proteins are modified in the
· 29 " golgi apparatus.
· 30 " Serine Goigi Apparatus

· 31 "
·32 "
· 33 "
Bottom line:
Be aware of the essentia l amino acids. Likewise, understand that arginine and histidine are
· 34 "
required during period of growth.
· 35 " Amino Acids ArgInine HIstidIne

· 36 "

· 37 "

·38 "
· 39 "
iii lif, ,
fiRST ... I D f ... C TS
., .. , for year: 2015
· 40 " FA1S. p. 104.3
Aminoac:ids alii)" I.."mino ~c ids arc fOllnd ill proteins
Essential Cl11cogc nic, methi onine (Met). ,,,lille (V;,I). All essential ;,mino ;,cids ne~l to he snppliell ill
histidill c (His). the dict.
,Pad "" 21 00 26« _ •

+ G R< UI usmle rx com/ap l#qmax1 + III :


Item' 38 of 44
QID 1815 i
L • Mark --<l
PreYlous
t:>
Next
_!
la b~ ues
~.
Not es
C';'\I
Ca k ul~ to r

· 24 " A 17-year-o ld boy complains of fever and painful, swollen cheeks. He has recently emigrated IAA[
from Mexico, and one of his relatives had a simi lar illness a few weeks ago. He finds it hard
· 25 "
to talk, eat. and swallow. He denies headache or neck pain.
· 26 "

·27 "
· 28 "
Which of the following findings might also be present on examination?
134415 ; zheng
· 29 " We value your feedback!
A. Hepatomegaly
· 30 "
8. Orchitis
· 31 "
·32 "
· 33 "
C. Papilledema

D. Peripheral edema
· 34 "
· 35 " E. Splenomegaly
· 36 "

· 37 "
· 38 "
· 39 " The correct answer is B. 74% chose this.
· 40 " Mumps is an infectious disease that can cause swollen cheeks. Although not often seen in the
United States because of the measles/m umps/rubella vaccine, mumps occasionally presents in
those who have not been vaccinated . Mumps is caused by an RNA paramyxovirus that replicates
in the upper respiratory tract and causes parotiditis and, frequently, orchitis. Pancreatitis and
meningitis also can be present. Respiratory droplets and direct contact are its means of
transmission. Treatment is usually supported with pain medication.

A is not correct. 7% chose this.


· 24 "
Some infectious causes of hepatosplenomegaly include infectious mononucleosis, acute viral
· 25 "
hepatitis, and cytomegalovirus. Mumps is not common ly associated with hepatosplenomega ly.
· 26 " Infectious MononucleOSIS Mumps

·27 "
· 28 "
C is not correct. 6% chose this.
Papilledema often is a marker of increased intracranial pressure and can be seen in encephalitis
· 29 " and meningitis. Although mumps can present as meningitis, given this patient's denial of
· 30 " headache or neck pain, it is unlikely that mumps would be the cause of increased intracranial
pressure that wou ld result in papilledema.
· 31 "
·32 "
· 33 "
Encepha litis MenlJ)9'tls Denial (Psychology) Headache Papilledema Intracranial Pressure Mumps Neck Pain

D is not correct. 5% chose this.


· 34 "
Peripheral edema has numerous causes. Infectious causes include filariasis and cellu litis.
Mumps is not known to be associated with peripheral edema.
· 35 "
FilariaSIS Cellu litis Mumps
· 36 "
E Is not correct. 9% chose this.
· 37 "
Infectious causes of hepatosplenomegaly includ e infectious mononucleosis, acute viral
· 38 "
hepatitis, and cytomegalovirus .
· 39 " Infectious Mononucleosis
· 40 "

Bottom Li ne:
Clinical manifestations of mumps in clude parotiditis, orchitis, and meningitis. Because of
routine measles/mumps/rubella vaccination, mumps is relatively uncommon in the
,Pad "" 21 00 26« _ •

+ G R< UI usmle rx com/ap l#qmax1 + III :


Item' 38 of 44
QID 1815 i
L • Mark --<l
PreYlous
t:>
Next
_!
la b~ ues
~.
Not es
C';'\I
Ca k ul~ to r

· 24 "
o is not correct. 5% chose this.
Peripheral edema has numerous causes. Infectious causes include filariasis and cellulitis.
· 25 "
Mumps is not known to be associated w ith peripheral edema.
· 26 "

·27 "
FilariasIs Cellu ll1i5 Mumps

E is not correct. 9% chose this.


· 28 "
Infectious causes of hepatosplenomegaly includ e infectious mononucleosis, acute viral
· 29 " hepatitis, and cytomegalovirus.
· 30 " Infectious MononucleosIs

· 31 "
·32 "
· 33 "
Bottom LI ne:
Cl in ical manifestations of mumps in clude parotiditis, orchitis, and meningitis. Because of
· 34 "
routine measles/mumps/rubella vaccination, mumps is relatively uncommon in the United
· 35 " States.
· 36 " Orch itis Meningitis Mumps Measles

· 37 "
· 38 "
· 39 " I ill if; , ., 1.1 for year: 2015
FIRS T AI D FAC TS
· 40 "

FAI5,p . 165.1
Mumps virus ,\ p<lramp o"irns tklt c;lnses Imllllp5. Mnmps m;lkes )'our parotid gland s alHI tesles ,IS
Im common due to erfecti n:nes5 of ~:I~·..I R big ,IS PQ;'\ 1 ~I)()m S

· 24 " A patient's hemoglobin level drops below 7 g/dL, and the hematocrit drops below 21 % IAA[
du ring a prolonged surgery. The patient receives a unit of autologous blood via a central line
· 25 "
in the internal jugular vein.
· 26 "

·27 "
· 28 "
Where did the anesthesiologist aim the needle when placing the central line?
~----------~. 13441 5; zh e ng
· 29 " We value your feedback!
A. Lateral to the common carotid artery
· 30 "
8. Lateral to the external carotid artery
· 31 "
·32 "
· 33 "
C. Medial to the common carotid artery

D. Medial to the external carotid artery


· 34 "
· 35 " E. Medial to the internal carotid artery
· 36 "

· 37 "
· 38 "
· 39 " The correct answer is A. 47% chose this.
· 40 " Common carotid artery Vagus nerve structure
,Pad"" 21 00 26 Y0 . •

+ G R< UI usmle rx com/ap l#Qmax1 + • :


Item ' 39 of 44
QID 2973
L

J.
• Mark -<l
PreYlDU'
t>-
NeKt
all
lab"" ues
~.
Notes
~'t
c .. ku l~tor

..
' 24 •
. 25 .
' 26 •

."
' 28 •

."
. 30 .
· 31 •
· 32 •
. 33 •
. 34 .
' 35 .
' 36 .
' 37 •
• 38 •
. 39 .
' 40 •
. 41 .

Image courtesy of Wikimedia Commons

.
In the neck, the int ernal jugular vein runs alongside the common carotid artery and the vagus
' 24 .
nerve within a fibrous, tubular sheath called the carotid sheath, shown in the red portion in the
.25 . drawing. Within this sheath, the common carotid artery is med ial, the internal jugular vein is
' 26 . lateral, and the vagus nerve runs posteriorly .
." B is not correct. 18% chose this.
' 28 . One could potentially enter the internal carotid artery and/or damage the carotid sinus by
.29 . aiming lateral to the external carotid artery, as these structures are adjacent and just lateral to
the external carotid artery .
. 30 .
Internal carotid artery structure
· 31 •
C is not correct. 16% chose this.
· 32 •
There are no major arteries or veins medial to the common carotid artery.
. 33 .
Common carotid artery
.34 .
D Is not correct. 11 % chose this.
' 35 .
There are no major arteries or veins medial to the external carotid artery.
' 36 .
E is not correct. 8% chose thi s.
' 37 .
The interna l jugular vein lies lateral and not medial to the internal carotid artery.
· 38 •
Internal carotid artery structure
.39 .
' 40 .
Bottom Line:
The common carotid artery is medial, the internal jugu lar vein is lateral, and the vagus nerve is
posterior within the carotid sheath.
,Pad "" 21 00 26« _ •

+ G R< UI usmle rx com/ap l#qmax1 + III :


Item ' 40 of 44
QID 4023 i
L • Mark --<l
Pre Ylous
t:>
Next
_!
la b~ ues
~.
Notes
C';'\I
Cakul~ tor

· 24 " Genetic material is exchanged between cells through several different mechanisms. IAA[
· 25 "
· 26 " In which of the following processes is DNA t ransferred directly from one bact erium to anoth er?

·27 "
· 28 " A. Conjugation
We value your feedback!
134415 : zheng

· 29 " B. Hybridization
· 30 "
C. Transduction
· 31 "
·32 "
· 33 "
D. Transformati on

E. Translocation
· 34 "
· 35 "
· 36 "

· 37 "
The correct answer is A. 68% chose this.
·38 "
· 39 "
Conjugation refers to the process by whic h DNA is transferred from one bact erium to anot her. It
involves prokaryotic cells and the transfer of chromosomal or plasmid DNA. It is the only
· 40 " mechan ism listed that involves direct ce ll-t o-ce ll contact.
B is not correct. 3% chose this.
Hybridization does not describe the exchange of genetic material between cells. It refers to the
act of mix ing different species or varieties of animals or plants.

· 24 " Hybridization does not describe the exchange of genetic material between cells. It refers to the
· 25 " act of mix ing different species or varieties of animals or plants .
· 26 " C is not correct. 14% chose this.
·27 "
· 28 "
In transduction, DNA is tra nsferred by a virus from one cell to another. It does not involve cell -
to-cell contact, and takes places in prokaryotic cells.
· 29 " o is not correct. 10% chose this .
· 30 " Transformation is the process by which purified DNA is taken up by a cel l. It occurs in
prokaryotic and eukaryotic cells, and does not involve cell -to-cell cont act.
· 31 "
·32 "
· 33 "
E is not correct. 5% chose this.
Translocation describes th e transfer of part of a chromosome to a new position on the same or
on a different chromosome with resultant rearrangement of the genes. It is not a mechanism by
· 34 " which bacteria eXChange DNA.
· 35 "
· 36 "
Bottom li ne:
· 37 "
·38 "
· 39 "
Bacterial DNA is transferred directly from one bacterium to another by conjugation.

· 40 "
Iii lif, , ., .. , for year: 2015
fIRST ""D f ... CTS

FA15,p.126.1
,Pad"" 2101 26« _ •

+ G R< UI usmle rx com/ap l#qmaxl + III :


Item ' 41 of 44 L • Mark --<l t:> _! ~. C';'\I
QID 15S4 i Pre Ylous Next la b~ ues Notes Cakul~ tor

· 24 " Investigators of a study are exam ining the reliability of a patient questionnaire. Both inter·
rater reliability and test-retest reliability were examined by the investigators.
· 25 "
· 26 "

·27 "
· 28 "
If the study is determined to have poor test-retest reliability, wh ich of the following can be
concl uded?
13441 5; zhe ng
· 29 "
A. This study suffers from low accuracy
· 30 "
8. This study suffers from low precision
· 31 "
·32 "
· 33 "
C. This study suffers from selection bias

D. This study's poor reliability is due to random error


· 34 "
· 35 " E. This study's poor reliability is due to systematic error
· 36 "

· 37 "
·38 "
· 39 "
We value your feedback!

· 40 "

· 24 " The correct answer Is B. 60% chose this.


· 25 " Test-retest is a statistical method used to determine a test's reliabil ity. To determine reliability,
· 26 "
the same t est is given to a group of part icipant s on two separate occas ions. A correlation of 0.7

·27 "
· 28 "
or higher between the two administrations of the test indicates good reliabil ity. Reliabi lity is
analogous to precision, and poor reliability suggests poor precision.
A is not correct. 17% chose this.
· 29 " Whereas reliabi lity is analogous to precision, validity is analogous to accuracy. One cannot use
· 30 " this test's poor reliability to judge its val idity/accuracy.
· 31 " C Is not correct. 5% chose this.
·32 "
· 33 "
Selection bias occurs when subjects are improperly assigned to two groups. There is no
evidence of selection bias in this question.
· 34 " o is not correct. 7% chose this.
· 35 " Random error shows up as different results for the same measurement (eg, weighing the same
object twice and getting different values). Random errors may be one but not t he sole cause of
· 36 "
low-precision. However, without further knowledge of the study design, it is impossible to
· 37 " concl ud e whether random error is responsible for the low precis ion of this test. Other causes of
·38 "
· 39 "
low precision may include poor quality of test design or inconsistency in the delivery of the test.
E Is not correct. 11% chose this.
· 40 " Systemati c error occu rs when measurements are consistently and incorrectly pushed in th e
same direction (eg, weighing objects with an improperly tared scale, resulting in 0.5 g added to
· 41 "
every measurement). Again, due to the vague nature of this question stem, systematic error
· 42 "
cannot be definitively blamed.
· 43 "
A 196/s
v 5336/s

8
l ock
0
End Block
,Pad "" 21 01 26« _ •

+ G R< UI usmle rx com/ap l#qmaxl + III :


Item ' 41 of 44 L • Mark --<l t:> _! ~. C';'\I
QID 15S4 i Pre Ylous Next la b~ ues Notes Cakul~ tor

E is not correct. 11% chose this.


· 24 "
Systematic error occurs when measurements are consistently and incorrectly pushed in the
· 25 "
same direction (eg, weighing objects with an improperly tared scale, resulting in 0.5 9 added to
· 26 " every measurement). Again, due to the vague nature of this question stem, systematic error
·27 "
· 28 "
cannot be definitively blamed.

· 29 " Bottom Line:


· 30 " Poor pretest reliability indicates a low ability to reproduce a result, or a low precision. Accurate
tests provide results that are close to correct, or "valid,"
· 31 "
·32 "
· 33 "
· 34 " I ill ;fi , ., 1.1 for year: 2015
F1II.S T A I D FACTS
· 35 "
· 36 " FA15, p . 51 . 1

· 37 " Precision VS. accuracy

·38 "
· 39 "
Precision Th e c{) llS istcll CY ,md reproduc ibilit y of 11 tes t
(reliahilit )·).
Th e ahsenC<' of random \'ariation in :J test
Random error . precision in a test.
t precis ion - • sta ndard deviation.
t precis ion - I stati stica l ]lOwer (1- il).
· 40 " Accu ra cy l 11e trueness of tcst measurement s (\'Ilidit )·). Systematic error I aeem 3ey in 3 tes t.
'J11e ahsenee of sys tcmatic erro r or bias in a tcs t

· 24 " A 24-year-old woman with type 1 diabetes mellitus undergoes a Mantoux test as part of a
physical examination for her new job. After 48 hours she is found to have 15 mm of
· 25 "
induration at the test site and treatment is started. Three months later she complains of a
· 26 " "pins and need les" sensation in her lower legs. Physical examination reveals a positive Romberg 's
·27 "
· 28 "
sign .

· 29 " Which of the following is the best next step in management?


· 30 " 134415 : z h e n g

· 31 " A. Discontinue her current medication

·32 "
· 33 "
B. I ncrease the dose of her current medication
We value your feedback!

· 34 " C. Start pyrazinamide


· 35 "
D. Start pyridoxine
· 36 "
E. Start rifampin
· 37 "
·38 "
· 39 "
· 40 "
The correct answer is D. 60% chose this.
This patient presents with latent tubercu losis (T8) infection, as evidenced by her positive
Mantoux (PPD) skin test, and was likely treated with ison iazid (IN H). INH competes with vitamin
86 (pyridoxine) as a cofactor in the synthesis of neurotransmitters, causing a relative
deficiency, and possibly neurotoxicity in the form of parasthesias, peripheral neu
,Pad "" 21 01 26« _ •

+ G R< UI usmle rx com/ap l#qmaxl + III :


Item' 42 of 44
QID 2034 i
L • Mark --<l
Pre Ylous
t:>
Next
_!
la b~ ues
~.
Notes
C';'\I
Cakul~ tor

Th is pati ent presents with lat ent tubercul osis (TB) infecti on, as evi denced by her positive
· 24 " Mant oux (PPD) skin test, and was likely treated with ison iazid (IN H). I NH competes with vitamin
· 25 " B6 (pyridoxine) as a cofactor in t he synthesis of neurotransmitters, causing a relative pyridoxine
· 26 " deficiency, and possibly ne urotoxicity in the form of parasth esias, periphera l neuropathy, and

·27 "
· 28 "
ataxia. Because INH signi fi cantly increases fasting glucose levels in diabetics and slows the
return to baseline, patients with diabetes are at increased risk of neurotoxic adverse effects,
and pyridoxine should be st arted to reverse and prevent neurotoxicity.
· 29 " Ataxia TuberculosIs Hypersensitivity skin testing Vitamin 86 Penpheral Neurop<lthy Glucose

· 30 " A is not correct. 19% chose this.


· 31 " Current recommendations for the treatment of latent T8 inc lude isoniazid t herapy for a
·32 "
· 33 "
min imum of 9 months.
B Is not correct. 6% chose this.
· 34 " Increasing the dose of isoniazid may only worsen her symptoms, and may cause additiona l
· 35 " adverse effects such as hepatotoxicity.
Hep<l totoxicity
· 36 "

· 37 " C is not correct. 8% chose this.

·38 "
· 39 "
Pyrazinamide is one of the drugs used to t reat active T8 infection. It is not recommended as
adj unctive treatment for latent T8 infection.
Pyraz inamide
· 40 "
E is not correct. 7% chose this.
Rifampin is one of the medications used to t reat active T8 infection. It is not recommended as
adj unctive treatment for latent T8 unless compliance is an issue and a short course of
treatment (3 months) is adopted.

E is not correct. 7% chose this.


· 24 "
Rifampin is one of the medications used to t reat active T8 infection. It is not recommended as
· 25 "
adj unctive treatment for latent T8 unless compliance is an issue and a short course of
· 26 " treatment (3 months) is adopted.
·27 "
· 28 "
lUfampin

· 29 " Bottom line:


· 30 "
Isoniazi d treatment can cause a relative pyridox ine deficiency, espec ially in patients with
· 31 " diabetes. Thus these patients should be started on supplementary pyridoxine to reverse and
·32 "
· 33 "
prevent neurot oxic adverse effects.

· 34 "
· 35 " iii lif, , ., 1.1 for year: 2015
FlII. S T AI O FA CTS
· 36 "

· 37 " FAI5.p . I891

·38 "
· 39 "
Isonia zid
MWIAHISM j s),nlhesis of 1I1)'colic ,Kids. IJactcrial cata lase-
peroxidase (l'ncoded b)' Kale) nced('d to
· 40 " COIllTrt IN l-llo aclive metabolite.
(lIN I(All/SE M),oobticte riwllilibercil/osis. Th .... onl)' ag.... nl Different IN H half-Ih'cs in fast \"S. slow
med as solo prophylaxis agai nst TB. acetrlatoTS.
1011(111 Nnlroloxicity. h .... P.lloloxicity. Pyridoxin .... IN II Iniures Neurons and I kp~tocytes.
(I·itmllin B6) .... an prevent n .... llrotoxicit)'.
i I I
,Pad"" 2101 26« _ •

+ G R< UI usmle rx com/ap l#qmaxl + III :


Item ' 43 of 44
QID 2049 i
L • Mark --<l
Pre Ylous
t:>
Next
_!
la b~ ues
~.
Notes
C';'\I
Cakul~ tor

· 24 " A 45-year-o ld homeless man is brought to the emergency room by ambulance after being IAA[
found intoxicated in a nearby park. Physica l examination reveals poor dentition and several
· 25 "
punctate areas on the buccal mucosa that are leaking a thick. yellow exudate. The patient
· 26 " also has a hard, tender, redd ish lump on his mandible.
·27 "
· 28 " Which of the following is the most likely causative organism?
· 29 "
· 30 " A. Actinomyces israelii
We value your feedback! 1 134415 : zheng

· 31 "
·32 " B. Nocardia asteroides

· 33 "
· 34 "
· 35 "
C.

D.
Pseudomonas aeruginosa
Serratia marcescens
LtlDi1 J
· 36 " E. Staphylococcus aUfeus
· 37 "
·38 "
· 39 "
· 40 " The correct answer is A. 75% chose this.
The sinus tracts on the buccal mucosa, the yellow exudate, and the mandibular lesion suggest
that this patient has an oral abscess caused by the gram-positive organism Actinomyces israefii.
A. israellf is an anaerobic bacillus that forms long branching filaments that resemble fungi but
are much th inner by comparison. This organ ism is part of the normal flora of the mouth,
and vagina and tends to cause infection in patients with dental carries, extractions, or

The si nus tracts on the buccal mucosa, the yellow exudate, and the mandibular lesion suggest
· 24 " that this patient has an oral abscess caused by the gram-positive organism Actinomyces israefii.
· 25 " A. israelif is an anaerobic bacillus that forms long branching filaments that resemble fungi but
· 26 " are much thinner by comparison. This organism is part of the normal flora of the mouth, colon,

·27 "
· 28 "
and vagina and tends to cause infection in patients with dental carries, extractions, or
gingivitis/gingival trauma. Men with poor oral hygiene are at highest risk. The characteristic
feature of this organism is the yellow clumps, known as sulfur granules. Treatment of A. israeJii
· 29 " infection is intravenous penicillin for 2-6 weeks, followed by oral therapy with penicillin or
· 30 "
amoxicillin for 6-12 months.
Gingivitis Amoxici llin Buccal mucosa Vagina Sulfur
· 31 "
·32 "
· 33 "
B is not correct. 6% chose this.
Nocardia asteroides is a gram-positive, weakly acid-fast organ is m that grows in a filamentous
pattern similar to some funQi. It is often confused with Actinomyces. However, N. asteroides
· 34 "
mainly causes pu lmonary infection in immunocompromi sed patients.
· 35 "
C is not correct. 5% chose this.
· 36 "
Pseudomonas aerugfnosa is an oxidase-positive, gram-negative bacillus that causes skin infection
· 37 " in burn victims and pneumonia in those with cystic fibrosis. This bacterium produces a blue-
·38 "
· 39 "
green pigment.
Pneumonia Skin Diseases. Infectious

· 40 " D is not correct. 5% chose this.


Serratia marcescens is a member of the Enterobacter fam ily, which is notable for the production of
a bright red pigment. It is a common cause of urinary tract infections, wound infections, or
pneumonia. While it cou ld infect the type of wound described in this question, it would not
produce yellow granules.
,Pad"" 2102 26« _ •

+ G R< UI usmle rx com/ap l#qmax1 + III :


Item ' 43 of 44
QID 2049 i
L • Mark --<l
Pre Ylous
t:>
Next
_!
la b~ ues
~.
Notes
C';'\I
Cakul~ tor

SerratlO marcescens IS a member of the Enterobacter fam il y, which 15 notable for the production of
· 24 " a bright red pigment. It is a common cause of urinary tract infections, wound infections. or
· 25 " pneumonia. While it cou ld infect the type of wound described in this question, it would not
· 26 "
produce yellow granules.

·27 "
· 28 "
Pneumonia Ur inary tract infection

E is not correct. 10% chose this.


Staphylococcus aureus is a catalase-pos itive. coagulase-positive, gram-positive coccus that is the
· 29 "
most common cause of skin and soft tissue infections. Although these bacteria produce gOld-
· 30 " colored colonies when cultured, they do not form the cha racteristic sulfur granules of
· 31 " actinomycotic infections.
·32 "
· 33 "
CATALASE Gold me<l5urement Su lfur

· 34 " Bottom Line:


· 35 "
Branching rods on culture from a patient with an oral infection are li kely Actinomyces israelli.
· 36 "

· 37 "
·38 "
· 39 "
iii I if.') , ., I,) for year: 2015
FIRST .0.10 FACTS

· 40 "
FAI5.p . 1331
Actinomyc~5 V5. Both form long. branching fil a m e nt ~ rescmblin g fungi .
Nocardia ActinomYC~5 Nocordia
G ram-posith-c a nacrobe r.J G rdm-pos iti \'c aerobe

· 24 " A 37-year-old man presents to the emergency department (ED) with difficulty breathing. He
notes that he has been feeling worse over the past few days, but he woke up this morning
· 25 "
gasping for breath. He also complains of a productive cough, which he suspects is
· 26 " preventing him from breathing better. On physical examination, bilateral rales and rhonchi are
·27 "
· 28 "
heard upon auscultation of the lungs. X· ray of the chest is ordered, and shows diffuse interstitia l
infiltrates. Sputum samples confirm a diagnosis of Pneumocystis jiroveci pneumonia. Additiona lly,
HIV tests come back positive. He is placed on several antiretroviral medications to manage his
· 29 " condition. Several weeks later, he returns to his physician for a routine complete blood cell count,
· 30 " which is normal except for a platelet count of 80,OOO/mm 3 • His physician suspects that this
laboratory finding is caused by one of his HIV medications, wh ich acts by blocking protease in
· 31 "
·32 "
· 33 "
progeny virions.

What medication could have caused his thrombocytopenia?


· 34 "
· 35 " 13441 5; zh eng
We value you r feedback!
· 36 "
A. Didanosine

· 37 " B. Enfuvirtide
·38 "
· 39 "
C. Indinavir

· 40 " D. Saquinavir

E. Zidovudine
,Pad "" 21 02 26« _ •

+ G R< UI usmle rx com/ap l#qmax1 + III :


Item ' 44 of 44
QID 4029 i
L • Mark --<l
Pre Ylous
t:>
Next
_!
la b~ ues
~.
Notes
C';'\I
Cakul~ tor

The correct answer IS C. 47% chose this.


· 24 " This patient presents to the ED with a typical picture of P. jiroveci pneumonia, an opportunistic
· 25 " infection. As prophylaxis, HIV patients can be placed on trimethoprim-sulfamethoxazole, or on
· 26 " pentamidine for those with sulfa allergies. Indinavir is a protease inhibitor used in the

·27 "
· 28 "
treatment of HIV that is known to have the associated toxicity of thrombocytopenia. Increased
frequency of kidney stones is another typical adverse effect associated with indinavir. Other
adverse effects of indinavir include gastrointestinal intolerance, hyperg lycemia, lipid
· 29 " abnormalities, change in sense of taste, dizziness or drowsiness, weakness, headache, and
· 30 "
trouble sleeping.
Pneumoni<l Pent<lm idine Indin.lllir Thrombocytopeni<l Hyperglycemia He<ld.lche Opportunistic Irlfections T<lste Perception Dizziness
· 31 "
·32 "
· 33 " A is not correct. 6% chose this.
· 34 " Didanosine is a nucleoside reverse transcriptase inhibitor that works by inhibiting reverse
transcriptase of HIV. This prevents the viral genome from being incorporated into the host DNA.
· 35 "
Like zidovudine, toxicities include bone ma rrow suppression, peripheral neuropathy, and lactic
· 36 " acidosis . This medication can be used as part of highly active antiretroviral therapy (HAART).
· 37 " Zidovudine Bone Marrow Periphera l Neuropathy Acidosis AntiretroYiral Tt~rapy. Highly Active

·38 "
· 39 "
B is not correct. 11% chose this.
Enfuvirtide works by binding to the viral gp41 subunit; thus inhibiting conformationa l change
· 40 " required for viral particles to fuse with (04+ cells. This drug is not a first-line therapy for HIV; it
is usually used in patients with persistent viral replication despite other types of antiretroviral
therapy. Thus it is used in combination with other drugs. Adverse effects include
hypersensitivity reactions and increased risk of bacterial pneumonia.
enflNirtide Hyperseml!lvity

· 24 " Saquinavir is a protease in hibitor used in the treatment of HIV. It is most often used in
· 25 " combination with ritonavir, which is considered the "boosted" formulation. Although protease
inhibitors are assoc iated with gastrointestinal intolerance, hyperglycemia, and lipid
· 26 "
abnormalities, thrombocytopenia is not a known complication of saqu inavir.
·27 "
· 28 "
Saqu,n.lVlr Riton.lvir Hyperglycemia Thrombocytoperlia

E is not correct. 18% chose this.


· 29 "
lidovudine (lDV) is a nucleoside reverse transcriptase inhibitor, and it prevents incorporation of
· 30 " DNA copies of viral genome into host DNA. It is not known to be specifica lly associated with
· 31 " thrombocytopenia, but is frequently associated with bone marrow suppress ion, such as

·32 "
· 33 "
neutropenia and anemia. It may also cause peripheral neuropathy and lactic acidosis. lDV is
also known to cause megaloblastic anemia. This medication is often used in conjunction with
others, as part of HAART. In pregnant mothers, lDV is also used to prevent maternal-fetal
· 34 " transmission of the virus.
· 35 " Zidovudine Thrombocytoperlla Neutropenia Anemia Bone Marrow Peripheral Neurop.lthy Acidosis

· 36 "

· 37 " Bottom II ne:


·38 "
· 39 "
Indinavir is a protease inhibitor used for HIV therapy. A unique adverse effect of indinavir is
thrombocytopenia .
1
· 40 " Ind'n.lvir Thrombocytopenia

iii lif, , ., I,' for year: 2015


FIII.S T .0.1 0 FA CT S
,Pad"" 2102 26« _ •

+ G R< UI usmle rx com/ap l#qmax1 + III :


Item ' 44 of 44
QID 4029 i
L • Mark --<l
Pre Ylous
t:>
Next
_!
la b~ ues
~.
Notes
C';'\I
Cakul~ tor

· 24 "
Zidovudine (ZOV) is a nucleoside reverse transcriptase inhibitor. and it prevents incorporation of
DNA copies of viral genome into host DNA. It is not known to be specifica lly associated with
· 25 "
thrombocytopenia, but is frequently associated with bone marrow suppression, such as
· 26 " neutropenia and anemia. It may also cause peripheral neuropathy and lactic acidosis. ZOV is
·27 "
· 28 "
also known to cause megaloblastic anemia. This medication is often used in conjunction with
others, as part of HAART. In pregnant mothers, ZOV is also used to prevent maternal-fetal
transmission of the virus.
· 29 " Zidovudine Thrombocytopenia Neutropenia Anemia Bone Marrow Penpherat Neuropathy ACidosiS
· 30 "

· 31 "

·32 "
· 33 "
Bottom li ne:
Indinavir is a protease inhibitor used for HIV therapy. A unique adverse effect of indinavir is
thrombocytopenia .
· 34 "
tndmaVir Thrombocytopen ia
· 35 "
· 36 "

· 37 " iii I ifi , .j 1.1 for year: 2015


·38 "
· 39 "
fiRST AI O fACTS

FAt5, p. 194.2
· 40 " High l)' acl i,'c ;ull iretro"i rJl theT;'p), ( HAt\ RT): often initi ated at the time of HI V diagn osi~.
HIVther"py
St rongl~t indi cation fOTpatien ts prcSl:ntin g with AlD S-dcfi ning ill ncss. low CD4+ cdl counts
« 500 ce ll s/mm J), or h igh I'im l load. Rcgimcn cons ists 00 dTllgs to prcl'cnt resistance:
2 N RTl s <l lId I ofthc followi ng: NN RTI or protca>e inh ibiloror inlcgmsc inh ib itor.

You might also like